<<

Criminal Law and Procedure

ESSAY QUESTIONS AND SELECTED ANSWERS

FEBRUARY 2003 CALIFORNIA BAR EXAMINATION

This publication contains the six essay questions from the February 2003 California Bar Examination and two selected answers to each question.

The answers received good grades and were written by applicants who passed the examination. The answers were prepared by their authors, and were transcribed as submitted, except that minor corrections in spelling and punctuation were made for ease in reading. The answers are reproduced here with the of their authors and may not be reprinted.

Question Number Contents Page

1. Civil Procedure

2. Wills/Real Property

3. & Procedure/

4. Professional Responsibility

5. Constitutional Law

6. Community Property QUESTION 3

Don was a passenger in Vic’s car. While driving in a desolate mountain area, Vic stopped and offered Don an hallucinogenic drug. Don refused, but Vic said if Don wished to stay in the car, he would have to join Vic in using the drug. Fearing that he would be abandoned in freezing temperatures many miles from the nearest town, Don ingested the drug.

While under the influence of the drug, Don killed Vic, left the body beside the road, and drove Vic’s car to town. Later he was arrested by police officers who had discovered Vic’s body. Don has no recall of the events between the time he ingested the drug and his arrest.

After Don was arraigned on a charge of first degree , the police learned that Wes had witnessed the killing. Aware that Don had been arraigned and was scheduled for a preliminary hearing at the courthouse on that day, police officers took Wes to the courthouse for the express purpose of having him to identify the killer from photographs of several suspects. As Wes walked into the courthouse with one of the officers, he encountered Don and his . Without any request by the officer, Wes told the officer he recognized Don as the killer. Don’s attorney was advised of Wes’s statement to the officer, of the circumstances in which it was made, and of the officer’s expected testimony at trial that Wes had identified Don in this manner.

Don moved to exclude evidence of the courthouse identification by Wes on grounds that the identification procedure violated Don’s federal constitutional rights to counsel and due process of law and that the officer’s testimony about the identification would be inadmissible hearsay. The court denied the motion.

At trial, Don testified about the events preceding Vic’s death and his total lack of recall of the killing.

1. Did the court err in denying Don’s motion? Discuss.

2. If the believes Don’s testimony, can it properly convict Don of: (a) First degree murder? Discuss. (b) Second degree murder? Discuss.

-18- Answer A to Question 3

1. Did the court err in denying Don’s motion?

The issue here is whether the court properly denied Don’s motion to exclude evidence of the courthouse identification.

Right to Counsel:

Don’s first ground for having the identification evidence excluded is that the procedure violated his federal constitutional rights to counsel.

Sixth Amendment: The Sixth Amendment of the US Constitution, which is applicable to the states through the Due Process Clause of the Fourteenth Amendment, affords citizens the right to counsel during all post-charge proceedings. The Sixth Amendment right to counsel only applies after a Defendant has been formerly charged. Here, Don was arraigned and therefore the Sixth Amendment right to counsel for his post-charge proceedings applies.

Don is arguing that the identification should be excluded on the grounds that it violated his federal constitutional grounds that the identification procedure violated Don’s federal constitutional rights to counsel. However, Don’s attorney was present with him during the identification. Don is going to argue that they were not made aware of the identification and given an opportunity to object to it. His lawyer was told of the identification and its methods, however, it is unclear as to when the attorney was advised of this information. It seems more likely that he was told after the identification had already been made.

However, the Sixth Amendment right to counsel does not apply to identifications of the suspect, since it’s not a proceedings for purposes of the Sixth Amendment right to counsel.

Fifth Amendment: Miranda warning: Miranda warnings also afford the defendant of right to counsel. This right is to have an attorney present during all interrogation or questioning by the police. Miranda warnings are given to someone upon arrest. They include the right to remain silent and that everything said can be used in court against him, the right to have an attorney present and the right to have an attorney appointed by the court if the arrestee cannot afford one. [In] this case the right to counsel issue did not arise as a Miranda violation, since there was no questioning or interrogation of the police, and the Defendant has already been arraigned.

-19- This case involves the Sixth Amendment right to counsel in all post charge proceedings. There are certain occasions where there is no right to counsel, for example, a photo identification of a suspect, taking of handwriting or voice samples, etc.

Because the identification of a suspect by a witness does not afford the Si[x]th Amendment right to counsel, and because Don’s lawyer was actually present with him during the identification, the court was probably correct in denying Don’s motion to exclude the evidence on this ground.

Due Process:

Don’s second ground for having the identification evidence excluded is violation of due process of law.

Identification

The police may use different methods wherein witnesses can identify suspects as the crime doer. These methods include photo identification, lineups and in-court identifications. The identification process must be fair to the suspect and not involve prejudice and therefore not violate his due process rights. For example, the lineup must include others of similar build and appearance as the suspect.

The police in this case were going to have the Wes [sic]identify Don (or the murderer) through photo identification. However, they took him to the courthouse knowing that Don was having his preliminary hearing that day. The photo lineup did not have to be at the courthouse, in fact it is usually at the police station. This questions the officers’ conduct and intent. Don is going to argue that this was done with the express purpose of having Wes see him at the hearing and associate him to the crime. This is prejudicial to Don and a possible due process violation.

The police will argue that it was mere coincidence that they ran into Don in the courthouse and that their intent was to have Wes identify the murderer [sic] through a photo identification. They will further argue that Wes told the officer he recognized Don as the killer without any request by the officer. Therefore his identification was spontaneous and not prompted. Therefore it did not violate Don’s due process rights.

However it is very suggestive to a witness to see a defendant charged with the crime and make the identification that way. If Wes had identified Don independent of that situation then the identification would have been valid and there would be no due

-20- process violation. However, that was Wes’ first and only identification of Don, and Don is going to argue that it was prejudicial and violated due process of law.

Officer’s testimony

Don is further claiming in his motion to exclude that the officer testifying to the identification would be inadmissible hearsay.

Relevance:

For any testimony or evidence to be admitted it must first be relevant. Here the officer’s testimony will be established as relevant since it involves a witness’ identification of the defendant as the murderer.

Hearsay:

Hearsay is an out-of-court statement made by a declarant that goes to the truth of the matter asserted. Hearsay is inadmissible generally because of the Defendant’s right to confront and cross-examine witnesses. The officer is going to testify that he heard Wes tell him that he recognized Don as the killer. The statement was made out of court and goes directly to prove that Don is the killer. Therefore officer’s testimony is hearsay. The question then is, is it admissible hearsay? There are exceptions to the hearsay rule depending on whether the declarant is available or unavailable to testify. There is no indication whether Wes is available or unavailable so we must look at the possible exceptions to the hearsay rule.

Present Sense Impression: Present sense impression is an exception to hearsay. This is when a declarant is expressing a present impression at that moment without an opportunity to reflect. The State will argue that Wes, upon seeing Don, merely expressed that he recognized him as the murderer. It was an impression at the present he was expressing. However this exception will probably not apply in this case since [sic].

State of Mind: The state of mind exception is a statement by the declarant that reflects the declarant’s state of mind. For example, if the declarant said he was going to Las Vegas this weekend, that statement would be admissible to show that defendant intended on going to Las Vegas for the weekend. This is an exception to hearsay and would be admissible. The state of mind exception does not apply to this case.

Excited Utterance: A statement made when the declarant is an excited state caused by

-21- an event and has not had a chance to cool down. Nothing in the facts here indicate that Wes’ identification of Don was an excited utterance and therefore this exception does not apply.

Admission by Party Opponent: Statements made by the opposing party are usually admissible as an exception to hearsay. Here, since the statement the officer is going to testify to is not that of Don’s but rather Wes, the exception does not apply here [sic].

Declaration Against Interest: When a declarant makes a statement that goes against his own interests, that statement is admissible as an exception to the hearsay rule. Again, Wes’ statement was not against his own interest but against Don’s interest and therefore this exception is not applicable here.

None of the other exceptions, including dying declaration, business record, are applicable here. It appears as though the officer’s testimony is inadmissible hearsay. Therefore the court erred in denying Don’s motion on this ground.

2. (a) First Degree Murder

Under common law, murder was with malice aforethought. There were three types: murder, voluntary and involuntary manslaughter. Statutes have categorized murder into de [sic].

The issue here is that if the jury believes Don’s testimony, can Don be convicted of first degree murder[?]

Murder is the killing of another human being. It requires an (physical act) and a mentus rea (state of mind). The defendant must have the requisite state of mind in conjunction with a physical act to be guilty of murder. The state of mind does not have to be the specific intent to kill; it could be a reckless disregard or an intent to seriously injure or harm.

First degree murder is murder with premeditation or murder during the commission of violent (felony murder).

Premeditation: Premeditation and thus first degree murder, is a specific intent crime. Premeditation involves the prior deliberation and planning to carry out the crime in a cold, methodical manner.

-22- In this case there are no facts to indicate that Don planned or premeditated Vic’s murder. In fact, according to the facts, Don was intoxicated and has no recollection of the killing.

Intoxication: There are two states of intoxication, voluntary and involuntary. Voluntary intoxication involves the voluntary ingestion of an intoxicating substance. It is not usually a to murder. Voluntary intoxication can be a defense to specific intent crimes, if it was not possible for the defendant to have the state of mind to form intent.

Involuntary intoxication is the involuntary ingestion of an intoxicating substance, such as with duress, without knowing of its nature, prescribed by a medical professional, etc.

In this case, Don was intoxicated since he ingested the hallucinogenic drug. Although Don was aware of what he was taking when he took it, he will argue that he was forced to take it under duress. Since Vic threatened Don that he would abandon him in freezing temperatures far from any town, Don was forced to take the drug. Although involuntary intoxication is not a defense to murder, it is a proper defense to the specific intent required for premeditation and thus first degree murder.

Since Don did not premeditate the murder nor have the specific intent for premeditated murder, he cannot be convicted of first degree murder.

Felony Murder: Felony murder is murder committed during the commission of an inherently dangerous felony. There are no facts to indicate that Don was committing an inherently dangerous felony, independent of the murder itself. Therefore felony murder probably does not apply in this case and Don cannot be convicted of First degree murder.

2. (b) Second Degree Murder

Second degree murder is all murder that is not first degree and is not made with adequate to qualify for . Second degree murder does not require specific intent.

The issue here is if the Jury believes Don’s testimony, can Don be properly convicted of Second degree murder?

Don is going to use the defense of intoxication. Although intoxication is not a defense to murder, involuntary intoxication can negate a required state of mind. Since it will

-23- probably be determined that Don’s intoxication was involuntary due to duress (see discussion above), Don will argue that he did not have the state of mind required to commit second degree murder. He will be compared to a person who is unconscious. An unconscious person cannot be guilty of murder. Don will argue that he was so heavily intoxicated that he has no recollection of the occurrences and therefore could not have had even the general intent to kill or seriously injure.

Voluntary manslaughter: in order for a murder charge to be reduced to voluntary manslaughter there must be adequate provocation judged by a reasonable standard and no opportunity to cool down and the defendant did not in fact cool down. Nothing in these facts suggests that Don acted under the heat of passion or was provoked in any way. In fact Don does not remember the killing and therefore there is no evidence of provocation.

Since was [sic] involuntarily intoxicated, he could not have the requisite state of mind for murder. Therefore he cannot be convicted of either first degree or second degree murder.

-24- Answer B to Question 3

I. Court’s Denial of Don’s (D’s) Motion

A. Violation of D’s right to counsel

The Sixth Amendment guarantees defendants the presence of counsel at all critical stages of a criminal proceeding which results in imprisonment, as well as providing that the police may not elicit information from a defendant in the absence of counsel once criminal proceedings have been initiated against the defendant, usually in the form of an arraignment. Among those stages of a criminal proceeding which are considered critical are a preliminary hearing, at trial, when making a plea, at sentencing, and at any lineup or show-up conducted following the filing of charges against the defendant.

In this instance, the identification of D occurred after he was arraigned, and thus D did have a right to have counsel present during any lineup or show-up. However, this right to counsel does not extend to photographic identifications, which are not considered adversarial proceedings, but instead only to in-person lineups or show-ups. Thus, the police in this instance will claim that they simply took Wes (W) to the courthouse for the express purpose of having him attempt to identify the killer from photographs of several suspects, something for which D was not entitled to the presence of counsel, and the fact that W witnessed D emerging from the courthouse was not part of their plan, and something for which they should not be held responsible. Further, the police will refer to the fact that when D emerged from the courthouse they made no request that W identify D, but rather W made such an identification completely of his own volition.

D’s counsel will most likely argue that the police were well aware that D would be at the courthouse at that particu[la]r time, and that bringing W to the courthouse ostensibly to view photographs was in reality simply a veiled effort to conduct a one-on-one show-up in which W could identify D, and that D thus had the right to counsel at such a proceeding.

In this instance, the court did not err in denying D’s motion based on grounds that the identification procedure violated D’s Sixth Amendment right to counsel. The Sixth Amendment guarantees the right to counsel at any post-charge lineup or show-up in part to ensure that the defendant’s attorney will be aware of any potentially unfair methods utilized in the identification process, and can refer to these inequities in court. Because D’s counsel was in fact present when W saw and identified D, D’s attorney would be able to raise any objections he had to the identification, and thus D was not ultimately denied his right to counsel. Thus, even if the court were to find that the police bring[ing]

-24- W to the courthouse amounted to a show-up in which D was entitled to the presence of counsel, D was with his attorney when the identification was made, and therefore his right to counsel was satisfied.

B. The identification as violative of due process of law

The Due Process Clause of the 14th Amendment, made applicable to the federal government by the Fifth Amendment, ensures that the prosecution bears the burden of proving each of a criminal case against defendant beyond a reasonable doubt, and also guarantees that a defendant will be free from any identification which is unnecessarily suggestive or provides a substantial likelihood of misidentification.

In this instance, D’s attorney would probably contend that the police bringing W to the courthouse on the date of D’s prelimi[na]ry hearing to view photographs of suspects in fact raised a substantial probability that W would in fact observe D emerging from the courthouse, which is exactly what occurred. D’s attorney would contend that any identification made in this context is extremely suggestive, as the fact that D is emerging from a court of law and was in the presence of an attorney places D in a situation in which he appears to be of a criminal nature, and is likely to lead an eyewitnesses to mistakenly identify D based solely on these circumstantial factors. Further, D’s attorney would argue that the situation was unnecessarily suggestive because the witness could believe the fact that criminal proceedings had already been initiated against D, thus warranting his appearance in court, sufficient evidence, perhaps even in the form of testimony by other eyewitnesses, exists which incriminates D, and may make W more likely to believe that D was the man he had seen commit the killing.

The court probably did not err in denying D’s motion based on the fac[t] that W’s identification was violative of due process of law. The 14th Amendment guarantees against unnecessarily suggestive identifications, or identifications posing a substantial likelihood of misidentification, are intended primarily to remedy lineups in which a criminal defendant is placed in a lineup with other individuals to whom he bears no physical similarities whatsoever. It is unlikely that a court would find that a witness seeing an individual emerging from a courthouse would be so prejudicial as to lead to an unnecessarily suggestive identification.

-25- C. Hearsay

Hearsay is an out-of-court statement being offered to prove the truth of the matter asserted. In this instance, the officer’s planned testimony that W had identified D at the courthouse would qualify as hearsay, as the officer would be testifying to a statement made by W ou[t] of court in order to prove that W identified D.

However, instances in which a witness has previously identified a suspect are admissible as exceptions to the hearsay rule even if the defense is not attacking the identification. Such statements of prior identification are considered to possess sufficient guarantees of trustworthiness that the party against whom they are offered is not denied his Sixth Amendment right to confront witnesses against him. Therefore, the court did not err in denying D’s motion to exclude the evidence of the courthouse identification because the officer’s testimony would in fact not be inadmissible hearsay. II. Crimes for which D may be properly convicted

A. First degree murder

In order to convict a defendant of first degree murder, the prosecution must prove beyond a reasonable doubt that the defendant unlawfully killed a human being with malice aforethought, and that the killing was either premeditated and deliberate or was committed during the commission or attempted commission of an inherently dangerous felony (felony murder). In order to prove malice aforethought, the prosecution must show that defendant acted with an intent to kill, an intent to inflict serious bodily harm, acted with a depraved and malignant heart, or was guilty of felony murder.

In this instance, D’s acts appear to be both the actual and proximate cause of Vic’s (V’s) death, as the facts indicate that D killed V and dumped his body beside the road. However, D would probably be found not to possess the requisite intent to kill or to inflict serious bodily harm by way of his raising the excuse of involuntary intoxication. Intoxication, whether voluntary or involuntary, may be raised to negate the presence of an essential element of a crime, generally intent. In this instance, D’s intoxication would be involuntary, as he did not wish to take the hallucinogenic drug V offered, but was forced to when he feared that if he did not, he would be abandoned in freezing temperatures and his life would be in jeopardy. Ingesting a drug under such circumstances is the virtual equivalent of being unknowingly slipped the drug, or being forced to ingest the drug upon threats of death. As such, D was involuntarily intoxicated, and his intoxication resulted in his having no recall of the events between the time he ingested the drug and his arrest. D thus will be found not to have posssessed the requisite intent to kill or intent to inflict serious bodily harm necessary for a finding of first

-26- degree murder. Further, even if D were not able to rely on the excuse of intoxication in order to negate a requisite mental state, there is no evidence that the killing was premeditated or deliberate, and because it did not occur during the commission or attempted commission of an inherently dangerous felony, there is no basis for finding D guilty of first degree murder.

2. Second degree murder

The jury most likely could not properly convict D of second degree murder, either. Second degree murder also requires the prosecution to prove beyond a reasonable doubt that the defendant intentionally killed a human being with malice aforethought, though it relieves the prosecution of proving the additional elements of premeditation and deliberation or felony murder.

In this instance, D’s involuntary intoxication resulting from his unwillingly ingesting a[n] hallucinogenic drug should sufficiently relieve him from being found guilty of second degree murder, as it negates the requisite mental states of intent to kill or intent to inflict serious bodily harm as discussed above. Further, D should not be convicted under a theory of depraved or malignant heart, as such a finding requires proof of reckless conduct which created a substantial and unjustifiable risk of death or serious bodily harm. A defendant must be consciously aware of the risk he is creating to be guilty of a depraved heart killing, and D’s involuntary intoxication would most likely relieve him of guilt, since he had no recall of the events between the time he ingested the drug and his arrest, and would most likely not be considered to have appreciated the risk of his conduct.

If D were found to have been intoxicated voluntarily, rather than involuntarily, he could be properly convicted of second degree murder for V’s killing. However, if the jury believes D’s testimony that he only ingested the hallucinogenic drug because he feared if he did not he would be left out in the cold and could potentially die, they must find that D was involuntarily intoxicated, which would relieve him of guilt for second degree murder.

-27- ESSAY QUESTIONS AND SELECTED ANSWERS

FEBRUARY 2004 CALIFORNIA BAR EXAMINATION

This publication contains the six essay questions from the February 2004 California Bar Examination and two selected answers to each question.

The answers received good grades and were written by applicants who passed the examination. The answers were prepared by their authors, and were transcribed as submitted, except that minor corrections in spelling and punctuation were made for ease in reading. The answers are reproduced here with the consent of their authors and may not be reprinted.

Question Number Contents Page

1. Criminal Law and Procedure 1

2. Community Property 12

3. Professional Responsibility 22

4. Real Property 29

5. Constitutional Law 36

6. Civil Procedure 45

i Question 1

Bank was robbed at 1 p.m. by a man who brandished a shotgun and spoke with a distinctive accent. The teller gave the robber packets of marked currency, which the robber put into a briefcase. At 3:30 p.m., the police received a telephone call from an anonymous caller who described a man standing at a particular corner in the downtown business district and said the man was carrying a sawed-off shotgun in a briefcase. Within minutes, a police officer who had been informed about the and the telephone call observed Dave holding a briefcase at that location. Dave fit the description given by the anonymous caller.

The officer approached Dave with his service revolver drawn but pointed at the ground. He explained the reason for his approach, handcuffed Dave, and opened the briefcase. The briefcase contained only the marked currency taken in the bank robbery. The officer said to Dave: “I know you’re the one who robbed the bank. Where’s the shotgun?” Dave then pointed to a nearby trash container in which he had concealed the shotgun, saying: “I knew all along that I’d be caught.”

Dave was charged with robbery. He has chosen not to testify at trial. He has, however, moved to be allowed to read aloud a newspaper article, to be selected by the judge, without being sworn as a witness or subjected to cross-examination, in order to demonstrate that he has no accent. He has also moved to exclude from evidence the money found in the briefcase, his statement to the officer, and the shotgun.

How should the court rule on Dave’s motions regarding the following items, and on what theory or theories should it rest:

1. Dave’s reading aloud of a newspaper article? Discuss.

2. The currency? Discuss.

3. Dave’s statement to the officer? Discuss.

4. The shotgun? Discuss.

1 Answer A to Question 1

1)

This question raises issues involving Dave’s rights under the 4th Amendment and 5th Amendment.

Dave’s Reading Aloud of a Newspaper Article

A criminal defendant may be required to give a voice sample. This does not violate a defendant’s right against self-incrimination.

A criminal defendant is allowed to submit evidence that will prove that he could not or did not commit the crime. Here, the alleged robber spoke with a distinctive accent. Dave seeks to read a newspaper article to the jury in order to show that he was not the robber because he does not have an accent. The key issue, however, is whether Dave may do this given that he does not want to be sworn in as a witness or subjected to cross- examination. By doing so, Dave is denying the prosecution the right to cross-examine him and to test whether he is being truthful. It is possible for Dave to fake an accent or to have taken voice lessons to change this previous accent. All of these are factors that the prosecution should be permitted to test on cross-examination. Because the prosecution will not be given the right to cross-examine Dave, Dave’s request to read to the jury should be denied.

THE CURRENCY

The 4th Amendment prohibits warrantless searches and seizures by a police officer in an area where a person has a reasonable expectation of privacy. The 4th Amendment applies to the states via incorporation into the 14th Amendment. Warrantless searches are permitted under certain circumstances.

State Action:

The 4th Amendment prohibits warrantless searches and seizures by a state actor. Here, the officer was conducting the search and seizure as a police officer and therefore state action is involved. In addition, the officer was searching Dave’s briefcase - - an area where Dave had a reasonable expectation of privacy.

Search Incident to a Lawful Arrest

An officer does not need a search warrant if the search is done pursuant to a lawful arrest. Under this exception to the warrant requirement, an officer may search the person arrested and search the area within the person’s immediate control if the officer suspects that the area would contain contraband or a weapon. In order for this exception to apply, the arrest

2 must have been lawful.

The officer arrested Dave after receiving a phone call from an anonymous caller stating that a man fitting Dave’s description was carrying a sawed-off shotgun in a briefcase. An officer may arrest a person in public without a warrant if the officer has probable cause to believe that the person has committed a crime. A tip from an anonymous informant can be used as a basis for establishing probable cause if the officer reasonably believes that the tip is reliable. Here, the officer knew that a Bank was robbed at 1 p.m. by a man who had a shotgun. The officer received a tip at 3:30 saying that a man was standing at a corner with a sawed-off shotgun in a briefcase. The combination of the call, with the circumstances surrounding the Bank robbery are sufficient to give the officer probable cause to arrest Dave in public without a warrant.

Because the arrest was lawful, the officer could search Dave and the area within his immediate control if the officer suspects that the area would contain contraband or [a] weapon. Here, the officer suspected that the briefcase would have a sawed-off shotgun and it was within Dave’s immediate control. Thus, the officer could search the briefcase. Any evidence found during this valid search could be admitted.

Plain View

Any evidence seen by an officer when the officer has a lawful right to search the area may be admitted. Here, the officer had a right to search Dave’s briefcase under the exception to the warrant requirement for searches incident to a lawful arrest. Because the marked currency was in the officer’s plain view during this search, the currency can be admitted as evidence against Dave.

Stop & Frisk

An officer who has reasonable suspicion to believe that a person is engaged in criminal activity may stop the suspect and conduct a warrantless frisk for weapons. An officer may not look inside containers during a stop & frisk. Thus, this exception to the warrant requirement will not be a basis for admitting the currency.

DAVE’S STATEMENT TO THE OFFICER

The 5th Amendment privilege against self-incrimination applies when there is state action and a custodial interrogation of a person. It gives a defendant a right to refuse to give testimonial evidence that would result in self-incrimination.

State Action

As discussed above, the action of the police officer involves state action.

3 Custodial Interrogation

Under the 5th Amendment, an officer must read a suspect his Miranda rights before conducting a custodial interrogation. A person is in custody if he believes that he is not free to leave the officer’s control. Here, the officer approached Dave with his service revolver drawn and handcuffed Dave. Under these circumstances, Dave was in custody because he was not free to leave the officer’s control.

An interrogation is any communication by the police to the suspect that is likely to elicit a response. Before engaging in a custodial interrogation, the officer must read the suspect his Miranda rights, which involves the suspect’s right to remain silent and the right to ask for counsel.

Here, the officer would argue that his statement to Dave “I know you’re the one who robbed the bank. Where’s the shotgun?” was not an interrogation and that Dave’s response to this statement was a voluntary statement. A statement by a suspect that is blurted out is admissible. Dave, however, would argue that the officer’s statement “I know you’re the one who robbed the bank” is a statement likely to elicit a response and that Dave would not have said anything had he not been prompted by the officer’s accusation. Dave would probably win on this argument because accusing a suspect who is in handcuffs of committing a crime is the type of statement likely to elicit a response.

As a result, Dave’s statement to the officer cannot be admitted because Dave was not read his Miranda warnings prior to the interrogation. Dave’s statement could be admitted for impeachment purposes if Dave takes the stand and could be admitted in a grand jury proceeding.

THE SHOTGUN

The admissibility of the shotgun also depends on an analysis of whether Dave’s 5th Amendment privilege against self-incrimination was violated when the officer asked Dave where the shotgun was without reading Dave his Miranda rights.

As discussed above, state action was involved and Dave was in custody when the officer asked him where the shotgun was. If the question to Dave was improper, the shotgun cannot be admitted because it is the fruit of a poisonous tree.

Dave will argue that he pointed to the trash container as a result of the officer’s interrogation and that he wouldn’t have done so but for the officer’s interrogation. The officer will argue that Dave’s “pointing” to the trash is not testimonial and therefore the 5th Amendment does not apply. The 5th Amendment does not typically apply to conduct but it may apply if the conduct is testimonial in nature. Here, Dave’s pointing to the shotgun could be considered testimonial in nature because Dave was telling the police the location of his weapon.

4 Courts, however, allow an officer to question a suspect about the location of the weapon without giving Miranda warnings if it is necessary because of exigent circumstances. In other words, if the officer thinks that there might be a weapon laying around that might pose an immediate danger to the public the officer can question the suspect immediately following the arrest and pre-Miranda as a means of securing the premises and protecting the public.

Here, the shotgun is probably admissible under this exception because the officer knew that there was a shotgun used in connection with the robbery and has reason to believe that Dave was connected with this robbery given the discovery of the marked bills. Thus, the officer could ask about the location of the gun to secure the premises.

5 Answer B to Question 1

1)

Dave’s Reading Aloud the Newspaper Article

The Fifth Amendment protects against self-incrimination. Therefore, the prosecution cannot compel D to testify against his will. Furthermore, the Sixth Amendment allows an accused to confront his accusers. Here, D wants to read aloud a newspaper article of the judge’s cho[o]sing to demonstrate that he does not have a distinctive accent, which is something that was described by the bank teller. D would like to do this without being sworn in or subject to cross-examination by the prosecution. The issues hinges [sic] on whether reading the statement aloud is testimonial in nature. If it is testimonial in nature than [sic] the judge will not allow Defendant to do this without being sworn in because he will be a witness.

Non-Testimonial

Here, Defendant wishes to demonstrate that he does not have an accent. The content of his speech is not testimonial in nature because he is not asserting this own thoughts, opinions, observations, or knowledge, which are things that a witness would do. Here, D is not making any statements of fact. The evidence is relevant to demonstrate that D doesn’t have an accent, but it is only the sounds of his speech that matters [sic] and not the content. It is akin to showing tattoos, needle marks, or hair color. Therefore, reading a newspaper is sufficiently nontestimonial and D will be allowed to do this.

The prosecution may argue that this is testimonial because D can alter the way that he is speaking and if they were allowed to cross-examine him this would come to light in front of a jury that he was faking. This argument would fail because there is no content for the prosecution to cross-examine him on and they can sufficiently argue in closing that he may be faking or offer a witness to counter his assertion that he does not have an accent.

Dave will succeed because his reading the newspaper aloud is sufficiently nontestimonial and will[,] therefore, be admitted at trial.

The Currency

The Fourth Amendment, incorporated to the states via the Fourteenth Amendment, protects against unreasonable searches and seizures. In order to bring an action under the Fourth Amendment, the defendant must have standing and the action must be done by a government actor.

6 Standing

In order to have standing one must have a reasonable expectation of privacy in the items seized or search[ed]. Here, Defendant was seized and his briefcase searched. Therefore, since D had a reasonable expectation of privacy in himself and his briefcase he has standing.

Government Actor

A police officer is [a] government actor for the purposes of the Fourth Amendment.

Seizure of D

In order to arrest a person an officer must have a warrant based on probable cause signed by a neutral magistrate. Absent a warrant a search or seizure is per se invalid absent an exception. Here, there was no warrant for D’s arrest.

Dave would argue that this was an illegal arrest and that the officer did not have probable cause based on this information first and foremost because of the amount of time passed between the robbery of the bank and the time that the officer contacted defendant two and half hours later. D would argue that it is unreasonable to think that a bank robber is going to just stand out in the middle of public [sic] with a gun two and a half hours later. Furthermore, D will argue that he was a man with a briefcase downtown, which is hardly a novel notion. Moreover, D will argue that the anonymous caller lacked any indicia of reliability and was not corroborated by anything other than the fact that D just happened to match the description of a man with a briefcase, but with no sawed-off shotgun. D will also point out that the bank teller described a shotgun whereas the anonymous calle[r] described a sawed-off shotgun, which are noticeably different. Therefore, D will argue that the officer had no probable cause to arrest D based on this information and therefore, the arrest was illegal.

The prosecution would like[ly] respond that the initial contact with D by the police officer was a detention based on reasonable articulable facts or if it rose to the level of an arrest that there was probable cause.

Detention based on Reasonable suspicion

The prosecution may argue that D was not arrested by [sic] merely stopped in order to investigate whether criminal activity was afoot. During a detention, an officer must have reasonable suspicion that criminal activity is afoot. Here, the officer had two basis [sic] as will be described in more detail below. The officer had the matching description of the bank robber with the briefcase and he had an anonymous caller who described D with a gun at the corner. Therefore, the officer had sufficient probable cause to contact D. The officer may detain a suspect long enough to investigate and determine if there is criminal

7 behavior or not. Here, the officer drew his weapon and handcuffed D because he believed that D had a gun based on the anonymous tip and the bank robbery information.

D will argue that this was an arrest and not merely a stop. D will argue that the officer approached him with a weapon drawn and handcuffed him and[,] therefore, it was an arrest because D was not free to leave.

The court will hold that this was a detention based on reasonable suspicion and was, therefore, not in violation of the Fourth Amendment.

Probable Cause

Moreover, the officer had probable cause to arrest D based on the information that he had. If an officer has probable cause to believe that someone has committed a felony they may arrest that person without a warrant as long as within 48 hours a magistrate makes a determination that there was probable cause for the arrest. If a person commits a it must be committed in the officer’s presence for an arrest.

Here, the officer had reason to believe that D robbed a bank. Robbery is a felony under the law. The information that the officer had at the time that he contacted the defendant was that a bank was robbed at 1 pm, by a man with a shotgun who spoke with a distinctive accent. The robber had in his possession marked currency given to him by the teller which he put into a briefcase. The officer received a tip from an anonymous caller who described a man standing at a corner with a sawed-off shotgun in a briefcase. The officer arrived to [sic] the corner within minutes of the call, saw Dave there holding a briefcase and matching the description given by the anonymous caller.

The prosecution will argue that under the “totality of the circumstances” the officer’s arrest was based on probable cause. Not only did the officer have reasonably articulable facts to contact D and investigate him to see if he had a weapon but also to arrest him in connection with the bank robbery. As the facts described above detail the officer had description of Defendant and just because minutes after the phone call he no longer had the weapon does not mean that the officer should just walk away without any investigation. The officer has a duty to investigate and determine if there is a safety issue and what is going on.

Therefore, based on the totality of the circumstances the officer has probable cause to arrest Dave and the seizure of D was not unlawful.

Search of Briefcase

Here, the search of the briefcase also requires and [sic] warrant exception because there was no additional warrant to search the briefcase. D had a reasonable expectation of privacy in his briefcase because it was something that was closed and not open to public

8 view or scrutiny.

Probable Cause

As stated above the officer had probable cause to believe that Defendant was armed with a shotgun and therefore had sufficient probable cause to search the bag to ensure for his own safety and the safety of others where the gun was. During a detention an officer may “pat down” an individual if they believe the person may have a weapon. Here, the officer did believe that D had a weapon which was something that could have easily fit in the briefcase. Therefore, the search of the briefcase was lawful.

Search incident to Arrest

Furthermore, as stated earlier there was sufficient probable cause for a lawful arrest. In a search incident to a lawful arrest, the arrest must be lawful, and the officer can search the Defendant and anything within the “wingspan” of the suspect under Chimel. Here, D was holding the briefcase which was sufficiently in his wingspan. Therefore, the search of the briefcase was a lawful search incident to arrest.

Finding the Currency

Although the officer had probable cause to search the briefcase for a weapon, he saw the currency in plain view when he opened the briefcase. Something is in plain view in a place the officer may lawfully be and without the officer touching or moving it around.

Conclusion: The currency found in the briefcase will not be suppressed.

Dave’s Statements to the Officer

Miranda Miranda protects against coerced confessions. It is a profalactic [sic] measure designed to provide additional protection for the 5th Amendment, incorporated to the states through the 14th Amendment, against self-incrimination. According to Miranda, if a suspect is interrogated and in custody, he is to be warned of his right to remain silent, that anything that he says can be used against him, that he has a right to an attorney and if he can’t afford an attorney one will be appointed for him.

Here, Dave made two statements to the police officer and each needs to be analyzed separately to determine the admissibility. The first statement was when Dave pointed to the nearby trash can and the second is when he said “I knew all along that I’d be caught.”

Pointing to the trash can

Statements can be express or implied. An express statement is an oral statement. An

9 implied statement is one made with assertive conduct or by silence. Here, Dave pointed to the trash can in response to the Officer’s question “Where’s the shotgun?” In custody

Custody occurs where the suspect is not free to leave. At this point Dave was handcuffed standing on a street corner. This is sufficiently in custody for Miranda.

Interrogation

Interrogation occurs where the officer asks questions in order to elicit a response. Here, the officer asked where the gun was and D pointed to the trash can. Therefore, this was interrogation.

Dave’s argument will succeed because the conduct of pointing to the gun should be suppressed and inadmissible at trial.

“I knew all along that I’d be caught”

This was an express statement made by Dave after he pointed to the gun. As stated above Dave was in custody, but the difference with this statement is that it was a spontaneous statement. The officer did not ask D if he knew that he would be caught. He asked him where the gun was. The prosecution would argue that the [sic] D’s statement was spontaneous and therefore, not a violation of Miranda and should be admissible. D would argue that this was a result of a custodial interrogation and the statement should not come in.

Dave’s argument will fail because this was a spontaneous statement and is, therefore, admissible.

Shotgun

The shotgun was found as a result of D’s pointing to where it was located and therefore D will argue that it is inadmissible as the result of a Miranda violation.

Fruit of the poisonous Tree

When there are violations of the Fourth Amendment the exclusionary rule helps to protect against unreasonable officer conduct by excluding the evidence. D would likely argue that as a result of his unmirandized statement the gun should be supressed. This argument would likely fail because courts have not readily applied the fruits of the poisonous tree doctrine to evidence resulting from Miranda violations. Furthermore, under the doctrine of inevitable discovery the officers would have likely found the shotgun independent of D’s pointing to it. Generally, when officers find the suspect of a crime who had only minutes before been seen with a weapon and now has no weapon to [sic] search the area around

10 where the defendant was found to see if he dumped the weapon.

Furthermore, D abandoned the gun before the officer even approached him so he had no expectation of privacy in the trash can.

Dave’s argument will fail and the gun will be admissible.

11 ESSAY QUESTION AND SELECTED ANSWERS

JULY 2004 CALIFORNIA BAR EXAMINATION

This publication contains the six essay questions from July 2004 California Bar Examination and two selected answers to each question.

The answers received good grades and were written by applicants who passed the examination. The answers were prepared by their authors, and were transcribed as submitted, except that minor corrections in spelling and punctuation were made for ease in reading. The answers are reproduced here with the consent of their authors.

Question Number Contents Page

1. Criminal Law 1

2. Constitutional Law 12

3. Wills/Trusts 22

4. Evidence 33

5. Professional Responsibility 44

6. Tarts 51

i Question 1

On August 1, 2002, Dan, Art, and Bert entered Vince’s Convenience Store. Dan and Art pointed guns at Vince as Bert removed $750 from the cash register. As Dan, Art, and Bert were running toward Bert’s car, Vince came out of the store with a gun, called to them to stop, and when they did not do so, fired one shot at them. The shot hit and killed Art. Dan and Bert got into Bert’s car and fled.

Dan and Bert drove to Chuck’s house where they decided to divide the $750. When Chuck said he would tell the police about the robbery if they did not give him part of the money, Bert gave him $150. Dan asked Bert for $300 of the remaining $600, but Bert claimed he, Bert, should get $500 because his car had been used in the robbery. Dan became enraged and shot and killed Bert. He then decided to take all of the remaining $600 for himself and removed the money from Bert’s pocket.

On August 2, 2002, Dan was arrested, formally charged with murder and robbery, arraigned, and denied bail. Subsequently, the court denied Dan’s request that trial be set for October 15, 2002, and scheduled the trial to begin on January 5, 2003. On January 3, 2003, the court granted, over Dan’s objection, the prosecutor’s request to continue the trial to September 1, 2003, because the prosecutor had scheduled a vacation cruise, a statewide meeting of prosecuting attorneys, and several legal education courses. On September 2, 2003, Dan moved to dismiss the charges for violation of his right to a speedy trial under the United States Constitution.

1. May Dan properly be convicted of either first degree or second degree murder, and, if so, on what theory or theories, for: a. The death of Art? Discuss. b. The death of Bert? Discuss.

2. May Chuck properly be convicted of any crimes, and, if so, of what crime or crimes? Discuss.

3. How should the court rule on Dan’s motion to dismiss? Discuss. Answer A to Question 1

1)

1. A. Dan - Liability for Art’s Death

Murder

Murder is the unlawful killing of a human being with malice aforethought. Malice can be shown by either intent to kill, intent to cause grevious bodily harm, or reckless indifference to human life. Here, Dan is probably not liable under any of these theories. Because Vince, the shopkeeper, shot Art, causing his death, Dan did not exhibit intent to kill or cause grevious bodily harm. Likewise, fleeing probably does not constitute reckless indifference to human life.

Felony Murder Rule

However, Dan might be convicted under the . The felony murder rule holds defendants liable for foreseeable killings committed during the commission of inherently dangerous . Here, Dan, Art, and Bert were engaged in a robbery. A robbery is the taking and carrying away of the personal property of another by force with the intent to permanently deprive the victim of the property. Dan, Art and Bert robbed Vince because they took $750 from him at gunpoint, with the intent to keep the money. A robbery - especially an armed robbery of a convenience store - is likely an inherently dangerous felony. Art’s death was the kind of death that frequently results from armed , and thus was foreseeable.

Limitation of Felony Murder Rule - Fleeing

Liability for felony murder generally ends when the felons reach a place of safety after the felony. Here, because Art was killed while fleeing - before the felons reached a place of safety - this limitation will not apply.

Limitation on Felony Murder Rule - Death of a Co-Felon

However, most states have enacted limitations on the felony murder rule when the death of a co-felon is at issue. Under states that follow the agency rationale, a defendant can be found guilty if the killing was done by a felon or his agent. Under this view, Dan is likely not liable for felony murder because it was Vince rather than Dan or Bert who shot Art.

Under the proximate cause view of the felony murder rule, any killing proximately caused by the felony can make a defendant liable for felony murder. Under this rule, it is arguable that Dan should be liable for Art’s death. Being shot while fleeing from a convenience store robbery is foreseeable. Thus, if the jurisdiction follows this view, Dan might be liable

2 for Art’s death under a felony murder theory.

First Degree Murder

In most states, first degree murder requires premeditation or deliberation. Many states also include that fall under the felony murder rule in the definition of first degree murder. Thus, if this jurisdiction adheres to that view, Dan may be liable for first degree murder for Art’s death.

Second Degree Murder

Second degree murder generally is murder that does not involve premeditation and deliberation, but also does not amount to any form of manslaughter. If the applicable statute defines felony murder as second degree murder, Dan may be liable for that crime instead.

Conspiracy

Conspiracy requires an agreement to commit a crime between two or more people, an intent to agree, an intent to commit a crime, and an overt act. A conspirator is liable for all reasonably foreseeable crimes committed in furtherance of the conspiracy. Here, Art, Dan, and Bert clearly agreed to rob Vince’s store with the intent to commit the crime. Conspiracy does not merge with the completed crime. Thus, if Dan was liable for conspiracy, and a court found that Art’s death was foreseeable, Dan could potentially be liable on these grounds as well. However, this is a stretch, especially since Vince killed Art.

B. Dan’s Liability for the Death of Bert

Murder

As mentioned, one potential grounds of liability for murder is intentional killing or killing with an intent to cause great bodily harm. Here, Dan probably intended to kill Bert or at least intended to cause him great bodily harm. Dan simply shot Bert - there is no indication that he was merely trying to scare him.

First Degree Murder

Dan may be liable for first degree murder. Although premeditation and deliberation are generally prerequisites to a charge of first degree murder, some courts have held that one can premeditate or deliberate in very short periods of time. However, Dan will argue that he was “enraged” and had no time to deliberate or premeditate. Due to the spontaneous nature of the crime, Dan will likely not be found guilty of first degree murder. In addition, as discussed below, he is likely not guilty of felony murder. Thus, even if the state murder statute includes felony murder as first degree murder, Dan will likely not be liable for this

3 crime.

Second Degree Murder

Dan is much more likely to be guilty of second degree murder. As discussed above, he intended to kill Bert, but likely did not premeditate or deliberate. As discussed below, he is unlikely to be guilty of voluntary manslaughter or felony murder.

Felony Murder

A felony murder charge against Dan would be problematic. For one, liability for felony murder generally ends when the perpetrators have reached a place of safety. Dan and Bert had reached Chuck’s house when Dan killed Bert. Indeed, they had begun to divide up the money. This would likely cut off any liability for felony murder based on the robbery of Vince’s store.

In addition, the prosecution might argue that Dan is liable for felony murder because he took $600 from Bert’s pocket. The prosecution might argue that this is a robbery, and that Dan’s killing was a foreseeable result of the robbery. However, this is a weak argument. Dan only decided to take the money from Bert after he shot him. In addition, Dan might also be able to argue that since Bert did not have lawful title to the money, no robbery took place. This is because one element of a robbery is that the money be “property of another.” Thus, Dan is likely not liable for felony murder for Bert’s death.

Voluntary Manslaughter

Dan may argue that he is only liable for voluntary manslaughter. Voluntary manslaughter is a killing that would be murder, but was conducted while the perpetrator was highly upset. The upsetting incident must be the sort that would upset a reasonable person, the defendant must have been upset, a reasonable person would not have had time to cool off, and the defendant must not have cooled off. Dan will argue that he was “enraged” by Bert’s demand of extra money. However, this argument is unlikely to succeed. For one, Bert’s actions do not rise to the type of extremely upsetting provocation that generally suffices to reduce a murder charge to voluntary manslaughter. Moreover, there is no indication that a reasonable person would have had such a violent reaction to Bert’s demand for money. Thus, Dan is likely not liable for voluntary manslaughter.

Conspiracy

As discussed above, any underlying conspiracy to rob Vince’s store had likely ended by the time that the robbers reached Chuck’s house.

2. Chuck’s Liability

4 After the Fact

Chuck is likely guilty of being an accessory after the fact. An accessory after the fact is one who shields, shelters, or assists criminals after a crime. Chuck is clearly aware that Dan and Bert have committed a robbery. He threatens to tell the police about the crime unless he receives some of the money. He provides his house as a safe haven for Dan and Bert. If found guilty of this charge, Chuck would not be guilty as an - he would simply be guilty of an independent, lesser offense.

Accomplice

Chuck is probably not an accomplice to either Dan’s killing of Bert or the robbery of Vince. To be an accomplice, one must assist a crime with the intent that the crime be committed. Here, there is no indication that Chuck had any idea that Dan, Art and Bert were going to rob Vince’s store. In addition, given the spontaneous nature of Dan shooting Bert, there is no indication that Chuck intended that crime either. Mere presence at a crime scene does not necessarily result in accomplice liability.

Extortion

Chuck perhaps is guilty of . Extortion involves the obtaining of property through threats. Here, Chuck threatened to tell the police about the robbery. As a result, he obtained $150 from Dan and Bert. Thus, because he obtained property through the use of threats, he might be guilty of extortion.

Conspiracy

There is no indication that Chuck was involved in any agreement - or even knew about - the convenience store robbery. Also, Dan seems to have acted alone when he shot Bert. Accordingly, Chuck is likely not be [sic] guilty of conspiracy.

Mispris[i]on of Felony

If the jurisdiction recognizes this crime, Chuck may be guilty because he aided and assisted Dan and Bert to cover up their crime.

3. Dan’s Motion to Dismiss

The Sixth Amendment to the United States Constitution protects an accused’s right to a speedy trial. When evaluating whether such a right has been violated, courts consider several factors. Among them are the reason for the delay, whether the defendant has objected to the delay, and the length of the delay.

5 Here, Dan’s strongest argument is that the prosecutor’s reasons for delaying the trial are simply not compelling enough to warrant impinging upon his constitutional rights. The prosecutor’s desire to go on vacation and attend meetings and legal education classes seems more like a personal pred[i]lection than a good reason to delay Dan’s trial. Dan will languish in jail during this time - nearly thirteen months after he was arrested and arraigned. Moreover, with the exception of the vacation, it is not at all clear why the prosecutor cannot attend the meeting or legal education courses on his own time. Finally, in any event, it is not clear why those events warrant delaying the trial from January 3 to September 1 - a delay of nine months. Dan will also note that he initially moved to have trial set in October, 2002. Finally, Dan will point out that the prosecutor’s motion was granted on Jan. 3, which was essentially the eve of trial. Waiting until the last minute to continue a trial so long seems unfair and may have prejudiced his ability to mount an effective defense.

However, the prosecution will counter that Dan should have moved to have his charge dismissed on Jan. 3. Indeed, Dan waited until September 2 to move to dismiss. Although he “objected” on Jan. 3, he should have moved to dismiss then. By waiting to move to dismiss until after the trial began, Dan likely waived his rights. Accordingly, Dan’s motion should be denied.

6 Answer B to Question 1

1)

May Dan (“D”) be convicted of murder.

The first question is whether Dan may be convicted of murder in the 1st or 2nd degree. At common law, murder was the unlawful killing of a human being with malice aforethought. Malice aforethought was committing murder with any of the following mental states (1) intent to kill, (2) intent to do serious bodily harm, (3) reckless indifference to the unjustifiably high cost to human life and (4) intent to commit a felony. The types of felonies included in felony murder were inherently dangerous felonies.

Murder in the first degree is a statutory creation that involves the unlawful killing of another human being with premeditation and deliberation. In addition, many state statues have also included in the definition of murder in the first degree murders committed while committing a felony -- also enumerating inherently dangerous felonies.

Voluntary manslaughter is the unlawful killing of a human being which would be murder but for the existence of adequate provocation, and involuntary manslaughter is the killing of another human being with or during the commission of an unenumerated felony or misdemeanor.

2d Degree murder is a residual murder category that covers the unlawful killing of another human being that does not fall within the Murder in the 1st Degree or Voluntary or Involuntary Manslaughter categories. With this in mind, we can investigate whether Dan is liable for murder in the first or second degree.

All homicide crimes also require actual and proximate as well as the result of death.

KILLING OF ART.

Here, Dan did kill Art. Vince killed Art. Thus, the only theory that could convict Dan of the murder of Art would be the felony murder. Here, Art and Dan and Bert were committing robbery, an inherently dangerous felony.

Robbery is the taking of personal property of another from their person or presence by force or threats of force with the intent to permanently deprive.

Here, Dan, Bert and Art entered the convenience story and pointed guns at Vince (the requisite threat of force) and took $750 (personal property) from Vince’s person. This, especially because of the existence of guns, qualifies as an inherently dangerous felony that should rise to the level of a felony that would qualify for Felony murder. Thus,

7 because the killing of Art took place whil[e] Dan was committing an inherently Dangerous felony, if this occurred in a jurisdiction where felony murder is included in the definition of first degree murder, Dan could be guilty of first degree murder.

There are however some limiting doctrines to felony murder. Notably in this instance, the killing must be a foreseeable result of the felonious conduct, and the redline view of felony murder provides that defendants cannot be guilty of felony murder for the murder of one of their co-felons by the police or by third parties. Thus, although the killing of Art certainly is a foreseeable result of committing a robbery, if this is a jurisdiction that follows the redline view, Dan will not be guilty of felony murder for Art, and will not be guilty of either first or second degree murder for Art.

It is noteworthy that Vince’s killing of Art was not lawful because one may never use deadly force in defense of property, and here, Vince chased Art out of the store (after the physical danger to him passed) and killed Art, when Art failed to stop.

FOR DEATH OF BERT

The next question is whether Dan can be guilty of murder in the first or second degree of Bert.

The standards for murder in the first and second degree are set forth above. Here, the question will revolve around whether (1) Dan possessed the requisite premeditation and deliberation to kill Bill, (2) whether Dan could be guilty of felony murder, since this happened right after the robbery, or (3) whether adequate provocation existed to reduce the killing to a charge of involuntary manslaughter.

Premeditation.

Dan can be guilty of first degree murder of Bert if he committed the murder with premeditation and deliberation. Here, the facts do not indicate that he possessed that premeditation. Dan and Bert just committed a robbery together and were returning to divide the money. There is nothing to suggest that he had a prior plan to kill Bert. In fact, he only became enraged when Bert insisted on taking the entire share for himself. Thus, on these facts, he cannot be convicted of first degree murder on a premeditation and deliberation theory.

Felony Murder

The next question is whether he could be convicted of felony murder for the murder of Bert. Dan did just commit a felony (robbery) as discussed above. He had the requisite intent to commit that felony and it was an inherently dangerous felony. Thus, could his killing of Bert qualify for felony murder?

8 The felony murder rule also has the limited doctrine that the killing must occur during the commission of the felony. Once the felons reach a point of temporary safety, they are no longer considered as carrying out the felony for purposes of the felony murder rule.

Here, Dan and Bert had reached the safety of Chuck’s house and[,] therefore, were no longer in the commission of a felony and[,] therefore, Dan cannot be guilty of felony murder.

2d Degree Murder and Voluntary Manslaughter

The next question is whether adequate provocation existed to make the killing a voluntary manslaughter. If not, the murder will fall into the residual category of Murder in the 2d degree. Here, since Dan acted with intent to do serious bodily damage to Bert (he shot and killed him), or at a minimum proceeded with reckless disregard for the unjustifiably high risk to human life, he will be guilty of second degree murder if the charge isn’t reduced to voluntary manslaughter.

Vol manslaughter requires (1) provocation aro[u]sing extreme and sudden passion in the ordinary person such that he would not be able to control his actions, (2) the provocation did in fact result in such passion and lack of control, (3) not enough time to cool off btwn the provocation and the killin[g] [gna] d (4) the defendant did not in fact cool off.

Here, Bert refused to give Dan his $300. While it is understandable that the failure to give such money would aro[u]se anger in an ordinary person that had just put their freedom and life on the line in a robbery attempt, we are only talking about $300. While understandably angry, it is hard to imagine that an average person would lose control over $300 to the point of taking another person’s life. Thus, Dan will not qualify for the reduction to voluntary manslaughter and will be convicted of 2d degree murder.

MAY CHUCK BE CONVICTED OF ANY CRIMES

The possible crimes Chuck could be convicted of is [sic] either all of the crimes that the principals committed (under an accomplice liability theory), or at a minimum an Accessory After the Fact.

ACCOMPLICE LIABILITY

If one aids, abets or facilitates the commission of a crime with the intent that the crime be committed, one can be found guilty on accomplice liability theories. The scope of liability includes liability for the crimes committed by the principals and all other foreseeable crimes. The common law used to distinguish between principals in the first and second degrees and accessories before and after the fact. Largely those distinctions have been discarded, although, most jurisdictions still do recognize the lesser charge of accessory after the fact.

9 Here, there is no evidence that Chuck aided, abetted or facilitated the crime until after it was committed. He provided a safehouse and subsequently demanded money. But mere presence or knowledge is not enough to ground accomplice liability.

ACCESSORY AFTER THE FACT

However, Chuck did assist after the crime happened (he provided a safehouse, and agreed not to tell the authorities in exchange for money), so at a minimum he will be guilty of accessory after the fact.

Extortion

Chuck may also be liable for extortion. Extortion is the illegally obtaining property through threats of force or threats to expose information. Here, he threatened to expose the criminals to the police if he didn’t get paid, and so he will be liable.

Receiving Stolen Property

Chuck also will be liable for receiving stolen property. The requirement for this crime are [sic] that you know the circumstances around the property (ie, that it is stolen) and that you willing [sic] receive it. Chuck knew this money was the fruit of a robbery and received it in exchange for his providing a safehouse. Thus he will be liable of receipt of stolen property.

CONSPIRACY

Chuck also could be guilty of conspiracy. Conspiracy is (1) an agreement between two or more people, (2) the intent to agree, (3) the intent to pursue an unlawful objective and (4) in some jurisdictions, some overt act. Conspiracy does not merge into the completed crime.

HOW SHOULD COURT RULE ON DAN’S MOTION TO DISMISS.

The 6th amendment provides each defendant the right to a speedy trial. The 6th amd is applied to the states through its incorporation into the due process clause of the 14th amendment. The right to a speedy trial attaches post charge. Whether the defendant has been given a speedy trial depends on an analysis of the totality of the circumstances.

Here, Dan was arrested on August 2, and immediately charged. Thus his right to a speedy trial attached sometime in early August. The initial trial date was set for January 5, 2003. It is not likely that the denial of Dan’s request for a trial 2 months after his charge is a violation of his constitutional rights since the court set a date very closely thereafter in January. However, the prosecutor’s delay subsequent to that date does not rise to the level of providing adequate excuse for moving Dan’s date (coupled with the fact that the request was made only days before the January trial was to commence). Here, the

10 prosecutor wanted to take a vacation cruise and take some legal education classes, and meet for a statewide meeting of prosecutors. First, none of these seem to rise to the level of an adequate excuse to delay a trial 9 months. Particularly since the defendant was denied bail and was sitting in jail. While the court could have granted a continuance for a short period of time for the meeting or to accommodate the prosecutor, given the defendant’s status (sitting in jail), it was improper for the court to grant this motion, and the court may dismiss Dan’s case.

It should be noted, however, that Dan should have moved earlier than September 2, as this would have permitted the court to fashion relief without having to dismiss the charge altogether. Accordingly, a court could find that he was not entitled to dismissal because of his delay.

11 ESSAY QUESTIONS AND SELECTED ANSWERS

FEBRUARY 2006 CALIFORNIA BAR EXAMINATION

This web publication contains the six essay questions from the February 2006 California Bar Examination and two selected answers to each question.

The answers received high grades and were written by applicants who passed the examination. Minor corrections were made for ease in reading. The answers are reproduced here with the consent of their authors.

Question Number Contents Page

1. 1

2. Wills and Succession 13

3. Real Property 21

4. Civil Procedure 28

5. /Remedies/Professional Responsibility 40

6. Criminal Law and Procedure 48

i Question 6

Deft saw Oscar, a uniformed police officer, attempting to arrest Friend, who was resisting arrest. Believing that Oscar was arresting Friend unlawfully, Deft struck Oscar in an effort to aid Friend. Both Friend and Deft fled.

The next day, as a result of Oscar’s precise description of Deft, Paula, another police officer, found Deft on the street, arrested him for and and searched him, finding cocaine in his pocket. After Paula gave proper Miranda warnings, Deft said he wanted to talk to a lawyer before answering any questions. Paula did not interrogate him. However, before an attorney could be appointed to represent Deft, Paula placed him in a lineup. Oscar identified Deft as his assailant. Deft was then charged with assault and battery of a police officer and possession of cocaine. Thereafter, he was arraigned.

The next day Paula gave Deft, who was without counsel, proper Miranda warnings, obtained a waiver, and interrogated him. He admitted striking Oscar.

How should the judge rule on the following motions made by Deft at trial:

1. To suppress the cocaine? Discuss.

2. To suppress Oscar’s identification during the lineup? Discuss.

3. To suppress Deft’s admission that he struck Oscar? Discuss.

4. For an instruction to the jury that Deft’s assault was justified on the basis of defense of another? Discuss.

48 Answer A to Question 6

6)

1. Deft’s Motion to Suppress the Cocaine

The Fourth Amendment of the Constitution protects individuals from unreasonable searches and seizures by government officials. If a defendant’s Fourth, Fifth, or Sixth Amendment rights are violated in connection with a criminal prosecution, the exclusionary rule, a judge-made doctrine, requires the exclusion of all evidence obtained in violation of such rights and all derivative evidence, or fruit of the poisonous tree.

Government Conduct

To make a Fourth Amendment claim, there must first be government conduct. Here, Larry was searched by Paula, a police ofiicer, which qualifies as government conduct.

Standing – Reasonable Expection of Privacy

A defendant also must have standing to challenge government action, which occurs if the defendant has a reasonable expectation of privacy in the item or place searched. Because Larry’s body was searched, this clearly qualifies Larry to contest the act since he had a reasonable expectation of privacy in his own body.

Requirement for Probable Cause and a Valid Warrant

Generally, a search will be considered unreasonable unless the officer has probable cause to conduct the search, and the search is supported by a valid warrant. However, a number of exceptions to the requirement for a search warrant exist.

Search Incident to a Lawful Arrest

Paula did not have a valid search warrant. However, one exception to the warrant requirement is for searches incident to a lawful arrest. A lawful arrest can be made in public, without a warrant, if the officer has probable cause to believe that the defendant has committed a felony.

Paula was making a lawful arrest because she knew that Oscar had been assaulted and battered and that Deft fit the description of the perpetrator. Thus, she had probable cause to believe that Deft was the perpetrator of these felonies. Because Paula made a lawful arrest of Deft, her search of his body was also lawful. Thus, the court should deny Deft’s motion to suppress the cocaine.

Hot pursuit

49 Paul[a] might also be able to argue that her search of Deft was lawful because Deft was a suspect who might get away. Her better claim, though, is that the search was incident to a lawful arrest.

2. Deft’s Motion to Suppress Oscar’s Identification During the Lineup

A defendant has a Fifth Amendment privilege against self-incrimination, which includes the right to counsel if the [the] defendant does not waive his right to such counsel. This right attaches whenever there is custodial police interrogation. A defendant also has a Sixth Amendment right to counsel, which attaches once the defendant has been charged with a crime. Here, Deft had not been charged with assault and battery by the time the lineup was conducted; thus, his Sixth Amendment right to counsel had not attached.

The facts show that Deft did not waive his Fifth Amendment right to counsel because he stated that he “wanted to talk to a lawyer before answering any questions.” The question is whether the lineup even violated Deft’s Fifth Amendment right.

A defendant is in custody when a reasonable person would believe he was not free to leave. Deft had just been placed under arrest; as such, he was in police custody at the time of the lineup.

Interrogation occurs whenever the police make a statement that is likely to elicit an incriminating response. During the lineup, there is no evidence that the police made any statements likely to elicit an incriminating response from Deft. Thus, Deft cannot be said to have been under interrogation during the lineup. For this reason, Deft’s Fifth Amendment right to counsel was not violated by the lineup.

Even if Deft’s Fifth Amendment right had been violated, the identification would likely still be admissible under an exception to the exclusionary rule, which allows evidence if it would have been discovered anyway. Oscar clearly saw Deft, his assailant, when Deft was committing the crime. Thus, the government can show that it would have had an independent source for the identification. Thus, the court should deny Deft’s motion to suppress Oscar’s identification.

3. Deft’s Motion to Suppress Deft’s Admission that He Struck Oscar

The issue is whether Deft’s Fifth and Sixth Amendment right to counsel were violated by Paula’s interrogation of Deft the day after Deft was arraigned. Paula did give Deft proper Miranda warnings, but she also obtained a waiver. A waiver of Miranda rights is valid if the defendant knowingly, voluntarily, and intelligently waived his rights. There are no facts to indicate that the waiver was not knowing, voluntary, and intelligent, so Deft’s Fifth Amendment right to counsel was not violated, even though he was subject to custodial interrogation.

A defendant’s Sixth Amendment right to counsel applies to all post-charge proceedings.

50 The question is whether Paula’s interrogation of Deft was a post-charge proceeding. Because Deft had been charged and arraigned, his Sixth Amendment right to counsel had attached. Once this right attaches, a defendant cannot be questioned about the crime charged without the presence of the defendant’s attorney, unless he explicitly waives his right to counsel. Although the facts show that Paul obtained a waiver of Deft’s Miranda rights, they do not clearly show that Deft explicitly waived his right to counsel. Thus, the court should grant Deft’s motion to suppress the admission. If, however, Deft testifies for himself in the criminal trial, then his admission can be used to impeach him on cross- examination.

4. Deft’s Motion for a Jury Instruction that Deft’s Assault Was Justified on the Basis of Defense of Another

A defendant may have a valid defense if he acts with reasonable force, with a reasonable belief that such force is necessary for self-defense or the defense of another. For the defense of others, courts are split on whether the defense exists in a situation in which the person being “defended” by defendant does not himself have the privilege of self-defense clothes against his “attacker.” For example, if an officer in plain clothes conducted a lawful arrest of another, a third party “defending” the arrestee might not have the privilege to assert the defense since the arrestee also did not have the privilege against the officer.

Here, however, Oscar, the party making the arrest[,] was not a plain clothes or undercover officer; rather, he was wearing a uniform when he attempted to arrest Friend. Deft clearly knew that Oscar was a police officer.

A person also can lawfully resist an arrest if an officer clearly does not have lawful basis to make an arrest. This privilege, however, is very limited even as to the person being arrested and would only attach where there is no basis whatsoever to make an arrest of the person. This privilege does not extend to onlooking third parties who witness the arrest. These rules are necessary to protect society and to assist officers in the enforcement of the law for the conduct of a lawful and orderly society.

The facts do not show the circumstances behind why or how Oscar was making the arrest. It would seem that Deft might have a defense if, for example, Oscar were conducting the arrest in an extremely physically abusive manner and was unwarranted in doing so. In plainer terms, if Oscar were “beating the crap” out of Friend for no reason, then Deft might be entitled to assert a privilege of defense. However, there are no facts to indicate that Oscar was acting unreasonably; further, because Friend was resisting arrest, this weighs in favor of not extending the privilege, even if Oscar did have to resort to some physical means to complete the arrest.

In Deft’s situation, absent additional extenuating facts just described, it simply was not reasonable for Deft to strike Oscar in an effort to aid Friend, even if Deft believed, reasonably or unreasonably, that Oscar was arresting Friend unlawfully. Accordingly, the court should deny Deft’s motion to instruct the jury that Deft’s assault was justified on the

51 basis of defense of another.

In short, the judge should deny all of Deft’s motions except for his motion to suppress Deft’s admission, which the court should grant.

52 Answer B to Question 6

Deft’s Motion to Suppress Cocaine

The issue is whether Paula properly seized the cocaine from Deft’s pockets. The Fourth Amendment protects individuals from unreasonable searches and seizures by government agents. It only applies to evidentiary searches when the individual has a reasonable expectation of privacy. Deft has a reasonable expectation of privacy in the contents of his pockets. Therefore the question is whether the government can show that Paula’s search satisfied the requirements of the 4th Am.

Warrantless Search

Paula searched Deft’s pocket without a warrant. Thus, the gov’t must show that Paula executed the search pursuant to a valid warrantless search exception.

Search Incident to Lawful Custodial Arrest

An officer may search a suspect as a consequence of a lawful custodial arrest. In order to fit within this exception, the underlying arrest must be lawful. An officer may not arrest a suspect for a misdemeanor without a warrant unless the officer saw the suspect commit the misdemeanor. An officer may arrest a suspected felon if the officer had probable cause to believe the suspect committed a felony.

The first issue here is whether Paula had probable cause to believe Deft committed a crime. She based her arrest on Oscar’s precise description of Deft. Since she knew Deft had assaulted Oscar the day before and because she was relying on Oscar’s “precise” description, Paul[a] had probable cause to believe Deft had committed assault and battery. Probable cause is satisfied if an officer has trustworthy facts that lead to the probability that a suspect committed a crime. Oscar’s description sufficed.

The second issue is whether Paula had probable cause to believe that Deft had committed a felony. In many states assault and battery are . However, battery is generally elevated to a felony when directed against a police officer under aggravated battery statutes. As long as this state makes battery of a police officer a felony. Paula’s arrest of Deft was lawful because she had probable cause to believe he had committed a felony. Under the SILCA doctrine, the judge should deny Deft’s motion to suppress the cocaine.

Other Warrantless Search Exceptions

If a judge determines that Paula’s arrest of Deft was unlawful, the judge must suppress the cocaine because no other warrantless search exceptions apply to these facts. The other exceptions are: plain view, consent, auto searches, searches in hot pursuit or to seize evanescent evidence, and pat down searches performed with reasonable suspicion

53 that a suspect is armed. There are no facts to support any of these doctrines.

2. Deft’s Motion to Suppress Oscar’s ID

The issue is whether Oscar’s pre-arraignment identification of Deft can be suppressed.

6Am Right to Counsel

Deft may argue that the identification should be suppressed because he did not have counsel present for it. Under the 6th Amendment, defendants have a right to counsel at all ‘critical stages’ of litigation following indictment/arraignment. Courts have ruled identification lineups are ‘critical stages’ under the Sixth Amendment.

Deft’s arguments must fail here because the lineup occurred before his arraignment. Therefore, his 6th Amendment right to counsel had not attached. This is true even though Deft properly invoked his right to counsel after being given his Miranda warnings. The 5th Amendment provides Deft with a limited right to have counsel present during custodial interrogation. It does not apply to Deft’s presence in a lineup because his physical appearance is not testimonial in nature.

Unnecessarily Suggestive

The only other argument that Deft may offer to suppress the identification is that the lineup was unnecessarily suggestive and resulted in a substantial likelihood of misidentification. Deft must pose this argument under the due process clause of the 14th Amendment, and a court would consider the suggestiveness of the lineup in the totality of the circumstances. There are no facts to suggest the lineup was unnecessarily suggestive, so Deft will likely lose this argument.

Thus, a court should not suppress Oscar’s identification of Deft.

3. Deft’s Motion to Suppress His Statement

This issue is whether Deft’s admission should be suppressed. It should be suppressed under both the 5th & 6th Amendments.

5th Amendment

On the day of his arrest, Paula gave Deft Miranda warnings and he unambiguously invoked his 5th Amendment right to counsel by saying he wanted to talk to a lawyer before answering questions.

54 Once a suspect invokes his 5th Amendment right to counsel, the police may not question that suspect on that charge or any other charge until the suspect has spoken with an attorney. The facts that new charges were brought against Deft and that Paula readministered Miranda warnings and obtained a waiver do not change this analysis. Deft’s invocation of the 5th Amendment right to counsel operates as a complete bar to questioning until he has a spoken with an attorney.

The proper remedy for testimony obtained in violation of the 5th Amendment is suppression except for impeachment. Therefore, the court should suppress Deft’s statement from the prosecution’s case[-]in[-]chief.

6th Amendment

As discussed above, defendants have the right to assistance of counsel at all “critical stages” of litigation after indictment/arraignment. Here, Deft’s admission came a day after he was arraigned. Therefore, his Sixth Amendment right to counsel had attached. The only issue is whether interrogation is a ‘critical stage’.

Courts have ruled that interrogation is a critical stage of ligation under the Sixth Amendment’s right to assistance of counsel. Thus, Deft had a right to have counsel present when he admitted striking Oscar.

The proper remedy for a statement gained in violation of a suspect’s 6th Amendment right to counsel is suppression of the statement. Thus, the court should suppress Deft’s admission under the 6th Amendment.

4. Jury Instruction re: Defense of Another

The issue is whether the court should provide a jury instruction on the defense of defense of [sic] another. A defendant may justify a battery on defense of another when he acted out of a reasonable belief that another person had the right to use force in his own defense. A defendant asserting a justification of defense of another cannot use force that is excessive in the circumstances.

Here, the first issue is whether Deft had a reasonable belief that Friend could use force in resisting arrest by Oscar. An individual may use nondeadly force in order to resist an unlawful arrest by a uniformed police officer. Here, we are told that Deft believed Oscar was unlawfully arresting Friend. We do not know why Deft believed the arrest was unlawful. However, if Deft had a reasonable basis for his belief then he had the right to use nondeadly force in Friend’s defense. This right stemmed from the fact that Friend has the right to use nondeadly force against a uniformed police officer making an unlawful arrest.

The second requirement is that Deft used reasonable force. We are told that he

55 struck Oscar. As long as this was a reasonable amount of force to use in the circumstances, then Deft can invoke the justification of defense of others.

Based on this analysis, the court should offer the jury instruction[s] on defense of others.

56 ESSAY QUESTIONS AND SELECTED ANSWERS

FEBRUARY 2007 CALIFORNIA BAR EXAMINATION

This publication contains the six essay questions from the February 2007 California Bar Examination and two selected answers to each question.

The answers selected for publication received good grades and were written by applicants who passed the examination. These answers were produced as submitted, except that minor corrections in spelling and punctuation were made during transcription for ease in reading. The answers are reproduced here with the consent of their authors.

Question Number Contents Page

1. Real Property 1

2. Corporations/Professional Responsibility 9

3. Criminal Law and Procedure 18

4. Wills and Succession 29

5. Constitutional Law 37

6. Evidence 46

i Question 3

Dan has been in and out of mental institutions most of his life. While working in a grocery store stocking shelves, he got into an argument with Vic, a customer who complained that Dan was blocking the aisle. When Dan swore at Vic and threatened to kick him out of the store, Vic told Dan that he was crazy and should be locked up. Dan exploded in anger, shouted he would kill Vic, and struck Vic with his fist, knocking Vic down. As Vic fell, he hit his head on the tile floor, suffered a skull fracture, and died.

Dan was charged with murder. He pleaded not guilty and not guilty by reason of insanity. At the ensuing jury trial, Dan took the stand and testified that he had been provoked to violence by Vic’s crude remarks and could not stop himself from striking Vic. Several witnesses, including a psychiatrist, testified about Dan’s history of mental illness and his continued erratic behavior despite treatment.

1. Can the jury properly find Dan guilty of first degree murder? Discuss.

2. Can the jury properly find Dan guilty of second degree murder? Discuss.

3. Can the jury properly find Dan guilty of voluntary manslaughter? Discuss.

4. Can the jury properly find Dan not guilty by reason of insanity? Discuss.

Answer A to Question 3

3) 1. Guilty of First Degree Murder

First degree murder is a specific intent crime typically statutorily provided for. Typically, first degree murder consists of: (1) intentional killing of a human, (2) with time to reflect upon that killing, and (3) doing so in a cool and dispassionate manner.

Here, while there appears to be no statute that provides for first degree murder, it is unlikely that Dan would be guilty of first degree murder just the same.

Intentional killing An intentional killing is one done with specific intent to take the life of another.

Here, the prosecutor will argue that Dan expressed a specific intent to kill Vic when he yelled he would kill Vic, which was accompanied by a striking of Vic with Dan’s fist.

Therefore, it is likely given Dan’s express words of intent, the prosecutor will meet her burden of proving a killing by intent.

Time to Reflect Upon the Killing First degree murder requires time to reflect upon the killing. This is commonly known as premeditation. Premeditation, not in keeping with the lay person’s understanding of it, however, requires merely a moment’s reflection upon the killing.

Here, the prosecutor will argue that Dan reflected upon the killing of Vic when he took the time to say to Vic, “I’m going to kill you.” However, Dan will argue that there was no time to reflect upon the killing of Vic because he “exploded” and then hit Vic. Such an intense anger coupled with a spontaneous statement “I’m going to kill you” will likely not be construed as sufficient time to reflect.

Therefore, a jury should not properly find this element of the crime established.

Cool and dispassionate manner The defendant must have committed the killing in a cool and dispassionate manner. That means that the defendant killed another person in a calm and calculated manner without passion.

Here, the prosecutor will argue that Dan’s action of striking Vic with his fist without an expression of sadness or fright may amount to cool and dispassionate. However, such an argument is tenuous.

Dan will successfully show that his actions were the result of an explosion, regardless of the reasonableness of those actions. Dan “exploded.” This could hardly be construed as

19 “cool.”

Therefore, a jury should not properly find this element of first degree murder established.

In sum, a jury would not properly find Dan guilty of first degree murder.

Defenses Even if a jury could find Dan guilty of first degree murder, such an offense will be subject to the defense of insanity (discussed below).

2. Second Degree Murder

Second degree murder or common law murder is the intentional killing of a person with malice aforethought. Malicious intent will be implied by: (1) the intent to kill a person, (2) the intent to inflict a substantial bodily harm on someone, (3) an awareness of an unjustifiably high risk to human life, and (4) the intent to commit a felony.

Intent to kill a person As discussed earlier, Dan could be found to have intentionally killed Vic as evidenced by his expressed words “I’m going to kill you.” While words alone are sufficient to manifest intent, this is a subjective standard and a jury will be allowed to look to the totality of the circumstances. The jury will be able to consider that Vic told Dan that he was crazy and should be locked up, which aroused such anger that would negate a malicious intent.

However, a jury could find that Dan intended to kill Vic by using words of that intent, coupled with an action that indeed killed Vic.

Therefore, Dan could properly be found guilty of second degree murder, malicious intent implied by the intent to kill Vic.

Intent to Inflict Substantial Bodily Harm If Dan is not found to have the intent to kill, the prosecutor will argue that he did manifest the intent to inflict substantial bodily harm on Vic.

Here, Dan used his fist to strike Vic. The striking of another person could inflict substantial injury on another, depending upon where the person made the strike. Dan used his fist to strike Vic on the head, causing a fracture to his skull. The prosecutor will argue that Dan must have intended substantial bodily harm because striking a person in the head is a place of extreme vulnerability.

On the other hand, Dan will argue that people get into fistfights all the time, whether it be on the streets or boxing. He will argue that fistfights are a common way for people to work out their arguments and no substantial injury is intended. This argument has little merit given the high susceptibility to injury from striking someone in the head.

20 Therefore, a jury could properly find that Dan intended to inflict substantial bodily harm.

Awareness of an Unjustifiably High Risk to Human Life Again, the prosecutor will argue that even if Dan did not intend to inflict death or substantial bodily harm, surely Dan was aware of the high risk of human life.

Here, the prosecutor will argue that Dan was aware of this unjustifiably high risk because striking another on the head with the force of fracturing his skull is a high risk of which Dan could be aware.

Therefore, a jury could properly find that Dan had an awareness of an unjustifiably high risk to human life.

Felony Murder There is no evidence that Dan was intending to commit a felony, the intent from which can be implied to the killing of Vic.

Therefore, there would be no second degree murder based on an intent to commit a felony.

In sum, a jury could properly find Dan guilty of second degree murder.

3. Voluntary Manslaughter

An intentional killing will be reduced to voluntary manslaughter by a provocation that arouses a killing in the heat of passion. Voluntary manslaughter consists of: (1) a provocation that would arouse intense passion in the mind of an ordinary person, (2) the defendant in fact was provoked, (3) no reasonable time for the defendant to cool between the provocation and the killing, and (4) defendant in fact did not cool [sic].

Sufficient provocation Sufficient provocation to commit a killing is one that would arouse intense passion in the mind of an ordinary person.

Here, Dan will argue that shouting to someone that they are crazy and should be locked up is sufficiently inciting to induce anger. This is subjectively true where Dan had spent so much time in and out of mental institutions. He will argue that he was highly vulnerable to such insults.

On the other hand, the prosecutor will rightfully point out that this is a reasonable person standard that does not take into consideration the surrounding circumstances. A reasonable person would not be incited to kill simply by an insult of insanity.

Based on this argument, the prosecutor will successfully refute Dan’s attempt to reduce his killing to voluntary manslaughter.

21 That being said, the other elements appear to exist.

Dan in fact provoked. Dan was in fact provoked when he “exploded” and simultaneously killed Vic.

No reasonable time to cool between provocation and killing Dan immediately struck Vic on the head after the insult. There was no reasonable time to cool and Dan did not in fact cool [sic].

4. Insanity

In order to be convicted of a crime, the defendant must complete a physical act (actus reus) contemporaneously with the appropriate state of mind (). Insanity is a defense to all crimes except strict liability because insanity negates the requisite intent necessary to be convicted of murder in all forms.

Insanity is a legal defense that must be set out by applying the requisite elements as opposed to expert testimony of a psychiatrist. There are four theories of insanity a defendant may set forth and will depend upon which theory a jurisdiction adopts. All four theories will be discussed below to determine which, if any, are proper.

M’Naughten Test Insanity under this test is defined as the defendant was unable to understand the wrongfulness of his conduct and lacked the ability to understand the nature and quality of his acts.

Here, Dan testified that the crude remarks were so incitant that he was unable to stop himself. However, the prosecutor will argue that Dan showed his ability to understand the wrongfulness of his conduct because he shouted he would kill Vic. In addition, the mere fact of being unable to stop yourself implies that you indeed know it to be wrong but were unable to control yourself.

Based on this evidence, Dan would not successfully raise a defense under this issue.

Irresistible Impulse Test Under this test, the defendant may prove a defense of insanity if he shows he lacked the capacity for self-control and free will.

Dan will probably be more successful to claim a defense of insanity under this test. As mentioned above, Dan “could not stop himself.” This specifically evidences his inability to control himself. His will was subjugated by the insanity.

Based on this evidence, Dan will likely successfully claim a defense of insanity under this test.

22 Durham Test The Durham Test subscribes to the theory that a defendant will have an if his unlawful conduct was the product of a sick mind.

Dan will argue that he has spent much time in and out of mental institutions. Indeed, several witnesses testify as to Dan’s history of mental illness. Such a history suggests that his conduct was a product of a sick mental condition rather than the product of his own free will.

Dan will likely succeed in bringing a defense of insanity under the Durham test.

Model Penal Code Test Finally, under the test adopted by the Model Penal Code, a defendant’s actions may be defended by way of insanity if he was unable to conform his actions to the requirements of the law.

Here, Dan will offer his history of mental illness and continued erratic behavior despite treatment as a way to prove that he lacked the ability to conform himself to the requirements of law, i.e. not to kill. This, however, seems to be a less compelling argument as Dan has been able to conform himself to the requirements of law in other aspects of his life. He was able to work in a grocery store and successfully stock the shelves.

Because Dan appears to have the ability to conform his actions to the requirements of law in all other instances, the prosecutor will likely defeat Dan’s claim of an insanity defense.

23 Answer B to Question 3

3)

1. 1st Degree Murder Murder is the killing of another human being with malice afterthought. The crime of murder is subdivided into degrees based on the intent of the accused. First degree murder is the most serious of the degrees of murder. A person is guilty of first degree murder if the prosecution can show beyond a reasonable doubt that he killed someone with deliberation and premeditation; or, in jurisdictions that recognize the felony murder rule, if someone was killed as the foreseeable result of his act, or of the act of a coconspirator, during the course of an enumerated felony. This is the felony murder rule.

Felony Murder Dan will not be found guilty of first degree murder under the felony murder rule because he did not commit one of the underlying felonies. To be guilty under the felony murder rule at common law, the accused must have committed either , , robbery, , or , and the victim must have been killed during the commission of the crime (before the accused had reached a place of safety). The facts indicate that this killing occurred as the result of either no crime, if he was insane, or a battery, because he struck Vic. Battery is not an enumerated felony. Hence Dan cannot be guilty of first degree murder under this theory.

Premeditated and Deliberate Premeditation requires that decision to kill have arisen when the accused was acting in a cool, composed manner, with sufficient time to reflect upon the killing. Deliberateness requires that the accused had the intent to kill when he engaged in the act that resulted in the death.

The facts indicate that Vic [sic] was stocking shelves before Vic encountered him. There is nothing to indicate that he had any animosity towards Vic prior to the incident, or even knew Vic. The facts indicate instead that Dan punched Vic after he exploded in anger in response to a comment Vic made. Vic’s death resulted from a skull fracture caused by his impact with the ground. At no time do the facts indicate that Dan calmly and cooly reflected on killing Vic. In addition, it is not clear that he had the intent to kill Vic, as he only hit him once, an act that does not usually cause death. Although he shouted that he would kill Vic right before he killed him, the jury could likely not find that this shouting alone immediately before throwing the punch was enough. Moreover, it does not evidence a cool dispassionate manner, but instead, evidences the opposite. Therefore, because Dan’s actions appear neither premeditated nor deliberate, he will likely not be found guilty of first degree murder.

Dan will also have the defense of insanity, discussed below, and the defense of diminished capacity if the jurisdiction recognizes it. Under diminished capacity, Dan will have to show that a disease of the mind prevented him from forming the intent required, even though it

24 did not raise to the level of insanity.

2. 2nd degree murder Second degree, at common law, murder is the killing of a human being with malice afterthought. The mens rea of malice is satisfied when the accused intended to kill, intended to cause great bodily injury, showed a reckless disregard for an unjustifiably high risk of death, or killed during the commission of a rape, burglary, robbery, kidnapping, or arson. Because there is no issue as to the cause of Vic’s death, the prosecution’s issue will be in proving that Dan killed with malice and not in the heat of passion, as discussed in section 3 infra, then it cannot convict him of murder because he will have lacked the intent, and therefore must instead convict him of manslaughter. Again, Dan will also have the defense of insanity, discussed below.

Intent to kill - As discussed above, the jury will likely not be able to find that the facts show that Dan formed the intent to kill Vic because the facts indicate that Dan was in a rage when he punched Vic. Although Dan’s testimony that he had been provoked to violence does not absolutely show that he lacked the intent to kill, if the provocation would have caused a reasonable person to become enraged, and did cause him to become enraged, and there was not enough time for a reasonable person to calm down, and Dan did not in fact calm down, then the jury will not be able to conclude that he had formed the intent to kill at the time he punched Vic. However, if the jury finds that Dan’s passion was not reasonable, or he was not in the heat of passion, it could conclude that he intended to kill Vic because he shouted that he would kill Vic right before he killed him. However, Dan’s actions are not consummate with the intent to kill. He only hit Vic once. He did not stomp his head in when he hit the ground or hit Vic with a weapon. Consequently, even if Dan was not in passion, it is likely that the jury would not find he had the intent to kill.

Intent to cause bodily harm - As discussed above in the intent to kill, it is likely that the jury would not find that Dan had formed the proper intent to cause bodily injury at the time he hit Vic because of his passion. Because it is the formation of the intent that matters, if Dan did not have the state of mind necessary to formulate the intent to cause substantial bodily injury because he was in the heat of passion as a result of the provocation, he cannot be found guilty under this theory either. However, if the jury does not find that the he [sic] satisfies the requirements for finding heat of passion, then it is likely that they will convict him for murder under this theory of malice. Not only did Dan yell that he intended to kill Vic, but Dan punched Vic, which is an act that presented the likely result of causing serious bodily harm. Thus, unlike above where he did not take an act that was likely to kill, Dan took the requisite act here. Thus, the jury could more reasonably find that he intended to cause great bodily harm when he punched Vic and because Vic died as a result of that action, Dan is guilty of murder.

Reckless disregard for an unjustifiably high risk to human life - To convict Dan under this theory, the jury would have to conclude that Dan appreciated the high risk of death caused by his actions, and that he proceeded to engage in reckless conduct in the face of it. As discussed above, if Dan was in the heat of passion, the jury cannot find that he

25 appreciated the unjustifiably high risk of his actions, and thus cannot convict under this theory. However, if the jury does not find that he acted in the heat of passion, then it would be possible to convict under this theory because Dan should have known that punching Vic could cause him to die, and Dan engaged in the actions anyway.

Felony murder - As discussed above, battery is not one of the crimes that satisfies felony murder, so he cannot be found guilty under this theory.

Dan will have the defense of insanity, discussed below.

3. Manslaughter To find Dan guilty of voluntary manslaughter, the jury will have to find that Vic’s provocation would have caused a reasonable person to become enraged, that it did cause Dan to become enraged, that there was not enough time for a reasonable person to calm down between the time the comment caused Dan to be enraged and the time he hit Vic, and that Dan did not in fact calm down during that time.

Although manslaughter is sometimes thought of as a defense, it is not Dan’s burden to prove these elements. Instead, the prosecution must show the lack of a heat of passion killing in order to establish the necessary intent to convict Dan of either 1st Degree or 2nd Degree murder, as discussed above.

Reasonable person - The first test is whether a reasonable person would become enraged. The typical instances are when someone finds his spouse in bed with another. Here, there was a simple altercation between Dan and Vic. Vic complained that Dan was blocking the aisle. Dan swore at Vic in response and threatened to kick him out of the store. Vic told Dan that he was crazy. Dan flew into a rage and punched Vic. Vic died. The jury would likely find that these facts do not meet the requirement for a heat of passion killing because a reasonable person does not fly into a rage because someone else tells them [sic] they [sic] are crazy during an altercation that they [sic] escalated. A reasonable person would expect the other party to make a snide comment in response to being sworn at by a store employee who might have been blocking an aisle. If the jury finds that a reasonable person would not have become so enraged as to have punched Vic under the circumstances, then Dan will not be convicted of the lesser crime of manslaughter and will instead likely be convicted of 2nd degree murder, as discussed above.

Dan’s particular mental issues or state of mind is [sic] irrelevant for this test. This is an objective test; it is based on what the reasonable person would do. Thus it is irrelevant if Dan is particularly sensitive to comments about being crazy; he only gets this defense if the comments would have engendered passion in a reasonable person.

Dan’s passion - If the jury finds that a reasonable person would have been enraged by Vic’s actions, then the next issue is whether Dan did. The facts are pretty clear on this point. They state that Dan exploded in anger, shouted he would kill Vic, and then punched him. This is exemplary of enraged behavior; therefore, the jury will almost certainly find

26 that Dan was enraged.

Cooling off time for a reasonable person - If the jury finds the first two elements are satisfied, they must also find that there was not enough time for a reasonable person to cool off between the provocation and the act. The facts indicate that the entire event occurred in a very short period of time, although it does not specify how long. Had Vic or Dan left the scene of the altercation, or had someone else intervened such that there was a delay between the exchange of words and the punch, then the jury could find that there was time to cool off. However, because the facts do not show any appreciable time lapse, the jury will likely conclude that a reasonable person would not have had time to cool off.

Dan did not cool off - Finally, the jury must find that Dan did not cool off. The facts are pretty clear on this as well, since he punched Vic immediately after going into his rage. Thus the jury will likely find this is the case.

Dan will have the defense of insanity here as well, discussed below.

Insanity All jurisdictions recognize an affirmative defense of insanity, although there are four different theories across the various jurisdictions. Because it is an affirmative defense, the accused has the burden of proving by preponderance of the evidence that he met the test for insanity at the time in question. His sanity at the time of trial is not an issue. The evidence that supports Dan’s defense of insanity is that he has been in and out of mental institutions most of his life, that he has erratic behavior, and that he could not stop himself from striking Vic. These facts tend to show that he has a mental disease that affects his ability to conform to the law, which is at the heart of all four of the insanity tests.

M’Naughten Rule - Under the M’Naughten Rule, an accused is not guilty by reason of insanity if, because of a disease of the mind, he lacks the capacity to understand the wrongfulness of his acts or cannot appreciate the character of his actions. This is basically a test of whether the defendant’s mental disease prevents him from understanding right from wrong. The facts indicate that the jury could find that Dan has a disease of the mind because he has a history of mental illness and engages in erratic behavior. Dan’s testimony explaining the punch, however, was that he could not stop himself from striking Vic. He did not indicate that he did not understand that he was striking Vic, or that striking Vic was wrong. Instead, he struck Vic because he could not control himself. Consequently, if the jurisdiction uses this test, then it cannot find him not guilty by reason of insanity.

Irresistible Impulse Test - Under this test, an accused is not guilty by reason of insanity if, because of a disease of the mind, he cannot exercise the self-control to conform his actions to the requirements of the law. The facts indicate that the jury could find that Dan has a disease of the mind because he has a history of mental illness and engages in erratic behavior. Dan also testified that he could not stop himself from striking Vic; in other words, he struck Vic because he could not control himself. Consequently, if the jurisdiction uses

27 this test and the jury believes that Dan could not stop himself from striking Vic, and that the reason he could not do so was because of his mental illness, then it should find him not guilty by reason of insanity.

Durham Rule - Under the Durham Rule, an accused is not guilty by reason of insanity if the mental disease is the actual cause of the criminal act. In other words, if the act would not have been done “but for” the disease, then he is not guilty. The facts indicate that the jury could find that Dan has a disease of the mind because he has a history of mental illness and engages in erratic behavior. Consequently, if the jurisdiction uses this test and the jury believes that the reason Dan could not stop himself from striking Vic was because of his disease of the mind, then it should find him not guilty by reason of insanity. However, if it finds that the mental disease was unrelated to the reason he could not stop himself from striking Vic, then it should not find him not guilty by reason of insanity.

Model Penal Code Test - Under this test, an accused is not guilty by reason of insanity if, because of a disease of the mind, he is unable to appreciate the criminality of his conduct, or to confirm his actions to the requirements of the law. This is basically a blend of the M’Naughten Rule and the irresistible impulse test. As discussed above with regards to the latter, if the jurisdiction uses this test and the jury believes that Dan could not stop himself from striking Vic, it should find him not guilty by reason of insanity.

Therefore, if the jury uses the irresistible impulse test, the Durham rule, or the MPC test, it could properly find Dan not guilty by reason of insanity. If it uses the M’Naughten rule, it could not.

28 THURSDAY MORNING MARCH 1, 2007

California Bar Examination

Answer all three questions. Time allotted: three hours

Your answer should demonstrate your principles. Instead, try to demonstrate ability to analyze the facts in question, to your proficiency in using and applying tell the difference between material and them. immaterial facts, and to discern the If your answer contains only a points of law and fact upon which the statement of your conclusions, you will case turns. Your answer should show receive little credit. State fully the that you know and understand the reasons that support your conclusions, pertinent principles and theories of law, and discuss all points thoroughly. their qualifications and limitations, and Your answer should be complete, but their relationships to each other. you should not volunteer information or Your answer should evidence your discuss legal doctrines which are not ability to apply law to the given facts and pertinent to the solution of the problem. to reason in a logical, lawyer-like Unless a question expressly asks you manner from the premises you adopt to use California law, you should answer to a sound conclusion. Do not merely according to legal theories and show that you remember legal principles of general application.

ESSAY QUESTIONS AND SELECTED ANSWERS

JULY 2007 CALIFORNIA BAR EXAMINATION

This publication contains the six essay questions from the July 2007 California Bar Examination and two selected answers to each question.

The answers received good grades and were written by applicants who passed the examination. The answers were prepared by their authors, and were transcribed as submitted, except that minor corrections in spelling and punctuation were made for ease in reading. The answers are reproduced here with the consent of their authors.

Question Number Contents Page

1 Real Property 1

2 Torts 10

3 Evidence 22

4 Criminal Procedure/Constitutional Law 36

5 Remedies 45

6 Community Property 56

i

Question 4

Dan stood on the steps of the state capitol and yelled to a half-dozen people entering the front doors: “Listen citizens. Prayer in the schools means government-endorsed religion. A state church! They can take your constitutional rights away just as fast as I can destroy this copy of the U.S. Constitution.”

With that, Dan took a cigarette lighter from his pocket and ignited a parchment document that he held in his left hand. The parchment burst into flame and, when the heat of the fire burned his hand, he involuntarily let it go. A wind blew the burning document into a construction site where it settled in an open drum of flammable material. The drum exploded, killing a nearby pedestrian.

A state statute makes it a misdemeanor to burn or mutilate a copy of the U.S. Constitution.

It turned out that the document that Dan had burned was actually a copy of the Declaration of Independence, not of the U.S. Constitution, as he believed.

Dan was arrested and charged with the crimes of murder and attempting to burn a copy of the U.S. Constitution. He has moved to dismiss the charge of attempting to burn a copy of the U.S. Constitution, claiming that (i) what he burned was actually a copy of the Declaration of Independence and (ii) the state statute on which the charge is based violates his rights under the First Amendment to the U.S. Constitution.

1. May Dan properly be found guilty of the crime of murder or any lesser-included offense? Discuss.

2. How should the court rule on each ground of Dan’s motion to dismiss the charge of attempting to burn a copy of the U.S. Constitution? Discuss.

Answer A to Question 4

1. Murder or Any Lesser-Included Offense

Elements of a Crime

The four elements of a crime consist of (i) a guilty act, (ii) a guilty mind, (iii) , and (iv) causation.

For a person to be found guilty of a crime, the guilty act must be voluntary. Here, Dan appeared to only want to burn the document, not let it go and have it drift away. On the facts, it seems like he only let the document go involuntary when the heat of the fire burned his hand. So it appears that Dan may not have committed the requisite guilty act. However, if we frame Dan’s actions on a broader level, Dan did voluntarily burn the document and set into motion the chain of events leading to the ultimate killing of the pedestrian. The element of a guilty act is satisfied.

As to concurrence and causation, Dan’s intentional act of igniting the parchment document set into motion a chain of events: he let go of the burning document, it settled in an open drum of flammable material, and it caused the drum to explode and kill a nearby pedestrian. On the one hand, it appears that there is no proximate causation because it is arguably unforeseeable for someone to die from an explosion as a result of burning a document. On the other hand, courts are generally flexible when it comes to foreseeability, and there is a viable argument that the result was foreseeable because playing with fire is a dangerous activity. A court will probably find causation.

However, what we need to establish is whether Dan possessed the requisite guilty mind. The discussion below addresses this element.

Murder

At common law, murder is the unlawful killing of a human being with malice aforethought, which is established by any one of the following states of mind: (i) intent to kill, (ii) intent to do serious bodily harm, (iii) reckless indifference to an unjustifiably high risk to human life (i.e., depraved heart murder), and (iv) intent to commit a felony underlying the felony-murder rule.

Intent to Kill

From the facts, it does not appear that Dan knew any of the following facts: the nearby presence of the open drum with flammable material, the pedestrian’s presence near the drum, or the pedestrian’s identity. Therefore, he could not have formed a specific intent to kill the pedestrian. Dan cannot be found guilty of intent to kill murder. Intent to Do Serious Bodily Harm

On the facts, Dan did not intend to do any harm, let alone serious bodily harm. He was merely burning the document as a form of symbolic speech and probably did not even want to let go of the document.

Reckless Indifference to an Unjustifiably High Risk to Human Life

Dan’s act of igniting the document and letting it go did not reflect reckless indifference to an unjustifiably high risk to human life. No reasonable person would think that a burning document could ultimately kill someone. For example, Dan did not carry a dangerous weapon such as a gun and fire it into a crowded room.

Felony Murder

Under the felony-murder rule, a person can be found guilty of a killing that occurs during the commission of an underlying felony that is inherently dangerous, usually burglary, arson, rape, robbery, or kidnapping. Dan did not have the intent to commit any of these felonies.

Lesser Included Offenses

Voluntary Manslaughter

Voluntary manslaughter is an intentional killing committed with adequate provocation causing one to lose self-control. We have already established above that Dan cannot be found guilty of an intentional killing, so we need not determine whether it can be reduced to voluntary manslaughter. In any event, Dan was not even provoked to begin with.

Involuntary Manslaughter

Involuntary manslaughter is an unintentional killing that results either from (i) criminal negligence or (ii) misdemeanor-murder, which is a killing that occurs during the commission of a misdemeanor that is malum in se or inherently dangerous.

Criminal negligence exceeds negligence but is less than the reckless indifference of depraved heart murder. Significantly, for a person to be criminally negligent, he must have been aware of the risk. Here, Dan could have been aware of a general risk that results from a fire, which is an accidental burning of another object that occurs from a strong wind carrying the flame. On the other hand, Dan was not aware of the particular risk that an open drum of flammable material was nearby, which could kill someone. Dan cannot be found guilty of criminal negligence. On the other hand, Dan may be found guilty of misdemeanor-murder, because he committed the misdemeanor of burning or mutilating a copy of the U.S. Constitution, and the commission of the misdemeanor caused the ultimate death of the pedestrian. On the other hand, the misdemeanor was not malum in se and not inherently dangerous. Dan should not be found guilty of involuntary manslaughter.

Conclusion: Dan cannot be found guilty of the crime of murder or any lesser-included offense.

(2) Dan’s Motion to Dismiss the Charge of Attempting to Burn a Copy of the U.S. Constitution

(i) What he burned was actually a copy of the Declaration of Independence

Dan is being charged with attempting to burn a copy of the U.S. Constitution, but what he actually burned was the Declaration of Independence. At common law, factual impossibility is not a defense for attempting a crime. For example, if a person intends to shoot another with a gun and the gun happened to be out of bullets, the man is still guilty. However, legal impossibility is a defense to attempt. That is, if what the person was attempting to do was actually not a crime even though he thought it was, then he could not be found guilty of attempt.

Here, Dan’s assertion that he actually burned the Declaration of Independence is a claim of factual impossibility. From the facts, we know that he had the specific intent to destroy a copy of the U.S. Constitution, so even though it was factually impossible for him to do it because he was holding the Declaration of Independence, he can still be found guilty of attempting to burn a copy of the U.S. Constitution.

Conclusion: The Court should deny Dan’s motion to dismiss based on this ground.

(ii) The state statute on which the charge is based violates his rights under the First Amendment of the Constitution

The First Amendment protects free speech, and it is applicable to the states through the Fourteenth Amendment. The state action requirement is easily met here because it is a state statute making the act of burning or mutilating a copy of the U.S. Constitution a misdemeanor.

Symbolic Speech

Dan’s act was a form of symbolic speech. For a regulation of symbolic speech to be valid and not violative of the First Amendment, the law must have a purpose independent of and incidental to the suppression of speech and the restriction on speech must not be greater than necessary to achieve that purpose.

Here, the state statute does not appear to have a purpose independent of and incidental to the suppression of speech. For example, the burning of draft cards was upheld, because it was found that the government has a valid interest in facilitating the draft, and that the suppression of the speech was incidental and no greater than necessary. Here, preventing the burning of the Constitution does not appear to serve any significant government interest other than to prevent people from showing their anger toward the government, which is within their rights under the First Amendment.

Unprotected Speech

The government may attempt to frame Dan’s acts as unprotected speech that presents a clear and present danger. Such speech is intended to incite imminent unlawful action and is likely to result in imminent unlawful action, so that the state can regulate it. On the facts, Dan stood on the steps of the state capitol and yelled to a half dozen people entering the front doors while destroying what he thought was a copy of the U.S. Constitution, so arguably, he was trying to incite those people and get them enraged. On the other hand, there was no indication of encouraging harmful acts in his statement and burning a document in and of itself does not promote violence.

Moreover, even if the government can show that what Dan was specifically doing was inciting imminent unlawful speech, the government still cannot show that the state statute at issue is designed to restrain this kind of unprotected speech. The state statute merely bans burning the Constitution, but does not, for example, limit such acts to the steps of the state capitol, where the state might have an argument that doing such acts so close to government activity is dangerous and disruptive. The statute is overbroad and does not strive to only limit unprotected speech that is likely to incite imminent unlawful action.

Conclusion: The Court should grant Dan’s motion to dismiss based on this ground.

Answer B to Question 4

Murder Charges Against Dan (“D”)

The first issue is whether Dan may properly be found guilty of murder or any other lesser included offense.

Murder

Murder is defined as the killing of another human being with malice aforethought. In order to be found guilty of murder a Defendant must have committed a voluntary act and must have possessed the requisite mental state at the time of the act. A defendant will be guilty of murder if he committed the act (1) with the intent to kill, (2) with the intent to inflict great bodily injury, (3) if he acted in such a way as to demonstrate a reckless disregard for human life (often termed as having an “abandoned and malignant heart”), (4) or if the murder resulted during the commission of a highly dangerous felony.

Here, D’s act of igniting the document constituted a voluntary act. The fact that the heat of the fire had burned his hand, and caused him to involuntarily let it go does not negate the fact that his act of burning the document in the first place was voluntary. However, an act, in and of itself, is not sufficient to convict D of a crime. The State must also prove that, at the time D committed the act of burning the document, he had the intent to commit murder.

On these facts, it is clear that Dan did not set the document on fire with an intent to kill. While an intent to kill may be inferred in cases where the D uses a deadly, dangerous weapon against a victim (a gun, knife, etc.), that is not the case here. Additionally, D did not act with an intent to inflict great bodily injury on anyone. Instead, his act of burning the paper was done to make a political point to those that were present nearby.

The State may try and argue that Dan’s acts were done with an abandoned and malignant heart because, by igniting the document around individuals, he acted in a way that demonstrated reckless and unjustifiable disregard for human life. The State will not be able to meet their burden of proof under this theory either. Here, D’s act of burning the paper is not the type of act that an individual could expect would lead to someone’s death. The law demands more in order to show a reckless disregard for human life.

Felony Murder Rule

The state may try and argue that D should be convicted of murder based on the Felony Murder Rule (“FMR”). Under this rule, a D is liable for all deaths that occur during the commission of a highly dangerous felony, whether he intended to cause them or not. Instead, the intent is inferred from his intent to commit the underlying felony. In addition, the deaths caused during the commission of the felony must be foreseeable and must result before D has reached a point of temporary safety. Generally, the FMR has been reserved for deaths that occur during highly dangerous felonies, such as rape, arson, kidnapping, robbery, and burglary.

Here, the issue is whether D can be found guilty of one of these underlying felonies so that the FMR applies. The only one that would be applicable would be the crime of arson. In order to show that D is guilty of arson, the State must prove that D (1) acted with the intent, or was at least reckless, (2) in burning, (3) the dwelling, (4) of another. Here it is clear that D did not intend to burn the nearby construction yard. Instead, the fire resulted because a wind blew the lit paper into an open drum of flammable material. However, the State may try and argue that the act of igniting a document on fire and allowing the wind to carry it away constituted a reckless act. However, the State will also have to prove that D burned a dwelling. Here, the paper did not cause a dwelling to burn, but rather flew into a construction site.

Thus, D could not be convicted of the murder of the Pedestrian based on the Felony Murder Rule because he did not commit a highly dangerous felony.

Voluntary Manslaughter

Voluntary Manslaughter is a killing of another human being while acting under the heat of passion. Voluntary Manslaughter is generally reserved for cases in which the D kills another because of an “adequate provocation”. Here, Voluntary Manslaughter does not apply because there was no provocation which would have caused D to act the way that he did.

Involuntary Manslaughter / Misdemeanor Manslaughter

The remaining consideration is whether the State could properly convict D of involuntary manslaughter. Involuntary manslaughter is appropriate where the D is criminally negligent. Criminal negligence is a higher standard than is used in the tort context for negligence cases. In the criminal context, while D may not have been acting with an intent to kill, he nonetheless acted in a way that was so extremely unreasonable that a reasonable person in his shoes would have recognized that such actions are performed with a reckless disregard for the life of others. Here, the State will have to prove that not only was D’s act criminally negligent, but also that the Death was caused by D’s actions.

The State will likely fail on these facts because D’s act of burning a document does not rise to the level of a criminally negligent act. D’s conduct was not reckless in the sense that a reasonable person could have contemplated that burning a document could eventually lead to another person’s death. Moreover, the State will have a tough time meeting the causation requirement because, while D was the but-for cause in P’s death, the death was not foreseeable. Here, the death was caused by the explosion when the paper settled into an open drum of flammable material at the construction site. Thus, D could not, nor could a reasonable person foresee that such an act would result in a death due to such an explosion.

The State may also try and argue for misdemeanor manslaughter, which is appropriate when a death is caused during the commission of a lesser-included felony or by those specified by state statute. Here, it is highly doubtful that the burning of the Constitution is the type of misdemeanor that would be included under such a rule. As a result, the State will not succeed on these grounds.

2. Dan’s Motions to Dismiss

Attempt Charges vs. Dan

In order to prove attempt, the State must show that (1) D intended to commit the crime, and (2 he took a substantial step towards completing the crime. Regardless of the underlying crime, attempt is always a specific intent crime.

Here, the State will be able to show that D’s burning of a document that he believed to be the U.S. Constitution demonstrates his intent to commit the crime. Additionally, because he actually ignited the document, the second element is also satisfied. The issue thus is whether D has any valid defenses to the charge.

Mistake of Fact

D’s motion to dismiss is based on a of fact defense. Namely, he is arguing that, because he actually burned a copy of the Declaration of Independence, not the U.S. Constitution as he thought, he should not be found guilty for attempt.

D will fail in this defense because mistake of fact is not a good defense to attempt. That is because, here, if the circumstances had been as D believed (to burn the Constitution), he would have been guilty of the misdemeanor. By way of analogy, a thief who to receive stolen goods may not later argue that, because the police had secured the drugs and transferred them to him undercover, he cannot be guilty because the goods were no longer “stolen”. The fact remains that, had the circumstances been the way he believed them to be, he would have been guilty of the crime of receipt of stolen goods. Here, D’s mistake of fact may be a defense to the actual misdemeanor itself, but will not provide a defense to attempt.

First Amendment

The First Amendment protects an individual’s freedom of speech. However, included in the First Amendment is a protection of expressive activities that constitute speech. Here, it is clear that D’s act of burning the Constitution was an act of expression as it was intended to convey his political views regarding the problems inherent with government- endorsed religion and the commingling of church and state.

Statutes my limit expressive activity if they are unrelated to the expression that constitutes speech and are narrowly tailored to serve such goals. Here, the State may have a difficult time proving that this act is unrelated to expression because it seems to want to prevent individuals from burning or mutilating the Constitution as a way of expressing their political views.

The State would likely try and analogize to the U.S. Supreme Court case of O’Brien. There, a statute made it a crime to burn draft cards. When the defendant burned his draft card as a way of protesting against the war, he was prosecuted under the statute. The Court held that the statute was constitutional because it was not aimed solely at curtailing individuals’ ability to express their viewpoints. Instead, the County had an interest in the administrative matters of the draft and that draft cards were essential to the country keeping track of its draft members, soldiers, etc. Thus, because this statute was content- neutral, the Court applied intermediate scrutiny and found that the statute was narrowly tailored to a compelling state interest.

However, as noted above, no such interest appears to exist for the state’s statute in this case.

D will likely point to the flag burning cases, such as Johnson, where the Court has held that statutes making it a crime to burn the U.S. flag are unconstitutional because they restrict speech under the First Amendment. In the flag burning cases, the Court has noted that these statutes are aimed at curbing an individual’s right to express his views and thus warrant strict scrutiny. Because they are not necessary to advance a compelling interest, they are violative of the First Amendment.

The present case seems much closer to Johnson than O’Brien because the statute is aimed at expression rather than activities unrelated to expression. As such, it is unconstitutional because it impermissibly burdens the freedom of speech under the First Amendment. The State will have to meet a very high burden because strict scrutiny would be applied and thus it would have to show that the statute is necessary to advance a compelling state interest. Because no compelling interest appears to exist, the statute will be struck down.

ESSAY QUESTIONS AND SELECTED ANSWERS FEBRUARY 2008 CALIFORNIA BAR EXAMINATION

This publication contains the six essay questions from the February 2008 California Bar Examination and two selected answers to each question.

The answers received good grades and were written by applicants who passed the examination. The answers were prepared by their authors, and were transcribed as submitted, except that minor corrections in spelling and punctuation were made for ease in reading. The answers are reproduced here with the consent of their authors.

Question Number Contents Page

1 Torts 4

2 Professional Responsibility 16

3 Criminal Law and Procedure/Constitutional Law 25

4 Trusts/Wills and Succession 39

5 Community Property 51

6 Corporations/Professional Responsibility 60

2

Question 3

Dan’s neighborhood was overrun by two gangs: the Reds and the Blues. Vic, one of the Reds, tried to recruit Dan to join his gang. When Dan refused, Vic said he couldn’t be responsible for Dan’s safety.

After threatening Dan for several weeks, Vic backed Dan into an alley, showed him a knife, and said: “Think carefully about your decision. Your deadline is coming fast.” Dan was terrified. He began carrying a gun for protection. A week later, Dan saw Vic walking with his hand under his jacket. Afraid that Vic might be about to stab him, Dan shot and killed Vic.

Dan was arrested and put in jail. After his arraignment on a charge of murder, an attorney was appointed for him by the court. Dan then received a visitor who identified himself as Sid, a member of the Blues. Sid said the Blues wanted to help Dan and had hired him a better lawyer. Sid said the lawyer wanted Dan to tell Sid exactly how the killing had occurred so the lawyer could help Dan. Dan told Sid that he had shot Vic to end the . Dan later learned that Sid was actually a police informant, who had been instructed beforehand by the police to try to get information from Dan.

1. May Dan successfully move to exclude his statement to Sid under the Fifth and/or Sixth Amendments to the United States Constitution? Discuss.

2. Can Dan be convicted of murder or of any lesser-included offense? Discuss.

25

Answer A to Question 3

1. Dan’s Motion to Exclude his Statement to Sid

5th Amendment

The 5th Amendment protection demands that Miranda warnings be provided to persons that are in the custody of government officials prior to any interrogation. The Miranda rights to remain silent and to counsel must be waived before any statement used against the person in court is obtained. Miranda is not offense-specific.

A person is in custody if they reasonably believe they are not free to leave. Interrogation is defined as conduct or statements likely to elicit an incriminating response.

In this case, Dan was in jail. He had been arraigned for murder and was being held, so he was clearly not free to leave. Thus, custody is satisfied.

As to interrogation, Dan was approached by Sid, and Sid informed Dan that he was a member of the Blues, a rival gang to the gang of Vic, and that the Blues had hired an attorney to assist Dan. He said that the lawyer needed Dan to inform Sid of what happened so that he could represent him. In fact, Sid was a police informant, who had been instructed by the police to try to get information from Dan.

Clearly, Sid was talking to Dan in such a way that was likely to elicit an incriminating response; he was asking him to give the details so that Dan would have better representation. He had lied to Dan and was tricking him into confessing.

However, the problem here is that Dan did not know that Sid was a police informant who was seeking a confession. The court has upheld the admissibility of statements obtained by police informants when the suspect did not know that the informant was working for the government. The rationale is that the

26

factor is not so high, because the suspect does not know the police are involved. In other words, the suspect is free to not speak to the informant.

In this case, the court will have to weigh the fact that Dan did not know that Sid was a police informant against the devious nature of Sid’s behavior in lying to Dan in determining whether the interrogation factor is met. Based on the prior cases admitting police informant confessions, interrogation is probably not satisfied and the confession will probably not be barred by the 5th Amendment.

6th Amendment

The 6th Amendment guarantees every person the right to counsel at all critical post-charge proceedings and events, including questioning. This right is offense-specific and must be waived prior to questioning.

In this case, the time frame for the 6th Amendment protection had been triggered, because Dan had been arrested, put in jail, and arraigned for murder, all before Sid approached Dan. In fact, Dan had been appointed an attorney by the court.

When Sid, a government informant posing to be a member of a rival gang interested in helping Dan, approached Dan and elicited the incriminating response, he violated Dan’s 6th Amendment Right to Counsel. Sid initiated the conversation, and lied to Dan, tricking him into giving up the information. All the time, Sid was working as an informant. This equates to questioning by the government.

Because it was post-arraignment and the government sought to initiate questioning of Dan, Dan would have to first waive his right to have counsel present, or have his attorney present. Dan did not waive this right, because he did not even know Sid was a government informant, and his attorney was not present.

Because Dan’s 6th Amendment right to counsel was violated, he can successfully move to exclude his statement to Sid from trial.

When he makes this motion, the government will have to prove by a preponderance of the evidence that the statement is admissible, a burden they will not be able to meet on the existing facts. Thus, the statement will be excluded.

2. Can Dan be Convicted of Murder or any Lesser-Included Offense

Murder is the unlawful killing of another human being with malice aforethought.

27

It requires actus reus, which in this case was Dan’s act of shooing Vic.

It also requires causation, both actual and proximate. Actual cause is easily satisfied because “but for” Dan’s act of shooting Vic, Vic would not have died. Proximate cause is the philosophical connection which limits liability to persons and consequences who [sic] bear some reasonable relationship to the actor’s conduct, so as to not offend notions of common sense, justice, and logic. Proximate cause is also easily satisfied, because Dan shot and killed Vic without any intervening cause or unforeseeable event. If one shoots a human being, death is a logical and foreseeable result.

Malice is satisfied under one of four theories: 1. Intent to kill; 2. Intent to commit great bodily injury; 3. Wanton and Willful disregard of human life (“Depraved Heart Killing”); or 4. Felony Murder Rule.

Intent to Kill

Intent to kill can be satisfied by the deadly weapon doctrine: where the death is caused by the purposeful use of a deadly weapon, intent to kill is implied.

In this case, Dan used a gun, pointed it at Vic, shot Vic, and killed Vic. A gun is a deadly weapon, so intent to kill is satisfied.

Intent to Commit Great Bodily Injury

Even if intent to kill were not satisfied, intent to commit great bodily injury would be apparent because the least that can be expected to occur when one points a gun at a human being and pulls the trigger is great bodily injury.

Wanton and Willful Disregard

In addition, wanton and willful disregard for human life is satisfied because the use of a gun against another human being shows a conscious disregard for human life. Guns can, and frequently do, kill people. In fact, killing things is one of their main purposes. The use of a gun against another human being shows disregard for the human being’s life.

Felony Murder Rule

The felony murder rule requires an underlying felony, that is not “bootstrapped” to the murder. In this case, Dan does not appear to have

28

committed any crime except for killing Vic, so the malice could not be implied under the felony murder rule.

Murder in the First Degree

Murder in the first degree at common law was the intentional and deliberate killing of another human being. It required deliberation, but deliberation can happen in a very short period of time.

In this case, Vic had “terrified” Dan, and Dan began carrying a gun for protection. Dan carried this gun for an entire week before he saw Vic. In obtaining the gun, or taking it from its storage place, putting it on his person, and carrying it around for an entire week, Dan acted intentionally and deliberately. When he saw Vic, he then pulled out the gun and shot and killed Vic.

These facts, especially the elapse of an entire week, are probably sufficient to show that Dan was intentional and deliberate in his use of the gun. It did not arrive there by chance, and once Dan saw Vic, he acted without pause.

Murder in the Second Degree

All murder that is not murder in the first degree is murder in the second degree.

If the prosecution was not able to establish Dan intentionally and deliberately shot Vic, because perhaps the jury believed that Dan did not deliberate before he shot Vic, then he could be convicted of second-degree murder.

Self-Defense

Self-defense is the use of reasonable force to protect oneself at a reasonable time. Deadly force may only be used to protect against the use of deadly force.

Dan will argue that he was engaged in self-defense when he shot Vic. Dan will point out that his neighborhood was run by two gangs, and as such it was very dangerous. He will testify that Vic was a Red, one of the gangs, and that he had tried to recruit Dan to the gang. When Dan refused, Vic said he “couldn’t be responsible for Dan’s safety,” implying that Dan might be injured.

Vic then threatened Dan for several weeks, and finally backed him into an alley, showed him a knife, and told him that “Your deadline is coming fast.” Dan will argue that the statement regarding Dan’s safety, the threats, the knife and the deadline statement cumulate to show that Vic intended to kill Dan if he wouldn’t join the gang, or at least that Dan reasonably believed Vic would do it.

29

Dan will argue that when he then saw Vic on the street, with his hand under his jacket, he was terrified and afraid that Vic might stab him with the knife he had threatened him with, and therefore he defended himself by shooting Vic.

The primary problem with Dan’s defense is that he carried around a gun for a week before seeing Vic, and then when he saw Vic with his hand under his jacket he pulled out the gun and shot Vic, without Vic producing any weapon or making any threat at that time. The state will argue that Dan is not entitled to a self-defense defense because he was under no threat when he shot Vic.

Unreasonable Self-Defense

Unreasonable self-defense is a defense available to one who engages in good faith but unreasonable self-defense. It is a mitigating defense which takes a murder charge down to voluntary manslaughter.

Dan will argue that if self-defense was not appropriate because of the timing of the threats and the shooting, then he is at least entitled to an unreasonable self-defense defense. Dan will argue that he acted in good faith and really believed Vic would stab him.

This is a very colorable defense for Dan, because although the timing of self-defense was inappropriate, Vic had been threatening Dan for several weeks, and had recently shown him a knife and said “Your deadline is coming fast,” so Dan’s fear was likely reasonable.

Heat of Passion

Heat of passion is a defense when circumstances evoke a sudden and intense heat of passion in a person, as they would affect a reasonable person, without a cooling off period, and the person does not cool off. Heat of passion is a possible defense during a fight.

In this case, however, it is likely not viable because Dan had not seen Vic for an entire week before the shooting, which is sufficient time for a reasonable person to cool off from the last incident with the knife in the alley. For that entire week, Dan carried around a gun, and then when he saw Vic he shot and killed him, without any prior interaction on that occasion. It appears unlikely that Dan’s response was “sudden” or “intense”.

Involuntary Manslaughter

Involuntary manslaughter is established by a killing with recklessness not so egregious as to satisfy wanton and reckless disregard for human life, but more serious than common negligence.

30

Involuntary manslaughter could be established by the reckless use of a gun, but because Dan intended to kill Vic, Dan will be convicted of a greater crime, or, if his self-defense defense is effective, of no crime at all.

Conclusion

Dan will likely be tried for first-degree murder under the intent to kill theory, and will allege the defenses of self-defense and imperfect self-defense. Dan is likely to be found guilty of voluntary manslaughter, by use of an imperfect self- defense defense.

31

Answer B to Question 3

Dan’s Motion to Exclude

Exclusionary Rule

The exclusionary rule prohibits the introduction of evidence obtained in violation of defendant’s 4th, 5th, and 6th Amendment rights, and under the “fruits of the poisonous tree” doctrine, also prohibits any evidence found as a result of violating defendant’s 4th, 5th, and 6th Amendment rights, with limited exceptions. Thus, if Dan’s confession violated his 5th or 6th Amendment rights, the statement cannot be admitted.

5th Amendment Right

The 5th Amendment provides that a defendant should be free from self- incrimination. The right applies to testimonial evidence coercively obtained by the police. Under the 5th Amendment, before the police conduct custodial interrogation, the police must give the defendant his Miranda warnings. Miranda warnings inform the defendant of his right to remain silent and the right to an attorney. The 5th Amendment right is non-offense specific, meaning that even if the defendant exercises his rights, the police can question him about an unrelated offense. If the defendant asserts his right to remain silent, the police must abide by defendant’s right, although they can later question him after a reasonable amount of time has passed. If the defendant unambiguously asserts his right to an attorney, the police cannot question him without either providing an attorney or obtaining a waiver of the right to counsel.

The 5th Amendment right to remain silent and to counsel only applies in custodial interrogation. A person is in custody if he or she is not objectively free to terminate an encounter with the government. A person is subject to interrogation if the police engage in any conduct that is likely to elicit a response, whether incriminating or exculpatory.

Dan will argue that he was subject to custodial interrogation because (1) he was in prison and not free to leave, and (2) the informant was planted in order to elicit statements from Dan. Clearly, Dan was in custody, as he was in jail. Dan may have a harder time proving he was subject to interrogation. Typically,

32

interrogation only occurs when the person is aware that he is in contact with a government informant. The prosecution will argue that Dan was not aware that Sid was a government informant, and believed that Sid was a gang member who was trying to help him. Thus, the prosecution will argue, the police were not required to give Dan his Miranda rights before commencing the questioning. The prosecution will argue that if Dan trusted Sid and willingly spoke to him, he cannot now claim that the statement constituted interrogation or was coercively obtained. As Dan did not know that Sid was a government informant, he will likely fail in arguing that he should have received his Miranda rights before Sid questioned him. Thus, he will not be able to exclude his statement on 5th Amendment rounds.

Impeachment Purposes

Even if Dan’s statement violated his 5th Amendment right, the statement may still be used to impeach Dan’s testimony if he testifies at trial.

Fruits of Miranda

If the police obtained any evidence as a result of Dan’s statement to the informant, these “fruits of Miranda” may be admissible. The Supreme Court has not conclusively determined whether such fruits are admissible, but they likely are.

6th Amendment Right

The 6th Amendment provides the right to counsel at all criminal proceedings. It applies once the defendant has been formally charged with a crime, and prevents the police from obtaining an incriminating statement after formal charges have been filed without first obtaining the defendant’s waiver of counsel. The right is offense-specific, meaning it only attaches for the crime(s) for which the defendant has been formally charged. It does not prevent the police from questioning the defendant about unrelated offenses.

Here, Dan had been [under] arraignment on a charge for murder, so formal charges had been filed by the government. Thus, Dan was entitled to counsel at any post-charge police interrogation. Dan will argue that by subjecting him to interrogation by a police informant after formal charges had been filed without obtaining a waiver of his right to counsel, the police violated his 6th Amendment right.

The police will argue that Dan was not aware that Sid was a government informant, but this awareness is not necessary for a 6th Amendment violation. Once Dan’s rights to counsel attached at his arraignment, Dan had a right to counsel during police interrogation to prevent the police from deliberately eliciting

33

an incriminating statement. The police used a government informant who lied to Dan about his identity, made a promise of a better attorney, and asked him about his involvement with the crime, in order to obtain a confession from Dan. The police did all of this without waiving Dan’s right to have his attorney present during the interrogation. Dan’s right to counsel under the 6th Amendment has been violated, and Dan is entitled to exclusion of the statement at his trial.

Like a violation of Dan’s 5th Amendment right, the prosecution may use a coercively obtained confession to impeach Dan’s testimony at trial.

Conclusion

Dan’s statement to Sid likely violated his 6th Amendment right to counsel at any post-charge interrogation, because he had already been arraigned. The police should have obtained a waiver of Dan’s right to counsel before sending Sid in, and it should not matter that Dan did not know that Sid was a police informant. However, because Dan did not know that Sid was working for the government, the questioning and subsequent statement did not likely violate Dan’s 5th Amendment rights to Miranda warnings.

Thus, Dan will likely be successful in his motion to exclude his statement under the exclusionary rule as a violation of his 6th Amendment right.

Dan’s Conviction for Murder or any Lesser-Included Offense

Murder

Murder is the unlawful killing of another human being with malice aforethought. Malice aforethought exists if there is no excuse justifying the killing and no adequate provocation can be found, and if the killing is committed with one of the following states of mind: intent to kill, intent to inflict great bodily injury, reckless indifference to an unjustifiably high risk to human life, or intent to commit a felony.

The prosecution will argue that Dan is guilty of murder because no excuse existed (duress is not an excuse to homicide), no adequate provocation exists, and he had any one of the three following states of mind: intent to kill, intent to inflict great bodily injury, or a reckless indifference to an unjustifiably high risk to human life.

The prosecution will argue that no excuse existed for Dan to kill Vic. The prosecution will argue that even though Dan may have felt he was under duress imposed by Vic, this does not justify the killing of Vic, for two reasons: (1) the duress was to join the Reds, not to kill Vic, and (2) duress cannot be used as an excuse for homicide. The prosecution will also argue that no excuse existed from Vic’s actions toward Dan during the incident where he was killed that would

34

give Dan the reasonable belief that he was about to be killed or seriously injured. The prosecution will note that there is no evidence that Vic was even aware of Dan’s presence, that Vic did not confront Dan with unlawful force, and that it was unreasonable that Dan thought he was about to be stabbed.

The prosecution will be required to show that adequate provocation did not exist for Dan’s killing of Vic, and that Dan had one of the required states of mind here. Adequate provocation is discussed in detail below, but the prosecution will argue that even if Dan was subjected to a serious battery, he had a week to cool off from the provocation of that battery, and thus was not still under the direct stress imposed by that battery when he killed Vic.

The prosecution will also argue that Dan had any of the states of mind listed above. By pulling out his gun and pulling the trigger, Dan intended to kill Vic. This intent was evidenced by an awareness that the killing would occur if he pulled the trigger, and a conscious desire for that result to occur. The prosecution can also argue that if he did not intend to kill Vic, he knew or acted recklessly as to whether Vic would suffer great bodily injury as a result of the shooting. Finally, the prosecution can argue that by pulling the trigger, Dan was acting with a reckless disregard to the unjustifiably high risk to Vic’s life that would occur from his actions. Dan, the prosecution will argue, clearly did not care whether Vic lived or died as a result of the shooting, and thus Dan had the requisite intent to be convicted of murder.

Because the prosecution can show that no excuse or adequate provocation existed, and that Dan acted with one of the states of mind required for murder, Dan can likely be convicted of murder unless he has a valid defense. In addition, if the prosecution can show that the killing was deliberate and premeditated, Dan may be guilty of first-degree murder. The prosecution will show that the killing was deliberate and premeditated because Dan was carrying a gun and shot Vic almost immediately after seeing him in the street.

Self-Defense

Self-defense is a complete defense to murder. Self-defense is justified when the defendant reasonably believes that the victim is about to kill him or inflict great bodily injury upon him. Deadly force may be used in self-defense if the defendant is not at fault, is confronted with unlawful force, and is subject to the imminent threat of death or great bodily harm.

Dan will argue that the defense of self-defense should completely bar his conviction for murder. Dan will point to the history between the parties as well as Vic’s actions at the scene of the crime to establish that he was justified in using deadly force against Vic. Dan will argue that Vic had subjected him to a serious battery when he pushed him into the alley, showed him a knife, and threatened him. Dan will argue that this battery made Dan aware that Vic was a serious

35

criminal (and that Dan already had knowledge of Vic’s criminality because he was involved in a gang), and that Vic would stop at nothing to injure Dan if Dan refused to join his gang.

With this history, Dan will argue that it was reasonable for him to believe that Vic was about to shoot him, because Vic was walking with his hand under his jacket, Dan will argue that the history between the parties and Vic’s suspicious behavior made it reasonably likely that he was about to be stabbed, and thus he was justified in using deadly force in self-defense.

The prosecution will argue that even if the history between the parties made Dan afraid of Vic, that Vic had not confronted Dan with any unlawful force before Dan shot him. There is no evidence that Vic even saw Dan walking down the street. In addition, the prosecution will argue that even if Vic had plans to harm Dan, he wanted Dan to join his gang and would have only injured him if Dan refused to join the gang once again. While Dan was obviously not required to join the gang, this evidence will support the prosecution’s defense that Dan’s belief that he was about to be subject to immediate harm was unreasonable. At the very least, Vic probably wanted to talk to Dan one more time before inflicting harm upon him, so Dan was not subject to an immediate threat of death or bodily harm. The prosecution will argue that Dan should have waited until Vic produced the knife before shooting, or, at the very least, approached Dan in a threatening manner. Because Vic did not do these things, Dan cannot use the defense of self-defense.

Duress

Dan may argue that he was under duress, and this resulted in his killing of Vic. Duress is a good defense when the defendant is coercively forced under threats from another to commit a criminal act. Duress may have been a good defense if Dan was forced to join the gang and commit criminal acts. However, duress cannot be used to defend against homicide. Thus, this defense will fail.

Voluntary Manslaughter

Dan may try to get his charge lessened to voluntary manslaughter. Voluntary manslaughter is a killing that would be murder but for the existence of adequate provocation. Adequate provocation will be found where: the provocation is such that it would provoke a reasonable person, the defendant was in fact provoked, the facts suggest that the defendant did not have adequate time to cool off, and the defendant did not in fact cool off.

Dan will argue that Vic’s repeated threats to him constituted adequate provocation. He will argue that being shoved into an alley, being shown a knife, and given basically a death threat is enough to provoke anger in the mind of a reasonable, ordinary person. Courts typically use an aggravated battery, as Vic

36

has committed here, as existence of adequate provocation. Dan will also argue that he was provoked, evidenced by carrying a gun for protection and living in fear of Vic.

However, Dan will have a harder time showing that a reasonable time to cool off could not be found, and that he did not in fact cool off. A week existed between Vic’s aggravated battery of Dan and Dan’s killing of Vic. While Dan may have still been frightened of Vic, a week is likely too long to find that Dan was still acting under the provocation supplied by Vic during the aggravated battery. Rather, Dan likely had cooled off, but was still upset by the incident and repeated threats.

It is likely that the prosecution can successfully argue that adequate provocation did not exist here because Dan was not acting under the direct stress imposed by the serious battery committed by Vic when he shot and killed Vic. However, if Dan can show such adequate provocation, his charge should be reduced to voluntary manslaughter.

Manslaughter

Dan may try to get his charge lessened to a manslaughter charge under the ‘imperfect self-defense” doctrine. Dan will argue that even though he may be ineligible to use the self-defense as a valid defense because Vic had not confronted him with unlawful force, he reasonably believed that it was necessary to shoot Vic to avoid being killed or subject to serious bodily harm. It is more likely that a court will accept Dan’s argument for a lesser charge of manslaughter under the imperfect self-defense doctrine, rather than accepting Dan’s total defense of self-defense, because Vic did not do anything during the incident where he was shot to suggest that he was about to kill Dan or subject Dan to great bodily harm.

Thus, Dan may likely be convicted of murder, voluntary manslaughter, or manslaughter.

37

THURSDAY MORNING FEBRUARY 28, 2008

California Bar Examination

Answer all three questions. Time allotted: three hours

Your answer should demonstrate your ability to analyze the facts in question, to tell the difference between material and immaterial facts, and to discern the points of law and fact upon which the case turns. Your answer should show that you know and understand the pertinent principles and theories of law, their qualifications and limitations, and their relationships to each other. Your answer should evidence your ability to apply law to the given facts and to reason in a logical, lawyer-like manner from the premises you adopt to a sound conclusion. Do not merely show that you remember legal principles. Instead, try to demonstrate your proficiency in using and applying them. If your answer contains only a statement of your conclusions, you will receive little credit. State fully the reasons that support your conclusions, and discuss all points thoroughly. Your answer should be complete, but you should not volunteer information or discuss legal doctrines which are not pertinent to the solution of the problem. Unless a question expressly asks you to use California law, you should answer according to legal theories and principles of general application.

38

THE STATE BAR OF CALIFORNIA OFICE OF ADMISSIONS 180 HOWARD STREET • SAN FRANCISCO CALIFORNIA 94105 1639 • (415) 538 - 2303 1149 SOUTH HILL STREET • LOS ANGELES CALIFORNIA 90015-2299 • (213) 765 – 1500

ESSAY QUESTIONS AND SELECTED ANSWERS JULY 2009 CALIFORNIA BAR EXAMINATION

This publication contains the six essay questions from the July 2009 California Bar Examination and two selected answers to each question.

The answers received good grades and were written by applicants who passed the examination. The answers were prepared by their authors, and were transcribed as submitted, except that minor corrections in spelling and punctuation were made for ease in reading. The answers are reproduced here with the consent of their authors.

Question Number Contents Page

1 Torts/Civil Procedure/Professional Responsibility 3

2 Professional Responsibility 16

3 Evidence 27

4 Constitutional Law 50

5 Civil Procedure/Remedies/Professional Responsibility 66

6 Criminal Law and Procedure 82

1

Question 6

Polly, a uniformed police officer, observed a speeding car weaving in and out of traffic in violation of the Vehicle Code. Polly pursued the car in her marked patrol vehicle and activated its flashing lights. The car pulled over. Polly asked Dave, the driver, for his driver’s license and the car’s registration certificate, both of which he handed to her. Although the documents appeared to be in order, Polly instructed Dave and his passenger, Ted: “Stay here. I’ll be back in a second.” Polly then walked to her patrol vehicle to check for any outstanding arrest warrants against Dave.

As she was walking, Polly looked back and saw that Ted appeared to be slipping something under his seat. Polly returned to Dave’s car, opened the passenger side door, looked under the seat, and saw a paper lunch bag. Polly pulled the bag out, opened it, and found five small bindles of what she recognized as cocaine.

Polly arrested Dave and Ted, took them to the police station, and gave them Miranda warnings. Dave refused to answer any questions. Ted, however, waived his Miranda rights, and stated: “I did not know what was inside the bag or how the bag got into the car. I did not see the bag before Dave and I got out of the car for lunch. We left the windows of the car open because of the heat. I did not see the bag until you stopped us. It was just lying there on the floor mat, so I put it under the seat to clear the mat for my feet.”

Dave and Ted have been charged jointly with possession of cocaine. Dave and Ted have each retained an attorney. A week before trial, Dave has become dissatisfied with his attorney and wants to discharge him in favor of a new attorney he hopes to select soon.

What arguments might Dave raise under the United States Constitution in support of each of the following motions, and how are they likely to fare:

1. A motion to suppress the cocaine? Discuss.

2. A motion to suppress Ted’s statement or, in the alternative, for a separate trial? Discuss.

3. A motion to discharge his present attorney and to substitute a new attorney in his place? Discuss.

82

Answer A to Question 6

1. Motion to suppress the cocaine

Standing: Dave has standing to bring this motion because he is being charged with possession of cocaine that was found in his car. He, unlike Ted, has a reasonable expectation of privacy in compartments within his car that are not visible in plain view, and can therefore assert a violation of the 4th Amendment if they are unlawfully searched, and assert the exclusionary rule to suppress evidence found that way.

Traffic stop A police officer has the right to stop and detain a car that is violating any provision of the vehicle code. Here, the car was speeding and weaving in violation of the code, so Polly had the right to cause the car to pull over. Upon such a stop, both the driver and passenger are considered detained according to the Terry v Ohio doctrine. The request for Dave’s driver’s license and registration were lawful, as was her intended search for arrest warrants.

Search However, instead of going to her patrol car, Polly saw Ted “slip something under the seat.” This must have been a very minimal viewing, and somewhat lacks credibility, because Ted was in the passenger seat, and Polly was walking away from the driver’s side back to her own vehicle. Anyway, assuming that she actually did [see] what she says she saw, her actions were still unlawful. Polly opened Ted’s car door, looked under his seat, and opened a bag found there. This action qualifies as a search, because a person has a reasonable expectation of privacy in the compartments of his car which are not visible in plain view. The contents of a paper bag under a car seat are certainly not in plain view. Therefore, to search it, Polly needed a warrant, or a warrant exception.

83

Auto Exception: The auto exception the warrant requirement allows an officer to search any compartment within a car in which the officer has probable cause to believe that she will find evidence of a crime. Here, Polly saw Ted “slip something under his seat.” Under these circumstances, that sight is not enough to generate probable cause. If asked, she could not articulate with particularity what it is she suspected she saw. There were no other facts to cause Polly to suspect that something under Ted’s seat would contain evidence of a crime. The mere fact that Ted appeared to be concealing whatever-it-was is not enough. A Supreme Court case involving a student on school grounds, who held a black pouch behind his back when approached by the , provides precedent that the mere inarticulate hunch or suspicion created when a suspect appears to be hiding something is not enough to create reasonable suspicion, much less the higher standard of probable cause.

Search incident to arrest: Before a Supreme Court decision [in] March of 2009, an officer would be allowed to search the passenger compartment of a car during or after the arrest of a car’s occupant, based on a search incident to arrest. However, this rule has been changed, and does not allow a search if the passenger has been removed and is no longer in arm’s reach of the contents of the car. Additionally, Polly had not chosen to arrest Ted and Dave at the time she made the search. Although she had the right to arrest Dave for a vehicle code infraction, she had not made the decision to do so, and therefore, even under the old rule, she would not have been able to use this exception to search under Ted’s seat.

Terry frisk As stated earlier, the traffic stop was a detention. When an officer detains a suspect because of a reasonable suspicion that a crime has occurred (here, the vehicle code infractions), she has the right to frisk the suspect for weapons to protect herself. This allows a visual scan, as well as a brief physical inspection of the outer garments by running her hands along them. To do this, the officer must have at least a reasonable

84 suspicion that the person might be carrying a weapon. Here, Polly went far beyond what was allowed. She wasn’t looking for weapons; she was simply indulging her suspicious curiosity when she checked to see what Ted put under the seat. As mentioned above, she had no reason to believe Ted would be concealing a weapon. Now, if perhaps she had run her check for warrants, and found a warrant out for Ted or Dave for a violent offense, that might have generated the necessary suspicion for some kind of frisk. But even then, the frisk would have required her to command Dave and Ted out of the car and she could frisk their clothing - not permitted her to look under their seats and inside bags.

Conclusion: Since no warrant exception permitted Polly to make the search, and she did so in violation of Dave’s reasonable expectation of privacy without a warrant, the search was unlawful, the cocaine that was found is “Fruit of the poisonous tree” and should be excluded.

2. Motion to suppress Ted’s statement or for a separate trial Confrontation Clause A statement by a coconspirator is not admissible against a defendant as an admission of a party opponent. Therefore it must be admissible under some other hearsay exception if it is hearsay. Even if it is admissible under evidence law, the constitution sometimes allows for suppression. The confrontation clause of the constitution requires that for any testimonial evidence offered against a defendant, the defendant must have the opportunity to confront and cross-examine the declarant. Here, Dave and Ted are being tried jointly, and Ted’s statement is offered substantially against both of them. Ted’s statement is not admissible against Dave unless Ted can be cross-examined. And because it is Ted’s trial too, Ted has the right not to take the stand because of his Fifth Amendment right against self-incrimination. If Ted exercises this right, then Ted cannot be cross- examined, and Dave’s right of confrontation is violated. The remedy is, as Dave requested, to either exclude the statement, or try Ted and Dave separately.

85

The prosecution, if it wishes to avoid both these remedies, can argue that the statement is not offered “against” Dave. The statement really doesn’t incriminate Dave in any way; in fact, it is more exculpatory than anything for both defendants. More facts would be needed to be sure of this, because if Dave’s defense is that Ted owned the cocaine, then the statement, while good for Ted, weakens Dave’s defense. Or if Ted has changed his story, this prior inconsistent statement may hurt Ted’s credibility, which may hurt Dave’s defense by association with Ted. So the prosecution‘s attempt to include the statement and maintain a joint trial will probably fail, but will succeed if Ted’s statement is not harmful to Dave’s defense. If the statement is helpful to [the] prosecution of Ted, the prosecution will not wish it to be excluded. Rather than exclude it, the prosecution will prefer to try Dave separately, and this remedy will be granted upon the prosecution’s agreement.

Miranda Even if Ted’s statement was obtained in violation of Miranda rights or 14th Amendment voluntariness rights, Dave cannot assert those rights as a reason to exclude the statement from use against him. A defendant can only assert his own constitutional rights in seeking to exclude evidence, not those of another person.

3 . Motion to discharge Dave’s attorney and substitute a new attorney in his place A criminal defendant has an absolute right to counsel at trial, as long as incarceration is a possible punishment. The issue is whether Dave has a right to discharge and replace his attorney a week before trial. Dave has retained an attorney, not used a publicly provided one, and this is helpful to his case, because no public financial hardship is involved. However, because [the] trial is so soon, the court has discretion to grant Dave’s motion only if it finds that the case will not be unduly delayed. The court will not permit Dave to delay the case so much that he will have a defense of a speedy trial violation; however, it may allow Dave the delay if he waives that defense. And, if the substitution will cause delay that will make a necessary witness unavailable, the court will be disinclined to grant it.

86

The court will balance Dave’s interests as well. If he has differences with his attorney that make it impossible for his attorney to provide him with competent representation, then the court will be strongly inclined to grant the substitution, because otherwise Dave may have a case for Ineffective Assistance of Counsel that could undo the court’s and prosecution’s time and efforts. If the only consequence of the substitution will be delay, the court will consider its calendar, and it will also consider the right to a speedy trial. But weighing all these considerations, the court will likely permit the substitution because no facts show that any undue burden on the court will occur.

87

Answer B to Question 6

Question 1: The Motion to Suppress the Cocaine

Fourth Amendment / Fourteenth Amendment Applicability: Any action by the state (a government official) that invades a person’s reasonable expectation of privacy (REOP) will trigger the applicability of the Fourth Amendment protections against unreasonable searches and seizures.

Here, assuming that Polly was a state police officer, the Fourth Amendment will apply to her actions through selective incorporation via the Fourteenth Amendment.

Fourth Amendment -- State Action: Private actors are not bound to constitutional norms. As mentioned above, any Fourth Amendment challenge to a search or seizure must involve “state action” in the searching and seizing. Here, there is no question that Polly, a police officer, is an agent of whatever state or local government she works for. Since her actions revealed the cocaine, the state action requirement is satisfied.

Fourth Amendment -- Reasonable Expectation of Privacy: To have standing to bring a Fourth Amendment claim to suppress seized evidence, the person asserting the claim must have standing.

To have standing under the Fourth Amendment, Dave must prove that he had a reasonable expectation of privacy in the contents of his passenger compartment. Under existing case law, because Dave is the owner of the vehicle that was stopped by Polly, Dave has a reasonable expectation of privacy in the contents of the passenger compartment of the vehicle, as well as the trunk and any other places that items could be stored.

Note also that the state cannot argue that Dave lacked a REOP due to the item being in plain view from the exterior of the car (placing an item in plain view in the passenger

88 compartment may indicate that the owner had no reasonable expectation of privacy), the item in question--the bag--was under a passenger seat, and not visible from the exterior of the car.

Therefore, Dave has standing (a REOP in the item seized) to move for its suppression.

The Traffic Stop -- Lawful Stop: A police officer may conduct a routine traffic stop if the police officer has reasonable suspicion that a law has, is, or will be violated by the occupants of the car, or if the police officer has probable cause that the car contains contraband, or the driver has violated the law.

Here, Polly personally observed Dave’s car “speeding” and “weaving in and out of traffic” in violation of the Vehicle Code. Therefore, Polly was justified under the Fourth Amendment in stopping the car, because she had at least reasonable suspicion, if not probable cause, that a law had been violated.

The Traffic Stop -- Lawful Seizure: The Supreme Court has made clear that a traffic stop seizes not only the driver, but any passengers, under the Fourth Amendment. However, because the stop was justified (as discussed above), this seizure is lawful under the Fourth Amendment.

The Search of the Passenger Compartment -- Improper Search

Warrant Requirement

The general rule, subject to a number of exceptions, is that any search by a state actor of any area that a person has a REOP in cannot be conducted without (1) probable cause, (2) supported by a validly executed warrant.

Here, it is clear that Polly did not have a validly executed warrant to search Dave’s car. Therefore, we must look to see whether any exceptions will apply to this general rule.

89

Automobile Exception Does Not Apply Because NO PROBABLE CAUSE

The automobile exception, which exists because items in an automobile may be quickly transported and disappear before a warrant can be applied for and issued, is only a replacement for the general warrant requirement. However, it does not absolve the state actor from having probable cause to search.

Probable cause to search means that the person has probable cause to believe that the place to be searched will contain specific items of contraband. It is determined based upon a totality of the circumstances, and must be based upon more than just mere suspicion, but reliable sources and articulate observations.

Here, Polly merely saw Ted slipping “something” under his seat as she was walking away. Polly had no other facts to support a belief that the item was contraband or a weapon, nor could she be sure that Ted was actually performing that act (she was walking when she observed it). Therefore, Polly did not have probable cause to perform the search of Dave’s car. Moreover, the basis for the stop itself was a routine traffic violation, and not something (perhaps intoxicated driving) that would provide probable cause to search the automobile compartment (perhaps for open liquor bottles).

Because Polly did not have probable cause to search Ted’s car, the automobile exception cannot apply.

An Exception to Probable Cause -- A Terry Search of the Car: An officer may conduct a “Terry Frisk” of a person if the officer has reasonable and articulable suspicions that the person may be armed. This is to ensure that officers are safe while conducting their duties.

Here, the state may argue that Polly’s observation created an articulable and reasonable suspicion that the occupants of the car were stowing weapons or other materials that might put her in danger. Therefore, pursuant to her lawful seizure of Ted

90 and Dave, she was within her rights to conduct a “Terry Search” of the automobile (only for weapons) to ensure her safety.

However, a Terry search is limited solely to a search of weaponry, and the paper lunch bag was likely clearly not a weapon (even if Polly conducted a plain feel of it, which she didn’t). Polly was not authorized to open the bag under a Terry search theory, because she did not first ascertain that it was contraband based upon a “plain feel.”

Therefore, this exception will also not apply.

Plain View Does Not Apply: As mentioned earlier, because the paper bag was beneath the passenger seat, the item was not in plain view of the officer from a lawful vantage point (outside the car), nor was the paper bag immediately incriminating on its face. Therefore, the discovery of the paper lunch bag does not meet either of the requirements for this exception.

Evanescent Exception Does Not Apply: The evanescent exception often applies to contraband that can be easily disposed of, or will easily disappear, thereby excepting officers from obtaining a valid warrant. However, it requires that the officer have probable cause to search the area in which the contraband is discovered. Because no probable cause existed, this exception does not apply.

No Consent: The seizure of a passenger vehicle in a routine traffic stop does not provide consent to the officer to search the passenger compartment, nor did Dave or Ted give such consent to Polly. Therefore, this exception will also not apply.

No Exception to the Warrant Requirement or Probable Cause Applies [To] The Cocaine: Because no exception to the warrant requirement or probable cause applies to the circumstances here, the search of the car and the discovery of the cocaine must be suppressed. Thus, Dave will likely succeed on this motion.

91

Question 2: Motion to Suppress Ted’s Statement or for a Separate Trial

State Action: Again, private actors are not bound to constitutional norms. Thus, the statement must have been obtained by a “state actor” for the suppression motion to be valid. Here, the statements by Ted were obtained by questioning by Polly, who as discussed above is a state actor. Therefore, this requirement is met.

Suppression of Statement After Unlawful Arrest -- No Standing to Bring: As discussed in Question 1, the arrest of Ted and Dave was the result of an improper search of Dave’s vehicle, because the probable cause to arrest Ted and Dave was based entirely upon the improperly seized cocaine. If probable cause to arrest is based solely on unconstitutionally obtained evidence, then the subsequent arrest is invalid and unlawful.

Any statements made by a suspect in custody following an unlawful arrest must be suppressed unless the state can show that the “taint” of the unlawful arrest has been purged. Case law is unclear whether Mirandizing a suspect unlawfully arrested is sufficient to “purge the taint” of the prior arrest, even if the suspect waives his Miranda rights following a properly administered warning. What is clear is that releasing the suspect would purge the taint (but that didn’t happen here).

However, regardless of the merits of this valid issue, Dave has no standing to bring a claim that Ted’s statement was improperly obtained as evidence of an unlawful arrest. This is because only the person who made such a statement can bring such a challenge. Thus, Dave would be wise to encourage Ted to bring this argument forward.

Co-Defendant Confession, Confrontation, and Self-Incrimination Rights -- Redact or Suppress: Because this is a criminal trial with co-defendants, special constitutional concerns arise when one defendant’s confession is being admitted against the other defendant. This is because of the intersection between the right of a defendant against self-incrimination (and the right to not take the stand) and the right of an accused to

92

“confront” the witnesses against him, meaning being able to put the witness under oath, cross-examine him, assess his demeanor, and physically be present for the process.

The Confrontation Clause only applies to “testimonial statements,” which case law clearly includes confessions to police officers within the definition. Here, Ted’s statement falls within this category, because his statement was made to Polly after waiving his Miranda rights. Therefore, the admission of the statement falls within the “testimonial” category of testimony.

Moreover, the testimony clearly implies that Dave is responsible for the contents of the bag, as Ted makes it clear that he--the only other passenger in the car--had nothing to do with the paper bag. This testimony will likely be used against Dave to show that he had true possession of the bag.

Under these facts, because Ted cannot be forced to take the stand and be confronted (because he can assert his Fifth Amendment right to not take the stand), the confession must be redacted as to not cast any negative light onto Dave, or be suppressed.

Conclusion on Suppression: Because it is unlikely that the statement can be redacted to not cast an accusatory light upon Dave, the court will likely grant its suppression.

Conclusion on Alternative -- Separate Trials: The Court may alternatively grant separate trials for Dave and Ted, and should do so in the interests of justice, since it appears under the facts that Dave and Ted will be asserting inconsistent defenses, and will likely attempt to implicate each other in the process.

This has the potential of prejudicing each defendant’s right to a fair trial, and confuse the issues to the jury, because the jury may be tempted to conclude that one defendant is “correct” and the other defendant is “wrong” in accusing the other of fault. This may violate the Fourteenth Amendment requirement that the state bear the burden of

93 proving the element of every crime charged, and, therefore, separate trials may be the only way to ensure that the state still bears this burden.

Under these circumstances, the court, in the interests of justice should grant the request for separate trial.

Question 3: Motion to Discharge Attorney

The Sixth Amendment Right to Counsel of Choice: The “root meaning” of the Sixth Amendment, per Supreme Court case law, is that the Sixth Amendment right to counsel also includes a Constitutional right to the counsel of one’s choice. This right, of course, does not apply to appointed counsel (which the Supreme Court has clarified), but only to retained counsel. Moreover, this right is not absolute. A criminal defendant cannot improperly delay criminal proceedings by abusing this right, constantly requesting permission to substitute counsel for no good reason.

Here, it is clear from the facts that Dave has retained counsel, and was not appointed counsel by the court. Therefore, Dave does have a Constitutional right to the counsel of his choice. However, it is also clear that the time frame in which Dave has requested a new lawyer is one week before trial.

Under these facts, the court must consider whether granting the request for substitution of counsel would be unfairly prejudicial to the other parties (both the co-defendant and the state), because it would likely have to grant time for the new counsel to become familiar with the details of the case.

Thus, under these facts, it is unlikely that the court would agree--at the eve of trial--to allow the defendant to exercise his Constitutional right to the counsel of his choice.

The Sixth Amendment Right to Go Pro Se: Note that the Sixth Amendment also guarantees the right of a defendant to represent himself (subject to competency

94 requirements and a knowing and intelligent waiver of the right to an attorney). Here, the Court could grant the discharge of the present attorney (but deny the substitution of a new attorney) if Dave would rather represent himself. However, the facts do not show such a desire, and therefore, the Court will likely not propose such an alternative.

The Sixth Amendment Right to Effective Counsel: The Sixth Amendment guarantees a defendant the right to effective assistance of counsel. The deficiency of counsel in representation, if it causes actual prejudice (a reasonable probability of a different outcome due to the deficiency), is a structural Constitutional error that is grounds for reversal of a conviction and retrial.

Here, the facts show that Dave was merely dissatisfied with his attorney’s performance. If Dave had alleged an actual conflict of interest (which would exist if the same attorney represented both Dave and Ted), and the court agreed with this claim of actual conflict, the court should allow Dave to discharge his present attorney and substitute a new attorney, or risk any conviction being reversed under the Sixth Amendment.

95

THE STATE BAR OF CALIFORNIA OFFICE OF ADMISSIONS 180 HOWARD STREET • SAN FRANCISCO CALIFORNIA 94105 1639 • (415) 538 - 2303 1149 SOUTH HILL STREET • LOS ANGELES CALIFORNIA 90015-2299 • (213) 765 – 1500

ESSAY QUESTIONS AND SELECTED ANSWERS JULY 2010 CALIFORNIA BAR EXAMINATION

This publication contains the six essay questions from the July 2010 California Bar Examination and two selected answers to each question.

The answers received good grades and were written by applicants who passed the examination. The answers were prepared by their authors, and were transcribed as submitted, except that minor corrections in spelling and punctuation were made for ease in reading. The answers are reproduced here with the consent of their authors.

Question Number Contents Page

1 Torts 3

2 Professional Responsibility 22

3 Evidence 34

4 Business Associations 54

5 Criminal Law and Procedure 64

6 Community Property 78

1

Question 5

Harriet was on her porch when Don walked up, pointed a gun at her, and said, “You’re coming with me.” Believing it was a toy gun, Harriet said, “Go on home,” and Don left.

While walking home, Don had to pass through a police checkpoint for contraband. Officer Otis patted down Don’s clothing, found the gun, confiscated it, and released Don. Later, Officer Otis checked the serial number and located the registered owner, who said the gun had been stolen from him.

A month later, Officer Otis arrested Don for possession of stolen property, i.e., the gun. During a booking search, another officer found cocaine in Don’s pocket.

Don was charged with possession of stolen property and possession of cocaine. He moved to suppress the gun and the cocaine, but the court denied the motion.

While in jail, Don drank some homemade wine. As a result, when he appeared in court with counsel, he was slurring his words. The court advised Don that if he waived his right to a trial, it would take his guilty plea and let him go on his way. Don agreed and pleaded guilty. Subsequently, he made a motion to withdraw his guilty plea, but the court denied the motion.

1. Did the court properly deny Don’s motion to suppress: a. the gun? Discuss. b. the cocaine? Discuss.

2. Did the court properly deny Don’s motion to withdraw his guilty plea? Discuss.

3. If Don were charged with attempted kidnapping against Harriet, could he properly be convicted? Discuss.

64

Answer A to Question 5

1) Whether the Court Properly Denied Don’s Motion to Suppress A) The Gun Officer Otis (O) discovered a gun on Don (D) while D was walking home and subsequently encountered a police checkpoint for contraband. Thus, whether the gun is admissible evidence depends on whether the checkpoint was constitutional. D will likely argue that the checkpoint violated his Fourth Amendment rights, which prohibits unreasonable searches and seizures.

The Checkpoint All Fourth Amendment violations must come from the hands of the government. This is easily satisfied because the checkpoint at which the gun was discovered was a police checkpoint. However, the general rule is that for a checkpoint to be outside the scope of Fourth Amendment protection, the checkpoint must be conducted in a nondiscriminatory manner, and must be for purposes other than the police investigation of criminal activity. In this case, the checkpoint was likely conducted in a nondiscriminatory manner. A nondiscriminatory checkpoint generally checks every person who passes through or some other equal rule, such as every third person that passes through.

However, D will likely argue that the checkpoint is invalid because it directly relates to the investigation of criminal activity. The United States Supreme Court has held that a constitutional checkpoint only occurs when the underlying purpose is not criminal investigation. Such examples include DUI checkpoints being motivated by the state interest of safety on public roads, and informational checkpoints, to investigate the occurrence of an accident that happened in the area recently. In this case, the police checkpoint is specifically looking for contraband, i.e., illegal materials. While O may argue that the checkpoint’s purpose of checking for contraband directly advances public safety, this argument will likely be rejected given the fact that it directly relates to criminal investigation. Thus, the checkpoint is unconstitutional.

65

Since D’s gun was discovered through an unconstitutional police checkpoint, the court improperly denied D’s motion to suppress the gun.

Terry Stop and Frisk O may attempt to argue that the gun is a valid seizure because it was performed pursuant to a Terry stop and frisk. A stop and frisk allows an officer to pat down a suspect when the officer has a reasonable suspicion that the suspect may be armed and dangerous. In this case, O will argue that he had a reasonable suspicion that D could be armed, thus giving O the ability to pat down D’s clothing, thus leading to a constitutional avenue towards discovery of the gun. However, this argument will likely fail because the Supreme Court has held that “reasonable suspicion” requires more than a “hunch,” but instead a set of articulated facts that give rise to the notion that criminal activity is afoot. In this case, O had no suspicion because he was merely checking people at the police contraband checkpoint. In other words, O had less than a hunch, and thus no reasonable suspicion that would give rise to a constitutional stop and frisk.

Thus, as discussed above, the court improperly denied D’s motion to suppress the gun.

B) The Cocaine At the checkpoint, O seized the gun from D. O subsequently checked the serial number and located the registered owner of the gun, who said that the gun had been stolen from him. One month later, O arrested D for possession of stolen property. During a booking search at the police station, another officer found cocaine in D’s pocket. Thus, the admissibility of the cocaine depends on whether the booking search was constitutional.

Booking Search As discussed above, the Fourth Amendment protects against unreasonable searches and seizures. The Supreme Court, however, has held that administrative searches, such as routine booking searches performed for safety and to ensure that suspects’ personal items are not lost, are not subject to the Fourth Amendment. Thus, the prosecution will likely argue that the cocaine was properly found and confiscated.

66

However, D will argue that the cocaine should be suppressed because the booking search was based on an arrest founded on probable cause from an illegal search, i.e., the checkpoint discussed above.

Fruit of the Poisonous Tree The fruit of the poisonous tree doctrine precludes the admission of evidence that was lawfully seized based on prior unconstitutional acts. As discussed above, D will argue that the gun which led to his arrest and subsequent booking search was unconstitutional, and therefore the cocaine is a fruit of the poisonous tree. In response, the prosecution will likely argue that the cocaine is admissible under the independent source and inevitable discovery doctrines.

The independent source doctrine makes evidence admissible because the police had an alternative, constitutional, avenue towards its discovery. This argument is likely to fail. The only avenue the police have to D’s cocaine is from a booking search based on an arrest founded on probable cause from an illegal search. There is no other source. While O may argue that his independent source is his research of the serial number and discussion with the registered owner, such an argument is likely to fail because O would not have performed those actions without the illegally confiscated gun. Thus the independent source doctrine does not apply.

The inevitable discovery doctrine makes evidence admissible because the police authorities would have eventually discovered the evidence through their investigation anyway. The argument is also likely to fail for the same reason that the independent source doctrine, discussed above, will fail: the only route towards the cocaine that O had was from a gun that was from the fruit of an illegal search.

Thus, the cocaine is the fruit of a poisonous tree, and should be suppressed unless the prosecution can show that the taint associated with the illegal search is attenuated.

67

Attenuation of Taint The attenuation of taint doctrine will admit improperly seized evidence if the police can show factors that have led to the attenuation of the taint. In this case, O will argue that, despite the fact that the gun was discovered at a police checkpoint, the probable cause for the arrest was for stolen property. Specifically, it was O’s investigation into the serial number of the gun and discussion with the true registered owner of the gun which led to the probable cause to arrest D for stolen property. Prior to this attenuation, the gun was merely the product of an illegal search, but now the gun is evidence in a claim of stolen property by the registered owner. Furthermore, O will argue that an entire month passed by, thus indicating that the illegal search was not the main motivating factor in D’s ultimate arrest for stolen property. A court would likely agree.

Thus, the court properly admitted the cocaine discovered in the booking search because, although the arrest was based on a gun discovered in an illegal search, there was a sufficient attenuation of the taint of that illegal search to support probable cause to [sic] for D’s arrest for stolen property.

2) Whether the Court Properly Denied Don’s Motion to Withdraw His Guilty Plea Whether the court denied D’s motion to withdraw his guilty plea depends on: (1) whether D’s initial guilty plea was knowing and voluntary, and (2) whether proper formalities were followed when D entered his guilty plea.

D’s Guilty Plea and Voluntary Intoxication The general rule is that a defendant’s plea of guilty must be knowing and voluntary. In this case, D drank homemade wine and as a result, he was slurring his words. This indicates that, even if counsel and the court advised him of the nature of his rights, it is likely that D lacked capacity to understand the material details associated with a guilty plea and subsequently D could not have made a knowing and voluntary guilty plea.

Formalities to Enter a Guilty Plea For a guilty plea to hold up under appellate review, at the time the defendant enters a guilty plea, the judge must inform the defendant: (1) the maximum possible sentence; (2) the mandatory minimum sentence; (3) that he has a right to a jury trial, and; (4) that

68 he has a right to plead not guilty. All of this information and dialogue must be on the record.

In this case, none of these formalities were followed. Instead, the court merely advised D that if he waived his right to a trial, the court would take his guilty plea and let him go on his way. Thus, although the court somewhat advised D regarding his right to a jury trial, it is clear that the court failed to inform D of the maximum possible sentence, the mandatory minimum, and that he has the right to plead not guilty.

Thus, the court improperly denied D’s motion to withdraw his guilty plea because: (1) it is highly unlikely that D lacked capacity through voluntary intoxication to making a knowing and voluntary guilty plea, and (2) the court failed to follow constitutionally required formalities for accepting and entering a guilty plea.

3) Whether Don May Properly Be Convicted of the Attempted Kidnapping of Harriet Whether D may be convicted of attempting to kidnap Harriet depends on whether D committed the criminal act (“actus reus”) simultaneously with the requisite mental intent (“mens rea”).

Mens Rea Since the jurisdiction is not identified, this analysis presumes that the common law is applied. Under the common law, a crime may either be a general intent crime or a specific intent crime. While there is no clear-cut rule delineating the two, suffice to say that a general intent crime requires a lower mental threshold, while a specific intent crime requires a higher threshold of mental acknowledgment, such as purposefully engaging in the crime or knowing the likely outcome of the defendant’s acts.

In this case, kidnapping is a general intent crime. However, if D were charged with attempted kidnapping, it would be a specific intent crime. The inchoate crime of attempt requires that the defendant have the specific intent to commit the crime. Thus, to be properly convicted a jury must find that D specifically intended to kidnap Harriet (H). It is likely that D intended to kidnap Harriet, as he pointed a real gun at her and said, “You’re coming with me.” While one act (pointing the gun) or the other (saying “You’re

69 coming with me”) alone may be insufficient to establish that D had the mens rea to effectuate a kidnapping, both acts together make it highly likely that D intended to kidnap H. However, D will point out that after H told him to go home, D obliged and left. Thus, it is unclear whether D had the requisite mental state to commit an attempted kidnapping.

Thus, because it is unclear whether D had the requisite mental state to commit an attempted kidnapping, required under the inchoate crime of attempt, D may not have the requisite mens rea to [be] convicted of attempted kidnapping. However, specific intent may be indicated by the actions that D took to effectuate the kidnapping, discussed below.

Actus Reus While the normal crime of kidnapping requires that D falsely imprison Harriet (H) and either move her location or conceal her presence from others for an extended period of time, since D is hypothetically being charged with attempted kidnapping, D need not go that far. Under the common law, to be convicted of an attempted crime the defendant must be in “dangerous proximity” of committing the crime, while in other jurisdictions the defendant need only take a “substantial step” towards the commission of the crime.

In this case, it is likely that D’s actions satisfy both the “dangerous proximity” and “substantial step” doctrines. Walking up to someone, pointing a gun at them, and saying “You’re coming with me” is within the dangerous proximity of committing the crime, as the defendant is face-to-face with the intended kidnapping victim coupled with the fact of oral communication threatening or coercing the intended victim. Likewise, the same actions are obviously a substantial step towards the commission of a kidnapping, as D has taken the time to approach H at her house, pull a gun on her, and coerce her to come with D, which would have the result of completing the kidnapping crime, i.e., by moving the victim.

Furthermore, these acts are extremely probative as to D’s mental state, as it is highly unlikely that someone who not only took a substantial step towards attempting a

70 kidnapping, but is also in the dangerous proximity of doing so, would have the requisite mental state to be convicted of attempt.

Thus, if D were charged with attempted kidnapping against H, D could properly be convicted for the reasons discussed above.

71

Answer B to Question 5

1a. Don’s Motion to Suppress the Gun Don’s motion to suppress will be based on the argument that the confiscation of his gun was an impermissible search-and-seizure in violation of his Fourth Amendment rights.

Governmental Conduct For Fourth Amendment rights to attach, the search-and-seizure must have been done by government actors. In this case, Otis stopped Don at a checkpoint, and was presumably on duty. Note that even if Otis had stopped and searched Don while he was off duty that would still be sufficient for governmental conduct.

Reasonable Expectation of Privacy In addition, the Fourth Amendment also requires that the individual have a reasonable expectation of privacy in the items or place searched. Here, the gun was located in Don’s clothing and on his person. The fact that the police had to pat down Don to find it alone evidences that he had a reasonable expectation of privacy. The fact the gun was stolen and that Don was not the proper owner is not sufficient to demonstrate that he lacked a reasonable expectation of privacy.

Warrant Generally, 4th Amendment search requires a valid warrant, where there must be particularity and probable cause. Here, there was no warrant. Therefore, Otis cannot have been in good faith relying on the warrant even if it was defective, so an exception to the warrant requirement must apply.

Checkpoint Don will first argue that the confiscation of the gun was invalid because the checkpoint was not authorized by law. A valid checkpoint requires a neutral reason for stopping or selecting people for the checkpoint. For example, if the officers stop every third person that passes through the checkpoint, that would be a sufficiently neutral basis for the checkpoint. In this case, there is no specific evidence of an improper police purpose in stopping Don and the officer’s actions are thus presumptively going to be valid. 72

A valid checkpoint also must address some legitimate government concern or interest. Again as an example, a checkpoint to stop drivers and watch for those that are driving under the influence is permissible because there is a valid interest in keeping dangerous drunk drivers off the road. Here, the checkpoint was to stop pedestrians carrying contraband. Don will argue that pedestrians, even if they are intoxicated, do not present inherently dangerous risks similar to that posed by drunk drivers.

In addition, Don will argue that while it may be permissible to stop pedestrians for specific reasons, there must be some sort of articulable purpose. Here, the officers are simply looking for contraband, which could be evidence of any offense. Officers are not allowed to stop every passerby without having any reason for the stop. Therefore, the checkpoint here is probably not valid absent some more articulable purpose.

Terry Stop and Frisk A secondary justification to stop Don would be on the basis of a Terry stop. A Terry stop requires reasonable suspicion that the individual stopped either be dangerous or have some improper purpose. If the officer has reasonable suspicion necessary for the stop, if the officer also has reasonable suspicion that the suspect is dangerous, then the officer may pat down or frisk the individual to look for weapons. If during the patdown the officer by “plain feel” thinks an item is either a weapon or drugs, then the officer is allowed to seize the item.

In this case, there is no evidence that Officer Otis had reasonable suspicion to stop Don. Don was simply “walking home” and while [he] had a weapon, the weapon was in his clothing and there is no indication Otis saw the gun, saw a bulge in Don’s clothing that could indicate he was armed, or some other reason that Don was acting suspiciously. Otis may point to the totality of the evidence here, that Don was leaving Harriet’s after what might have been an attempted kidnapping, but even given this fact there is no indication from the way that Don was walking home that he had just tried to kidnap someone.

Therefore, the seizure of Don’s gun was probably not valid under either the justification of a checkpoint or a Terry Stop and Frisk.

73

1b. Don’s Motion to Suppress the Cocaine

Fourth Amendment Attachment The search of Don that found the cocaine was done by a government official after Don had been arrested and Don had a reasonable expectation of privacy of items contained in his pocket. Therefore, 4th Amendment protections attach.

Booking Search Don will first argue that the booking search was impermissible. A booking search is valid as long as it is conducted as a result of and in accordance with the regular practice of the police office. If so, the search does not require probable cause, nor does it require reasonable suspicion. In this case, the cocaine was found during a booking search of Don, in Don’s pocket. Because there is no evidence of anything other than the fact that this was a routine booking search, the search-and-seizure was proper.

Fruit of the Poisonous Tree Even though the booking search itself was valid, Don will argue that it is impermissible because the booking search only arose as the result of the impermissible search-and- seizure that led to the gun. The booking search was conducted after Officer Otis arrested Don for possession of stolen property in the gun found at the checkpoint search.

Evidence that is discovered through impermissibly tainted evidence is also invalid. In this case, because the gun was improperly seized, the prosecution will have to show some alternative means of acquiring the evidence. If the prosecution can show that they had an independent source for the evidence, would have inevitably discovered it anyway, or that the secondary evidence arose from intervening acts of free will by the defendant, then the evidence is valid anyway.

Independent Source If the police can derive the evidence from an independent source, that will be sufficient to cleanse the taint of the impermissible evidence. In this case, the officers found the cocaine as a result of the booking search, which only arose directly from the seizure of

74

Don’s gun. After the officers seized the gun, they checked the serial numbers and located the registered owner, who informed the officers that the gun had been stolen. The officers then followed up on the owner’s statements and arrested Don for possession. There was thus only one source for the evidence that led to the cocaine, and that source was impermissibly tainted.

Inevitable Discovery If the police can show that they would have inevitably discovered the cocaine that would also be sufficient to cleanse the taint of the seizure of the gun. Again, there is no evidence here that the officers would have discovered the cocaine without the information obtained from the gun. Without the gun, the officers probably never would have discovered the cocaine, and thus the inevitable discovery exception is inapplicable.

Intervening Acts of Free Will by Defendant Finally, if the officers show that there had been some intervening act of free will by Don that led to the discovery of the cocaine that could lead to its admissibility as well. The prosecution will point out the fact that the police did not arrest Don for one month after the initial search, and they will thus argue that time was sufficient to clear the taint. This is probably the prosecution’s best argument; however, it still fails to show any direct relationship to the evidence from anything other than the illegal search. Therefore, the cocaine will probably have to be excluded as well.

2. Don’s Motion to Withdraw His Guilty Plea Before a judge can accept the defendant’s guilty plea, the judge must inform the defendant that the defendant has a right to plead not guilty and demand a trial. The judge must also inform the defendant of any mandatory minimums that will result from the guilty plea as well as the possible maximum penalty. The judge should also inform the defendant of his ability to secure an attorney or alternatively proceed per se. Finally, the judge must inform the defendant that all of this information and the defendant’s plea itself must be on the record.

75

In this case, the judge did not do any of this. The court advised “Don that if he waived his right to a trial, it would take his guilty plea and let him go on his way.” Don then pled guilty. The judge did not inform Don of the possible results of pleading guilty, nor did the judge tell him that his plea would be recorded. Arguably, the judge satisfactorily met the requirement of informing Don of his right to trial by telling him about his ability to waive it, but the judge still should have expressly stated his right, instead of simply discussing his ability to waive trial.

Furthermore, Don will point to the fact that the judge should have been aware of Don’s lack of capacity when making the decision. As a result of drinking wine in jail, Don “was slurring his words” when he went into court. The judge at this point should have been even more careful than normal to comply with the various requirements in taking a defendant’s guilty plea. However, the judge failed to meet these requirements. Therefore, the court improperly denied Don’s motion to withdraw his guilty plea.

3. Attempted Kidnapping Kidnapping requires refraining a person’s ability to move or leave along with either concealment or movement of the person. Here, there was no actual kidnapping because even if Harriet’s ability to leave was briefly restrained by Don pointing the gun at her, because Harriet didn’t believe the gun was real and Don left, there was no concealment or movement.

Attempted kidnapping requires the specific intent to kidnap as well as a substantial step towards completion of the act. In this case, while there is no direct evidence of Don’s state of mind, his actions demonstrate that he probably had the requisite specific intent to kidnap. First, as evidenced by his later arrest, Don had brought a real gun with him, pointed it at Harriet and made a demand of her. This is all relevant to show Don’s state of mind, that he did intend the outcome he stated that she come with him. Furthermore, had Harriet believed that it was a real gun she probably would have gone with him, sufficient for kidnapping. Therefore, while more evidence would be helpful, there is a sufficient amount of evidence to conclude that Don had the requisite intent.

76

In addition to the specific intent to kidnap, Don must also have completed a substantial step towards completion of the kidnapping. This test is not the most restrictive. If Don had simply brought the gun to Harriet’s home and at the point was arrested, the fact that he brought a gun with him that far would probably be a substantial step. Here, however, Don not only brought the gun, he pointed it at Harriet and made a demand. There was not much more left for Don to do. Don may point to the fact that the act itself was not completed, or the fact that Harriet was not scared, but neither of these outcomes is required for an attempt. Therefore, Don would be convicted of attempted kidnapping.

The minority rule would require not that Don completed a substantial step towards kidnapping but rather that Don was dangerously close to succeeding in kidnapping. Here, the acts of drawing the gun and demanding that Harriet come with him were probably sufficient to be dangerously close to success. Don will again raise the fact that Harriet did not come with him, and will have a better argument by pointing to the fact that Harriet was not in fact even scared of him, but again this argument goes to the result of the actual crime of kidnapping. Don had done everything required to complete the act besides Harriet acquiescing to his demand. Therefore, because Don had done everything he could besides trying to further convince Harriet the gun was real, he would probably be convicted even under the minority rule.

77

ESSAY QUESTIONS AND SELECTED ANSWERS JULY 2011 CALIFORNIA BAR EXAMINATION

This publication contains the six essay questions from the July 2011 California Bar Examination and two answers to each question that were written by actual applicants who passed the examination after one read.

The selected answers were assigned good grades and were transcribed for publication as submitted, except that minor corrections in spelling and punctuation were made for ease in reading. The answers are reproduced here with the consent of their authors.

Question Number Contents Page

1 Criminal Law and Procedure 4

2 Civil Procedure 18

3 Contracts 32

4 Professional Responsibility 46

5 Real Property 57

6 Community Property 70

2 Question 1

Vicky operates a successful retail computer sales business out of the garage of her house. Vicky told Dan that she intended to go on vacation some days later. Dan subsequently informed Eric of Vicky’s intended vacation and of his plan to take all of her computers while she was away. Eric told Dan that he wanted nothing to do with taking the computers, but that Dan could borrow his pickup truck if Dan needed it to carry the computers away.

While Vicky was scheduled to be away on vacation, Dan borrowed Eric’s pickup truck. Late that night, Dan drove the truck over to Vicky’s house. When he arrived, he went into the garage by pushing a partially open side door all the way open. Vicky, who had returned home early from her vacation, was awakened by noise in her garage, opened the door connecting the garage to the house, and stepped into the garage. When she saw Dan loading computers into the back of the truck, she stepped between Dan and the truck and yelled, “Stop, thief!”

Dan pushed Vicky out of the way, ran to the truck, and drove off. He immediately went to Fred’s house where he told Fred what had happened. In exchange for two of the computers, Fred allowed Dan to hide the truck behind Fred’s house.

What crimes, if any, have Dan, Eric, and/or Fred committed? Discuss.

4

Answer A to Question 1 I. Dan's Crimes

By plotting to break into Vicky's home to steal her computers and then actually doing so, Dan committed the crimes of burglary, , robbery, and battery. He may have also conspired to commit burglary and/or larceny with Eric.

Burglary

At common law, burglary was defined as the unlawful breaking and entering of the dwelling house of another at night with the intent to commit a felony therein. Most modern jurisdictions have amended the elements to include burglary of any structure and have not limited it to nighttime .

Here, Dan committed burglary when he entered Vicky's home to steal the computers.

Breaking and Entering

Burglary requires that the burglar break and enter into the structure. "Breaking" constitutes any form of forcible entry, including pushing open a partially open door. "Entry" requires physical entry by any part of the burglar's body or a tool under his control.

Here, Dan pushed a partially open side door to V's garage fully open in order to gain entry. This is evidence of breaking. Further, Dan entered the garage, which is a part of Vicky's residence. Thus, the elements of breaking and entering are satisfied.

Structure of Another

Dan entered into Vicky's garage, both the location of her retail sales business and part of her home (her dwelling place). This is sufficient to constitute a protected structure for purposes of burglary, which belonged to another (Vicky). Therefore, this element is met.

With the Intent to Commit a Felony Therein

5

Burglary requires the intent to commit a felony (or a misdemeanor in some jurisdictions) inside the structure at the time of the breaking and entering.

In this case, Dan had the intent to commit larceny of Vicky's computers when he entered her garage. He had previously expressed this desire to Eric, and nothing in the facts suggests he changed is mind prior to entering. In fact, his actions of actually taking the computers demonstrates that the intent was present.

Therefore, Dan committed burglary.

Larceny

Larceny at common law was the trespassory taking and carrying away of the personal property of another with the intent to permanently deprive the victim of the property.

Trespassory

Trespass is the unpriviliged, nonconsensual invasion of another's protected space.

Here, Dan did not have the consent of Vicky to enter the garage at night and therefore his decision to do so was a trespass. While Dan might argue it was not trespassing because Vicky opened her business up to the public and her business was located in the garage, this argument will fail because he clearly did not have implied or actual authorization to force his way into the garage at night when Vicky was not operating her business and was in fact supposed to be on vacation.

Asportation

Asportation is the taking and carrying away of another's property. For larceny purposes, even slight movement of the property is sufficient.

In this case, Dan took computers from Vicky's garage, loaded them into his truck and drove off with them. Thus, he moved the computers and this element is satisfied.

Personal Property of Another

6

The computers were the tangible, moveable personal property of Vicky and her business. The computers did not belong to Dan and he had no claim of right to the computers. Therefore, this element is satisfied.

Intent to Permanently Deprive

At the time of the taking, the defendant in a larceny case must have the intent to permanently deprive the owner of the property.

Here, Dan had the intent to permanently deprive because he planned to steal the computers and presumably sell them for value. Nothing in the facts indicates a contrary intent on Dan's part, so this element is satisfied.

Therefore, Dan also committed larceny.

Robbery

Common-law robbery requires that the defendant take and carry away the personal property of another from their person or presence by force or threat of force, with the intent to permanently deprive.

The requirements that Dan took and carried away the computers belonging to Vicky with the intent to permanently deprive have been described above. The remaining elements follow.

Person or Presence

Robbery requires that the items be taken from the victim's person or presence, which has been broadly defined to include anything the victim is holding or, indoors, items from the same room that the victim was in at the time of the taking.

Here, Vicky was present in the garage when Dan loaded some of her computers into the truck. In fact, she stepped between Dan and the truck as he was attempting to flee with the computers, so it suggests that she was immediately present when her property was taken. Therefore, this element is likely satisfied because the computers were taken from within a very close proximity to Vicky. As such, they were taken from her immediate presence.

7

Force or Threat of Force

A robber must use physical force or threaten to use physical force to commit robbery.

Here, as he was attempting to flee, Dan physically pushed Vicky out of the way. Shoving another person is physical force, which Dan used to accomplish and complete his taking of Vicky's computers.

Dan will argue that he did not accomplish the taking by force because he already had the computers in his possession before Vicky confronted him. He will defend by saying that the force was only used to effectuate his escape, and not the robbery itself. However, because the robbery would not have succeeded but for the physical force to the victim, it's likely to satisfy the requirement of forcible robbery.

For those reasons, Dan also robbed Vicky.

Battery

Battery is the intentional unlawful application of physical force to another person. Battery is a general intent crime, meaning there is no requirement that the defendant intend to cause injury to the victim. He must only intend to commit the physical action that constitutes the force.

Here, Dan physically shoved Vicky out of the way as he was escaping. He intended to complete the shoving action because it allowed him to get Vicky out of his way and proceed to the truck. Therefore, Dan committed a battery.

Conspiracy to Commit Burglary/Larceny

Conspiracy is an that required at common law an agreement between two or more people to accomplish the same unlawful objective with the intent to complete that objective. Many jurisdictions require proof of an "overt act" to establish conspiracy. In a majority of states, only bilateral conspiracies are permissible, but a minority of states recognize the idea of a "unilateral conspiracy," where the defendant

8

believes he is conspiring with another "guilty mind" who in fact shares a different objective.

The prosecution may attempt to argue here that Dan conspired with Eric to rob Vicky because he discussed his plans with Eric in advance and Eric loaned Dan his truck for purposes of the robbery. However, as will be addressed below, it is not clear that Eric had the intent for the robbery to be completed. If Eric lacked the requisite intent to accomplish the robbery, then Dan can only be convicted of conspiracy in a jurisdiction that recognizes unilateral conspiracy.

II. Eric's Crimes

Conspiracy to Commit Burglary/Larceny

The issue is whether Eric had the intent to enter into an agreement with Dan for an illegal purpose (the burglary/larceny) and if Eric intended for the illegal object to transpire as planned. Here, the facts suggest that Eric lacked that intent, so he is likely not guilty of conspiracy.

The prosecution will argue that Eric's decision to loan his truck to Dan knowing that Dan intended to use it to burglarize Vicky's business is evidence that Eric conspired to commit that crime. However, Eric specifically told Dan that he wanted "nothing to do with taking the computers." Although the prudence of nonetheless letting Dan use his truck to commit the robbery is questionable, the facts do not prove that Eric intended to participate in the burglary or that he shared Dan's goal for the burglary to succeed. He may have been indifferent to the being committed or even favorable to the idea, but this is not persuasive evidence that he intended for Dan to succeed in the burglary. Since the prosecution will have the burden to show intent beyond a reasonable doubt, this is unlikely to be a persuasive argument.

Therefore, it's likely that neither Dan nor Eric could be convicted of conspiracy.

Accomplice Liability

An accomplice is someone who aids, abets, counsels or encourages the principal to commit a crime with the intent that the principal succeed. A majority of jurisdictions

9

hold liable for all reasonably foreseeable crimes that the principal committed.

Burglary and Larceny

Here, Eric was likely an accomplice to the burglary and larceny committed by Dan, and he should be convicted of those offenses. By offering to let Dan use his truck to carry away the computers after he stole them, Eric aided Dan by giving him a getaway vehicle. Without Eric's participation in loaning Dan his truck, it's not clear that Dan would have been able to commit the crimes. Therefore, if it was foreseeable that Dan would commit burglary and larceny, Eric is liable therefor.

In this case, Eric knew that Dan intended to enter Vicky's business and take her computers. Therefore, he was personally informed of Dan's intent to commit larceny and burglary. In fact, he specifically told Dan that he could use Eric's truck "if Dan needed it to carry the computers away." Therefore, Dan is liable as an accomplice to burglary and larceny.

Robbery

Eric will argue he is not an accomplice to the robbery of Vicky because it was unforeseeable that Vicky would be home and therefore that Dan would take anything from her person or presence. He will claim that he thought Vicky was on vacation, and that therefore, the most that Dan could be guilty of is burglary and/or larceny.

On balance, however, this argument is likely to fail. Eric had no personal knowledge of Vicky's travel plans, and by agreeing to lend Dan his truck for the purposes of escaping with Vicky's computers, he assumed the risk that Dan might have erred in determining Vicky's travel plans. Further, because the business was in Vicky's garage and therefore on her property, it would not be unforeseeable that someone might be either on Vicky's property for business purposes or that someone else besides Vicky was living there. As such, the presence of another person was reasonably foreseeable, and so was the robbery of the computers from that person's presence.

Eric is therefore guilty of robbery as an accomplice.

10

Battery

Similarly, Eric will argue that it was not reasonably foreseeable that Dan would commit battery against Vicky because he didn't even know that Vicky would be present. For the reasons discussed above, this argument will likely fail. Committing a always carries with it inherent risks that someone will be present, and breaking into a business carries similar concerns. It was foreseeable that Vicky or another person might be there during the burglary, and therefore, that Dan might use force against them in order to effectuate his escape.

As such, Eric is guilty as an accomplice to battery.

III. Fred's Crimes

Accessory After the Fact

Most jurisdictions will label an individual who aids, abets, counsels or encourages a criminal in avoiding apprehension to be an "accessory after the fact" if they did not play any role in the crimes before they happened. Such a defendant is an accomplice, but is generally only punished for his own behavior in obstructing justice rather than the crimes of the principal.

Here, Fred knew that the computers Dan brought to his home were stolen from Vicky by Dan. Nonetheless, in exchange for two of them, he agreed to let Dan hide his truck on Fred's property. This action aided Dan in covering up the crime and aiding detection. Hiding the getaway vehicle that Vicky had seen Dan driving away increased the chances that Dan would get away with the theft of her property, and therefore Fred acted as an accessory after the fact.

Receipt of Stolen Property

If the jurisdiction in this case recognizes knowing receipt of stolen property as a criminal offense, Fred is likely guilty of that crime as well.

Dan specifically informed Fred that the computers were stolen, but Fred agreed to take them in exchange for hiding Dan's truck. Therefore, the scienter requirement is

11

met here because Fred had firsthand knowledge of the computers' stolen status but agreed to take them into his possession.

Answer B to Question 1 Dan's criminal liability:

Burglary:

Burglary is the breaking and entering at nighttime into the dwelling house of another with the intent to commit a felony therein.

Breaking and Entering:

A person must physically enter the dwelling house of another to commit a burglary. Here, Dan entered into the garage of Vicky's house by pushing a partially open side door all the way open. Although he did not literally break anything to enter into the garage because the door was already open, this element is still met. Only the slightest movement is required to "break" into the house. The door need not be locked either. Thus, by pushing the partially opened door to the garage open and subsequently entering the garage, Dan committed a breaking and entering.

At nighttime:

Although modern statutes have eliminated the requirement that a burglary be committed at night, the common law crime of burglary required that the burglary happen at night. Here, the facts indicate that Dan drove over to Vicky's house at nighttime. Thus, the common law element and any modern statutory elements are met.

Dwelling house of another:

The common law definition of burglary required that the breaking and entering be of the dwelling house of another, that is, where the person lived and slept. Modern statutes have expanded this element to include any structure such as an office building. Here, Dan broke into the garage of Vicky's house. Vicky did not sleep in her garage, but she did conduct her computer business out of her garage and frequently spent time in there. Additionally, the garage was connected to the house by the door that Vicky entered when she heard the noise. Thus, the garage is part of Vicky's dwelling house, and this

12

element is met under the common law definition of burglary. The element is also met under a modern statutory definition because a garage would be considered a structure.

Intent to commit a felony therein:

A person must have an intent to commit a felony inside the dwelling house at the time that they committed the breaking and entering. Here, when Dan learned that Vicky was going away on vacation, he informed Eric that he planned to take all of her computers. Thus, Dan intended to commit larceny, analyzed below, once he broke into Vicky's house. He had this intent at the time he pushed the partially open side door. Thus, Dan had the requisite intent to commit a felony once inside the garage, and his intent was simultaneous with his breaking and entering.

Because Dan broke and entered into Vicky's garage, at nighttime, with the intent to commit a larceny, he has committed burglary.

Larceny:

Larceny is the trespassory taking and carrying away of the personal property of another with the intent to permanently deprive.

Trespassory taking and carrying away:

A person must take the personal property from the possession of another and move the property, if only the slightest bit. Here, Dan loaded Vicky's computers into the back of the truck. The computers were in Vicky's possession because they were stored in her garage as part of her retail computer sales business. Thus, Dan has met the element of a trespassory taking and carry away

Personal Property of another:

Here, the computers belonged to Vicky as she ran a retail computer business out of her garage. Thus, this element is met.

Intent to Permanent Deprive:

13

A person must intend to permanently deprive the victim of the possession of the personal property or act knowing that there actions will result in a substantial risk of loss. Dan intent to take all of her computers, which he told Eric. Although the facts do not indicate what he was going to do with the computers once he took them, it is unlikely that he was going to return them to Vicky, especially after he pushed her out of the way and drove off with them. Thus, Dan acted with the intent to permanently deprive Vicky of the computers. Because all the element for larceny are met, Dan committed larceny when he took Vicky's computers.

Robbery:

Robbery is the trespassory taking and carrying away of the personal property of another by the use or threat of force from the person of another. Here, Dan took the computers from Vicky's garage and loaded them into his truck meeting the requirement of a trespassory taking and carrying away. The computers where Vicky's personal property, which she stored in her garage. Although Dan though Vicky was away when he entered the garage, Vicky heard him and stepping into the garage as Dan was loading the computers into the back of the truck. She stepped in between Dan and the truck, at which point Dan pushed her. Although the computers were not on Vicky's person, the computers were in the immediate area. When she yelled at Dan, he pushed her by using force. Therefore, Dan used force to take the computers from the area in Vicky's immediate control. Because of the use of force when he took Vicky's computer, he has committed robbery as well.

Battery:

Battery is the unlawful application of force on the person of another, committed with the intent to cause the application of force to another. Here, Dan pushed Vicky out of the way when she stepped in between him and the truck. This was the unlawful application of force on Vicky. He acted with the intent to push Vicky out of the way because he was trying to move her to escape. Thus, Dan committed a battery as well.

Eric's Criminal Liability:

Conspiracy:

14

A conspiracy is the agreement of two or more person for an unlawful objective, with the intent that the unlawful objective be obtained. Additionally, statutes now include that an overt act be committed in furtherance of the conspiracy. Here, Dan told Eric of his plan to take all of Vicky's computers while she was away on vacation. Eric told Dan that he wanted nothing to do with the theft although he let Dan borrow his truck knowing Dan would use the truck to take the computers away. Eric did not agree with Dan to commit the burglary of Vicky's home. He did not have the same unlawful as Dan. Although he handed Dan his keys, which would qualify as an overt act, he did not have the intent to burglarize Vicky's home and steal her computers. Thus, he did not enter an agreement with Dan for the unlawful purpose of stealing from Vicky. Eric is not liable for conspiracy.

Accomplice Liability:

An accomplice to a crime aids, encourages, counsels, or abets a person committing the crime, with the intent that the person commit the target crime. Here, Eric gave Dan his keys to his pickup truck so that Dan could use the truck to move the computers. This was aid to the principal, Dan, who actually committed the burglary because Dan was able to move the computers once he could use Eric's truck. Although Eric wanted nothing to do with Dan taking the computer away, he told Dan that he could borrow his truck if he needed it to carry the computers away. Thus, although Eric did not want to actually take part in the burglary, he acted knowing that burglary would take place. He knew that Dan would use the truck to burglarize Vicky's house. Eric had the requisite intent for accomplice liability. Because he both aided Dan in committing the crime against Vicky, and acted with the intent to aid Dan, Eric is liable as an accomplice.

Vicarious Liability for the Target Crime:

An accomplice is liable for the crimes committed by the principal if the principal's crimes were foreseeable. It was completely foreseeable that once Eric gave Dan the keys to his car, Dan would steal all of Vicky's computers and Dan would use Eric's truck to move them. Additionally, it was foreseeable that Vicky might be home even though she told Dan that she would be on vacation; it is possible that her vacation plans had to be cancelled, as it turned out. If Vicky or anyone else was in the house, it was foreseeable

15

that Dan would use some measure of force to take the computers. Thus, Eric is liable for Dan's crimes of burglary, larceny, robbery and battery because all of these crimes were foreseeable once Eric gave Dan his keys to his truck knowing Dan would try and steal the computers.

Fred's Criminal Liability:

Accessory after the fact:

Under the common law, accomplices were liable as accomplices in the first degree or in the second degree based on how they aided the principal and when their aid occurred. Modernly, a person who aids a felon in his escape is liable as an accessory after the fact. This is a separate crime, and an accessory is not liable for the principal's target crime. Here, Dan immediately went to Fred's house after he drove off from Vicky's house. He immediately told Fred what he had done. Thus, Fred knew that Dan was a felon and that he was trying to escape after he stole Vicky's computers. He aided Dan because he allowed Dan to hide the truck behind Fred's house. This would make it harder for the police to spot that truck that Vicky would report, and thus help Dan in his escape. Fred is liable as an accessory after the fact. Unlike Eric who acted as an accomplice, Fred's liability as an accessory does not mean that he is also liable for the separate crimes that Dan committed.

Receipt of Stolen Property: Receipt of stolen property requires that the person receive, buy, or accept property knowing that the property was stolen. Here, Dan immediately told Fred what he had done once he arrived at Fred's house. Fred was aware that the computers belonged to Vicky, and that Dan had just unlawfully taken them from Vicky's garage. When Fred accepted two of the stolen computers in exchange for allowing Dan to hide his truck behind Fred's house, he accepted the property knowing that it was stolen from Vicky. Thus, Fred is criminally liable for the crime of receipt of stolen property.

16

ESSAY QUESTIONS AND SELECTED ANSWERS JULY 2012 CALIFORNIA BAR EXAMINATION

This publication contains the six essay questions from the July 2012 California Bar Examination and two answers to each question that were written by actual applicants who passed the examination after one read.

The selected answers were assigned good grades and were transcribed for publication as submitted, except that minor corrections in spelling and punctuation were made for ease in reading. The answers are reproduced here with the consent of their authors.

Question Number Contents Page

1 Civil Procedure 4

2 Community Property/Professional Responsibility 22

3 Evidence 40

4 Contracts 58

5 Wills and Succession 71

6 Criminal Law and Procedure 85

2

Question 6

Dan worked at a church. One day a woman came to the church, told Dan she wanted to donate some property to the church, and handed him an old book and a handgun.

Dan had originally intended to deliver both the book and the gun to the church’s administrators, but he changed his mind and delivered only the book. He put the gun on the front seat of his car.

The next day, as he was driving, Dan was stopped by a police officer at a sobriety checkpoint at which officers stopped all cars and asked their drivers to exit briefly before going on their way. The police officer explained the procedure and asked, “Would you please exit the vehicle?”

Believing he had no choice, Dan said, “Okay.”

After Dan got out of his car, the police officer observed the gun on the front seat and asked Dan if he was the owner. Dan answered, “No. I stole the gun. But I was planning to give it back.”

Dan is charged with theft and moves to suppress the gun and his statement to the police officer under the Fourth Amendment to the United States Constitution and Miranda v. Arizona.

1. Is Dan likely to prevail on his motion? Discuss.

2. If Dan does not prevail on his motion, is he likely to be convicted at trial? Discuss.

85

ANSWER A TO QUESTION 6

1. Is Dan ("D") likely to prevail on his motion? A. On Fourth Amendment Grounds. The Fourth Amendment protects the citizenry from unreasonable searches and seizures by the government. Thus violations require government action. They also require that the search or seizure be unreasonable, something that may be an issue for D. A search is a violation of a reasonable expectation of property; a seizure is an instance in which a person does not feel "free to leave" based on governmental presence. Generally, for a search to be reasonable, there must be a warrant. A warrant is granted by a neutral judge and must be based on articulable facts shown in an affidavit and must be reasonable and particular in terms of scope and time. In this case, there was no warrant to search D's car or to seize D. Thus, the search and seizure is presumptively unreasonable, subject to certain exceptions. One important exception is the checkpoint search; another such exception is consent. As an initial matter, a person must have standing to challenge the search. Because Dan was driving his own car, he will have standing. i) The Checkpoint Search: Warrantless, even suspicionless, road checkpoints have been upheld by the Supreme Court under certain circumstances. First, the search must be supported by the justification of highway safety - including prevention of DUI, etc. Second, the checkpoints must be administered in such a way that officer discretion is very limited. This means that an officer must go through a protocol driven method of stopping the cars - i.e., either every car, or one of every ten cars, etc. The officer may not stop whatever car he subjectively thinks looks criminal. Third, the search must be reasonable in scope - it must not exceed the degree necessary to check for whatever the search is aimed at.

Here, it does appear that the checkpoint search is aimed at a valid justification - a sobriety checkpoint. This has been expressly held as constitutional by the Supreme Court. However, there are some other issues. For one, all cars are being stopped. While this is not presumptively unreasonable, it will be an issue, as it basically allows a

86

policeman to stop and seize every single person driving down the expressway. Secondly, the police required D to step out of his car. Under Supreme Court precedent, police only have been allowed to stop people. If sobriety or another criminal violation seem likely, then the people can be asked to exit their car. Because of the stopping of every car, and the demand that the drivers exit the car, this may be found to be an unreasonably long stop than what is necessary to meet the highway safety justification.

Conclusion: There is a chance that this checkpoint too far exceeds permissible protocol based on Supreme Court precedent. However, it is a close call. I will consider this to be a reasonable and permissible warrantless search, though the court may be convinced otherwise. ii) Consent to Search: A person may validly waive his right to be free from unreasonable search and seizure by giving consent. Because it is likely that the stop and seizure was permissible up until the time that D was removed from his car, his consent to get out of the car would completely remove any potential objection to the search and seizure. The question will be whether the consent was freely and voluntarily given. Courts have found that when police attempt to search a person's house on the basis of consent, they do not have to tell that person that he or she has the right to refuse consent. This does not remove the "voluntary" aspect of consent. Here, Dan subjectively thought that he had no choice, but he still consented to getting out of the car. Assuming that the court would apply the consent rule used in home searches to a car search, this consent should be found to be voluntarily given.

Conclusion: Thus, the search for the gun was likely reasonable based on consent, regardless of whether or not it was legitimate based on checkpoint rules for the cops to remove him from his car. iii) The Plain-View Doctrine: It appears, either because the entire checkpoint process was constitutional, or because D gave his consent to be moved from the car after a constitutionally permissible checkpoint stop, that the stop and seizure was constitutional

87

at the time Dan got out of the car. Thus, the police were constitutionally on solid ground when Dan was out of the car. The plain-view doctrine allows police who are legitimately in a place and see something criminal in plain-view to use that plain-view finding in court. The justification is that a person does not have a reasonable expectation of privacy in something the person lets the public see. Here, the gun will qualify under the plain-view doctrine. The police need not rely on any Terry type frisks of automobiles, or the automobile exception, because they do not apply. The gun was in plain-view, and to the extent that the officer "searched" the car by looking in the window, the plain-view exception applies. iv) CONCLUSION: The search and seizure was reasonable and the gun should be admissible. The checkpoint rule may validate the entire process, but even if it doesn't then the checkpoint rule was at least legitimate up until the time D was asked to exit the car. Because he consented, there is no violation of the 4th amendment. The gun is admissible based on the plain-view doctrine.

B. Will D prevail on 5th Amendment Miranda Grounds? The 5th Amendment protects the right against self-incrimination. Miranda v. Arizona, a case based on this right, holds that a person's statements made cannot be used against him in court if the Miranda warning is not given. However, Miranda applies only to custodial interrogations, and not when a person is not in custody or voluntarily offers information. Miranda warnings include the right to remain silent, the right to counsel, the knowledge that counsel will be provided to a person, and the knowledge that anything said while in custody may be used against that person in court. i) No Miranda Warnings were given. Here, the cops gave no warnings. Thus, D's statement is protected if it was made during a custodial interrogation. a. Custodial. Custodial situations are those in which a reasonable, innocent person does not feel free to terminate the encounter and leave at will. Here, D was out of his car being asked in the company of some police. It seems up to this point to have been

88

a pretty friendly encounter, with the cops not showing much force or . Still, it's hard to say whether someone would reasonably feel at this point justified and correct in telling the police that this interview has to stop, and that the person is just going to drive away; especially before the sobriety check is performed. Thus, it's a close call. However, as D is out of his car, speaking to police, and about to be subject to a sobriety test, I would conclude that this is a custodial situation as a reasonable person would not feel free to terminate the questioning and leave. b. Interrogation: An interrogative question is one that is reasonably likely to elicit an incriminating response. This is a pretty close call as well. On one hand, the officers had no indication that the gun was criminally possessed, and thus a mere question about it may not be enough to reasonably expect an incriminating response. On the other hand, if the gun was criminally possessed, then a truthful response would be incriminating. However, because the officer questioned D about the gun without any suspicion at all of it being stolen, I would find this to be a non-interrogative question. I.e., if they knew that there was a stolen gun around, and then they asked, that would be more likely to be an incriminating response. Here, this just seems like the officers inquiring about a gun in the car without any suspicion whatsoever. Thus, Dan's statement should be admissible. It also appears that even if he had denied the ownership of the gun, the bit about him admitting to the crime was completely volunteered. I.e., the cops did not ask him whether he stole the gun. They asked him if he owned it. Thus, D's answer could have been "No." Instead, and completely unprompted, D volunteered that he stole the gun. ii) CONCLUSION: This was likely a custodial situation. The situation probably not interrogative, but it may have been. Even if it was not an interrogative scenario, D's statement that "I stole the gun" was not in response to any questioning by the police, and is voluntary and admissible. If it is found to be an custodial interrogative situation, the only part of the statement that will be inadmissible will be the answer to the policeman's question: "No."

89

2. Which theft crime will D be convicted of? A. Theft crimes are specific intent crimes. This means that the thief must specifically intend the proscribed conduct - i.e., the thief must have the mens rea to permanently deprive the true owner of the object possession. Theft crimes include larceny (trespassory taking and carrying away of the personal property of another with intent to permanently deprive); larceny by (larceny, plus getting actual title to the property by intentional and legitimate ); larceny by trick (larceny, but obtaining mere possession of the property by trick or deception); and (the fraudulent conversion of the personal property of another by one legally in possession of that property).

B. No larceny crime lies: This will be an embezzlement, if it's anything. The reason is because the larceny crimes all require an intent to steal the item at the moment of possession. Here, Dan did not form the intent to keep the gun until he had already been in legitimate and lawful possession - as a courier for the church, and holding it for the church. The continuing trespass doctrine will not apply, because that applies to scenarios where a person has borrowed something against the owner's intent, but doesn't plan to steal it until later. That person is never in lawful possession. Because Dan's specific intent mens rea was not formed at the moment of possession of the gun, no larceny crime will lie.

C. Embezzlement: Embezzlement is: i) Fraudulent: I.e., wrongful. Here, D was supposed to deliver the gun to the church, but has kept the gun. Thus, he is in wrongful possession of the gun at the time the gun was found on him. ii) Conversion: This means the intent to permanently deprive the owner (Church) of possession. This will be the major issue. Dan tells the cops he wanted to give the gun back; further we have no indication that he ever meant to keep the gun forever - maybe he just wanted to drive around with it for a little bit. Because this is a specific intent crime, the prosecution will have a tough job proving that Dan subjectively and

90

specifically intended to keep the gun forever when he decided to not turn it in. It is important to note that once he kept the gun with intent to steal it, the crime was complete - it doesn't matter if he later developed the intent to return it. The prosecution could point to the fact that he was driving around with it and didn't turn it in when he was supposed to, which may help; so will the statement that "I stole it." This will be the issue at trial, right now it looks only probably proven at best. iii) Of the personal property of another: The woman gave the gun to the church. As such, the gun was the property of the church. iv) By someone in legal possession: Dan worked for the church, and it was his job in this instance to deliver the gun to the church. Thus, he has legal possession of the gun when the woman gave it to him. She gave it to him thinking he was going to give it to the church, because he was an employee of the church. The church charged him with the duty of taking donations and delivering them to it. Thus, this possession was legal. It is akin to a bank manager stealing money that he or she is supposed to be counting. D. CONCLUSION: Embezzlement may lie, but only if the prosecution can prove specific intent to steal the gun, which will be tough.

3. General conclusion: Gun and statement ("I stole it.") admissible. Embezzlement if there is specific intent, which there likely is.

91

ANSWER B TO QUESTION 6

1. Motion to suppress The fourth amendment prohibits unreasonable searches and seizures by the state. Miranda v. Arizona requires that warnings be given to an individual subject to "custodial interrogation" in order to protect the individual's right to be protected from self-incrimination. This is clearly state action, so the issues here are whether the gun was seized pursuant to an unreasonable search or seizure, or whether the statement was obtained in the context of custodial interrogation.

Exclusionary Rule and Fruit of the poisonous tree doctrine The exclusionary rule requires that a court exclude evidence seized pursuant to an unlawful search or seizure. The fruit of the poisonous tree doctrine also provides that evidence that is obtained as a result of an lawful search must also be excluded, subject to certain exceptions. The exclusionary rule also requires the suppression of statements obtained in violation of Miranda, although the fruit of the poisonous tree doctrine does not apply to Miranda. Here, if the gun was seized during an unlawful search or seizure, or if the statement was obtained in violation of Miranda, this evidence must be suppressed.

Gun Expectation of privacy An individual has standing to challenge a search or seizure when they have a reasonable expectation of privacy in the place or property being searched. When an individual knowingly exposes something to the public, he no longer has standing to challenge a search of it. In this case, Dan placed the gun on the front seat of his car. It is not clear if his windows were tinted, or if someone could see easily into the car and see the gun. However, typically an individual has an expectation of privacy as to the inside and contents of their car, so Dan probably has standing to challenge the search. He certainly has standing to challenge any detention of his person, which would constitute a seizure if a reasonable person would not feel free to leave.

92

Routine checkpoint Routine sobriety checkpoints are not considered seizures under the 4th amendment, so long as they are administered in a nondiscretionary manner and do not detain individuals for an unreasonable period of time. In this case, the officers at the checkpoint were stopping all cars, and were asking all drivers to briefly exit before going on their way. As a result, this checkpoint was not a seizure of Dan or his car, and did not implicate the 4th amendment.

Consent In addition, a search or seizure is not unreasonable if an individual to the search. Valid consent must be knowingly and voluntarily given. Whether an individual validly consented is determined objectively, and the court considers whether a reasonable police officer would believe that the individual consented to the search or seizure. In this case, the police officer explained the procedure and asked if Dan would exit the vehicle. As a result, Dan appears to be informed about the procedure and his consent was knowing. His consent was also voluntary because he said okay, and stepped out of the car. A reasonable police officer would consider this to be valid consent.

Plain-View The plain-view doctrine provides that where a police officer has a right to be in the place that he is, any objects in plain view may be validly searched or seized if there is probable cause to believe that the objects are products or instrumentalities of a crime. In this case, the officer had the right to be in the place that he was, as discussed above, because he had the right to stop Dan pursuant to the nature of the checkpoint and Dan's consent. At this time, the gun was in plain-view. The officer then asked Dan if the gun was his, and he responded that it was stolen. At that time, the police officer had not yet searched or seized the gun because he had not touched it or moved it in any way. However, when Dan confessed that it was stolen, probable cause arose for the officer to seize it, and the seizure was therefore lawful under the plain view doctrine.

93

Even if the statements were elicited in the context of a Miranda violation (to be discussed below), because the poisonous tree doctrine does not apply to Miranda, and because the gun was in plain view, the seizure of the gun was still lawful. Dan's motion to suppress the gun is likely to fail.

Statement A statement is obtained in violation of Miranda where an individual is in custody, and an officer is interrogating the individual without first providing the appropriate Miranda warnings. Here, it is clear that the officer did not provide Miranda warnings, so the question is whether Dan was in custody and whether the police officers question as to whether Dan owned the gun constituted interrogation.

Custody An individual is in custody for the purposes of Miranda where a reasonable person in his position would not feel free to leave and end the detention. However, the supreme court has specifically held that routine traffic stops did not constitute custody for the purposes of Miranda. In this case, therefore, the routine security checkpoint would not be considered custody for Miranda purposes. It does not matter that Dan thought that he had no choice, because the test is objective, and not subjective. When the police officer asked Dan if he would consent, it is also possible that a reasonable person in Dan's position would have interpreted this question as indicating that he was free to not consent.

Because Dan was not in custody at the time that he made the statement, it was not illicit in violation of Miranda and is admissible.

Interrogation A police officer is considered to be interrogating an individual where his questions are reasonably likely to illicit incriminating statements. Here, the officer asked Dan if he was the owner of the gun. This question does not seem designed to lead to an incriminating statement, only to determine who was the owner of the gun. In

94

responding to the question, Dan would have been expected to give a simple yes or no. In the event of a non, probably a statement about who it belonged to would be expected. From the perspective of the officer, it probably seemed unlikely that this question would illicit a confession to the theft of the gun.

Because Dan was not being interrogated at the time he made the statement, it was not obtained in violation of Miranda for this reason as well. Dan's motion to suppress the statement is likely to fail.

2. Likelihood of conviction Elements of theft Larceny, or theft, is the taking or concealing of the property of another with the intent to permanently deprive the owner or rightful possessor of that property of the property. The issue here is whether Dan took property that belonged to the church, and whether he intended to permanently deprive the church of the gun.

Taking A taking of the property of another occurs where the defendant physically moves the property of another, or conceals it on his person. In this case, although Dan may have had a right to possess the gun at the time that the woman handed it to him, it belonged to the Church as soon as the woman handed it over and told Dan that she wanted the Church to have it. Although Dan may have intended to give the gun to the church, a taking of the gun occurred when he did not give it to the church and instead placed it in his car. When he turned over the book and mislead the church as to the donation, his right of possession did not continue to exist and his action met the first element of larceny.

Intent to permanently deprive A defendant need not have had the intent to permanently deprive the owner or rightful possessor at the time that the taking of the property occurred. It is enough that the intent to permanently deprive arose after the taking. In this case, it is not clear if

95

Dan had the intent to permanently deprive. It would appear that he did not intend to ever give the gun to the church when he gave them only the book and placed the gun in his car. This is circumstantial evidence of an intent to permanently deprive and may be sufficient to meet the requirements for this element. On the other hand, he also told the officer that he was planning on giving it back. If he merely later changed his mind about the gun, this would be irrelevant, because if he had the requisite intent even this would be enough. However, this statement could also be circumstantial evidence indicating that he never had the required intent. This is a question for the jury to decide, depending on whether they believe the defendant’s statements.

Mistake of law Dan appears to believe that he "stole the gun." His beliefs about the illegality of his actions are immaterial however. His statement would be relevant only to determine whether he had an intent to permanently deprive. This is because belief that one completed an unlawful act that is actually lawful does not render the act unlawful.

Embezzlement Embezzlement is a type of theft, and is the taking of a piece of property that the defendant had a right to possess at the time of the taking. Therefore, even if Dan had a right to possess the gun at the time, Dan could still be convicted of embezzlement, as opposed to basic theft. This conviction would turn on whether the jury found that placing the gun in the car was sufficient to indicate that Dan intended to convert the Church's property into his own and permanently deprive the church of it.

Because Dan took a gun that he did not have a right to possess, and because circumstantial evidence indicates he intended to permanently deprive the church of the gun, he is likely to be convicted at trial for theft.

96

THE STATE BAR OF CALIFORNIA

OFFICE OF ADMISSIONS 180 HOWARD STREET • SAN FRANCISCO, CALIFORNIA 94105-1639 • (415) 538 - 2303 1149 SOUTH HILL STREET • LOS ANGELES, CALIFORNIA 90015-2299 • (213) 765 – 1500

ESSAY QUESTIONS AND SELECTED ANSWERS

FEBRUARY 2013

CALIFORNIA BAR EXAMINATION

This publication contains the six essay questions from the February 2013 California Bar Examination and two selected answers to each question.

The answers received good grades and were written by applicants who passed the examination. The answers were prepared by their authors, and were transcribed as submitted, except that minor corrections in spelling and punctuation were made for ease in reading. The answers are reproduced here with the consent of their authors.

Question Number Contents

1 Criminal Law and Procedure

2 Professional Responsibility

3 Remedies

4 Torts

5 Civil Procedure

6 Business Associations

Question 1

Max imports paintings. For years, he has knowingly bought and resold paintings stolen from small museums in Europe. He operates a gallery in State X in partnership with his three sons, Allen, Burt, and Carl, but he has never told them about his criminal activities. Each of his sons, however, has suspected that many of the paintings were stolen.

One day, Max and his sons picked up a painting sent from London. Max had arranged to buy a painting recently stolen by Ted, one of his criminal sources, from a small British museum.

Max believed the painting that they picked up was the stolen one, but he did not share his belief with the others.

Having read an article about the theft, Allen also believed the painting was the stolen one but also did not share his belief.

Burt knew about the theft of the painting. Without Max’s knowledge, however, he had arranged for Ted to send Max a copy of the stolen painting and to retain the stolen painting itself for sale later.

Carl regularly sold information about Max’s transactions to law enforcement agencies and continued to participate in the business for the sole purpose of continuing to deal with them.

Are Max, Allen, Burt, and/or Carl guilty of:

(a) conspiracy to receive stolen property,

(b) receipt of stolen property with respect to the copy of the stolen painting, and/or,

(c) attempt to receive stolen property with respect to the copy of the stolen painting? Discuss.

ANSWER A TO QUESTION 1

(a) Max, Allen, Burt, and Carl's liability for conspiracy to receive stolen property

Max

The issue is whether Max is liable for conspiracy to receive stolen property.

Conspiracy requires (i) an agreement, express or implied, to accomplish an unlawful objective or to accomplish a lawful objective with unlawful means, (ii) an intent to agree to commit conspiracy, (iii) an intent to achieve the unlawful objective, (iv) an overt act in furtherance of the objective of the conspiracy.

(i) Agreement

There was no express agreement among Max and any of his sons, Allen, Burt, and Carl that the paintings were stolen. Max has knowingly bought and resold paintings stolen from small museums in Europe, and operates a gallery in State X with his sons. Max never told them about his criminal activities; thus there was no way they could have expressly agreed to commit the conspiracy. However, Max and Ted have an agreement, because Max had arranged to buy a painting recently stolen by Ted, one of his criminal sources.

There was no implied agreement among Max and his sons because there is no circumstance or conduct to indicate that they were in agreement. Max never affirmatively ensured that his sons were additionally compensated for keeping it a secret that they were undergoing criminal acts, nor had any of them given Max an indication confirming their understanding even if no explicit words were exchanged regarding the conspiracy. Here, each of his sons suspected that many of the paintings were stolen. However, Max had no idea that his sons might be aware. When Max picked up the painting that he thought was stolen, he did not share this belief with the others.

(ii) Intent to agree to the conspiracy

There must be at least two guilty minds to be liable for conspiracy. Under the minority jurisdictions, unilateral intent is sufficient if the guilty mind genuinely believed that the other non-guilty mind had the intent to agree to the conspiracy. There was no intent to agree to commit the conspiracy because Max never shared his beliefs with the others that he was dealing with stolen paintings. Here, Burt did not share his knowledge about the theft of the painting. Nor did Carl have an intent to agree, because he was solely continuing to participate in the business for the sole purpose of selling the information to the police. Thus, there could not have been an intent to agree to the conspiracy with either Burt nor Carl based on the majority rule. Under the minority approach, there is still no intent to agree because the facts indicate that Max did not tell Carl about his illegal activities and nothing suggests Carl shared his information with Max. Because there was no agreement in the first place among Max and any of his sons, Max did not have the intent to agree to commit the conspiracy.

Max and Ted have the intent to agree to the conspiracy, as evidenced by Max's arrangement to pick up the painting that Ted stole.

(iii) Intent to achieve the unlawful objective

There must be an intent to achieve the objective, which here is the intent to receive stolen goods. Max had the intent to receive the stolen goods because he has knowingly bought the paintings stolen from small museums in Europe.

(iv) Overt act in furtherance of the objective

There must be an overt act in furtherance of the objective, which is anything including mere preparation. Here, Max committed an overt act when he picked up the painting which he thought was the stolen painting.

Max is guilty of conspiracy with Ted.

Allen

See rule above.

(i) Agreement

Allen did not enter into an agreement to commit the conspiracy because even though he suspected that many of the paintings were stolen, and that he believed the one stolen by Ted was stolen, he did not share his belief with others.

(ii) Intent to agree

Allen did not intend to agree to the conspiracy because he did not share his belief that the painting may have been stolen with others. He only learned that the painting was stolen from reading an article and not from the other members.

(iii) Intent to achieve the objective

Allen may have had the intent to achieve the objective because he did nothing to stop the receipt of the stolen paintings.

(iv) Overt act

An overt act was the picking up of the painting sent from London.

Thus, Allen is not liable for conspiracy.

Burt

See rule above.

(i) Agreement

Burt made no agreement to enter into the conspiracy, because even though he suspected that they were stolen, and knew about the painting, he did not share his knowledge with the others. However, Burt has an agreement to enter into the conspiracy with Ted, because he arranged for Ted to send Max a copy of the stolen property and to retain the stolen painting itself for sale later.

(ii) Intent to agree

Burt had no intent to agree with the others, because he did not tell Max, and he arranged for Ted to send Max a copy of the stolen painting and to retain the stolen painting itself for sale later. However, Burt had the intent to agree with Ted, given that Ted was the other end of the deal and he arranged for Max to receive the stolen painting.

(iii) Intent to achieve the objective

Burt had the intent to achieve the objective because he knew the painting was stolen, and was going to sell it later at a more convenient time to gain a personal benefit.

(iv) Overt act

Overt act was committed when they picked up the painting from London.

Thus, Burt is liable for conspiracy with Ted.

Carl

See rule above.

(i) Agreement

Carl made no agreement to enter into the conspiracy.

(ii) Intent to agree

As discussed under Max's discussion, in the majority jurisdiction, because two guilty minds are necessary, there is no intent to agree since Carl was acting solely to sell the information to the police, and not to actually engage in the unlawful conduct. However, under the unilateral approach, one guilty mind, Max's guilty mind, would be sufficient for Max to be guilty of conspiracy. However, Carl would not be liable because he has no intent to agree himself.

(iii) Intent to achieve the objective

Carl has no intent to steal property, but is only participating to sell the information to the police.

(i) Overt act Overt act was committed when the painting was received from London.

Conclusion

Because there is no agreement to conspire, neither are liable for conspiracy with each other, but Burt and Max are liable for conspiracy as a result of their individual agreements with Ted.

(b) Max, Allen, Burt, and Carl's liability for receipt of stolen property with respect to the copy of the stolen painting

Co-conspirators are liable for the target crime and any crimes committed in furtherance of the conspiracy. As above, anyone who was liable for the conspiracy would be liable for the crime of receipt of stolen goods. However, the target crime of receipt of stolen goods did not occur because it was a copy of the stolen painting. Thus, no liability for the target crime at this point.

Receipt of stolen property requires (i) receipt or control of stolen property, (ii) of personal property by another, (iii) with the knowledge that the property was obtained in a way that constitutes a criminal offense, (iv) with the intent to permanently deprive.

Max

Max knew the property was obtained in a way that constituted a criminal offense, because he arranged to buy the painting recently stolen by Ted, one of his criminal sources. A painting is personal property, and it was stolen by another, Ted. He had the intent to permanently deprive because his motivation was to resell the stolen paintings. However, he did not actually receive or come into control of the property because the one he received was actually not stolen. Thus, he is not liable.

Allen

For the same reasons as Max, Allen is not liable because he did not actually receive the stolen painting.

Burt

For the same reasons as Max, Allen is not liable because he did not actually receive the stolen painting.

Carl

For the same reasons as Max, Allen is not liable because he did not actually receive the stolen painting. Further, Carl did not have the intent to permanently deprive because he was only working with the police so that the police could regain the stolen property and return it to its rightful owner.

Conclusion

Because no one actually came into receipt or control of the stolen property, they cannot be liable for the copy of the stolen painting.

(c) Max, Allen, Burt, and Carl's liability for attempt to receive stolen property with respect to the copy of the stolen property

Attempt requires the specific intent to achieve the criminal act and a substantial step in the direction of the commission of the act or dangerously close to the commission of the act.

Max

Max had the specific intent to receive stolen property. He believed that the painting was the stolen one. Even an unreasonable mistake would negate specific intent. However, if the facts were as he believed them to be, it would have been a crime, and thus, his intent cannot be negated. Mistake of fact is no defense. He committed a substantial step when he picked up the painting from Ted.

Allen

Allen also believed the painting was stolen because he read an article about the theft. Even if the stolen painting was not actually stolen, mistake of fact is no defense, and the

act would have been criminal had the facts been as he believed them to be, and thus, he is also liable for attempt.

Burt

Burt knew about the theft of the painting. He had specific intent to receive the stolen painting. But as to this copy, he had arranged for it to be simply a copy, and had told Max to retain the stolen painting for sale later. Thus, he had no specific intent to receive stolen property when he picked up the copy of the painting. Thus, he is not liable for attempt.

Carl

Carl suspected that many of the paintings were stolen. However, he did not have the specific intent to receive stolen property. He did not intend to permanently deprive because he was merely working with the police.

Conclusion

Max and Allen are liable for attempt, but Burt and Carl are not.

ANSWER B TO QUESTION 1

A. Conspiracy to Receive Stolen Property

The crime of conspiracy requires: (1) an agreement between two or more people to accomplish an unlawful or fraudulent purpose, and (2) an overt act taken in furtherance of the conspiracy. Under the majority rule, all parties to the conspiracy must agree to pursue the unlawful or fraudulent purpose; however, under the minority rule, the agreement of only one participant is sufficient to establish the conspiracy (for instance, in circumstances where one participant conspires in an effort to commit a crime and the other is an undercover law enforcement officer). Regarding the overt act requirement, nearly any act taken by any co-conspirators in furtherance of the unlawful objective will suffice.

Co-conspirators are liable for both conspiracy as a separate crime, for and all foreseeable crimes committed by any co-conspirators in furtherance of the unlawful objective. There is no doctrine of merger applied to conspiracy, and thus one may be convicted of both conspiracy and the underlying crime(s) committed in furtherance of it. A co-conspirator need not personally participate in an underlying crime committed by a co-conspirator in furtherance of the conspiracy, so long as the crime was a foreseeable result of the unlawful objective.

In this case, there was no express or implied agreement between M, A, B, and C to receive the painting stolen by and acquired from T. Agreement among co-conspirators need not be in writing and need not even be expressed orally, but rather can be implied from conduct and knowledge under the circumstances. However, there must be some evidence of an understanding and meeting of the minds among the parties of the conspiracy that they will pursue an unlawful objective for conspiracy liability to occur. Here, while M certainly had the requisite knowledge and intent to receive stolen property, he did not do anything to obtain the agreement of A, B, or C to do anything in furtherance of that objective. In fact, M never told any of his sons that he regularly

bought stolen paintings from Europe, nor did he share his belief as to the specific painting in question being the stolen one. Far from agreeing with them to receive stolen property, he was trying to shield them from that fact. Moreover, the mere fact that A. B, and C suspected their father's nefarious activities does not suffice to create an implied agreement between any or all of them and him to pursue that common unlawful objective, as they neither shared those suspicions and/or knowledge with M or with each other. Nor does it matter that A believed the painting was stolen (and that the one they picked up was the stolen one), as he never did anything, through words or conduct, to share that belief. The same is true for B and C -- though each independently suspected or knew of their father's activities, there is nothing to suggest that through words or conduct, an agreement was reached between M, A, B, and C (or any subcombination of them) to receive stolen property. Thus, there is no conspiracy liability for M, A, B, and C here.

Moreover, if evidence of an agreement existed, there would also be a question as to whether C's role sufficed to show an agreement among the co-conspirators. As noted above, under the majority rule, all co-conspirators must agree to pursue an unlawful objective. Thus, C's status as informant to law enforcement and participation for the sole purpose of continuing to deal with law enforcement would destroy his agreement to further the objective in question. As a result, under the majority rule there would be no conspiracy for this reason as well. Under the minority rule, however, the agreement of only one participant will do, and thus there would be an agreement, if evidence of it existed, notwithstanding C's status.

If evidence of such an agreement did exist, however, the overt act requirement would be satisfied. The four of them going to pick up the painting that T had sent from London would qualify as an overt act in furtherance of a conspiracy, as nearly any conduct that is in furtherance of the objective in question will qualify.

Further, if an agreement existed, the defense of impossibility would not be available to M and his sons. While a defense of legal impossibility would work (i.e., if the objective of

the conspiracy is not actually illegal, there can be no conspiracy liability for agreeing to commit a lawful act), here the defense would be factual impossibility (i.e., that though they had hoped to receive a stolen painting, it was not in fact the stolen one but rather a copy). Factual impossibility is not a defense to crimes in general, nor is it to the crime of conspiracy, and thus if evidence of an agreement had existed it would not prevent their guilt.

Lastly, M and T may well be guilty of conspiracy to steal and/or receive the stolen painting. M and T agreed for T to sell the stolen painting to M, and T took the act of sending the copy and arranging for payment in furtherance of the conspiracy. Similarly, B has conspired with T, and if he receives the stolen painting from T, he may face conspiracy liability for the theft and/or receipt or sale of the painting as well.

B. Receipt of Stolen Property

The crime of receiving stolen property requires that the defendant: (1) receive property that has been wrongfully taken from the rightful owner with the intent not to return it to its true owner, and (2) know that the property in question was wrongfully taken from its rightful owner. A defendant's knowledge may be express or implied under the circumstances, and, furthermore, the knowledge requirement may be met if the defendant under the circumstances is "willfully blind" to the fact that the property has been stolen.

In this case, however, the painting that M, A, B, and C received was not in fact stolen. Thus, they will not be guilty of having received stolen property based on their receipt of the copy. However, if B later does receive the true stolen painting from T, he would be guilty of this crime. With regard to receipt of the copy, however, B is not guilty for the reason that the copy was not stolen and for the additional reason that he knew that it was not the stolen item in question, and thus could not be found to have known or be willfully blind to the fact that it was stolen.

M, A, B, and C might also argue factual impossibility, as discussed above. However, since one of the prima facie elements of this crime is that the property is in fact stolen and that element is not met under these facts, there is no need to apply this defense here.

If M and his sons had received the authentic stolen painting, even in the absence of a conspiracy agreement among them, each of M, A, B, and C would be guilty of this crime. M and B plainly knew it was stolen, and A believed it was from the article, making his knowing receipt of the true article a crime (absent his immediately returning it to the authorities). C regularly sold information about M to the authorities, and thus also likely knew the painting was stolen. Thus, if they had received the true painting, each would be guilty of receipt of stolen property.

C. Attempt to Receive Stolen Property

Attempt is a specific intent crime. It requires: (1) that the defendant take sufficient action toward the completion of a crime, and (2) specifically intend to commit that crime. There is a split of authority as to the appropriate test to use for determining whether a defendant has done enough to constitute an attempt. While all courts agree that "mere preparation" for the crime is not sufficient to impose criminal attempt liability, some courts require that the defendant take a substantial step toward the commission of the crime. Other courts require instead that the defendant come dangerously close to succeeding in committing the underlying crime in question. Unlike conspiracy, the crime of attempt is subject to the doctrine of merger, meaning that if a defendant actually does commit the underlying crime, the attempt merges into the completed crime, and the defendant thus cannot be liable both for attempt and for the completed crime.

M and A: In this case, M knew the painting had been stolen and believed the copy was the real thing, and A also knew it had been stolen and believed that this one was the real thing. Thus, M and A each specifically intended to commit the crime of receiving stolen property. Moreover, each took a substantial step toward doing so, and came

dangerously close, by picking up the copy of the painting. But for B's dirty double- crossing of his father and brothers, M and A would have succeeded in committing this crime. Thus, each of M and A is guilty of attempt to receive stolen property, regardless of the fact that the painting they picked up was a copy.

M and A will argue factual impossibility, as discussed above. However, this defense will fail, as factual impossibility is not a defense in general, nor is it a defense to attempt. After all, if M had tried to pickpocket someone's wallet but that person had left their wallet at home, M would nonetheless be liable for attempted larceny. So it is here with regard to attempt liability.

B: B presents a different case. Clearly he took a substantial step toward and came dangerously close to committing the crime, but he did not specifically intend to commit the crime of receiving stolen property by taking the copy of the painting. He in fact knew that the painting they picked up was a copy, and had not been stolen, and thus lacked specific intent. Thus, B would not be guilty under these circumstances for attempted receipt of stolen property by taking the copy of the painting sent from London. As noted above, he may be guilty for other conduct -- such as actually receiving the true stolen painting if T sends it to him, or for receiving proceeds of the sale of the true stolen painting under his agreement with T.

C: C, however, did believe that the painting that he picked up with the others was in fact stolen, and thus, like M and A, would be guilty for attempt. The fact that he was participating with law enforcement would not change this fact. C might be able to obtain immunity from prosecution as a result of his assistance, but absent a grant of immunity, he would be guilty along with M and A of attempted receipt of stolen property.

The State Bar Of California Committee of Bar Examiners/Office of Admissions

180 Howard Street • San Francisco, CA 94105-1639 • (415) 538-2300 845 S. Figueroa Street • Los Angeles, CA 90017-2515 • (213) 765-1500

ESSAY QUESTIONS AND SELECTED ANSWERS

JULY 2014

CALIFORNIA BAR EXAMINATION

This publication contains the six essay questions from the July 2014 California Bar Examination and two selected answers for each question.

The answers were assigned high grades and were written by applicants who passed the examination after one read. The answers were produced as submitted by the applicant, except that minor corrections in spelling and punctuation were made for ease in reading. They are reproduced here with the consent of the authors.

Question Number Subject

1. Contracts/Remedies

2. Evidence

3. Business Associations / Professional Responsibility

4. Criminal Law and Procedure

5. Trusts / Community Property

6. Torts

Question 4

One summer afternoon, Officer Prowl saw Dan, wearing a fully buttoned-up heavy winter coat, running down the street. Officer Prowl ordered Dan to stop. Dan complied. As Officer Prowl began to pat down Dan’s outer clothing, a car radio fell out from underneath. Officer Prowl arrested Dan and took him to the police station.

At the police station, Officer Query met with Dan and began asking him questions about the radio. Dan stated that he did not want to talk. Officer Query responded that, if Dan chose to remain silent, he could not tell the District Attorney that Dan was cooperative. Dan immediately confessed that he stole the radio.

Dan was charged with larceny. He retained Calvin as his attorney. He told Calvin that he was going to testify falsely at trial that the radio had been given to him as a gift. Calvin informed Dan that he would make sure he never testified.

Calvin filed motions for the following orders: (1) suppressing the radio as evidence; (2) suppressing Dan’s confession to Officer Query under Miranda for any use at trial; and (3) prohibiting Dan from testifying at trial.

At a hearing on the motions a week before trial, Dan, in response to Calvin’s motion for an order prohibiting him from testifying, stated: “I want to represent myself.”

1. How should the court rule on each of Calvin’s motions? Discuss.

2. How should the court rule on Dan’s request to represent himself? Discuss. QUESTION 4: SELECTED ANSWER A

1. Ruling on Calvin's Motions

Motion to Suppress the Radio as Evidence

Fourth Amendment Protections The Fourth Amendment, incorporated to the states through the Fourteenth Amendment, protects individuals against unreasonable searches and seizures of their person, home, and personal effects. A seizure occurs when an individual's freedom of movement is limited by an officer such that the person would not feel free to leave the officer's presence. A search occurs when an officer gathers information in which the individual has a reasonable expectation of privacy, such as a physical search of the person's body, a search of the person's home, or eavesdropping on private conversations through wiretapping. However, if the officer is in a location in which he is entitled to be, he may observe the person's conduct or identify contraband that is within plain view, since people do not have a reasonable expectation of privacy for things they disclose to the public, such as speaking on a public street. The general standard for reasonableness to affect a search or seizure is probable cause, although lesser standards apply in certain circumstances, as discussed below. The Fourth Amendment generally requires that police officers obtain a search warrant before searching a person and an arrest warrant before an arrest to ensure that the probable cause standard is met.

Terry Stop Under the Supreme Court decision in Terry, an officer may stop and search an individual based on less than probable cause. A "Terry stop" is a reasonable search under the Fourth Amendment when two conditions are satisfied. First, the officer must have reasonable suspicion, based on specific and articulable facts, that the individual is engaged in criminal activity in order to stop the person. The officer may then question the individual. In order to search the person, the officer must have reasonable suspicion, based on specific and articulable facts, that the person is armed. This is reasonable because if the person is armed, the officer is in possible danger.

Seizure A seizure occurs when an officer restricts the freedom of movement of a suspect such that the individual would not be free to leave the officer's presence. The court will take into account all of the circumstances, including the officer's language and tone and the setting in which the confrontation took place. However, merely being in a physically confined area (such as a bus) will not make the officer's interaction with a person into a seizure. If the officer orders the individual to stop, the seizure does not occur until the person complies with the officer's instructions and his movement is actually restrained. Here, Officer Prowl ordered Dan to stop while he was running down the street. He did not approach Dan and ask him to voluntarily speak with him. Rather, ordering "stop" would be interpreted by a reasonable person to be a use of police authority to restrain Dan's movement such that Dan could be subject to penalty if he refused. Dan complied with Prowl's order and actually stopped. Thus, a seizure occurred.

Reasonable Suspicion to Stop The seizure of Dan will be reasonable under the Fourth Amendment, per Terry, if Prowl had reasonable suspicion to stop Dan. In order to satisfy the Fourth Amendment, Officer Prowl must have reasonable suspicion that Dan is engaged in criminal activity. This must be more than a mere hunch or an anonymous tip that the officer has no reason to trust. The officer must be able to identify specific facts that demonstrate objectively the reasonable suspicion to stop the person. Here, Dan was running down the street wearing a fully buttoned-up heavy winter coat on a summer afternoon. It is objectively unusual to see someone wearing such a coat during the summer, and Prowl's experience would likely indicate to him that people use such coats to conceal contraband, such as stolen property or drugs. Further, Dan was running. Because of the coat, it would seem unlikely that Dan was running for exercise, since he would be overly hot during the summer. Because these facts, taken together, indicate that Dan was acting objectively suspiciously, Prowl had reasonable suspicion to stop Dan.

Search A search occurs when an officer infringes upon an individual's reasonable expectation of privacy. The individual's person is always an area in which the person has a reasonable expectation of privacy unless that expectation has been reduced for some reason, such as in prisoners and parolees. We do not have any indication that Dan was a parolee or on . Thus, when Officer Prowl patted Dan down, a search occurred.

Reasonable Suspicion to Perform Pat-Down Under Terry, Prowl's search of Dan will be reasonable if he had reasonable articulable suspicion that Dan was armed. Although Dan's activity was objectively suspicious, he did not do anything and we have no indication that Prowl had prior knowledge that would make it objectively likely that Dan was actually armed. Prowl did not even speak with Dan after ordering him to stop, but immediately began a pat-down. Prowl would argue that Dan's bulky coat could easily have concealed a weapon, and Prowl's search was thus for self-protection. However, a physical search based on no independent facts suggesting that the person is armed is only reasonable following an arrest. Here, Dan was not arrested when Prowl performed the search. Prowl's search of Dan was not based on reasonable articulable suspicion and was therefore a violation of Dan's Fourth Amendment rights.

Exclusion of Evidence Evidence seized in violation of an individual's Fourth Amendment rights will generally be excluded in any subsequent criminal prosecution of that individual. The exclusionary rule operates as a deterrence mechanism to discourage police officers from committing constitutional violations. Although there are some circumstances in which the Supreme Court has concluded that the deterrent effect of the exclusionary rule is too inadequate to justify exclusion (such as knock-and-announce violations), the exclusionary rule operates in the Terry stop circumstances. Any contraband that was discovered as a result of an illegal search subject to the exclusionary rule will be excluded from evidence. Here, Prowl violated Dan's Fourth Amendment rights when he unreasonably searched Dan. Therefore, the court should order that the radio be suppressed.

Motion to Suppress Dan's Confession

Fourth Amendment First, Dan would argue that the Fourth Amendment violation directly led to his confession, and thus the confession should be excluded under the "fruit of the poisonous tree" doctrine discussed above. However, the Fourth Amendment exclusionary rule operates to exclude physical evidence rather than statements. Thus, Dan's confession would not be excluded by the Fourth Amendment.

Fifth Amendment Protections The Fifth Amendment right against self-incrimination protects suspects from being compelled to make statements against their own penal interests. The Supreme Court in Miranda interpreted this protection to require the police to effect certain warnings to individuals who are subject to custodial interrogation at the hands of police to offset the inherently compelling pressures of police interrogation.

Miranda Warnings Police officers must give each suspect warnings about his rights once he is subject to custodial interrogation. The warnings must inform the suspect of his right to remain silent, his right to an attorney, and that the attorney will be provided for him if he cannot afford to pay.

Custodial The "custodial" element is satisfied if the person is subject to police custody at the time of questioning. Once the individual is arrested, he is generally understood to be in police custody. Even before an arrest, the suspect may be subject to custody if he is being restrained in a formal setting, such as a police station, and is not told that he is free to leave at any time. The suspect need not have been indicted or charged for the custody element to be satisfied. Here, Dan had been arrested and taken to the police station, where Query began questioning him. Because Dan was in a formal setting and had actually been arrested, the custodial element is satisfied.

Interrogation The "interrogation" element requires that the police actually be asking the defendant questions that would be reasonably likely to lead to an incriminating response. A question such as whether the suspect would like a drink of water or whether he was comfortable would not constitute interrogation. Here, once Dan was in custody, Query began asking him questions specifically about the radio. Thus, Dan was being interrogated.

Because both elements of Miranda are satisfied here, Query violated Dan's Fifth Amendment right against self-incrimination by failing to read him Miranda warnings.

Dan's Statement That He Did Not Want to Talk Once an officer has read the suspect his Miranda rights, any express invocation of those rights must be strictly honored by the officers, who must then stop interrogating the suspect. Here, Query should have read Dan his rights. Dan's explicit statement that he "did not want to talk" likely qualifies as an invocation of his right to remain silent. Because Query continued to interrogate Dan following Dan's express invocation of his right to remain silent, Query violated Dan's Fifth Amendment rights.

Exclusion of Statement under Fifth Amendment The remedy for a Fifth Amendment violation is an exclusion of the improperly obtained confession. However, generally speaking, any physical fruits of the confession, such as evidence seized in reliance on statements made in the confession (such as the location of contraband) are not excluded. Further, the statement may still be used to impeach the suspect if he were to testify in the criminal case. Here, Dan confessed that he stole the radio. Because Dan's Fifth Amendment rights were violated, the statement should be excluded from the prosecution's case-in- chief, although it may still be used to impeach Dan.

Voluntariness The Fifth and Fourteenth Amendments of the Constitution also protect individuals against compulsory statements. A statement is compulsory if it was made involuntarily. An involuntary statement could be made as a result of legal compulsion (such as a subpoena to testify before a grand jury) or by improper police tactics, such as physical violence, threats, or promises that the suspect will not be prosecuted if he confesses. Although Calvin did not move to suppress the statement on voluntariness grounds, Dan would be wise to do so, since exclusion on voluntariness grounds would prevent the statement from being used against Dan on cross-examination. Here, Query told Dan that he "could not tell the District Attorney that Dan was cooperative" if he refused to speak. Although this statement does not explicitly promise Dan that he would not be prosecuted based on the statement, Dan would argue that Query suggested that he could guarantee different penal consequences based on whether Dan confessed. Query would say that he merely suggested a statement he could make to the prosecution, not that the prosecution would react in any specific way. Because Query did not make any actual promise that Dan's penal outcome would be different, the statement was likely voluntarily made.

Exclusion of Statement for Voluntariness If Dan's statement were involuntarily made, the statement itself would be excluded for all purposes, including impeachment. Further, any physical fruits of the statement would be excluded as well. Thus, because Dan wants to testify at trial, he should still argue that the statement was involuntary, even if this argument is likely to fail. Motion to Prohibit Dan from Testifying

Defendant's Right to Testify Each defendant has a constitutional right to testify in his own trial. Although an attorney has a professional ethical obligation to counsel his client not to lie on the stand, the lawyer cannot prevent the client from doing so. Under the ABA authorities, the attorney must seek to withdraw from the representation if he knows that the client intends to perjure himself. The court could then grant leave to withdraw, but may also decide that efficiency and justice require continued representation. Thus, the court should rule against Calvin's motion to prevent Dan from testifying. However, it would be proper under the ABA rules for Calvin to seek to withdraw from representing Dan.

2. Dan's Request to Represent Himself

Sixth Amendment Protections The Sixth Amendment right to counsel protects a criminal defendant's right to be represented by an attorney in all critical stages of prosecutory action by the state. The Sixth Amendment right includes the right to counsel of choice or to decline the right of representation if the defendant is competent to refuse.

Right of Self-Representation The Sixth Amendment includes a right of self-representation. The court must grant the right if the defendant is competent.

Competence to Stand Trial The general rule is that if the defendant is competent to stand trial, he will be found competent to represent himself. To be competent to stand trial, the defendant must understand the nature of the proceedings against him and be aware of the consequences of the proceedings. Here, we have no facts suggesting that Dan has a mental defect that would affect his competence. Thus, the competency to stand trial is satisfied.

Competence for Self-Representation The Supreme Court has stated that competence for the purpose of self- representation does not require the defendant to be legally sophisticated or be able to do an objectively good job representing himself. Although the Court has recognized that most defendants would be better served by counsel than by self-representation, the Sixth Amendment guarantee requires the court to allow the defendant to represent himself, regardless of whether the court finds that his action is in his own best interest. Thus, although Dan does not appear to have any particular legal knowledge or skills, such knowledge is not required to trigger the constitutional right to self- representation. Therefore, the court must allow Dan to represent himself.

Advisory Counsel The court may require that the individual be assigned advisory counsel to assist him. The role of advisory counsel is to provide the defendant with legal advice and information, but advisory counsel is not allowed to make the strategic decisions that appointed or retained counsel may, such as choosing to call only certain witnesses (other than the defendant) or present certain evidence. The advisory counsel role serves as a layer of protection for a self-representing defendant in order to protect the integrity and efficiency of the judicial process. Thus, although the court must allow Dan to represent himself, it could choose to appoint Calvin or another attorney as Dan's advisory counsel. QUESTION 4: SELECTED ANSWER B

1. HOW SHOULD THE COURT RULE ON EACH OF CALVIN'S MOTIONS

(1) Suppressing the Radio as Evidence

Exclusionary Rule Where evidence is obtained unlawfully under the Fourth, Fifth, or Sixth Amendments, that evidence is generally inadmissible against the accused. In Mapp v. Ohio, the Supreme Court held that the exclusionary rule is incorporated against the states. Moreover, under the fruit of the poisonous tree doctrine, all evidence obtained as a result of an invalid search or confession is also suppressed unless the government can prove (i) an independent basis; (ii) inevitable discovery; or (iii) an intervening act of free will.

Fourth Amendment Search and Seizure The Fourth Amendment provides that a person be free from unreasonable searches and seizure of their persons, homes, papers, or effects. To that end, Dan (D) should be able to successfully argue that he was unlawfully seized and that the radio must be excluded as the fruit of an invalid seizure.

(1) State Action The Fourth Amendment is only triggered by state action. Thus, a state or federal police officer or a private officer that has been deputized by the city or state must be the actor in order to render the Amendment applicable. Here, Officer Prowl (OP) appears to be a state police officer and hence the state action requirement is satisfied.

(2) Search / Seizure A "seizure" occurs under the Fourth Amendment where the circumstances of the encounter are such that a reasonable person would not feel free to decline the encounter. A "search" under the Fourth Amendment only occurs where the D has a reasonable expectation of privacy in the area and thing searched, or where there is a government intrusion into a constitutionally protected area.

Seizure. Here, D was ordered to stop by OP. A police officer may ask a person if they are willing to talk, at which point the person is free to decline and is not seized. However, where an officer commands a person to stop, their authority as a police officer is such that a reasonable person does not feel free to decline the encounter. Thus, D was seized by OP when he was commanded to stop and he did, in fact, stop.

Search. Here, D does not have a reasonable expectation of privacy in his movement on the streets. OP is free to follow him as much as he wants. However, D does have a reasonable expectation of privacy in the things he keeps out of public view, hidden under his coat. Merely stepping out onto the street does not render everything in D's possession "public." In this case, OP also intruded upon a constitutionally protected area, i.e., D's person. By patting down the outer clothing that D was wearing, OP intruded on his person and searched him under the Fourth Amendment.

Thus, if there is not a valid basis under the Constitution for this search and seizure, the evidence was obtained in violation of the Fourth Amendment and must be suppressed.

(3) Warrant Requirement A search or seizure is generally unreasonable unless the police have a warrant, or an exception to the warrant requirement applies. A warrant must be founded on (i) probable cause; (ii) state with particularity the persons and places to be searched; and (iii) be executed in a valid manner. Where a warrant that is otherwise invalid is relied upon in good faith by the arresting officers, the search or seizure will be upheld as long as the warrant was not: (i) so lacking in probable cause or particularity as to render reliance unreasonable; (ii) obtained by fraud on the magistrate; or (iii) the magistrate was impartial. Here, there was no warrant to arrest or search D. Thus, the search and seizure are unconstitutional unless an exception to the warrant requirement applies.

(4) Warrant Exceptions Terry Stop. An officer may engage in what is known as a temporary "investigative detention" under the Supreme Court's Terry framework, provided the officer has reasonable suspicion of criminality on the part of the D which is based on "articulable facts."

Here, the only facts that are given is that D was running down the street one summer afternoon wearing a fully buttoned, heavy winter coat. The fact that it was summer and D was wearing a fully buttoned up winter coat is certainly suspicious. Indeed, a reasonable person would almost have to assume that the purpose of wearing such a coat would be to hide evidence of contraband. If it is warm outside, as it usually is in the summer, a coat would be unnecessary. On the other hand, D may live somewhere like San Francisco where summers can be quite cold; D may have had a cold or some condition that makes him cold; or D may have been training for a sporting event such as wrestling where people force themselves to sweat more. The Court has held that headlong flight from an officer after seeing the officer is evidence sufficient to help support reasonable suspicion, but merely running has never been held to be reasonable suspicion absent additional facts.

Nevertheless, given that D was running down the street and wearing a coat that was fully buttoned during the winter, a court would likely find that the officer had reasonable suspicion--but certainty not probable cause--to detain D for a short period of time to investigate the potential criminality.

Terry Search. An officer that has reasonable suspicion of criminality based on articulable facts may also conduct a Terry search of the D, provided he has reasonable grounds for believing that the D is armed and dangerous. A Terry search must be limited to a pat-down of the outer clothing of the D, and must be limited to a search for weapons. In order to remove evidence that is not a weapon, the officer must have probable cause to believe the other evidence, e.g., drugs or a car stereo, is illegal.

Here, there is no real evidence that D is armed and dangerous. He was running wearing a coat, which--as discussed above--is sufficient to find reasonable suspicion that D just committed some type of theft offense and is trying to conceal the contraband in his coat. However, D will argue there is really no reason to believe that he was armed at this point. OP cannot simply claim he thinks D is armed because he seemed sketchy. On the other hand, OP might be able to convince a court that many theft offenses are committed with a weapon and hence that D could reasonably have been carrying a weapon. The fact that D was not actually carrying a weapon will not undermine this argument. While this is a close call, a court would likely permit OP to conduct a Terry search here.

The scope of the search seems permissible in this case, as OP merely patted down D's outer clothing. As he did so, a car radio fell out. The car radio is not a weapon, but may be admissible under the plain view doctrine, discussed below. In any event, the search and seizure itself was not unconstitutional.

Plain View. The Plain View doctrine applies where (i) the police have a right to be where they are viewing; and (ii) they see evidence and it is immediately apparent the evidence is contraband. Here, as discussed above, OP had the right to stop D under Terry, and hence he had a right to be where he was viewing the radio as it fell from D's coat. Moreover, it was immediately apparent to OP that the car radio was contraband. Indeed, D was running down the street, in a coat, in the summer, with a car radio hidden inside his coat. The radio was quite apparently stolen and hence admissible under the plain view doctrine.

Consent. While D has a constitutional right not to be searched or seized, the right is subject to waiver, i.e., the search or seizure is not unreasonable if D consents to the search or seizure. Consent must be knowing and voluntary. However, it is not required that one know they have the right to decline the encounter.

Here, D is not likely to be deemed to have consented to either the seizure or the search by OP. Indeed, as discussed above, he was seized. A defendant is not deemed to consent when seized. Moreover, with respect to consent to search, OP just started patting down D's outer clothing. Consenting to questioning is not within the scope of consenting to search. Thus, even if D were deemed to consent to questioning he would not be deemed to consent to the search. In any event, the search and seizure are valid under Terry.

Conclusion The evidence of the radio is admissible given that the search and seizure were valid under a Terry stop and frisk and the radio fell out of D's coat and was in plain view.

(2) Suppressing Dan's Confession to Officer Query

The Fifth Amendment protects a person from being compelled to be a witness against his or her self. Due to the inherent risks of coercion in police custodial interrogations, the Supreme Court has held that a defendant must be given Miranda warnings before any confessions by the defendant are admissible against the defendant, unless used to impeach.

Miranda Warnings Miranda is triggered where the D is: (i) in custody; and (ii) interrogated.

Custody. For purposes of Miranda, custody is defined as a place where a reasonable person would not feel free to leave. Moreover, custody is assessed by looking to whether the situation involves the same inherently coercive pressures as stationhouse questioning. Here, D was arrested and taken to a police station where he was then met by Officer Query (OQ). D had no ability to leave, and no reasonable person would feel free to leave in this situation. Moreover, this is stationhouse questioning, so the inherent pressures that Miranda is meant to protect against are at their pinnacle here. Thus, D is in custody.

Interrogation. Interrogation is defined as any line of questioning that a reasonable officer would find likely to illicit an incriminating response. Here, OQ was asking D questions about the radio. This is clearly questioning that is likely to generate an incriminating response. Thus, D was interrogated.

As both elements of Miranda are met, D was required to receive Miranda warnings. OQ ought to have told him he had the right to remain silent; that anything he said could be used against him in court; that he had the right to an attorney; and that he had the right to have an attorney appointed if he could not afford one. Since D was not warned, his confession is inadmissible against him (unless it is used to impeach him).

Invoking Miranda D was not warned, but in this case it even seems that he attempted to invoke his Miranda rights. To invoke the right to remain silent, the D must clearly and unequivocally indicate his intent to invoke. Here, D stated to OQ that he "did not want to talk." That may not use the word "remain silent" but no reasonable officer could think that "not want[ing] to talk" means anything other than remain silent. After having said that, OQ tried to coerce him into talking. This is not permitted. OQ must honor D's request and stop talking. By badgering him after he invoked, any later confession is in violation of Miranda. In this case, since D was not even Mirandized, his is irrelevant. However, even if D were Mirandized, the fact that OQ failed to honor his request to remain silent is a separate basis for excluding this statement. Conclusion The confession must be suppressed (except for purposes of impeachment). Thus, the court should grant the motion in part, subject to use for impeachment.

(3) Prohibiting Dan From Testifying At Trial

Constitutional Right to Testify in Defense

All defendants have a constitutional right to testify in their defense at a criminal trial. This right trumps any ethical obligation that Calvin (C) has to the court or the profession. Indeed, neither C nor the court can prohibit D from testifying in this situation.

[NOTE: The proper response by C would have been to inform D that he cannot testify falsely and persuade him to testify truthfully. If that failed, C should have tried to withdraw from the representation. If the court failed to allow him to do so, under the ABA C should have then informed the tribunal and allowed the tribunal to take the necessary steps. Under the California rules, no disclosure is permitted. Instead, C should have let D testify and questioned him up until the point he knew he was going to testify falsely, then, at that point, allow D to testify in the narrative and in no way rely upon D's narrative in closing. Under any ethical rule and the Constitution, the prohibition on D testifying is not permitted.]

Conclusion The court should rule that D be permitted to testify, as a criminal defendant has a constitutional right to testify. The tribunal may take necessary steps to remedy the false testimony, such as requiring narrative testimony.

2. HOW SHOULD THE COURT RULE ON DAN'S MOTION TO REPRESENT HIMSELF Faretta Motion The right of a criminal defendant to be represented by counsel was held to require the right of self-representation in Faretta. Where a Faretta motion is timely made, and the court is satisfied that the defendant is competent enough to represent himself, the court is required to respect the dignity of the defendant and allow him to have the right to choose for himself and represent himself. A court may also appoint back-up counsel to assist (but not actually control) the representation, but that is not constitutionally required.

Competence. The Supreme Court recently held that a defendant may be competent to stand trial but nevertheless incompetent to represent himself.

In this case, we have very little information on whether D is capable of representing himself. It appears he was found competent to stand trial, or at least that no such hearing has been conducted to this point. Thus, given no facts indicating that D cannot represent himself, he would likely be deemed competent to stand trial. The judge would have to verify that D was able to understand the charges and the legal issues, but--again--there is nothing in the facts indicating D cannot handle this. The court would also look to the issues between D and C and use this as a further justification for allowing D to represent himself.

Timeliness. A court need not allow a defendant to represent himself if doing so would cause an undue delay in the case. The request must be timely.

Here, D made the request to represent himself after an attorney was appointed and various pretrial motions were made. Indeed, the motion came just a week before trial. To allow D to testify would likely require giving D extra time to prepare the case himself, which would mean that the trial would have to be pushed back. That would interfere with availability of witnesses and with the efficiency of the court and the ability for the prosecution to put on its case. D might also win sympathy from the fact C is not permitting him to put on his case. However, that is more of a reason to substitute counsel than to let D represent himself. In this situation, D would need to show he was immediately prepared to go to trial. Delay of any sort would be sufficient to permit the court to deny his Faretta motion.

Conclusion Although D is likely competent to represent himself, but the court is likely to deny the motion as untimely, given that the trial date is set for only one week from the date of the motion and given that D would likely need a good amount of time to fully prepare himself for trial. Civil Procedure

ESSAY QUESTIONS AND SELECTED ANSWERS JULY 2001 CALIFORNIA BAR EXAMINATION

This publication contains the six essay questions from the July 2001 California Bar Examination and two selected answers to each question.

The answers received good grades and were written by applicants who passed the examination. The answers were prepared by their authors, and were transcribed as submitted, except that minor corrections in spelling and punctuation were made for ease in reading. The answers are reproduced here with the consent of their authors.

Question Number Contents

1. Civil Procedure

2. Real Property

3. Evidence

4. Constitutional Law

5. Torts

6. Wills/Trusts

1 QUESTION 1

Pam took an indefinite leave of absence from her job, sublet her apartment in State A, and went to care for her elderly mother in State B. Approximately six months later, while Pam was walking to her car in the parking lot of Don's Market in State B, Rita, a resident of State C, struck Pam with her car. In Rita's car were three friends from State C who were traveling through State B with Rita. The friends told the police officer called to the scene of the accident that Pam was reading a magazine as she walked across the parking lot and was therefore not watching where she was going. Pam told the police officer that she had just walked out from behind a large concrete column in the parking lot when Rita's car struck her.

Pam sued Rita and Don's Market in federal court in State B. Pam's complaint sought $60,000 in damages against each defendant. It also asked the court for an injunction ordering Don's Market to tear down the concrete column in the parking lot.

Don's Market moved to dismiss Pam's complaint on the ground that the court lacked subject matter jurisdiction. The court denied the motion.

Rita then moved for a change of venue of the action to federal court in State C on the grounds that she is a citizen of State C and that it would be a hardship for her and her witnesses to travel to State B for trial. The court denied Rita's motion for change of venue.

Rita then filed a notice of appeal of the court's denial of her venue motion. The appellate court dismissed Rita's appeal.

1. Was the trial court correct in denying the motion of Don's Market to dismiss the complaint on the ground that the court lacked subject matter jurisdiction? Discuss. 2. Was the trial court correct in denying Rita's motion for change of venue? Discuss. 3. Was the appellate court correct in dismissing Rita's appeal? Discuss.

3 ANSWER A TO ESSAY QUESTION 1

I. Trial Court's Denial of Don's Market's Motion to Dismiss for Lack of Subject Matter Jurisdiction Federal courts are courts of limited subject matter jurisdiction (SMJ). Generally, a plaintiff's cause of action must be based on a federal question or on diversity of citizenship for a federal court to have SMJ.

A. Federal Question Jurisdiction A federal question exists when plaintiff sues to vindicate a federal right, often under a federal statute or the Constitution. Here, it is not clear what Pam's lawsuit is specifically about. However, since the incident was a car accident in a private parking lot, it is probably a negligence action. Accordingly, this is not a federal question since no federal issue is raised, and so the court does not have federal question SMJ.

B. Diversity of Citizenship SMJ For a federal court to have SMJ based on diversity, each plaintiff must be diverse from each defendant and the amount in controversy must exceed $75,000. a. Diversity of Citizenship Rita (R) is a resident of State C. An individual's citizenship is that of their domicile; since R appears to be domiciled in C, where she resides, R is a citizen of C.

Don's Market is probably a corporation. A corporation's citizenship includes its state of incorporation and the state in which it has its principal place of business. We are not told in which state Don's Market (DM) is incorporated. The market itself is in State B. If this is the only store DM operates, then its principal place of business is in State B and so it is a citizen of State B. Accordingly, DM is probably a citizen of B.

Pam, as an individual, is a citizen of the state of her domicile. Pam originally lived in State A, but left her job there indefinitely, subletting her apartment, to come to care for her mother in State B. Domicile is determined by physical residence combined with intent to make the state a permanent home. Pam (P) is physically residing in B. Her indefinite leave of absence from her job in State A may indicate she intends to eventually move back to A. If she intended to make B her permanent home, she probably would have quit her job in A and terminated her lease rather than subletting it. Accordingly, P probably does not have the intent to make B her permanent home. She is therefore still domiciled in State A and is a citizen of A.

Because P is a citizen of A, and R is a citizen of C, and DM is a citizen of B, complete diversity exists. b. Amount in Controversy For diversity SMJ, the amount in controversy must exceed $75,000. Here, P is claiming

4 $60,000 from each defendant. A plaintiff's good faith claim in excess of the required amount is sufficient.

P may aggregate her claims for $60,000 against each defendant. A plaintiff may only aggregate claims against multiple defendants if they are joint tortfeasors. Here, P appears to be claiming that R and DM jointly caused her injury through their individual negligence: R's negligence in driving and DM's negligence in placing the concrete column. Since these acts of negligence combined to cause P's injury, DM and R are joint tortfeasors. Accordingly, P may aggregate her separate $60,000 claims together, making $120,000, in excess of $75,000.

Additionally, P is seeking an injunction. An injunction may be valued by either the value of the benefit to plaintiff or the cost of compliance for defendant. The value of removing the column to P is probably not great. However, if the cost to DM of removing the column is over $15,000, then the injunction against DM plus the damages claim would exceed $75,000. Note P may then argue she has supplemental jurisdiction over the claim against R. However, as a plaintiff in a diversity case, she may not join additional claims under the supplemental jurisdiction statute.

Accordingly, since the parties are all diverse, and the amount in controversy exceeds $75,000 either through aggregation or the injunction, subject matter jurisdiction was proper.

The motion was therefore properly denied.

II. Denial of Motion for Change in Venue A. Proper Venue in State B To determine if R's motion should have been granted, we must see if venue was originally proper.

In a diversity case, venue is proper in any district where all defendants reside; or where a substantial part of the claim arose; or, if neither is possible, any district where any defendant is subject to personal jurisdiction.

5 1. Residence A corporation resides where it is subject to personal jurisdiction. Jurisdiction is proper under traditional bases, such as presence or citizenship in a state, or under minimum contacts analysis, in which the exercise of jurisdiction would not offend traditional notions of fair play and substantial justice. Here, DM is a citizen of B since it has its principal place of business there. Personal jurisdiction will be proper under traditional grounds over a corporation present as a citizen in a state. Accordingly, DM is subject to personal jurisdiction in B as a citizen. Additionally, DM certainly has minimum contacts with State B. It does substantial business there, purposefully availing itself of State B's laws, since its market is in State B. Also, the accident arose directly out of DM's contacts with B, since its market parking lot is in State B and it was certainly foreseeable that DM could be sued in State B arising out of incidents involving its market in State B. Accordingly, DM is subject to personal jurisdiction in State B, and so, for venue purposes, it resides in State B as well.

R, as a citizen and domiciliary of State C, resides in State C.

Accordingly, since DM resides in B and R resides in C, there is no district in which all defendants reside.

2. Substantial Part of the Claim The accident occurred entirely in State B, in the DM parking lot. Accordingly, a substantial (indeed, all) part of the claim arose in the district in which DM's store is located.

We are not told whether P's suit was filed in the district encompassing DM's market (sic). If State B only has one district, then venue is proper since the accident necessarily occurred within that district. If the accident and lawsuit are in different districts, then venue may not be proper where filed. More information is needed.

3. Any District Where Any Defendant is Subject to Personal Jurisdiction Here, R, as a citizen of C, is subject to personal jurisdiction in C; as noted above, DM is subject to personal jurisdiction in B. Accordingly, if no district fulfilling either of the first two requirements exists, then venue would be proper in R's home district or in B. However, as explained above, venue is proper in the district encompassing DM's market (sic), where the accident occurred.

Accordingly, assuming the lawsuit was filed in the same district encompassing DM's market (sic), venue was proper. Thus R's motion was properly denied on this basis.

B. Transfer of Venue Even if venue is originally proper, a court may still transfer venue to another court where the suit could originally have been brought, if the interests of justice so require.

R will argue that the interests of justice require transfer to C because her three witnesses

6 reside in C, and travel to B would be highly inconvenient. Certainly R's witnesses are very important, since their testimony presumably will state that P, in reading a magazine while walking, was at least contributorily negligent. The convenience of witnesses is normally a valid reason to transfer venue.

However, the action must have been bringable in the transferee district. Here this means all defendants must be subject to personal jurisdiction in C, the C court must have had SMJ, and venue must be proper in C.

1. SMJ in C Diversity of citizenship would provide valid SMJ in C.

2. Personal Jurisdiction R, a citizen and domiciliary of State C, is subject to personal jurisdiction in C under traditional bases.

DM may not be subject to personal jurisdiction in State C. We are not told State C's long-arm statute, but it does not appear that DM is a citizen of C, or even that it does any business there. Without any contacts with State C, DM cannot be said to have purposefully availed itself of C's laws, nor is it foreseeable that DM would be sued in C if it has no contacts there. Finally, the accident occurred in State B, so not only does State C have little interest in exercising jurisdiction over DM, but there is no relationship between DM and State C and the cause of action. Accordingly, it is highly unlikely that State C could exercise personal jurisdiction over DM, since no traditional bases exist, and there are apparently no contacts between DM and C, much less the constitutionally required minimum contacts.

Since DM is not subject to personal jurisdiction in State C, then the action could not have been brought originally in State C. Therefore, venue cannot be transferred to State C.

3. Venue As explained previously, a district exists where a substantial part of the claim arose: the district encompassing DM's market (sic), in State B. Therefore, since such a district exists, venue would not be proper in State C, since the only means of proper venue in State C would be under the "last resort" option of any district where any defendant is subject to personal jurisdiction. This option is unavailable where, as here, a district exists where a substantial part of the claim arose.

Accordingly, since transfer of venue to State C could not have been proper since neither venue nor personal jurisdiction over DM existed in C, the trial court properly denied R's motion to transfer venue.

III. Did the Appellate Court Correctly Dismiss Rita's Appeal? Appellate courts generally review final judgments. Here, the denial of R's motion to transfer

7 venue was not a final judgment. An appellate court may consider interlocutory appeals on certain matters, particularly if the matter is of great importance and, if not settled immediately by the appellate court, will substantially affect subsequent litigation.

Here, the denial of the motion to transfer venue was not a final judgment. In addition, since venue was proper in State B, since the accident occurred there and so the claim arose there (see previous analysis), the denial of transfer of venue did not confer improper jurisdiction on the trial court. Accordingly, there was no compelling need to consider the denial of the motion on immediate appeal.

Even had the appellate court heard the appeal, it would have reviewed the denial on an abuse of discretion basis. While the requirements of personal jurisdiction must be properly met, and cannot be waived, the determination of whether transfer would be in the interests of justice is for the discretion of the trial court. The court could have found that, while R and her witnesses would be inconvenienced in State B, that P and DM would be more inconvenienced in State C, especially since P cares for her elderly mother in State B. Since this decision would be one of the trial court's discretion, the appellate court would have been unlikely to overturn it.

Therefore, the dismissal of R's appeal was proper.

8 ANSWER B TO ESSAY QUESTION 1

(1) Subject Matter Jurisdiction Pam brought suit in federal court in State B. For a federal court to have subject matter jurisdiction, there must be a federal question or complete diversity of citizenship between the parties. Because no federal statute or constitutional claim is involved, jurisdiction can only be based on diversity.

Diversity Jurisdiction Federal court jurisdiction based on diversity requires complete diversity of citizenship between the parties and an amount in controversy over $75,000.

A. Complete Diversity of Citizenship For Pam to sue Don's Market and Rita in federal court, she must be a citizen of a different state than each of them.

1. Pam's Citizenship An individual's citizenship is based on their domicile, or where they intend to make their permanent home. For Pam to be diverse from Don's Market, her domicile cannot be in State B. Pam will argue that she is domiciled in State A, because that is where she was living until 6 months ago. Pam will argue that she left only for a temporary period to care for her elderly mother in State B, and that her intent to return is evidenced by the fact that she did not give up her apartment, only subletted it. Also, she did not quit her job, but only took a leave of absence from it.

Don's Market will argue that Pam is a citizen of State B because she is living there presently. He will argue that Pam's subletting her apartment was giving up her residence there, and that it was subletted just so Pam could avoid breaking her lease. Don will argue that she did not merely take a vacation from her job in State A, but has left it indefinitely.

Although it is a close question, the fact that Pam has retained both her apartment and her job in State A shows her intent to keep her permanent home there. The court should find that she is domiciled in State A.

2. Don's Market's Citizenship The citizenship of a business is its principal place of business and, if it is incorporated, where it is incorporated. The facts do not state whether Don's Market is a corporation, but its principal place of business is in State B, so it is a citizen of State B. Therefore Don is diverse from Pam.

3. Rita's Citizenship Since Rita is an individual, her citizenship, like Pam's, is based on her domicile. Since the facts state that she is from State C and was just driving through State B, her domicile can be

9 assumed to be in State C, where she lives. Therefore, Rita is diverse from.

Since both the defendants, Don's Market and Rita, are diverse from the plaintiff, Pam, complete diversity of citizenship exists.

B. Amount in Controversy Diversity jurisdiction requires an amount in controversy of over $75,000. The amount is based upon the plaintiff's good faith allegation and can only be challenged if it is clear to a legal certainty that she cannot recover that amount.

1. $60,000 Here, Pam claims $60,000 in damages against each defendant. Presuming that is her total claim against each one, including punitives and attorneys' fees if available, it does not satisfy the jurisdictional amount.

However, a plaintiff can aggregate her claims to meet the $75,000 requirement in certain circumstances. A plaintiff can aggregate her claims against the same defendant, but cannot aggregate her claims against different defendants unless they are joint tortfeasors against any of which she could recover the full amount. Here, there are facts to indicate that Don's Market and Rita are jointly liable, since they each caused the accident (Don's Market by placing a column improperly and Rita by driving carelessly). If they are jointly liable, Pam has met the jurisdictional amount because her claim is $120,000. If they are not, she cannot meet the requirement solely through her claimed damages.

2. Injunction However, Pam is also asking for an injunction to make Don's Market tear down the offending column. In a majority of states, injunctions are valued at their value to the plaintiff. Here, the injunction has little value for Pam, as she has already been injured and is unlikely to be injured by the column again. In the majority of states, then, this would not help Pam reach the jurisdictional amount against Don.

A minority of states allow injunctions to be measured by their cost to the defendant. Here, the cost to Don of tearing down the column may be high enough to raise her $60,000 claim to the required $75,000. If it does, the injunction will give the court diversity jurisdiction over Pam's claim against Don, but not over her claim against Rita. Nor is supplemental jurisdiction available over the claim against Rita based on the claim against Don, because this is not a federal question claim, and it is being brought by the plaintiff.

In conclusion, if Don's Market and Rita are jointly liable, the court's denial of Don's motion to dismiss for lack of jurisdiction was proper because there is complete diversity of parties and the amount in controversy exceeds $75,000 when the claims are aggregated. If they are not jointly liable, jurisdiction over Don may still be proper due to the injunction, but not as to Rita.

10 (2) Change of Venue A. Venue in State B Federal Court Venue is proper where any defendant resides if all reside in the same state, or where a substantial part of the events forming the basis for the claim arose.

Here, Don's Market resides in State B, where it is located and does business, but Rita resides in State C. However, since the accident that is the basis for the claim took place in State B, venue is proper there even though not all defendants reside there.

B. Transfer to State C Where venue is proper to begin with, a court may transfer to any other venue where the case could originally have been brought for the convenience of the parties or witnesses or in the interests of justice.

1. Convenience Here, Rita argues for transfer to State C on convenience grounds because that is where she resides, and it would be a hardship for her and her witnesses to defend in State B.

It is true that Rita and the three primary eyewitnesses, who also reside in State C (her friends who were in the car at the time of the accident and allege they saw Pam reading a magazine and not watching her step), would incur hardship in coming to State B to defend.

However, this hardship will be balanced against the hardship Pam and Don's Market will face in having to defend in State C, a foreign state for them. Pam is caring for her elderly mother and will find it hard to leave, and it will be hard for Don's Market to leave its business, especially as it is likely a sole proprietorship. Also, witnesses regarding the construction of the column, police who were called to the scene afterwards, and doctors who treated Pam are all located in State B. These factors weigh in favor of denying the motion to transfer.

2. Venue Proper in State C Moreover, venue may not be proper in State C because the case could not have originally been brought there, nor did the claim arise there. Although Rita resides there, Don's Market resides in State B, so venue cannot be supported on this basis. Also, the only event involved in the claim, the accident, occurred in State B, so venue is not proper on that basis either.

In conclusion, because the convenience to the parties and witnesses weighs in favor of State B and because venue would not be proper in State C, the court was correct to deny Rita's motion.

(3) Dismissal of Appeal A. Final Judgment A case can only be appealed from a final judgment on the merits in the lower court. If there are issues remaining for the lower court to decide, appeal will not be taken.

11 Here, the lower court has dismissed Rita's motion for change of venue, but that is not a final judgment. The court has not dismissed the underlying case, which still must be tried and decided.

B. Interlocutory Appeal A party may appeal before a final judgment on certain matters by right, such as a granting of an injunction, or if the lower court certifies that the issue is a close one and the appellate court agrees.

Because there is no right to an interlocutory appeal for a denial of a change of venue motion and the lower court did not certify, the dismissal of the appeal was proper.

12 California Bar Examination

Essay Questions and Selected Answers

February 2002 Question 1

Pam, a resident of State X, brought suit in state court in State X against Danco, a corporation with its principal place of business in State Y. The suit was for damages of $90,000 alleging that Danco breached a to supply Pam with paper goods for which she paid $90,000 in advance. In her complaint, Pam requested a jury trial. State X law provides that contract disputes for less than $200,000 must be tried to a judge.

Danco removed the case to federal court in State X. Danco moved to strike the request for a jury trial. The federal court denied the motion.

A few days before trial, Pam learned for the first time that Danco was incorporated in State X. She moved to have the case remanded to state court on this ground. The federal court denied the motion.

At trial, Pam testified that she paid for the goods but never received them. Danco admitted receiving Pam’s payment and then presented evidence from its dispatcher that it had sent a truck to Pam’s office with the paper goods. Danco also called as a witness Rafe, who works in a building next to Pam’s office. Rafe testified he saw a truck stop at Pam’s office on the day Danco claimed it delivered the goods. Rafe also testified he saw the truck driver take boxes marked “paper goods” into Pam’s office that same day.

At the close of all the evidence, Pam moved for judgment as a matter of law. Danco opposed the motion, and the court denied the motion. The jury returned a verdict in favor of Pam.

Danco then moved for judgment as a matter of law, which Pam opposed. The court denied Danco’s motion.

Did the court rule correctly on:

1. Danco’s motion to strike the request for a jury trial? Discuss.

2. Pam’s motion to have the case remanded to state court? Discuss.

3. Pam’s and Danco’s motions for judgment as a matter of law? Discuss.

-1- ANSWER A TO ESSAY QUESTION 1

I. Danco's motion to strike request for jury trial

Because this is a diversity case *(see below) and involves common law questions, Erie comes into play. Under Erie, in such a case, the federal court must use federal procedural law and state substantive law.

The question is whether a jury trial (versus a bench trial) is a procedural or substantive question. The state and federal laws on the subject conflict -- the law of State X provides for only a bench (or court) trial when the contract dispute is over an amount less than $200,000, and Pam is claiming only $90,000 in damages. The federal constitution, in the Seventh Amendment, provides for a jury trial in civil cases "for suits at common law" when damages exceed a mere

$20.

Therefore, in state court, Pam would have a trial in front of a judge, while if following federal law, she would have a trial in front of a jury. The Seventh

Amendment is not incorporated to the states through the Fourteenth

Amendment, so that does not control.

Some issues that at first may not seem substantive -- such as statutes of limitations -- are in fact considered such because of the effect they may have on suits. Because a jury trial is handled somewhat differently than a bench trial, it

2 would probably be considered a substantive issue, so the federal judge should

have applied the law of State X and denied the motion.

Diversity jurisdiction

This has to be a diversity case in federal court. Federal courts have two types of subject matter jurisdiction -- the power to hear cases regarding certain issues. The first is federal question jurisdiction, where federal courts may hear cases "arising under" a federal statute or the constitution. This is a contracts case, arising under state contracts law (or possibly the common law). Therefore it is not a federal question case.

Federal courts also have jurisdiction over diversity cases, where there is complete diversity among the parties -- all plaintiffs have different citizenship from all defendants, and where claimed damages exceed $75,000. Citizenship of a corporation for these purposes is its principal place of business and the (or all) place(s) of its incorporation. When Pam brought the suit and when Danco removed, there seemed to be diversity jurisdiction because P is a resident of State X, and D has its principal place of business in State Y, and damages exceed $75,000.

2. Pam's motion to have the case remanded to state court

Removal

Even if a plaintiff properly brings suit in state court, the defendant may remove it to federal court. However, the defendant may not do so if it is a resident of the state in which the case was brought. Therefore, because Danco (D) is a resident of State X -- since it is incorporated there -- it cannot properly remove to federal court.

3 (If removal were proper, it would be proper to remove to the federal court in the same state and district in which the case was brought.)

For analysis of citizenship of corporations, please see #1 above.

The federal court may have discretion to keep the case because Pam's motion was brought just a few days before trial, but in the end it cannot do so, as it lacks subject matter jurisdiction.

Lack of subject matter jurisdiction is not a waivable defense -- that is, even though Pam didn't raise this defense in her first pre-answer motions, she did not forfeit the defense. It may be brought at any time, even throughout trial.

The court simply lacks subject matter jurisdiction over this case, so it may not hear it. Therefore the court should have granted P's motion to remand.

3. (A) Pam's motion for judgment as a matter of law

Timing

A motion for judgment as a matter of law may be brought after the close of plaintiff's evidence or at the close of all evidence. Therefore Pam's (P's) timing was fine, as she brought the motion at the close of all evidence.

4 Substance of motion

A motion for judgment as a matter of law is granted if no reasonable person could differ as to the outcome of the trial. That is, it asks the judge to take the case out of the jury's hands and decide it as a matter of law.

Evidence

Dispatcher's evidence

The issue is whether the dispatcher's evidence was admissible and whether it makes a difference to Pam's motion.

We do not have enough information to decode this issue for sure, and we do not know how this evidence was offered at trial -- by the dispatcher orally, through notes or from someone else. It could be subject to the hearsay rule, if it is an out-of-court statement offered for the truth of the matter asserted -- particularly if D offered a written document or someone else to testify as to what the dispatcher said. If written, it could possibly come in under the business record exception to the hearsay rule -- if it was made in the normal course of business, known to the dispatcher at the time he made it, and timely made.

In any event, it wouldn't help the jury all that much because it doesn't show

(alone) that the truck ever showed up at Pam's place of business.

Rafe's evidence

5 This is not an out-of-court statement, and it is relevant because it might show

(especially when put together with the dispatcher's evidence) that the delivery from D was in fact made to P. This is Rafe testifying to his own personal knowledge.

The written statement "paper goods" could, however, be hearsay -- it is an out- of-court statement and D is offering it for the truth of the matter asserted -- that the boxes did in fact contain paper goods. That written statement would not fall into any exception or exclusion to the hearsay rule, so it should not be admitted.

With or without the "paper goods" statement, Rafe's evidence does not make clear whether the truck belonged to D. Together with the dispatcher's evidence, however, it does seem enough that reasonable people could differ as to the outcome.

Therefore the judge was correct in denying P's motion.

(B) Danco's motion for judgment as a matter of law

The issue is whether D's motion should have been granted.

In order to be able to make a renewed motion for judgment as a matter of law -- which is what it is called when made after the jury has returned its verdict -- the party must have made a motion for judgment as a matter of law at the close of all evidence.

Danco failed to make that motion, so its renewed motion is barred.

6 (If D had made the proper motion, its renewed motion would be subject to the same standard as discussed above.)

Therefore, the court was correct in denying D's motion.

7 ANSWER B TO ESSAY QUESTION 1

1. Danco's motion to strike request for jury trial

According to the US Constitution, any plaintiff who is suing in federal court for damages has a right to a trial by jury. If the suit involved is one of equity, there is no right to a jury trial.

Here, at the time that Danco made its motion, the case was in federal court. The state law that provides that contract disputes for less than $200,000 be tried by a judge does not therefore apply. Federal courts generally follow federal rules of procedure and are bound by the US Constitution. Therefore, the plaintiff does have a right to have a trial by jury and the federal court properly denied Danco's motion.

2. Pam's motion to have the case remanded to state court

If a plaintiff sues a defendant in state court and the case at hand is one in which a federal court would have subject matter jurisdiction over it, a defendant may properly remove the case to federal court. Therefore, if this case either posed a federal question or there was diversity between the claimants, the court could properly be heard in federal court.

8 In order to have diversity jurisdiction, the case at hand must involve an amount of at least $75,000 and there must be complete diversity between the plaintiff and defendant. For a person, his or her place of residence is the state to which he or she belongs. For a corporation, it is the principal place of business or place of incorporation where it is a resident.

At the time of removal, the federal court appeared to have diversity jurisdiction over this case. Pam was suing for $90,000, an amount over the required $75,000.

Furthermore, Pam was a resident of State X and all that was known was that Danco had its place of business in State Y. Therefore, it appeared that there was complete diversity.

After a case is removed to federal court, it can be remanded back down to state court if the federal court does not in fact have proper jurisdiction. Here, before the trial took place, Pam learned that Danco was incorporated in State X. This would ruin the complete diversity requirement and would be grounds for the federal court to remand back down to state court because it would not be proper for the federal court to hear the case since it did not have jurisdiction. If a federal question was involved it would be different, but that is not the case here. This is a simple contract claim. Danco would argue that Pam could have easily found out where it was incorporated by simply asking.

However, that does not have anything to do with the federal court's actual power to hear a case.

9 Therefore, the Federal Court was mistaken in denying the motion to have the case remanded.

3. Pam's and Danco's motions for judgement as a matter of law.

A motion for judgement as a matter of law can be made after the other side to a lawsuit has presented its case. That is, the Defendant can make the motion after the plaintiff has presented its side and after the defense has presented its side (or after all evidence has been presented). Furthermore, a renewed motion for a judgement as a matter of can be asked for after a jury has rendered its verdict. However, in order for the defendant to be able to make such a motion, he or she must have first made the motion at the end of all of the evidence being presented.

Pam's motion

When determining whether to grant a motion for a judgement as a matter of law, the court must consider the evidence and be able to determine that reasonable minds could not differ as to the outcome. This is an extremely difficult standard to meet. The judge in essence would be ruling that this would be the only reasonable verdict that could be reached if he or she granted the motion.

Here, Pam claimed that Danco breached the contract by not supplying the paper goods to her. Her only evidence was her testimony that she paid for the goods, but never received them. Danco on the other hand, admitted to receiving payment, but

10 claimed to have delivered the goods. He presented several pieces of evidence, including evidence from his own dispatcher that it did deliver the product and testimony of a neutral witness that claimed he saw a truck parked at Pam's office that day and a box marked "paper goods" on the front being delivered to her.

However, the witness did not mention whether the truck belonged to Danco and

Pam could have received different paper products from another company. Therefore, it could be reasonable to believe Pam's story over Danco's. The other evidence offered was simply that from their own records or own recollection. At the same time, it could be reasonable to believe Danco's story because he offered credible testimony from two different sources. Therefore, it would be reasonable to believe either side and a motion for judgement as a matter of law should not have been granted.

Therefore, the court properly denied Pam's motion.

Danco's motion

Danco first moved for a judgement as a matter of law after the jury verdict.

Again, this would normally be a renewed judgement as a matter of law and could only have been brought after Danco made a motion for a judgement as a matter of law after the close of all evidence or at the time Pam made the motion. Instead of simply denying

Pam's motion, Danco should have brought its own motion at that point.

11 Furthermore, as mentioned before, reasonable minds could have come to different conclusions in this case and such a motion would not have been warranted anyway.

Therefore, court properly denied Danco's motion.

12 ESSAY QUESTIONS AND SELECTED ANSWERS

FEBRUARY 2003 CALIFORNIA BAR EXAMINATION

This publication contains the six essay questions from the February 2003 California Bar Examination and two selected answers to each question.

The answers received good grades and were written by applicants who passed the examination. The answers were prepared by their authors, and were transcribed as submitted, except that minor corrections in spelling and punctuation were made for ease in reading. The answers are reproduced here with the consent of their authors and may not be reprinted.

Question Number Contents Page

1. Civil Procedure

2. Wills/Real Property

3. Criminal Law & Procedure/Evidence

4. Professional Responsibility

5. Constitutional Law

6. Community Property QUESTION 1

Petra, a State W resident, recently patented a new design for a tamper-free bottle cap for soft drinks. She contracted with Dave, who lives in State X, to design a manufacturing process to mass-produce the newly patented bottle caps. Under the contract, Dave was required to relocate to State W, where Petra had leased research and development facilities, and to keep confidential all design and production information concerning the bottle cap.

Dave promptly found someone to rent his home in State X. He moved all his belongings to State W. After working for six months in State W, Dave had perfected the manufacturing process, but when Petra denied Dave’s request for additional compensation he quit his job and disclosed the bottle cap manufacturing process to Kola, Inc. (“Kola”).

Kola is a regional soft drink bottler incorporated in State Y, with its principal place of business in State W. Kola flooded the market with bottled soft drinks capped with Kola’s version of Petra’s bottle cap months before Petra could begin production.

When Petra discovered what had happened, she filed suit against Dave and Kola in state court in State W for violation of State W’s patent infringement law. Petra’s complaint sought damages of $50,000 from Dave and $70,000 from Kola. Unknown to Petra’s lawyer, a federal patent law enacted shortly before Petra filed suit encompasses the type of claim pleaded by Petra and expressly preempts all state laws on the subject.

Six weeks after being served with the complaint, Kola removed the entire action to the federal district court in State W. Petra immediately filed a motion to remand the case to state court in State W. The district court denied Petra’s motion.

Petra immediately filed an appeal of the court’s ruling denying Petra’s motion to remand with the appropriate federal court of appeals.

1. Did the federal district court rule correctly on Petra’s motion to remand the case to state court in State W? Discuss.

2. Should the federal court of appeals entertain Petra’s appeal? Discuss.

-1- Answer A to Question 1

I. Did Federal District Court Correctly Rule On Petra’s Motion to Remand Case to State W?

Petra filed suit in State W Court against Dave (D) & Kola based on a State W cause of action. State courts are courts of general jurisdiction and thus if State Court had personal jurisdiction over D & Kola the claim was properly filed in State Court W.

A court has pers[onal] juris[diction] if the defendant is a resident, consents to juris, or is subject to the State’s long arm statute and meets the constitutional minimum contacts test. Here Kola is a corporation and thus a resident of its state of incorp (Y) and its state of principal place of business (W).

Dave’s residence is determined by his domicile and intent. He begins as a resident of State (X). Because of his contract with Petra he agrees to move to State W. It does not appear he intended to make W his domicile as he only rented his home rather than selling it. Also it was uncertain how long his job would take; thus Dave is likely still a resident of X.

State W may still have jurisdiction over Dave under the Const. minimum contacts analysis. Dave moved to State W to do business there and enjoyed the benefits of State W’s laws. He received compensation and performed services there. Because of the close contact between the claim and his contacts with State W, personal jurisdiction is fair provided he receives notice.

Removal of Case

A defendant may remove a case to the federal court in the state where the claim was filed provided the case could have initially been filed in federal court and the claim for removal is brought by all defendants within 30 days of filing of the complaint or the pleading which triggered the right of removal.

A Federal District Court is a court of limited jurisdiction, thus it may only hear claims based on federal questions (arising under the U.S. Constitution or statutes) or claims based on diversity of citizenship.

Diversity Jurisdiction

For a federal court to have diversity juris the plaintiff must be of diverse residency from

-2- all defendants and the amount in controversy must exceed $75,000.

In the instant matter Petra is a resident of State W and so is Kola because of its principal place of business in W. Thus diversity does not exist and federal subject matter jurisdiction doesn’t exist.

Additionally, the amount in controversy must exceed $75,000 based on plaintiff’s well- pleaded complaint (reasonable). A plaintiff may aggregate claims against multiple defendants provided the defendants are jointly and severally liable.

Petra’s complaint lists damages of $50,000 against Dave and $70,000 against Kola. Because the total damages exceed $75,000 and it is foreseeable that Dave or Kola could be liable for the full amount the $75,000 jurisdictional amount is met.

Kola cannot remove the action based on diversity because it is not diverse from Petra. Kola may, however, remove based on federal question jurisdiction because of Petra’s complaint, though pled under State W law, is really a claim under the new federal patent act. A plaintiff may not avoid federal question jurisdiction, knowingly or inadvertently, by failing to plead the federal statute.

In this case the claim is completely preempted by an express federal law and thus Petra has no claim based on the state statute.

Kola may therefore seek removal to federal court based on federal question jurisdiction.

The only remaining limitation is Kola’s failure to remove within 30 days and Dave’s failure to join in the removal action. Kola may be excused from the 30 day limitation because it was unknown initially that the case arose under a federal statute as opposed to the alleged state law basis.

Because federal juris is based on federal question and not diversity, not all the defendants must join in the removal. There Kola alone could remove.

A last limitation to removal is that a defendant may not seek removal if the case was initially filed in the state court of defendant’s residence. Here, that rule doesn’t apply because of federal question jurisdiction.

District Court’s Refusal to Remand to State Court

The federal district court with proper jurisdiction may refuse to remand a case to state

-3- court. In this instance, the federal court had federal question jurisdiction, thus properly retained jurisdiction.

II. Should a Federal Court of Appeals Hear Petra’s Appeal?

A federal court of appeals may only hear an appeal from a final judgment. A final judgment is one where all matters before the district court have been resolved by a final order. The only exception to this rule is for certain interlocutory appeals based on denial or granting of injunctive relief or failure to certify a class in a class action.

Here the denial of remand to state court was not a final judgment. The plaintiff still had ample opportunity to pursue his case in chief against Dave and Kola.

Upon final judgment, if Petra then loses [s]he may raise lack of subject matter jurisdiction of the federal court on appeal, because SMJ is never waived.

-4- Answer B to Question 1

ISSUE I: Did the federal district court rule correctly on Petra’s motion to remand?

In this case, plaintiff Petra (“Petra”) sued defendants Dave (“Dave”) and Kola (“Kola”) in state court, alleging violation of state W’s patent infringement law. Six weeks later, Kola removed the case to federal court, and Petra immediately moved to remand. The court denied Petra’s motion. At issue is whether this ruling denying the motion was proper.

Federal courts are courts of limited jurisdiction, and only have subject matter over cases that either (i) involve a question of federal law (statutory, constitutional, etc), or (ii) diversity jurisdiction exists. Cases that were originally filed in state court (like this case), can only be removed to federal court if (i) they could have originally been filed in federal court, (ii) all defendants agree, (iii) defendant is not a resident of the forum state, and (iv) removal is sought within 30 days of learning of the grounds for removal. It appears that the court was wrong on all of these grounds.

A. Could Case Have been Originally Filed in Federal Court?

This case likely could not have been filed in federal court, because there is likely lack of subject matter jurisdiction. First, there is no federal question subject matter jurisdiction (“SMJ”). Petra’s complaint is based on state patent infringement law. It is true that, as an affirmative defense, Dave and Kola will likely claim that Petra’s claims are pre-empted by the federal patent law. However, for federal question SMJ, the court looks to plaintiff’s well-pleaded complaint to determine whether a federal question is pled. Dave’s and Kola’s affirmative defenses - - even if they arise under federal law - - are irrelevant for federal question SMJ purposes. Because preemption by the federal patent law is an affirmative defense, it is irrelevant to federal question SMJ. Insofar as Petra’s complaint raises no federal question, there is no federal question SMJ.

Second, there are potential problems with diversity. As a rule, diversity jurisdiction exists where: (i) plaintiffs have diverse citizenship from EVERY defendant, and (ii) the amount in controversy is $75,000. It appears that Petra has met the second element: she has claimed damages of $50,000 from Dave and $70,000 from Petra. Amount in controversy is determined by (a) the amount pleaded in good faith in the complaint, and (b) plaintiff can aggregate her claims against multiple defendants to reach the amount in controversy threshold. Because Petra has claimed $120,000 in damages against both D’s combined, and we have no reason to suspect that this damages request was

-5- not made in good faith, Petra has met the amount in controversy requirement.

However, Petra may not be diverse from BOTH Dave and Kola. Dave’s residency: To be diverse from Dave, Petra and Dave must be residents of different states. Residency is determined by domicile - - where you live with intent to stay indefinitely. Although present living location is one factor, it may be offset by other factors that suggest that your current state is not your “domicile.” In the facts, we are told that (i) Petra is a resident of state W, and (ii) Dave lived in state X (and had a home in state X), and because of the contract, was required to relocated [sic] to State W. The issue is, assuming that state X was Dave’s domicile prior to the contract (and we have no facts to suggest otherwise, particularly because he owned a home in state X), did he change his domicile to state W?

Factors in favor of change of domicile: (i) he physically relocated to state W, and presumably got new living quarters; (ii) he moved all of his belongings to state W, suggesting that he was in it for the long haul; and (iii) this was not a short term project - - there are no facts to suggest that when Dave relocated to state W, he would only be there for a short time period. Although he quit his job after six months due to a contract dispute, this is not evidence that he had not intended to live in state W indefinitely. Factors against change of domicile: Dave retained his house in state X, and he only rented it out to someone else. This is strong evidence that Dave still considered state X his domicile, and even though he was moving out for a long period of time (as suggested by moving all of his belongings), there is no intent to change domicile. Conclusion: Dave is probably a resident of state X, because of lack of intent to change domicile. Factors that would help, but are not present, are: where is Dave registered to vote, driver’s license, etc. In the absence of more facts suggesting that Dave intended to live in state W indefinitely and make it his domicile, he should still be considered a resident of state X.

Kola’s residency: A corporation is a resident of two states: (i) its state of incorporation, and (ii) its state where its principal place of business is located. Moreover, principal place of business is defined differently by different courts, and can mean either (i) where its headquarters are located, or (ii) where its main manufacturing plants are located. In the facts, we are told that Kola is incorporated in state Y, and that its principal place of business is state W. Assuming that by “principal place of business” the facts mean that either Kola’s HQ or manuf. plants (as the case may be, depending on the jurisdiction) are located in state W, then Kola is a resident of BOTH state Y and W.

Mini-conclusion: There is no federal SMJ. In addition, there is no diversity jurisdiction, because of a lack of complete diversity between plaintiff and defendants: Petra is

-6- resident of state W and Kola is a resident of both states Y & W (and, Dave may be a resident of state W, but likely resident of state X). Because the case could not be originally brought in federal court, removal was improper, and the court should have granted the remand request.

B. All D’s must agree

In addition, all defendants must agree to a removal. We have no facts to suggest that Dave consented to the removal. If he didn’t, then removal was improper. If he did, this element is satisfied (but, still lose[s] because no jurisdiction). The case should have been remanded to state court as per Petra’s timely motion.

C. D cannot be resident of forum state

An additional reason for remand is that the defendant cannot be a resident of the forum state. Removal is a process to protect defendants against “hostile” foreign state courts. Here, Kola, and possibly Dave (though less likely, see above) are residents of state W. As such, removal of this case to state W federal court, with state W defendant(s), was improper. The case should have been remanded to state court as per Petra’s timely motion.

D. Motion must be brought within 30 days

A removal motion must be brought within 30 days of discovering the grounds for removal. In this case, Kola moved for removal 42 days after being served with the complaint. Assuming that Kola knew of the grounds for removal at the time it was served, its motion was untimely, and so the court should not have granted removal in the first place. (If Kola did not immediately know of the grounds, which is unlikely, then the original removal may have been timely, but case still should have been remanded because of lack of jurisdiction). The case should have been remanded to state court as per Petra’s timely motion.

CONCLUSION: The court erred when it denied the remand motion, because (i) no subject matter jurisdiction, and (ii) proper procedure not followed (all D’s didn’t agree, untimely motion, D’s resident of forum state).

ISSUE II: Should the Federal Court of Appeals entertain Petra’s appeal?

Typically, the federal court of appeals can only entertain appeals from final judgments - - i.e., from a judgment disposing of the matter, whether because of dismissal, grant of

-7- summary judgment, trial verdict, and the like. There are certain exceptions, however: the federal appeals court can hear certain interlocutory (i.e. not final) appeals involving grants of TRO’s and preliminary injunctions (and other pretrial remedies, e.g. attachment), collateral issues, as well as issues where the parties or court would be severely prejudiced - - or the right would no longer exist - - if they had to wait until final disposition to bring their appeal. In such an extraordinary case, where the parties or the court’s resources would be wasted, the court can use its inherent writ power to force the trial court to act.

This is one such case. A party can attack subject matter jurisdiction at ANY point in the proceedings - - even on appeal for the first time. Likewise, the court can raise SMJ at any point. If, at any point, the court discovers that it lacks subject matter jurisdiction, the case MUST be dismissed. Moreover, this is a good cause to use the extraordinary writ power, because Petra’s entitled to relief is [sic] clear.

In this case, it is conceivable that the parties could go through trial, never raising SMJ, and only on appeal the court discovers the issue and dismisses the case. This would result in a tremendous waste of judicial resources, a waste of the party’s resources and time, and could severely prejudice Petra’s ability to obtain relief, esp. if the proceedings are lengthy and there is a tremendous delay between now and when the SMJ problem is discovered. As such, the appellate court should entertain the appeal, either through its ability to award collateral relief, or more likely, through its inherent power to grant a writ of mandate in extraordinary circumstances.

-8- QUESTION 2

Olga, a widow, owned Blackacre, a lakeside lot and cottage. On her seventieth birthday she had a pleasant reunion with her niece, Nan, and decided to give Blackacre to Nan. Olga had a valid will leaving “to my three children in equal shares all the property I own at my death.” She did not want her children to know of the gift to Nan while she was alive, nor did she want to change her will. Olga asked Bruce, a friend, for help in the matter.

Bruce furnished Olga with a deed form that by its terms would effect a present conveyance. Olga completed the form, naming herself as grantor and Nan as grantee, designating Blackacre as the property conveyed, and including an accurate description of Blackacre. Olga signed the deed and Bruce, a notary, acknowledged her signature. Olga then handed the deed to Bruce, and told him, “Hold this deed and record it if Nan survives me.” Nan knew nothing of this transaction.

As time passed Olga saw little of Nan and lost interest in her. One day she called Bruce on the telephone and told him to destroy the deed. However, Bruce did not destroy the deed. A week later Olga died.

Nan learned of the transaction when Bruce sent her the deed, which he had by then recorded. Nan was delighted with the gift and is planning to move to Blackacre.

Olga never changed her will and it was in effect on the day of her death.

Who owns Blackacre? Discuss.

-9- ESSAY QUESTIONS AND SELECTED ANSWERS

FEBRUARY 2004 CALIFORNIA BAR EXAMINATION

This publication contains the six essay questions from the February 2004 California Bar Examination and two selected answers to each question.

The answers received good grades and were written by applicants who passed the examination. The answers were prepared by their authors, and were transcribed as submitted, except that minor corrections in spelling and punctuation were made for ease in reading. The answers are reproduced here with the consent of their authors and may not be reprinted.

Question Number Contents Page

1. Criminal Law and Procedure 1

2. Community Property 12

3. Professional Responsibility 22

4. Real Property 29

5. Constitutional Law 36

6. Civil Procedure 45

i Question 6

Paul and Tom, both State X residents, were involved in an auto accident in State X. At the time of the accident, Tom, who was working as a delivery truck driver for Danco, was driving through State X to make a delivery to a customer located in State Y. Danco is incorporated in State Y and has its principal place of business in State Z. State Z is located adjacent to State X. Danco does no business in State X.

Paul filed a complaint against Danco in federal district court in State X on the basis of diversity jurisdiction, alleging $70,000 in property and personal injury damages. Danco was properly served with the complaint at its principal place of business.

Appearing specially in the State X federal district court, Danco filed a motion to dismiss the complaint on the grounds that the district court lacked both subject matter and personal jurisdiction and that Paul’s action could not proceed without joining Tom. The district court denied Danco’s motion.

Danco then filed a counterclaim against Paul to recover $20,000 in property damage to the truck Tom was driving at the time of the accident. Paul moved to dismiss Danco’s counterclaim on the ground that the district court lacked supplemental jurisdiction to hear the counterclaim. The district court granted Paul’s motion.

State X law provides that its courts may exercise jurisdiction over nonresidents “on any basis not inconsistent with the Constitution of the United States.”

1. Did the district court rule correctly on Danco’s motion to dismiss Paul’s complaint? Discuss.

2. Did the district court rule correctly on Paul’s motion to dismiss Danco’s counterclaim? Discuss.

45 Answer A to Question 6

6)

Question 6

(1) Motion to Dismiss Paul’s Complaint

Personal Jurisdiction (PJ):

Personal jurisdiction refers to the court’s power to bind the person of the defendant. The traditional basis [sic] of personal jurisdiction are (1) domicile; (2) personal service in state; and (3) consent - either expressly through a forum clause or impliedly by failing to raise lack of PJ in your first response to the court. Paul filed a complaint against Danco in federal district court in State X. Danco denies that State X has personal jurisdiction over it. Danco is a corporation which is incorporated in State Y and has its principal place of business in State Z. Therefore, Danco’s residence would be considered State Y and Z.

Due Process:

To have personal jurisdiction over a defendant who is not a resident of the forum, the forum state must have a long arm statute and meet the requirements of International Shoe to meet due process requirements. To have personal jurisdiction, due process also requires that defendant be given notice and have the opportunity to be heard. Defendant must be served with the summons and complaint within 120 days of filing of the complaint. In this case, Danco was properly served with the complaint at its principal place of business.

Long Arm Statute:

A long arm statute is a statute that allows the state to assert jurisdiction. States may have specific or nonspecific long arm statutes. State X has a long arm statute that provides that its courts may exercise jurisdiction over nonresidents “on any basis not inconsistent with the Constitution of the United States.” This is a nonspecific long arm statute because it does not specific[y] the circumstances under which the forum may exercise personal jurisdiction. Therefore the court may exercise jurisdiction to the limits allowed by due process.

International Shoe:

To meet the test in International Shoe, the forum must show that defendant has such minimum contacts with the forum that assertion of personal jurisdiction would not offend traditional notions of fair play and substantial justice.

46 Minimum Contacts:

To have minimum contacts, the courts will analyze the (1) D’s purposeful availment of the forum; and (2) D’s foreseeability of a lawsuit.

Purposeful Availment:

In analyzing purposeful availment, the court will consider (1) the nature and quality of D’s actions; (2) voluntary acts of D directed at the forum; (3) whether D intentionally placed a good in the stream of commerce; and (4) where injury is show[n], jurisdiction is established. Here, Danco does no business in State X. However, at the time of the accident Danco’s driver was driving through State X to make a delivery to a customer located in State Y. Danco is incorporated in State Y and has its principal place of business in State Z. State Z is located adjacent to State X. Although Danco does not do any business directly in State X, it appears that Danco must make regular use of State X’s roads to conduct its business. Also, Paul was injured by a Danco driver in an accident in State X. Therefore, it appears that Danco did purposefully avail itself of State X.

Foreseeability of Lawsuit:

The court must also determine whether Danco could reasonably foresee that its actions could lead to a lawsuit, i.e., it being ha[u]led into court in State X. It appears that Danco drivers regularly traveled State X’s roads to conduct business. Therefore, it would be reasonable for Danco to foresee that one of its drivers may get into an accident while in State X and cause damage.

Traditional Notions:

The court must balance the minimum contacts of defendant against traditional notions of fair play and substantial justice. This means that the court will look at (1) the relatedness between the claim and D’s conduct; (2) P’s interest in obtaining relief; (3) D’s burden v. benefit; and (3) the state interest. Here, Danco’s driver drove through State X and this conduct lead [sic] to the car accident, P has a high interest in seeking relief for his injuries and property damage, D benefits from being able to drive on State X roads and it would not be a heavy burden to require D to be responsible for any accidents which this may cause, and finally State X has a strong interest in holding drivers who cause accidents on its roads, especially to State X citizens, responsible.

Conclusion: The district court was correct in its decision to deny D’s motion because State X may assert PJ over D.

Subject Matter Jurisdiction:

Subject matter jurisdiction refers to the court’s power to hear the kind of claim being

47 brought. P filed a suit against D on the basis of diversity jurisdiction, alleging $70K in property and personal injury damage. For diversity jurisdiction, plaintiff must show that (1) amount in controversy (AIC); and (2) complete diversity.

AIC:

To meet the AIC requirement, plaintiff must have a good faith claim exceeding $75K. Here, P is only seeking $70K. Therefore, he has not satisfied the AIC requirement. If P were seeking some sort of injunction, the value of the injunction could be added to the AIC requirement. However, it does not appear that P is seeking an injunction. Therefore, P has failed to satisfy the AIC requirement.

Complete Diversity:

Complete diversity requires that no plaintiff and defendant be from the same state. This will depend on where the parties were domiciled at the commencement of the lawsuit. P was domiciled in X. As discussed above, D was domiciled in Y and Z. Therefore, there appears to be complete diversity.

Conclusion: The court erred in denying D’s motion as to lack of SMJ. State X does not have SMJ to hear this claim because P has not satisfied the AIC requirement. Also, the federal court does not have any other SMJ over this case because it does not involve a federal question (it is a personal injury action) and it is not a matter within the federal court’s exclusive jurisdiction.

Joinder:

P claims that the matter cannot proceed without joining Tom. Under compulsory joinder of parties, the court will first look to see if the party is a necessary party. A party is necessary where the court cannot afford complete relief without the party or there is a danger that the absentee will be harmed, there may be an inconsistent judgment or there may be a possibility of double liability. Here, it is arguable whether Tom is a necessary party because although he may be liable to Danco for the accident, P may get a judgment solely against D for the accident because Tom was an agent of D when the accident occurred and because the accident was within the scope of Tom’s employment, D will be liable for Tom’s negligence.

However, if Tom is a necessary party, the court will next determine whether he is an indispensable party. An indispensable party is one whose joinder will destroy diversity. Here, Tom’s joinder will destroy diversity because Tom is also a State X resident and this would destroy complete diversity because P is also from State X. Where the party is indispensable, the court may dismiss the case or proceed without the party. The factors the court will use to determine that are the following: (1) alternative forum; (2) likelihood of prejudice; (3) chance of inconsistent judgment. Here, State X appears to be the best forum

48 for the case because the claim arose here and it would be highly inconvenient to require P to travel to State Y or Z. Also, there is not a high chance of prejudice because State X will likely fairly administer its laws. There is also not a chance of inconsistent judgment because as discussed, P can sue D alone for her damages. Therefore, the court may continue the case without joining Tom.

Conclusion: The court was correct in denying D’s motion for failure to join. Had the court had SMJ, it could proceed with the case without joining Tom.

(2) Motion to Dismiss Danco’s Counterclaim:

D filed a counterclaim against P to recover $20K in property damage to the truck Tom was driving at the time of the accident. Paul moved to dismiss D’s counterclaim on the ground that the district court lacked supplemental jurisdiction to hear the counterclaim.

Supplemental Jurisdiction:

Where the court has original jurisdiction over a matter, the court may also assert supplemental jurisdiction over other claims that are so related that they form the same case or controversy as the original claim. The same case or controversy means that the claims arose out of the same transaction or occurrence and arise out of a common nucleus of operative facts.

Same transaction/occurrence:

D is bringing a counterclaim to recover for property damage it suffered in the accident between P and Tom. The initial claim by P is for damages suffered as a result of the accident between P and Tom. Therefore, the counterclaim arises out of the same transaction or occurrence as the original claim.

Common Nucleus Operative Facts:

As discussed above, D’s counterclaim relates to the accident between P and Tom and P’s initial claim is for the same accident. Therefore, the counterclaim arises out of the same common nucleus of operative facts.

Counterclaim:

In cases where a counterclaim arises out of the same transaction or occurrence as the original claim, the counterclaim is considered compulsory and must be brought or it will be waived. Here, D had to assert the counterclaim or it would have been waived because the counterclaim arose out of the same transaction or occurrence. As discussed above, where a counterclaim is compulsory because it arises out of the same transaction or occurrence, the court will assert supplemental jurisdiction. The claim need not have an

49 independent basis for SMJ.

Conclusion: The court erred in granting P’s motion because the district court had supplemental jurisdiction to hear the counterclaim.

50 Answer B to Question 6

6)

Personal Jurisdiction in Federal Courts

Personal jurisdiction is the court’s power over the individuals in the case: the power to compel them to appear and to bind them to its judgment. The federal court[‘]s personal jurisdiction applies to state law (of the state it’s in) regarding domicile of the defendant, where the defendant was served (whether in state or not), and whether the defendant consented, either expressly or impliedly, to the jurisdiction of the court.

A corporation is a resident of every state in which it is incorporated and the state of its principal place of business.

Here, Danco (D) was incorporated in Y and its principal place of business is in Z. Thus, it is not domiciled in X. D was served in Z.

D filed a motion challenging personal jurisdiction pursuant to rule 12b prior to filing an answer. A 12b motion can allege, inter alia, improper personal jurisdiction, subject matter jurisdiction, process, service of process, as well as failing to state a claim upon which relief can be granted. By filing a 12b motion challenging personal and subject matter jurisdiction, a party does not consent to that jurisdiction by the appearance. Thus D did not consent to personal juris in X by filing the 12b motion.

Minimum Contacts

Personal juris may also be had over a defendant if he had minimum contacts w/ the forum state. The minimum contacts test states that the exercise of personal jurisdiction cannot offend traditional notions of fair play and substantial justice and must be reasonable. In applying this test, the court will look to whether the defendant had systematic and continuous presence in or contact w/ the forum state; whether the cause of action arose in the forum state; whether the defendant could reasonably have foreseen being sued in and being subjected to personal jurisdiction in the forum state; and whether the defendant purposefully availed himself of the privilege of doing business in the forum state.

Here, D’s delivery driver was driving through the forum state, X, in order to make a delivery in Y. However, D does no business in X. Furthermore, the facts do not indicate any contact by D w/ X except this driver driving through X to go to Y. While D is incorporated in Y, the facts do not indicate a large amount of business w/ Y requiring D’s employees to regularly cross through X. On the facts given, D has had 1 contact w/ X. This is not systematic and continuous contact. However, the cause of action arose in X. If D’s trucks were in X at all (which they were on at least 1 occasion), D could foresee an

51 accident requiring it to defend a lawsuit in X. D didn’t purposefully avail itself of doing business in X, but it did purposefully avail itself of the use of the roads of X. And not just a little bit of roadway use, but D’s driver was going all the way through X to get to Y. This is a close call, but given that the accident occurred in X and that D’s truck was purposefully driving through X it would not offend traditional notions of fair play and substantial justice to subject D to personal jurisdiction in X.

State Z abuts X. Thus, it would be convenient for D to defend the suit in X. Also, X has a strong interest in protecting its citizens from injuries and negligent drivers. In addition, it would be easier for a corporation (with more assets and personnel) to defend in the neighboring state than it would be for an individual (P) to prosecute the claim in another state. Thus, it is reasonable to subject D to personal juris in X.

Because D meets the minimum contacts requirements, the court had proper personal jurisdiction over D and this part of D’s motion should be denied.

Subject Matter Jurisdiction

Subject matter jurisdiction is the court’s power of the subject matter of the lawsuit. In federal court, subject matter jurisdiction can be based on a federal question properly plead [sic] in the complaint or on diversity jurisdiction. For diversity jurisdiction to be proper, there must be complete diversity (all plaintiffs diverse from all defendants) and the plaintiff must in good faith (subject to Rule 11) plead damages of more than $75,000. (Diversity is where 1 plaintiff resides in a different state from 1 defendant.)

Here, P resides in X. As stated above, D resides in Y and Z. Thus, there is complete diversity. However, P only alleged $70,000 in damages in his complaint. This does not meet the $75,000 minimum. The fact that D counterclaimed for $20,000 doesn’t matter; the 2 can’t be added to cross the $75,000 minimum. Thus, the court does not have subject matter jurisdiction over the case. That part of D’s 12b motion should be granted.

Compulsory Joinder/Indispensable Parties

An indispensable party is one which a current party alleges must be included in the case 1) to grant complete relief; or 2) because the current party’s interests would be prejudiced if it was forced to defend the case w/o the indispensable party. The current party can force the indispensable party to join the case through compulsory joinder. By doing so, the current party is alleging the indispensable party is the one responsible to the plaintiff (not the current party). First, the current party must meet 1 of the above 2 requirements. Second, the joinder of the indispensable party cannot destroy diversity in the case. The rationale for this requirement is that defendant should not be allowed to torpedo the plaintiff’s proper diversity jurisdiction by bringing in a non-diverse party.

Here, D wants to join T. T is an employee of D. Through the doctrine of respondeat

52 superior, D can be held liable for T’s actions that were in the scope of and during the course of T’s employment. Thus, whether T is joined or not, P will be suing D and attempting to collect his judgment (should he win) against D, the party with the deep pockets. Complete relief can be granted to P w/o T’s presence. D is not going to sue its own employee and obtain relief from him. D may need T as a witness in the case, but it will suffer no damage if T is not a party to the case. Furthermore, T is a state X resident. By joining T, D would destroy diversity because P is a state X resident. Thus, the court should deny D’s motion regarding joinder of T.

Counterclaims

A counterclaim is when the defendant asserts a claim against the plaintiff that is suing him. Compulsory counterclaims are claims against the suing party that arise out of the same conduct, transaction, or occurrence. Compulsory counterclaims must be plead [sic] or the claim is lost. (The defendant cannot sue on that claim later as a plaintiff.)

Here, D alleges that P damaged its truck as a consequence of the same accident P is suing for. This is the same transaction and occurrence. Thus, D’s counterclaim is compulsory.

Supplemental Jurisdiction

Supplemental jurisdiction is the federal court’s power to hear cases associated with the main claim (the plaintiff’s claim which must meet all jurisdictional requirements) even though the associated claims may not meet all jurisdictional requirements. For a plaintiff w/a valid federal case, the federal court can hear a plaintiff’s state claim if it comes from the same common nucleus of operative facts and has a common question of law or fact. Supplemental jurisdiction also covers a state law claim by the defendant against the plaintiff if the defendant’s claim arose out of the same conduct, transaction, or occurrence. In a diversity case, supplemental jurisdiction includes compulsory counterclaims. The rationale is that it would not make sense to make a defendant sue in state court on a claim that arose from the same conduct, transaction, or occurrence for which the plaintiff is suing in federal court. It would help the parties and serve judicial economy to hear both claims at one time.

Here, D’s counterclaim is compulsory. Thus, the federal court has supplemental jurisdiction to hear that claim.

However, P’s claim will be dismissed from federal court due to D’s 12b motion, as above. Once that happens, the federal court will not hear D’s counterclaim because it is no longer associated w/a plaintiff’s valid complaint. D’s counterclaim would have to meet its own jurisdictional requirements, which it does not. So the court will, after dismissing P’s claim, dismiss the whole case.

53 ESSAY QUESTIONS AND SELECTED ANSWERS

FEBRUARY 2006 CALIFORNIA BAR EXAMINATION

This web publication contains the six essay questions from the February 2006 California Bar Examination and two selected answers to each question.

The answers received high grades and were written by applicants who passed the examination. Minor corrections were made for ease in reading. The answers are reproduced here with the consent of their authors.

Question Number Contents Page

1. Torts 1

2. Wills and Succession 13

3. Real Property 21

4. Civil Procedure 28

5. Contracts/Remedies/Professional Responsibility 40

6. Criminal Law and Procedure 48

i Question 4

Pat, a resident of State A, received a letter from Busco, a tour bus company that had been in business for about two months. Busco was incorporated and had its principal place of business in State B. The letter invited Pat to go on a tour of State C at a special introductory price. After Pat sent in her money, Busco sent Pat a tour brochure and ticket.

Ed, also a resident of State A, saw an ad that Busco had placed once a week for the last several weeks in Ed’s hometown newspaper for the same State C tour. The ad listed a State A telephone number to call for tickets. Ed called the telephone number and ordered and bought a ticket for the same tour as Pat and for the same price.

Pat and Ed boarded the tour bus in State B. Upon entering State C, the bus veered off the road and hit a tree. Ed was not hurt, but Pat suffered serious injuries. The tour was canceled. Busco refused to reimburse passengers the price of their tickets.

Ed sued Busco for breach of contract in state court in State A to recover the price of his ticket. Busco moved to dismiss the suit based on lack of personal jurisdiction. The court denied the motion. After trial, judgment was entered in favor of Ed.

Thereafter, Pat sued Busco in state court in State A for breach of contract to recover the price of her ticket and for tort damages for her personal injuries. After Busco filed its answer, Pat filed a motion for summary judgment on both claims on grounds of res judicata and collateral estoppel. The court denied Pat’s motion.

State A has a long-arm statute that authorizes the exercise of personal jurisdiction over nonresident defendants on any basis not inconsistent with the Constitution of the United States.

1. Did the court rule correctly on Busco’s motion to dismiss Ed’s suit for lack of personal jurisdiction? Discuss.

2. Did the court rule correctly on Pat’s motion for summary judgment on each of her claims on grounds of res judicata and collateral estoppel? Discuss.

Answer A to Question 4

28 4)

I. The Court CORRECTLY Ruled in Finding Personal Jurisdiction Over Busco in State A.

A. An Overview of Jurisdiction

In order for a court, be it state or federal, to gain jurisdiction over an individual or entity (such as Busco (“B”)), it must either assert in personam jurisdiction (jurisdiction over the person or entity itself), in rem jurisdiction (jurisdiction over property within the jurisdiction in which the court has authority and is related to the cause of action), or quasi-in rem jurisdiction (jurisdiction over property within the jurisdiction which the court sits but is not related to the cause of action). It must comply with due process requirements of state and federal constitutions such that a defendant is not called into a court in a jurisdiction in which it would be deemed to be unfair or unforeseeable.

B. In Personam Jurisdiction Requirements

Here, the Court in State A, upon Ed (“E”) bringing his claim in State A, asserted jurisdiction over B using in personam jurisdiction. In personam/personal jurisdiction comes in two types: general and specific.

1. General Personal Jurisdiction

A court can exercise general personal jurisdiction over a defendant when it has systematic and continuous contacts with the forum. This requires more than just drawing on a bank account in the forum, or communicating with entities or individuals in the forum. The defendant’s involvement must be intentional. The reason why the standard for contact here is so strict is that the [sic] under general jurisdiction, a court can take jurisdiction over a defendant even if the cause of action is completely unrelated to its contacts with the forum. Thus, the law has been crafted such that a defendant is deemed to have subjected itself to the laws of the forum, and benefited from their protection, to such an extent, as to be susceptible to suit there[,] no matter where the cause of action arises.

Without knowing what grounds upon which the Court in E’s suits against B used to sustain personal jurisdiction, I must assume that general jurisdiction was not the grounds upon which the court relied. B had only been in business for a few months, and having been incorporated in State B, as well as having its principal place of business (“PPB”) in state B, it’s unlikely to have “systematically and continuously” availed itself to State A.

The US Sup Ct has also shown that general jurisdiction can also be attained over a defendant in several more “traditional” ways. For instance, if the defendant was served with process in the jurisdiction (absent fraud or coercion), if the defendant consented to jurisdiction (either expressly or by failing to timely raise an objection to jurisdiction), or if a

29 defendant is deemed to have consented constructively by nominating an agent to accept service. In this case, B meets none of these standards, assuming it challenged jurisdiction in a timely fashion.

2. Specific Personal Jurisdiction

The more likely way the Court in E’s claim asserted jurisdiction over B in this case is through the use of specific personal jurisdiction. Although not available historically at common law (only general jurisdiction was available), this developed in the face of a more mobile and economically integrated society. Under this doctrine, specific personal jurisdiction requires two elements. First, jurisdiction must be permitted under state law, which usually means it must fall under a state long-arm statute, permitting courts within the state to “reach out” of the forum to grab defendants for suit within the forum’s courts. As we’re told, State A has a long-arm statute (similar to that of California’s) which permits jurisdiction to the same extent that the due process requirements of the US Constitution will allow. Thus, this takes [us] two [sic] our second requirement: the jurisdiction is permitted if the due process requirements are satisfied pursuant to the US Constitution.

Under due process, a defendant’s contacts need not be “systematic and continuous” to sustain jurisdiction. However–and this is a major caveat–the cause of action must be related to the defendant’s contacts with the forum. What contacts are required? Specific personal jurisdiction requires the satisfaction of two elements: 1) the defendant must have minimum contacts with the forum, and 2) exercising jurisdiction must not offend traditional notions of fair play and substantial justice.

A. Minimum Contacts

First, for the Court in state A to get jurisdiction over B, it must show that B had minimum contacts with state A. This requires an investigation into whether B purposefully availed himself of state A and whether it would be foreseeable for B to be haled [sic] into court there.

Under these facts, B, although incorporated in state B, had reached out to state A by placing an advertisement in the newspaper in that state, by providing a phone number for folks in that state to call to order tickets, and purposefully solicited patronage from state A. These facts are not all that different from the facts in the Asahi case in which the US Sup Ct found jurisdiction when a company provided phone numbers for customers and advertised in the forum. B was taking advantage of the laws of state A by directing its business to that state, and thus it was quite foreseeable that it would be haled [sic] into court there. Thus, minimum contacts are likely satisfied here.

B. Fair Play and Substantial Justice

30 Second, it must also be fair to exercise jurisdiction over B, even if B had minimum contacts with state A. This requires an investigation into several factors, including the interests of state A in protecting its citizens from “foreign” tortfeasors, the interests of the plaintiff in being able to seek the protection of the laws of his domicile, the fairness accorded to B in forcing him to a different state to defend himself, and administrative details associated with litigating in state A, such as the location of witnesses, etc. Here it’s not entirely clear how far State B or State C is from State A, but it’s likely quite close in distance. Moreover, State A has a significant interest in defending its citizens from foreign tortfeasors, or in the case of E, contract breachers. Granted, because the price of the contract is likely not great, it’s possible the interest here is not all that significant. However, on the other hand, there is likely to be little unfairness in pulling B into State A, so I would conclude that it comports with traditional notions of fair play and substantial justice to find jurisdiction here. Again, these facts are quite similar to [a case] where the Sup Ct did find jurisdiction. Granted, there were forum non conveniens issues in that case, but an international defendant was involved there – which is not the case with B.

Finally, a note should be made again about the action arising out of the contacts with the forum. Because this is specific personal jurisdiction (ie, not general), it requires that the minimum contacts with the forum be related to the cause of action. Here, B’s contacts with the forum–its attempt to get business from E–are directly related to the breach of contract action.

II. The Court did NOT Rule 100% Correctly on Pat’s SJ Motion[.]

When Pat (“P”) brought claims against B, [s]he brought two causes of action: breach of contract and tort damages. Although the former had already been litigated, the latter had not yet been litigated. This makes a significant difference when applying rules of Res Judicata and Collateral Estoppel.

A. Res Judicata

In order to invoke the doctrine of Res Judicata (“RJ”), three elements must be satisfied. First, the claim at issue must be related to the same transaction or occurrence of a previously litigated claim. Second, it must involve the same parties as the previously litigated claim. Finally, the previously litigated claim must have resulted in a final judgment on the merits.

Here, although the contract claim that P brings against B is the same as that brought against B by E, and although there was a final judgment on the merits, P was not [a] party to the earlier litigation. This means that RJ is not applicable to P’s claims. In other words, collateral estoppel (below) is all that [s]he has available to try to sustain [her] summary judgment motion.

B. Collateral Estoppel

31 The doctrine of Collateral Estoppel (“CE”) requires 5 elements. First, it must involve a previously litigated issue of law or fact. Second, the previously litigated issue must have been actually litigated, and third, it must have been litigated to a final judgment on the merits. The issue must have been a central (non-collateral) issue in the previous litigation, and at least one of the parties from the previous litigation must be present.

1. P’s Tort Claim

First, here the issues that P would like to assert CE over pertain to the liability of B to its patrons resulting from the bus accident. Although the issue of liability resulting from B’s breach of contract for not refunding the ticket price has been litigated to a final judgment on the merits by one of the parties, the issue pertaining to tort liability has not! Thus, so far as P’s summary judgment motion regarding B’s liability for P’s injuries, the court did rule correctly because the issue of B’s liability had not been previously litigated (it wasn’t even an issue in E’s litigation against B). Thus, showing that B was negligent, or that the driver was drunk, or whatever the tort claim may rely upon must still be shown by P. Given that P brought his [sic] SJ motion immediately following B’s answer [s]he [is] asking the court to find that there is no material issue of disputed fact with regard to B’s tort liability. Obviously, without affording B an opportunity to provide evidence to the contrary, and without P fulfilling his [sic] prima facie obligations as to his negligence claim, the Court ruled correctly here.

2. P’s Breach Claim

On the other hand, P’s second claim, his [sic] action for breach of contract was actually litigated in the previous action by E against B, and it would satisfy all of the above requirements for CE. However, there are two additional requirements that must be noted.

First, the party against whom CE is being asserted must have been the party to the previous action. This is satisfied here–B was a party to E’s litigation.

Second, there’s a question as to whether there must be mutuality. Traditionally, both parties had to have been party to the earlier litigation for CE to apply. However, modernly this is changing. The US Sup Ct has recognized that non-mutual defensive CE can be used quite easily so long as the other requirements of CE are satisfied. What that means is that, if B had won in his earlier litigation against E because the trier of fact had found that there was no breach (for example), then B would be entitled to assert CE against P’s claim here. The more questionable assertion of non-mutual CE is when it’s used offensively, as P is attempting to do here. Although this is less likely to be permitted, courts have begun to permit it more often so long as the defendant had an opportunity to litigate the issue competently in the previous case, it was foreseeable to the defendant that CE may arise in the future from the issue, that it’s fair to the defendant, and that the plaintiff who’s trying to assert CE could not have been joined in the previous litigation.

32 In applying this to the facts before us, it’s likely that the court in State A decided that because P could have brought his[sic] contract action at the same time as E, CE should not apply. If that’s the case, then the court was correct. On the other hand, it seems as though, for judicial efficiency[’s] sake, the court could have at least granted CE on the breach of contract claim in this case, leaving the tort claim to go to the trier of fact.

33 Answer B to Question 4

4)

1. Did the court rule correctly on Busco’s motion to dismiss Ed’s suit for lack of personal jurisdiction?

Personal jurisdiction

Personal jurisdiction (“PJ”) refers to the power of the court to render a judgment that will be binding on the defendant. The exercise of PJ is proper if it is authorized by statute and does not violate Due Process.

Traditional bases

States usually have a PJ statute that authorizes personal jurisdiction where the defendant (1) is domiciled in the forum state[,] (2) is personally served with process while physical present in the forum state[,] or (3) expressly or impliedly consents to jurisdiction in the forum state. A corporation is domiciled in any state in which it is incorporated and in which it has its principal place of business.

Here: (1) defendant Busco was incorporated in and had its principal place of business in State B. It was therefore not domiciled in State A. (2) No evidence suggests that any representative of Busco was personally served while physically present in State A. (3) Ed might argue that Busco consented to jurisdiction when it appeared in State A court, but this argument will fail if Busco’s appearance was specially limited to the sole purpose of contesting the court’s PJ over it.

Thus, no traditional bases for PJ are present.

Long-arm statute

State A’s long-arm statute provided for PJ over any non-resident defendant if such PJ is not inconsistent with the Constitution of the United Sates. The issue therefore becomes whether State A’s exercise of PJ was constitutional.

Constitutional limitations on personal jurisdiction

The Due Process clause of the Fourteenth Amendment limits a state’s power to exercise PJ over a non-resident defendant to those cases where (1) the defendant has minimum contacts with the state[,] and (2) exercising PJ would not offend traditional notions of fair play and substantial justice. International Shoe v. Washington.

Minimum contacts

34 Minimum contacts analysis focuses on (1) whether the defendant’s contacts with the forum were systematic and continuous (in which case the state has general personal jurisdiction over the defendant and the defendant is subject to PJ in the state for any act[)]; (2) whether the defendant purposefully availed itself of the benefits and protections of the laws of the state; and (3) whether the defendant could foresee being haled into court in the state.

Here, Busco’s contacts with State A were the following. It sent a letter to Pat, a State A resident, inviting Pat to go on a tour of State C at a special introductory price. After Pat sent in her money, Busco sent to Pat in State A a tour brochure and ticket. Busco also placed an ad once a week for several weeks in a hometown newspaper in State A advertizing the same tour. The ad listed a State A telephone number to call for tickets. Ed called the telephone number and ordered and bought a ticket for the tour. Although it is not stated, Busco probably also sent Ed a ticket to his residence in State A.

(1) Systematic & continuous

General jurisdiction is found where the contacts are systematic and continuous. Here, Busco is a State B corporation which has only been in business for two months. It has placed an ad several times in a State A newspaper. It has sent tickets to two State A residents. It sent a letter to a State A resident. These sporadic and short-term contacts are not the sort of continuous activity sufficient to find general jurisdiction.

(2) Purposeful availment

The next issue is whether Busco purposely availed itself of the benefits and protections of State A law.

Ed would point out that Busco deliberately placed an ad in a State A newspaper. This was a contract with the newspaper and was likely governed by the contract law of State A, on which Busco would rely for protection if the newspaper were to breach the agreement. Moreover, Busco maintained a State A telephone number for potential customers to call. This involved contracting with a telephone service provider in State A and again Busco would have availed itself of the protections of State A law in negotiating this agreement. Finally, after receiving an order from two State A law residents, Busco sent tickets to both Ed and Pat in State A. These were contracts and again would likely have been governed by State A law.

On the other hand, Busco did not conduct its business in State A - the tour began in State B and went to State C. The state A telephone number might simply have connected to a call center in State B. Busco also has only been in business for two months so there has not been much opportunity for purposeful availment.

On balance, Busco did purposely avail itself of the benefits and protections of State A law.

35 (3) Foreseeability

The third element of minimum contacts is whether the defendant should reasonably have foreseen being haled into court in State A.

Ed will argue that, based on the analysis under ‘purposeful availment’, Busco should have foreseen the possibility of a contract dispute based either on the ad, the telephone number, or the tour contracts with Pat and Ed. Busco knew that these contracts were negotiated with State A entities and that there was a strong likelihood that any dispute might be litigated in State A. Moreover, Busco was offering a tour service which involved the possibility of causing personal injury to tour participants if there was an accident. Busco knew that at least two persons on the tour were State A residents, and thus should have foreseen that any tort suit they brought might well be brought in their state of residence, State A.

Busco will counter that it was domiciled in State B and that any contract actions would probably have been brought there. Further, the tour never visited State A, so tort suits in State A were unforeseeable.

On balance, however, it was reasonably foreseeable that Busco might have been sued in State A.

Fair play and substantial justice

Even if minimum contacts are found, personal jurisdiction is only proper if it does not offend traditional notions of fair play and substantial justice. The court will consider three factors here: (1) the relation of the contact and the claim[;] (2) the convenience of the parties[;] and (3) the forum state’s interest in providing a forum for resolving the dispute.

(1) Relation of contact and claim

Personal jurisdiction is more likely proper if the claim arose out of the contact with the forum state. Here, Ed is claiming for breach of contract for the tour. This contract was entered into as a direct result of Busco’s placing an ad in Ed’s State A hometown newspaper. Thus, this element is met.

(2) Convenience of parties

The court will not impose personal jurisdiction where requiring the defendant to defend in the forum would impose an unreasonable burden on the defendant.

Busco would argue that the witnesses to the formation of Ed’s contract are its employees in its State B principal place of business, that the records relating to the contract are there, and that it would be unreasonable to require Busco to produce these in State A.

36 Ed would counter that Busco is a corporation which can surely spare a few employees for the limited purpose of testifying. Further, much evidence is in State A - the newspaper in which the ad ran, the telephone number through which Ed placed his order, and the tickets.

Since the inconvenience to Busco is not extreme, the convenience of parties favors State A.

(3) State A’s interest in providing a forum

The forum state must have an interest in providing a forum for the dispute.

E will assert that he is a resident of State A and negotiated a contract from his residence in State A using a State A telephone number after seeing an ad in a State A newspaper. This contract action will probably be governed by State A law. State A has a strong interest in providing a forum for its residents to obtain damages.

Busco will argue that the action likely raises no new issues of contract law, and since no new law is to be made, State A has little interest in having the issue litigated there.

On balance, State A’s interest favors PJ in State A.

Conclusion

In view of the factors in favor of and against finding PJ, the court probably was correct to deny Busco’s motion to dismiss for lack of personal jurisdiction.

2. Did the court rule correctly on Pat’s motion for summary judgment on each of her claims on grounds of res judicata and collateral estoppel?

Summary judgment

Summary judgment is a ruling that the moving party is entitled to judgment as a matter of law. It is proper where there is no triable issue of material fact, and, after viewing the evidence most favorably to the non-moving party, the court concludes that no reasonable trier of fact could find in favor of the non-moving party.

Here, Pat will argue that Busco’s previous action against Ed should result in judgment as a matter of law for Pat on both her contract and tort claims on theories of res judicata and collateral estoppel. Each claim will be examined in turn.

Breach of contract claim

Res judicata

37 Res judicata, or claim preclusion, bars relitigation of an action (1) by the same plaintiff against the same defendant (or their privies) (2) when the previous action ended in a final judgment on the merits and (3) the previous action involved the same claim (it arose out of the same transaction or occurrence, or series of transactions or occurrences).

Here, Busco will argue that Pat’s contract action related to a different contract from the one negotiated by Ed because the parties were different. Pat will argue that it was the same contract because the terms and the price were the same. Ed’s action ended in a final judgment on the merits because after trial, judgment was entered in favor of Ed. But the earlier lawsuit was between Ed and Busco, and this claim is between Pat and Busco.

Since the plaintiff is not the same in each case, res judicata will not apply.

Collateral estoppel

Collateral estoppel, or issue preclusion, bars relitigation (1) of the same issue (2) against a party to the previous action (3) when the issue was actually litigated[,] (4) the resolution of the issue was essential to the judgment[,] (5) and the previous action ended in a judgment on the merits.

Here: (1) the issue of whether Busco’s refusing to reimburse the tour passengers the price of their tickets after the tour was cancelled was a breach of contract is the same issue in Pat’s case as in Ed’s, because both likely had the same contract with Busco for the tour, and both were on the same bus. (2) Busco was a party to the previous action by Ed. (3) The issue of Busco’s breach was actually litigated in Ed’s action and (4) was essential to the judgment, because Ed could not have won his contract suit without a finding that Busco’s refusal to reimburse was a breach of contract. (5) The judgment was on the merits because after trial, judgment was entered in favor of Ed.

Non-mutual offensive collateral estoppel

Since Pat was not a party to the previous action, traditional mutuality rules should bar her use of collateral estoppel. But modernly, courts will allow non-parties to use collateral estoppel against parties to a prior action because mutuality is not required by Due Process. Use of non-mutual collateral estoppel against a defendant (‘offensive’) is permissible under Parklane Hosiery where the defendant had a full and fair opportunity to litigate the issue in the prior action and the forum of the previous action did not unfairly limit the defendant’s litigation strategies or use of evidence.

Here, Busco was a party to the prior action, and had the same opportunity and motive to argue that its actions were not a breach of contract against Ed as it had to argue this against Pat. Both actions were brought in State A court so the forum rules of litigation and evidence were the same.

38 Conclusion

Since offensive collateral estoppel is allowed under these circumstances, the court incorrectly denied Pat’s motion for summary judgment on her contract claim.

Tort claim

Res judicata

For the same reasons as the breach of contract claim, res judicata will not apply to the tort claim.

Collateral estoppel

The issue of Busco’s tort liability for the accident when the bus hit a tree was not actually litigated in Ed’s action, which was solely for breach of contract because Ed was not hurt. Accordingly, collateral estoppel will not apply to Pat’s tort action.

Conclusion

The court correctly denied Pat’s motion for summary judgment on the tort claim.

Question 5

Marla is a manufacturer of widgets. Larry is a lawyer who regularly represents Marla in legal matters relating to her manufacturing business. Larry is also the sole owner and

39 THE STATE BAR OF CALIFORNIA OFFICE OF ADMISSIONS 180 HOWARD STREET • SAN FRANCISCO CALIFORNIA 94105-1639 • (415) 538 - 2300 1149 SOUTH HILL STREET • LOS ANGELES CALIFORNIA 90015-2299 • (213) 765 -1500

ESSAY QUESTIONS AND SELECTED ANSWERS FEBRUARY 2009 CALIFORNIA BAR EXAMINATION

This publication contains the six essay questions from the February 2009 California Bar Examination and two selected answers to each question.

The answers received good grades and were written by applicants who passed the examination. The answers were prepared by their authors, and were transcribed as submitted, except that minor corrections in spelling and punctuation were made for ease in reading. The answers are reproduced here with the consent of their authors.

Question Number Contents Page

1 Professional Responsibility 3

2 Civil Procedure 16

3 Evidence 33

4 Torts 50

5 Contracts 62

6 Business Associations 77

1 Question 2

Copyco, Inc. (―Copyco‖), a maker of copy machines, was incorporated in State A. Most of Copyco’s employees work in State B at its sole manufacturing plant, which is located in the southern federal judicial district of State B. Copyco also has a distribution center in the northern federal judicial district of State B. Sally is a citizen of State B. Sally was using a Copyco copy machine at Blinko, a copy center within the northern federal judicial district of State B, when the machine started to jam. When Sally tried to clear the jam, she severely injured her hand. She underwent several surgeries at a nearby hospital. Her physician believes she may never recover the full use of her hand. Sally filed a lawsuit against Copyco as the sole defendant in the State B northern district federal court. Her complaint alleges that Copyco was negligent and that she has suffered physical injury, and also seeks damages of $100,000, exclusive of costs and interest. The federal court has subject matter jurisdiction to hear Sally’s lawsuit on the basis of diversity of citizenship. Copyco, however, moved for a change of venue to the southern federal judicial district of State B. The court denied Copyco’s motion. Sally wishes to obtain from Blinko a copy of the maintenance records for the copy machine that caused her injuries. Questioning the extent of the injuries Sally alleged, Copyco wishes the court to compel Sally to appear for an examination by both a physician and a psychologist of Copyco’s own choosing. 1. Was the federal court correct to deny Copyco’s motion for change of venue? Discuss. 2. (a) Is Sally entitled to a copy of the maintenance records? Discuss. (b) If so, how must she proceed to obtain them? Discuss.

3. (a) Is Copyco entitled to an order to compel Sally to appear for an examination by a physician and an examination by a psychologist chosen by Copyco? Discuss. (b) If so, how must it proceed to obtain such an order? Discuss.

16 Answer A to Question 2

1. Change of Venue

Proper Venue Under the federal rules of civil procedure, venue is proper in any district where (1) all defendants reside or where a substantial portion of the claim arose, (2) there is subject matter jurisdiction over the claim, and (3) there is personal jurisdiction over the parties. If there are multiple defendants and they reside in different districts, the venue may be satisfied in any district where one of the defendants resides.

Residence of Corporations A corporation is subject to special rules with regard to its residence for venue purposes. Unlike a person, who is a resident of whichever district that he/she is domiciled in, a corporation is considered a resident of any district where there is a personal jurisdiction over the corporation. Personal jurisdiction may be specific or general. General jurisdiction requires substantial, continuous and systematic contacts with the forum. Specific jurisdiction requires that the defendant have sufficient minimum contacts with the forum so as not to offend traditional notions of fair play and substantial justice.

Copyco (C) will argue that venue is not proper in the northern district (ND) of State B because it lacks personal jurisdiction over C. This argument will likely fail, however, because C arguably has substantial, continuous and systematic contacts with ND by the fact of its distribution center. C’s contacts are clearly continuous and systematic because C maintains a permanent presence in the district and presumably the distribution is an integral part of C’s overall business operation. Thus, the only real question is whether C’s presence in ND is substantial. The better argument is that C’s permanent physical presence in ND,

17 which presumably requires it to transport materials in and out of the district on a daily basis, is substantial.

Moreover, even if ND does not have general jurisdiction over C, the court will in all likelihood conclude that it has specific jurisdiction. Specific jurisdiction requires minimum contacts, which consist of purposeful availment and foreseeability, and basic fairness, which requires relatedness. Here, C purposefully availed itself of the ND by establishing a distribution center there. That C could be sued in ND is clearly foreseeable because it regularly transacts shipping/distribution operations there. Thus, the minimum contracts prong is satisfied. On these facts, the relatedness prong of the test may be debatable as it is difficult to determine if C actually sells any of its copy machines to businesses or consumers in the ND. Blinko may have obtained the copy machine outside C’s normal chain of retail/distribution, in which case the relatedness inquiry may cut in C’s favor. However, even where this [is] the case, if C took any action in ND to advertise its copy machines or otherwise availed itself of customers in ND, then relatedness is satisfied.

Where Injury Arose Of course, venue is also proper because the Northern District is where S was injured in the district while using a C copy machine; thus, a substantial portion of S’s personal injury claim arose in the Northern District.

Personal Jurisdiction The Northern District has personal jurisdiction over C – see discussion above re residence of corporations.

Conclusions – Northern District is Proper Venue Because (1) a substantial portion of the claim arose in the Northern District (and also because C is a resident of the Northern District), (2) the Northern District has specific jurisdiction and probably general jurisdiction over C, and (3) the facts

18 state the diversity subject matter jurisdiction is present, then venue in the Northern District is proper.

Change of Venue Where venue is improper, the defendant may move for dismissal of the plaintiff’s claim. The court may grant the dismissal or order that venue be transferred (assuming there is a federal district court with proper venue) if the transfer is in the interests of justice. Here, C did not seek dismissal. Moreover, venue in ND is proper (see discussion above). When a defendant seeks to transfer venue from a proper forum, a three part test is applied: (1) the transferee court must have subject matter and personal jurisdiction; (2) the transfer must be convenient; and (3) the transfer must be in the interests of justice. District courts are afforded great discretion when deciding permissive venue transfer requests.

Jurisdiction The Southern District, where C seeks to have the case transferred, may assert both personal jurisdiction and subject matter jurisdiction. Subject matter jurisdiction is present because the district is the same state as where the plaintiff filed her lawsuit, and therefore there is no disruption of the requirement of complete diversity. Personal jurisdiction is also present because C has its principal place of business in the Southern District and therefore C satisfies the general personal jurisdiction requirement of substantial, continuous, and systematic contacts.

Note: On the facts present, it is unclear how the federal court may assert the presence of subject matter jurisdiction based on diversity of citizenship. Diversity required complete diversity – no plaintiff may be a resident of the same state as any defendant. S is a citizen of B. As a corporation, C is a resident of (1) the state of its incorporation, and (2) the state where it maintains its principal place of business (PPB). The PPB is determined using either the muscle center test (where most of the corporation’s operations are located) or the nerve center test

19 (where most of the corporation decision making occurs). It would appear on these facts that C’s PPB is in State B because that is where most of its employees work and where it maintains its sole manufacturing plant. Yet the facts state the federal court has subject matter jurisdiction based on diversity, so perhaps the court applied the nerve center test (assuming C’s decision making occurs in A).

Convenience A court may transfer venue if it promotes convenience, and courts frequently focus the convenience issue on questions of witness availability. S will oppose the transfer and argue that convenience favors keeping the case in ND. That is the site of C’s injury and is also where she received medical treatment. Thus, virtually all of the key witnesses, and presumably the plaintiff, are located. On the other hand, C will argue that [the] machine in question was manufactured in SD, and thus, there [are] a number of witnesses present in that district (presumably, witnesses who will testify regarding any design or manufacturing defect).

Interest of Justice Normally, a plaintiff’s choice of forum is entitled to deference and should not be disturbed absent compelling reasons of fairness. At most, C has demonstrated that the convenience issue is a close call. Generally, a marginal difference in convenience will not be sufficient to overcome the deference afforded to the plaintiff’s forum choice.

Conclusion Because C will be unable to demonstrate that [it] is significantly more convenient to try the case in SD or that fairness issues dictate transferring the case to SD, the court acted appropriately in denying the motion to transfer venue.

20 2. Maintenance Records

(a) Relevant/Discoverable Unlike the admission of evidence at trial, the test for what information is discoverable is extremely broad and is not limited to simply that information which is deemed relevant (defined as having any tendency to make a fact of consequence more or less likely than in the absence of the evidence). Information is discoverable if it is relevant or if it is reasonably calculated to lead to the discovery of relevant information.

S will argue that the maintenance records are directly relevant to whether the copy machine was maintained in a manner and on the schedule established by C, the manufacturer. She is likely [to] attempt to preempt any possibly defense by C of an intervening supervening cause for her injury – namely, the lack of maintenance by Blinko or negligent maintenance by Blinko or a third party service contractor. Thus, the information sought by S is discoverable and, indeed, C would likely not oppose the discovery. S is therefore entitled to the discovery subject to the discussion below re third parties.

(b) Third Party Discovery Here, S seeks to obtain the records not from C, a party to the litigation, but rather from Blinko, who is a third party. As such, S is not entitled to many of the discovery devices set out in the FRCP, such as interrogatories or requests for production. Yet the rules do provide for limited discovery of third parties through use of a subpoena. Thus, a third party [may] be subpoenaed to appear for deposition. In this case, S seeks discovery of documents as opposed to live testimony. She must therefore serve a subpoena duces tecum to obtain the documents. Note that she does not have to seek court approval to serve the subpoena on Blinko, although she must include in the subpoena a list of Blinko’s third party rights under the FRCP, including the right to file a motion to quash the

21 subpoena. The subpoena must specify a time and location when Blinko will make the requested records available for inspection and copying by S.

Ideally, S will serve a subpoena duces tecum on Blinko in which she requests that Blinko produce its records custodian at deposition along with the actual records. This way, S can examine Blinko under oath to establish both the authenticity of the documents and attempt to establish any exceptions under the hearsay rules (such as business records). Note that if S simply wishes to secure the authenticity of the documents, she can simply negotiate with Blinko to have the records custodian provide an affidavit certifying the authenticity in lieu of a deposition.

If Blinko objects to the subpoena, Blinko may file a motion to quash or may simply respond to the subpoena duces tecum with written objections along with a refusal to produce the records. In this case, the burden shifts to the moving party (S) to establish the need for the discovery. Although courts generally try to protect the interests of third parties to be free from discovery, the maintenance records are highly relevant to S’s claims, and therefore, the court will in all likelihood overrule any objections to the discovery by Blinko.

Of course, S is always free to simply negotiate the production of the discovery with Blinko with the need to use any formal discovery devices.

Conclusion The maintenance sought by S is discoverable and she is entitled to use third party discovery devices, including subpoena duces tecum, to obtain the records.

3. Medical/Psychological Examination (a) C will be entitled to an order compelling S to a medical examination by a physician chosen by C because S, by the filing of her claim for personal injuries,

22 placed her physical condition at issue in her case. In personal injury cases, defendants have a right to examine the injured plaintiff upon a showing of good cause, In this case, C challenges the extent of S’s injuries and therefore the good cause condition is likely met. However, S will probably not be able to establish good cause with respect to the need for S to submit to a mental examination. The issue depends on the extent to which S is alleging any special emotional/mental damages. Generally, courts will permit a party to recover for emotional distress as a result of physical injuries and will not require any special expert testimony on this issue, largely because a jury is competent to understand this issue. However, if S intends to offer any expert testimony regarding her mental/emotional distress, then C will be able to show good cause as to why it should be entitled to have its own expert examine S.

(b) There is a specific rule under the FRCP which addresses requests by one party to conduct a physical/mental examination of the other party. Under this rule, the party seeking the examination [must] first serve a written discovery request on the party to be examined. The written request must identify the time and place for the examination as well, the amount of time the examination is expected to take, and the person who will be conducting the examination. The request must set forth good cause as to why the examination should be permitted to proceed. Preapproval of the court is not required. However, a party may object to the discovery request, in which case court involvement is necessary.

23 Answer B to Question 2

1. Denying Copyco’s motion for change of venue

The first question to determine is whether the original venue was proper, because in federal court this determines which law the court should apply if a transfer is granted.

Original Venue in Northern District Federal Court

Venue in federal court is proper (1) in any district where any defendant resides if all reside in the same state, or (2) where a substantial amount of the action or the property involved in the lawsuit is located. If neither applies, then in a diversity case, venue is proper where any defendant is subject to personal jurisdiction, and, in all other cases, where any defendant can be found. In local actions venue is proper where the land is located.

Here, since the property involved in this location is located in the Northern Federal District, venue was originally proper.

However, if this wasn’t the case, then we must look at the residence of C.

Residence of Copyco

A corporation is a resident of any place where it is subject to personal jurisdiction. It is not like citizenship for the purposes of diversity, which is its principal place of business and its state of incorporation.

Thus, we must do a personal jurisdiction analysis.

24 Personal Jurisdiction Over Copyco

Personal jurisdiction requires both that the state statute must allow jurisdiction and that it meet the constitutional requirements.

Statutory Requirements

In general, states allow jurisdiction when (1) the defendant is domiciled in the state, (2) the defendant is personally served in the state, (3) the defendant consents, or (4) if the long-arm statute applies.

Here, Copyco is domiciled in State B because it has its sole manufacturing plant in State B, and this would be considered its principal place of business. It is unclear where C was served. It appears that C has consented to venue in State B since it is simply asking for a transfer to any other district in the state. The long-arm statute probably allows this as well.

However, the real question is whether it is domiciled in the Southern or Northern District. Although it has a distribution center in the Northern District, this might be a very small operation. The facts are not clear on this. But assuming that the state statute allows this, which it might, the next question is whether this is appropriate for the Constitution.

Constitutional Limitations

This requires that the defendant have minimum contacts such that jurisdiction does not offend traditional notions of fair play and substantial justice. This requires (1) minimum contacts, which in turn requires (a) purposeful availment, and (b) foreseeability, and (2) fairness, which requires (a) relatedness of claim to contact, which can either be general or specific, (b) no severe inconvenience to defendant, and (c) weighing the interests of the forum.

25 However, the traditional bases have been found by the Supreme Court to satisfy this standard, and these are (1) domicile, (2) service in state, (3) consent.

As explained above, it is unclear if C is domiciled in the Northern District; thus we must do a minimum contacts analysis.

1. Minimum Contacts

(a) purposeful availment

C has a distribution center in the Northern District; thus, it is making use of the privileges and protections of the law of the Northern District. And it likely had knowledge +, as explained immediately below, that its machines would end up in a place like Blinko or actually in Blinko, since it might have personally done the distribution to the shop, and the Supreme Court is unanimous that knowledge plus is enough for purposeful availment.

(b) foreseeability

It was foreseeable that C might be haled into court in the Northern District since it sold machines to Blinko, which is in the Northern District, and thus they would probably sue there. The Supreme Court is split between knowledge and knowledge + requirement for PJ. It can be shown that C knew that B had some machines most likely, and C most likely purposefully sent them to Blinko or caused them to be distributed there; thus this was foreseeable.

2. Fairness

(a) relatedness

26 This suit is directly related to the contact between C and the Northern District since this was the machine that was sold in the Northern District and Sally was injured there.

(b) inconvenience

It must be severely inconvenient for C to defend there; this seems unlikely unless State B is incredibly large.

(c) state’s interest

The Northern District has an interest in protecting its citizens from defective products and the injuries they cause.

Conclusion Original venue was proper in the Northern District.

Transfer of Venue

The court will transfer to another district in the federal court if (1) it could have originally been bought there, (2) the interests of justice and the convenience of the parties require it. The court has discretion to grant or deny the motion.

Could Have Been Brought

This requires (1) subject matter jurisdiction, (2) personal jurisdiction, and (3) venue.

27 Subject Matter Jurisdiction

Federal courts are courts of limited jurisdiction. They can only hear diversity of citizenship jurisdiction cases and federal question cases. Diversity requires complete diversity of citizenship between defendant and plaintiffs, and that the claim exceed $75,000, exclusive of interest and costs.

Here, the claim that Sally is asserting is negligence. This is a state law claim; thus, there is no federal question jurisdiction.

However, the facts stipulate that there is diversity of citizenship jurisdiction (although this seems questionable since C is incorporated in State A and seems to have its principal place of business in State B, and the facts state that Sally is a citizen of State B; thus, it would seem that there is not complete diversity between plaintiff and defendant; however, since the facts stipulate it, that is settled.) The amount in controversy is $100,000, which exceeds $75,000.

Thus there is a subject matter jurisdiction.

Personal Jurisdiction

There is this over C; see above.

Venue

See rule above.

As analyzed above, because C had its sole manufacturing plant in the Southern Federal Judicial District, there is personal jurisdiction over it in this district, and thus venue is proper.

28 Interest of Justice and Convenience of the Parties

Here, the claim is for negligence. In order to prove the negligence there must be (1) duty, (2) breach, (3) actual cause, and (4) proximate cause. The defenses are (1) contributory negligence, (2) comparative negligence, and (3) assumption of the risk.

Here, the claim arose from a machine that is located in Blinko, a copy center in the Northern Federal District of State B. The property is thus located in the Northern District. All maintenance records and employees and witnesses to the use of the machine will likely be at or near the Northern District, since other customers might be called to testify as to whether they noticed anything or how Blinko maintained the machine. This is important because C will not be found liable if the defect was not present when the machine left its control; thus many Blinko employees might have to testify. Furthermore, S had her surgeries in a nearby hospital, and its staff and doctors might have to testify, and they likely live in the Northern District. It wasn’t just one surgery, it was several; thus many doctors could be involved, and staff and they might all have to appear as witnesses.

On the other hand, C could argue that its sole manufacturing plant is located in the Southern District, and it will have to call its employees to testify as to their manufacturing procedures and how they check their products for defects. However, on balance, it seems likely most of the witnesses and the records will come from the Northern District; thus, it seems like the most appropriate place.

Moreover, the Northern District has a big interest here because this was a severe injury, and it does not want this to happen to others.

29 Conclusion

Thus, the court did not err in denying the motion.

Original Venue Law

Note that because the original venue was proper, the original venue law would apply if the court had granted the motion, i.e,, the law of the Northern District of State B.

Note

This is not a motion for forum non conveniens since the federal court can transfer to another federal court.

2. (a) copy of maintenance records

Discovery in federal court is allowed as to anything that is nonprivileged and relevant to a claim of defense. It does not have to be admissible in court; it just had to be reasonably calculated to lead to the discovery of admissible evidence.

Here, the maintenance records will be necessary for Sally to prove her negligence claim against C. If B has an excellent record for maintaining its machines, then this circumstantial evidence that her negligence claim is viable because the defect in the machine must have been there when it left C’s control. On the other hand, the records could also show that the machine always had problems, which would also indicate that there was a defect from the start. However, if the machine had been tampered with by a customer, this would hurt her case. Thus, the records would likely lead to discovery of admissible evidence (customer names, maintenance company name). Note the records themselves are probably admissible as a business record.

30 (b) how can she obtain them

Since B is not a defendant, Sally will have to send a request to produce along with a subpoena duces tecum. This requires a non-party to produce documents in its possession.

It is not possible for this to be obtained by deposition or interrogatory since these are simply questions that are asked and interrogatories are just for [a] party.

3. (a) (i) physician

As above, discovery is allowed as to anything relevant to a claim of defense.

A physical examination will be relevant for C to disprove the amount of damages that S is claiming or to prove that she did not mitigate, or was perhaps herself negligent in seeking help for her injuries in a strange manner. Thus, an examination by a physician should be allowed by the court. While C can request that the court allow it to use a physician of its choosing, the court is not required to do this. The court is free to choose a neutral physician or to order the parties to decide together.

(ii) psychologist

A psychologist examination does not appear reasonably calculated to lead to the discovery of admissible evidence, and it does not appear relevant to any claim or defense by C. Here, Sally is not suing for emotional trauma; she appears to only be suing for her physical injuries. Thus, an examination by a psychologist will not determine the extent of her physical injuries. However, if Sally is claiming some pain and suffering or emotional scarring from the fact that she may never recover the full use of her hand, then a psychological examination would be appropriate.

31 (b) how does it proceed

Unlike in state court in California, where one physical examination is granted as a matter of right, if the physical condition of the party is in issue, in federal court, the requesting part must take a motion to the court to compel a physical examination and issue an order. The court then allows a hearing where both sides present their case, and decides whether it should issue an order. This is a form of discovery called a request for physical or mental examination. It must occur during the discovery period in accordance with the discovery schedule that the court has determined, although the court has discretion to allow it past the date if it would not prejudice the parties and the interests of justice don’t require otherwise.

32

THE STATE BAR OF CALIFORNIA OFICE OF ADMISSIONS 180 HOWARD STREET • SAN FRANCISCO CALIFORNIA 94105 1639 • (415) 538 - 2303 1149 SOUTH HILL STREET • LOS ANGELES CALIFORNIA 90015-2299 • (213) 765 – 1500

ESSAY QUESTIONS AND SELECTED ANSWERS JULY 2009 CALIFORNIA BAR EXAMINATION

This publication contains the six essay questions from the July 2009 California Bar Examination and two selected answers to each question.

The answers received good grades and were written by applicants who passed the examination. The answers were prepared by their authors, and were transcribed as submitted, except that minor corrections in spelling and punctuation were made for ease in reading. The answers are reproduced here with the consent of their authors.

Question Number Contents Page

1 Torts/Civil Procedure/Professional Responsibility 3

2 Professional Responsibility 16

3 Evidence 27

4 Constitutional Law 50

5 Civil Procedure/Remedies/Professional Responsibility 66

6 Criminal Law and Procedure 82

1

Question 1

Patty is in the business of transporting human organs for transplant in City. She is paid only upon timely delivery of a viable organ; the delay of an hour can make an organ nonviable.

David transports gasoline over long distances in a tank truck. Recently, he was hauling gasoline through City. As David was crossing a bridge in City, his truck skidded on an oily patch and became wedged across the roadway, blocking all traffic in both directions for two hours.

Patty was delivering a kidney and was on the bridge several cars behind David when the accident occurred. The traffic jam caused Patty to be two hours late in making her delivery and made the kidney nonviable. Consequently, she was not paid the $1,000 fee she would otherwise have received.

Patty contacted Art, a lawyer, and told him that she wanted to sue David for the loss of her fee. “There isn’t a lot of money involved,” she said, “but I want to teach David a lesson. David can’t possibly afford the legal fees to defend this case, so maybe we can put him out of business.”

Art agreed and, concluding that he could not prove negligence against David, decided that the only plausible claim would be one based on strict liability for ultrahazardous activity. Art filed a suit based on that theory against David on behalf of Patty, seeking recovery of damages to cover the $1,000 fee Patty lost. The facts recited in the first three paragraphs above appeared on the face of the complaint.

David filed a motion to dismiss. The court granted the motion on the grounds that the complaint failed to state a cause of action and that, in any event, the damages alleged were not recoverable. It entered judgment in David’s favor.

David then filed suit against Patty and Art for malicious prosecution.

1. Did the court correctly grant David’s motion to dismiss on the grounds stated? Discuss.

2. What is the likely outcome of David’s suit for malicious prosecution against Patty and Art? Discuss.

3

Answer A to Question 1

Patty instituted a suit via her lawyer Art for losses incurred due to Patty’s inability to deliver a kidney on time owing to a traffic jam. The traffic jam occurred when David’s truck skidded on an oily patch and became wedged across the roadway. There are two issues that need to be determined. First, the validity of the court’s decision to dismiss Patty’s cause of action for damages based on strict liability owing to an ultrahazardous activity. Secondly, whether David will be successful in recovering against Patty and Art in a claim of malicious prosecution.

1. David’s motion to Dismiss based on Failure to State a Cause of Action

David has instituted a motion to dismiss for failure to state a cause of action upon which relief can be granted. In the alternative, David argues that damages would not have been recoverable against David for strict liability from malicious prosecution. A motion to dismiss based on a failure to state a cause of action upon which relief can be granted is a 12(b)(6) motion in federal court. This motion can be filed as a preliminary motion to the filing of a complaint or contained within the answer. Along with failure to include an indispensable party it can be raised at any time prior to trial or at trial itself. The motion charges that the plaintiff has failed to adequately state a cause of action upon which relief can be granted. It requires the judge to accept that all the facts that are stated by the plaintiff are taken to be true and then requires a determination as to whether there exists an adequate basis for relief. In other words, even if everything that plaintiff asserted in the complaint is true, would that be sufficient to allege a cause of action against the defendant?

In the current case, in order to determine whether the emotion to dismiss was appropriately granted in Art’s favor, it is necessary to examine Patty’s allegations against David. Patty’s lawyer, Art, determined that a negligence claim would not be viable against David. Likely because there is nothing to indicate in the facts that David engaged in any activity whereby he breached the standard of care towards a

4 foreseeable plaintiff. There is nothing to indicate that he was negligent in driving his truck, but rather he skidded on an oily patch in the middle of the road and then his truck swerved to block all lanes of traffic. As a result, Art decided to pursue Patty’s claim on a strict liability theory for transporting an ultrahazardous activity.

Strict Liability for an Ultrahazardous Activity

Strict liability for transporting an ultrahazardous activity is an action whereby the defendant is engaged in an ultrahazardous activity. This is where the activity is so dangerous that the danger of its harm cannot be mitigated even with the exercise of reasonable care. Secondly, the activity has to be one that is not of common usage in the community. In a strict liability claim for ultrahazardous activity, in jurisdictions that still retain contributory negligence, this is not a valid defense to a strict liability claim.

In the current case, David transports gasoline over long distances in a tank truck. In the current case, he was transporting gasoline through the City. It is important to note that transporting gasoline through residential parts of a city is inherently an ultrahazardous activity because of the dangers that can occur if any gasoline spills, owing to the fact that gasoline is highly combustible and can cause serious injuries and damage to property in a matter of seconds. No amount of care can mitigate against these risks, and transporting gasoline through a residential community is not a matter of common usage in the community.

However, in the current case, when David was transporting the gasoline across the bridge, he skidded on an oily patch. There is no indication that he is responsible for the oily patch, rather, it was already spilled on the road when he arrived at the scene. As a result he skidded on the spill and his truck wedged across the roadway and blocked traffic in all directions. This blockage caused a traffic jam to develop in both directions and the delay of two hours caused Patty to be late in making her organ delivery. Yet the crucial distinction in this case is that the ultrahazardous nature of the gasoline was not the cause of Patty’s damages. Even if David had been transporting a truck filled

5 with benign materials, such as flowers or children’s toys, he still would have skidded on the oily patch and his truck would have wedged across the highway and caused the traffic jam. For strict liability to attach for transporting ultrahazardous activity, the nature of the harm or loss has to emanate from the ultrahazardous activity. This is not met in this case. There is nothing about the inherently dangerous nature of transporting gasoline that is the cause of Patty’s harm.

As a result, even if the judge was to take all of the allegations that Patty made in her complaint to be true, she has failed to state sufficient facts necessary to constitute a cause of action for strict liability for transporting dangerous materials. Therefore, the judge was correct to grant David’s motion to dismiss.

Patty’s Damages are not recoverable

Moreover, David claimed that the damages that Patty claimed in her complaint were not recoverable. In this case, Patty sought to recover the $1,000 fee she would have been paid had she been able to deliver the kidney while it was still viable.

As already noted, under strict liability the damages have to accrue from the inherent dangerousness of the activity - which in this case would have been transporting gasoline. However, in this case, the nature of Patty’s damages resulted from the truck skidding on the oily patch, and as previously mentioned this could have occurred to any truck, even one transporting regular household goods. As a result, Patty is not entitled to recover for damages based on a theory of strict liability.

Her only viable claim would have been under a negligence theory which requires a duty under the applicable standard of care to all foreseeable plaintiffs (which under the majority Cardozo theory is to all plaintiffs in the zone of danger). There has to be a breach of the duty, causation (both factual and proximate), as well as damages. In this case, David would be held to the standard of care of a reasonable person driving a big truck along a bridge. The facts do not indicate that he was negligent in any manner,

6 such as driving too fast, or driving while distracted. As a result, Patty would be unable to establish a prima facie case for negligence and would be entitled to no damages. It is likely that Art realized that the negligence claim would be a non-starter and as a result he decided not to pursue the claim.

In conclusion, the court was correct to grant David’s motion to dismiss for failure to state a cause of action and, in any event, the damages alleged were not recoverable because Patty failed to assert an appropriate and viable cause of action.

2. David’s Suit for Malicious Prosecution against Patty and Art.

David decided to file suit for malicious prosecution against both Patty and Art. To establish a prima facie case for malicious prosecution, the plaintiff is required to show that there was an institution of civil proceedings against the plaintiff. Second, there was a termination of the proceedings in favor of the plaintiff. There also has to be a lack of probable cause. Moreover, the institution of the civil proceedings has to be for an improper purpose and the plaintiff has to show damages.

David’s suit for Malicious Prosecution against Patty In David’s suit against Patty, David can show that Patty instituted a claim against him for strict liability based on transporting an abnormally dangerous activity. Since the judge granted the motion to dismiss, there was a termination in his favor.

The third prong requires David to show that the proceedings were instituted for an improper purpose. In the current case, when Patty came to Art for advice she was clear that she wanted to sue David for the loss of her fee, i.e., the $1,000 she would have received if she could have successfully delivered the kidney. In her mind, she believed that she had suffered damages and that David was to blame because he had caused the traffic jam on the bridge. As a result, it is unclear whether her motive to bring the suit was based on lack of probable cause. As a layperson, she likely did not have the legal knowledge to ascertain the proper basis for determining probable cause, and she

7 came to her lawyer for advice to determine the merits of her case. As a result, it is likely that the court will find that Patty’s decision to bring suit against David was based on her relying on the legal expertise of Art and she might have honestly believed that there was sufficient probable cause to bring the action.

The fourth prong requires bringing the suit for an improper purpose. This requirement is likely met in this case, because Patty acknowledged that there was not a lot of money involved in the action; however, she wanted to teach David a lesson and try to run him out of business. As a result, the primary motivation behind the suit was not to recover damages, but rather to seek revenge and damage to David. This is an improper purpose because the legal system is not to be used in a civil proceeding in order to extract a revenge against a defendant or for an improper purpose.

Lastly, the plaintiff has to show sufficient damages. In the current case, David was forced to respond to an action for strict liability and although the matter was dismissed under a motion for failure to state a cause of action, this still might have resulted in David losing days at work because of the lawsuit. There is also the loss of professional and social reputation from being forced to defend against a lawsuit. However, David would have to present evidence of any such pecuniary loss in order to meet the damages prong.

In conclusion, David would likely not succeed in his suit for malicious prosecution against Patty because he cannot show that she instituted the proceedings without probable cause. Patty likely relied on Art’s advice that there was a viable claim for strict liability and, as a result, she thought there was sufficient merit in the action to proceed to court.

8

David’s suit for Malicious Prosecution against Art David also filed suit against Patty’s lawyer Art for malicious prosecution.

Again, the first two prongs are easily met, because Art was the attorney that brought the strict liability action against Patty and there was a termination in Art’s favor with the court’s decision to grant the motion to dismiss based on failure to state a cause of action.

In the current case, the third prong, whereby the plaintiff has to show that the action was brought with a lack of probable cause, is likely to bring David more success against Art. An attorney is held to possess the required duty of competence, whereby he has to possess the legal skill, knowledge, preparedness and ability to pursue the case. In this case, Art realized that a negligence action would not be successful, but he still decided to pursue a claim for strict liability. This was the only plausible claim that he could bring against David and if he failed to adequately research the facts of the case based on the elements of strict liability, then he will be held liable for bringing a cause of action based on lack of probable cause. On the other hand, if Art honestly believed, with sufficient preparation and research in the case, that a strict liability cause of action might be viable in this case, then arguably there is sufficient probable cause. However, as previously noted under the first part, there was no connection between the ultrahazardous nature of transporting the gasoline and the accident that occurred in this case, and, as a result, Patty would be unable to recover damages based on a strict liability theory. As a result, Art should have realized this and counseled Patty against filing suit, and therefore, David will be able to successfully demonstrate the lack of probable cause in a suit for malicious prosecution against Art.

The fourth prong requires the plaintiff demonstrating that the suit was brought for an improper purpose. In the current case, Patty told Art that she knew that there was not a lot of money involved in the case, but that she simply wanted to teach David a lesson and run him out of business. A lawyer is held to a duty of candor and fairness to the court and an adversary. He is required to properly research the cause of action to

9 ensure that there is a viable cause of action. A lawyer signs Rule 11 motions asserting that there is a proper factual basis to the claim and legal contentions are accurate and that a claim is not being brought for an improper purpose. In the current case, Art should have counseled Patty against bringing a lawsuit for an improper purpose and made her aware of the legal basis of the claim and whether there were sufficient facts to bring a cause of action. Attorney representation can be expensive, and Art should not have taken a frivolous claim simply as a means of earning fees and wasting time. As a result, David will be able to show that the cause of action was brought for an improper purpose.

As previously noted, as long as David can show damages in the form of lost wages from days missed from work owing to the need to defend the lawsuit or other pecuniary losses, he will have sufficiently demonstrated the damages prong.

In conclusion, David will be successful in a claim for malicious prosecution against Art. Even though his case against Patty is not likely to be successful owing to the inability to demonstrate that Patty consciously knew that there was a lack of probable cause to the action. However, as an attorney, Art will be held to a higher professional standard, and he had an ethical duty to ensure that he only brings suit where there is a sufficient legal and factual basis and that the suit is not being brought for a frivolous purpose or to waste time or embarrass an opponent. As a result, he should be entitled to damages, based on the damages he incurred due to the inappropriate suit brought against him for strict liability.

10

Answer B to Question 1

1. Patty (P) v. David (D) – Motion to Dismiss Suit for Strict Liability

A motion to dismiss for failure to state a claim looks at the facts in a light most favorable to the party it is being asserted against. The court will then see if sufficient facts have been pled to sustain a prima facie case of the cause of action alleged. The court does not evaluate the merits nor go beyond the complaint.

In the present case, P filed a claim of strict liability for ultrahazardous activity against D. Therefore, the elements of the claim must be evaluated in light of the complaint to see if grant of the motion was proper. Additionally, the court noted the case would be dismissed because the damages alleged were not recoverable.

Strict Liability – Ultrahazardous Activity Strict liability is tort liability without fault. It applies in cases of products liability, ultrahazardous activities, and wild animals. Here, the allegation is one of ultrahazardous activity. The elements of strict liability are 1) an absolute duty of care, 2) breach of that duty, 3) causation, and 4) damages.

Absolute Duty of Care – Is the activity an ultrahazardous activity? For there to be an absolute duty of care (a duty that may not be met by reasonable protective measures), a court must decide if an activity is in fact ultrahazardous. An ultrahazardous activity is one where the activity is 1) highly dangerous even with remedial measures, and 2) not within common usage within the community. This is a question of law to be decided by the trial judge.

In the present case, D was driving a tanker truck filled with gasoline. P will argue that this is a dangerous activity, because no matter how safe D behaves the tanker is a giant gas bomb waiting to explode. D can argue that it is not that dangerous because, as the facts show, there was no explosion when the tanker crashed. However, because the

11 court will view the facts in a light favorable to P, the tanker is probably sufficiently dangerous.

However, the second element poses a problem for P. The activity must not be in common usage within the community. Here, D’s tanker truck was transporting gas. This is an activity in common usage within all US communities, because gasoline is the primary fuel for automobiles, which is the most common method of transportation in the US. Additionally, gasoline must be transported by some means to service stations. Tanker trucks are the most common, if not [the] exclusive method of delivering gas to service stations in the US. Therefore, driving a tanker truck is an activity of common usage in City.

Therefore, the duty element has not been met, because driving a tanker truck is not an ultrahazardous activity.

Breach: if the duty element had been met, any damage caused by the ultrahazardous activity would be sufficient breach. Here, the truck crashed and blocked traffic for 2 hours.

Causation Causation has 2 parts: 1) actual (factual) cause and 2) legal (proximate) cause. Both must be met for the causation element to be sustained.

Factual Cause The test for factual cause is the “but for” test. This asked but for the defendant’s conduct the injury would not have occurred. In the present case, but for D crashing the tanker on the bridge, P would not have been late for her delivery, the kidney would have been viable, and P would have been paid $1,000. Viewing the facts in a light most favorable to P, factual cause is met.

12

Proximate Cause Proximate cause is a question of foreseeability. First, the court must ask what is dangerous about the activity. Here, a tanker truck filled with gas is dangerous because it could explode or cause a fire. Second, the court will isolate the breach. Here, the breach was a crash that resulted in blocked traffic on the bridge. Lastly, the court will match up the danger of the activity to the breach; if they do not match up, then the injury is not the type of harm that would result from the ultrahazardous activity. Therefore, it would not be foreseeable. In the present case, the danger of explosion or fire does not match the breach of mere traffic jam. Thus, P’s injury was not foreseeable.

Damages Strict liability compensates damages from personal injury or property damages. In the present case, the type of harm is economic damages. Economic damages are those damages which result from the loss like lost wages or lost business opportunity. Therefore, there is not sufficient damage that P may be compensated for. While she may argue that the breach damaged the kidney. However, the kidney did not belong to her. At the very least it belonged to the kidney donor or the recipient. Additionally, one cannot have ownership interest in human tissue (see 13th Amendment). Thus, there is no personal injury or property damage that P has pled to sufficiently make a prima facie case.

Conclusion The motion to dismiss was proper, because P did not sufficiently plead facts to sustain a cause of action of strict liability for an ultrahazardous activity. Tanker driving is not an ultrahazardous activity. There is no proximate causation between the crash and the loss of $1,000. Additionally, the damages requirement is not met because it is mere economic damages. Additionally, the trial judge was correct to assert that P’s alleged damages are unrecoverable.

13

2. D v. P and Art (A) – Malicious Prosecution Malicious prosecution is a tort that protects the interest of only having process instituted against a party for proper purpose and only when there is a valid case. The elements are 1) institution of legal proceeding, 2) termination of case in plaintiff’s favor, 3) absence of probable cause, 4) improper ulterior purpose for bringing legal process, and 5) damages.

Institution of proceedings: Typically, malicious prosecution involves the institution of criminal proceedings. However, institution of civil proceedings will sustain a cause of action as well. Here, P (under the advisement and representation of A) filed a civil claim for $1,000 in lost damages in strict liability for an ultrahazardous activity (see above). A civil complaint was filed against D. This is sufficient to meet the first element/institution of legal proceeding.

Termination: The second element, termination of the case in plaintiff’s favor, is met because the case was dismissed on failure to state a cause of action. This was a termination in D’s favor, because he filed the motion to dismiss. The case was terminated on the granting of the motion.

Absence of probable cause Probable cause is the reasonable belief that there was a valid cause of action. In the present case, P relied on A’s advice as her attorney to form her basis of probable cause. A told her that he believed there was a plausible claim for strict liability. Reliance on counsel will sustain a finding of probable cause. Therefore, this element is not met, as to P.

A, on the other hand, probably did not have probable cause. As discussed above, the claim of strict liability lacked sufficient facts to make a prima facie case. The complaint was just so bad that an attorney with minimal competence could not have a reasonable belief that there was a valid cause of action based on strict liability. Therefore, this element is met as to A.

14

Improper purpose is any purpose except that of justice. Here, the just purpose would be to make P whole again, after the loss of her $1,000. This is the point of tort liability: to make the plaintiff whole. In the present case, she wanted to “teach D a lesson.” P and A will argue that this is not improper because D should be a safer driver. D may argue that strict liability has no punitive damages; therefore, strict liability is not to punish. Therefore, teaching a lesson is an improper purpose.

Additionally, and more flagrantly, P believed that D could not afford the legal fees, and bringing the strict liability case would cause him to go out of business. A acquiesced in assisting her in the case. This is an improper purpose because the $1,000 was not a lot of money to her, but it would be a total loss of D’s livelihood. This is not a proper basis for suit because it is merely to harass and damage D.

Defenses: A may assert that he would qualify for immunity based on the prosecutor exemption. However, this will not happen because of the exception for state prosecutors filing criminal charges.

Conclusion: D will probably prevail against A. However, he will probably not prevail against P, because she had probable cause.

15

THE STATE BAR OF CALIFORNIA OFICE OF ADMISSIONS 180 HOWARD STREET • SAN FRANCISCO CALIFORNIA 94105 1639 • (415) 538 - 2303 1149 SOUTH HILL STREET • LOS ANGELES CALIFORNIA 90015-2299 • (213) 765 – 1500

ESSAY QUESTIONS AND SELECTED ANSWERS JULY 2009 CALIFORNIA BAR EXAMINATION

This publication contains the six essay questions from the July 2009 California Bar Examination and two selected answers to each question.

The answers received good grades and were written by applicants who passed the examination. The answers were prepared by their authors, and were transcribed as submitted, except that minor corrections in spelling and punctuation were made for ease in reading. The answers are reproduced here with the consent of their authors.

Question Number Contents Page

1 Torts/Civil Procedure/Professional Responsibility 3

2 Professional Responsibility 16

3 Evidence 27

4 Constitutional Law 50

5 Civil Procedure/Remedies/Professional Responsibility 66

6 Criminal Law and Procedure 82

1

Question 5

Diane owns a large country estate to which she plans to invite economically- disadvantaged children for free summer day camp. In order to provide the children with the opportunity to engage in water sports, Diane started construction to dam a stream on the property to create a pond. Neighbors downstream, who rely on the stream to irrigate their crops and to fill their wells, immediately demanded that Diane stop construction. Diane refused. Six months into the construction, when the dam was almost complete, the neighbors filed an application in state court for a permanent injunction ordering Diane to stop construction and to remove the dam. They asserted causes of action for nuisance and for a taking under the United States Constitution. After a hearing, the state court denied the application on the merits. The neighbors did not appeal the ruling.

Thereafter, Paul, one of the neighbors and a plaintiff in the state court case, separately retained Lawyer and filed an application for a permanent injunction against Diane in federal court asserting the same causes of action and requesting the same relief as in the state court case. Personal jurisdiction, subject matter jurisdiction, and venue were proper. The federal court granted Diane’s motion to dismiss Paul’s federal court application on the basis of preclusion.

Infuriated with the ruling, Paul told Lawyer, “If the court can’t give me the relief I am looking for, I will take care of Diane in my own way and that dam, too.” Unable to dissuade Paul and after telling him she would report his threatening comments to criminal authorities, Lawyer called 911 and, without identifying herself, told a dispatcher that “someone is on his way to hurt Diane.”

1. Was the state court’s denial of Diane’s neighbors’ application for a permanent injunction correct? Discuss. Do not address substantive property or riparian rights.

2. Was the federal court’s denial of Paul’s application for a permanent injunction correct? Discuss. Do not address substantive property or riparian rights.

3. Did Lawyer commit any ethical violation when she called 911? Discuss. Answer according to both California and ABA authorities.

66

Answer A to Question 5

I. Was the State court’s denial of Diane’s neighbors’ application for a permanent injuction correct?

A permanent injunction is an equitable remedy which is appropriate where there is an inadequate remedy at law, the plaintiff has a protectable property interest, enforcement of the injunction is feasible, balancing of the hardships, and there are no applicable equitable defenses to enforcement of the injunction.

Inadequate remedy at law – A remedy at law is inadequate where monetary damages are insufficient to compensate the plaintiff, or where they are unlikely to be recovered because the plaintiff is insolvent. Furthermore, a legal remedy may be inadequate. In this case, the neighbors are going to argue that an award of monetary damages will be inadequate because they rely on the stream that Diane is diverting to irrigate their crops and fill their wells. While an award of damages would give them money, it would in no way help them in dealing with this problem. Furthermore, they will also argue that because the use and enjoyment of their real property is involved, this is a situation where their land is unique and legal damages will be inadequate because of the irreparable harm that will occur to the neighbors if they lose access to the water.

Protectable Property interest – A plaintiff may only seek a permanent injunction where they have a property interest that a court in equity will protect. While the traditional rule was very strict, the modern rule provides that an interest in property will suffice. The plaintiffs will argue that as landowners living downstream, they have a protectable property interest in the water. The court is likely going to accept this argument because they had been using the water before Diane came into the area and likely have at least some rights to continue using some of the water.

Feasibility of enforcement – Enforcement problems arise in the context of mandatory injunctions which requires the defendant to do something. Negative injunctions which

67 prohibit the defendant from performing certain actions create no enforcement problems. In the enforcement area, courts are concerned about the feasibility of ensuring compliance with a mandatory injunction and also with the problem of continuing supervision.

Under these facts, Diane’s neighbors initially asked for a partial mandatory injunction and partial negative injunction, ordering Diane to stop construction and remove the dam. With regard to the mandatory part (removing the dam), Diane has to affirmatively take this action, rather than being required simply to stop building the dam. Because this is a mandatory injunction, this creates an enforcement problem for the court. It will have the problem of continually supervising Diane to make sure that she in fact takes the dam down. The part of the injunction regarding stopping construction is a negative injunction because all that is required is that Diane stop construction. As such it creates no enforcement problems. While the part of the injunction that requires Diane to take down the dam creates some enforcement problems, the court could solve this problem by couching it as a negative injunction.

Balancing of the hardships – In balancing the hardships, the courts will always balance the hardships if the permanent injunction is granted on the defendant with the hardship to the plaintiff if the injunction does not issue. The only time that courts will not balance the hardships is where the defendant’s conduct is willful. Finally, in balancing the hardships, the court can take the public interest into account.

Was the plaintiff’s conduct willful so as to prohibit balancing of the hardships – In this case, while Diane willfully continued the construction and used the dam to divert the water, there is no indication that when she was doing this that she knew that her conduct was wrong or was intentionally violating the rights of the plaintiffs. While the neighbors demanded that she stop, there is no indication that she believed that she was not entitled to continue. Consequently, the hardships should be balanced because the defendant’s conduct was not willfully in violation of the plaintiffs’ rights.

68

Balancing the hardships – The plaintiffs are going to argue that they will suffer great harm if an injunction does not issue. Under these facts, the plaintiffs need the water from the stream for their crops’ irrigation and to fill their wells. Thus if a permanent injunction does not issue their crops are likely to die and they will not have a water supply in their wells. This is a great showing of hardship. The defendant is going to counter that she is trying to construct a free summer day camp for poor kids and that she cannot do so if she is forced to halt construction and if she cannot use the water diverted by the dam for her pond. However, in this case, these hardships do not seem so great compared to the hardships faced by the plaintiffs. There is no indication that she cannot get the water from her pond from somewhere else; furthermore, it seems likely that she could continue constructing her property in a way that does not interfere with the rights of the plaintiffs. The direct balancing of the hardships thus favors the plaintiffs.

Consideration of the public interest in balancing the hardships – Courts may also consider the public interest in balancing the hardships. Diane is going to argue that the public interest favors her because she is doing this project to create a free summer day camp for children who do not have a lot of money. This certainly indicates that her action is in the public interest. However, the neighbors can also make a public interest argument. Assuming that they sell their crops for consumption by the general public, they also have public interest factors on their side. Thus this factor does not seem to favor either side very strongly.

On balance, thus, it seems that the balancing of the hardships favors the plaintiffs when taking the direct hardships and the public interest into account.

Equitable Defenses – Courts in equity will not issue an injunction in favor of plaintiffs where they have unclean hands, where laches applies, or where the claim is barred by estoppel.

69

Unclean hands – is a defense in equity where the plaintiffs have committed acts of bad faith with regard to the subject matter before the court. In this case, there is no indication that the plaintiffs have unclean hands, so this argument by Diane will be unsuccessful as a defense.

Laches – Laches applies where a plaintiff or group of plaintiffs unreasonably delay in instituting a cause of action or claim against a defendant and this delay prejudices the defendant. In this case, Diane is going to argue that the plaintiffs’ delay in this case was unreasonable. When Diane refused the neighbors’ initial request to stop construction, they waited six months before filing an application with the state court for an injunction. Furthermore, she is going to argue that she was harmed by this delay because she continued construction and expended substantial funds during this delay. While Diane can make a pretty compelling argument, it does not seem that a delay of six months is enough time that the plaintiffs’ claim should be barred by laches.

Estoppel – applies as a defense in equity where plaintiffs take a course of action that is communicated to the defendant and inconsistent with a claim later asserted, and the defendant relies on this to their detriment. In this case, estoppel will not bar the claim by the plaintiffs because once they became aware of the construction, they immediately indicated that they did not approve. They commanded Diane to stop so the plaintiffs’ claim is not barred by estoppel.

Conclusion – The state court was incorrect in denying the permanent injunction because it appears that the permanent injunction should have issued because of the factors discussed above.

II. Was the federal court’s denial of the permanent injunction correct?

Claim Preclusion (Res Judicata) – The equitable doctrine of res judicata stands for the proposition that a plaintiff should only have one chance to pursue a claim against the same defendant. This doctrine applies and bars relitigating of a claim where (1) the 70 claim is asserted by the same claimant against the same defendant in case #2 as in case #1, (2) where the first case ended in a valid final judgment on the merits, and (3) where the same claims are being asserted in case #2 as in case #1. In federal court these claims arise from the same conduct, transaction or occurrence.

Same Claimant Against Same Defendant in Case #2 as in Case #1 – In this case, second case, Paul is suing Diane in federal court. The facts indicate that he was one of the neighbors and a plaintiff in the first case in state court. Consequently this element is met, because Paul was also a claimant against Diane in the first case.

Case #1 ended in a valid final judgment on the merits – The facts indicate that in the first case, the court denied the application for a permanent injunction on the merits. The facts also indicate that the neighbors did not appeal. A judgment on the merits is clearly a valid judgment and because no appeal was made, this judgment is also final. Consequently, this element of res judicata is also met. The one issue that Paul may raise on this point is that if the time for appeal has not run in state court, he may argue that he could file a notice of appeal in state court. However, taking up this suit in federal court is improper because absent an appeal in state court, there has been a valid final judgment on the merits that the federal court should adhere to.

Are the same claims asserted in case #2 as were asserted in case #1? Under federal law there is a theory of merger whereby a plaintiff is deemed to have asserted all claims pertaining to a prior claim that arise from the same conduct, transaction, or occurrence. In this case, the facts indicate that Paul asserted the same causes of action and requested the same relief in the second case as in the first case. Consequently, this element is met. California follows the primary rights theory which gives the plaintiff a cause of action for each right that this invaded. However, in this case, because there is no indication that any of the causes of action are different than the ones in the first case, the result in California would not be different.

Conclusion – The court was correct to dismiss Paul’s application for permanent injunction because the doctrine of claim preclusion (res judicata) precluded relitigating claims that had already been asserted in a prior case.

71

III. Ethical Violations of Lawyer in reporting Paul’s communications to the 911 Dispatcher

Duty of Confidentiality – Under the ABA Model Rules, a lawyer has a duty of confidentiality to a client which precludes disclosing any information obtained during the representation. Under the California rules, while there is no express duty of confidentiality, a lawyer is required to keep his client’s confidences and this is a strict duty.

In this case, Paul is going to argue that lawyer violated this duty when he revealed the information that he was told after the ruling to the 911 dispatcher. While he is correct that this raises an issue with regard to the duty of confidentiality, he may be incorrect that Paul has violated this duty because both the ABA Rules and the CA Code recognize that there are certain situations whereby the duty of confidentiality is overridden by other concerns.

Exceptions to the Duty of Confidentiality – Under the ABA Model Rules, a lawyer may reveal client confidences where he believes necessary to prevent reasonably certain death or serious bodily injury. The California Code has the same requirements but also requires that where reasonable a lawyer should first try to talk the client out of committing the act and then tell them that they will reveal confidences if they are not assured that the client will not commit the act. Under both the ABA and California rules, this type of disclosure of client confidences is permissive; it is not mandatory. Under the federal rules, there is also an exception to the duty of confidentiality where the client has used or is using the client’s services to commit a crime or fraud which will result in substantial financial loss. California has no such exception, but this exception will not be applicable anyway because there is no indication that Paul will be using Lawyer’s services if he acts against Diane or the dam.

Federal Rules – Under the federal rules, the main issue is whether Lawyer reasonably believed that his disclosure was necessary to prevent reasonably certain death or substantial bodily injury to Diane. If this is the case then he was entitled to reveal client confidences and will not have breached his duty of loyalty. The facts indicate that Paul

72 was infuriated with the ruling that the federal court had made in dismissing his claim and that he said “If the court can’t give me the relief I am looking for, I will take care of Diane in my own way and that dam too.” The question is whether the belief that he was going to get Diane made it reasonable to believe that she was threatened with death or serious bodily injury. Based on the facts of this case, this may not be met here because Paul had just lost his case and was upset. People often say things when they are upset, but don’t necessarily act on them. Lawyer will argue that he tried to talk Paul out of hurting Diane and that he only reported the comments then. However, under these circumstances, it seems like this disclosure may have been unreasonable and violated Lawyer’s duty of confidentiality, particularly because such a disclosure is permissive.

California Code – In addition to the federal requirements discussed above, before revealing any client confidences based on a reasonable belief of a reasonable threat of death or substantial bodily injury, Lawyer was required to first try to talk Paul out of committing the violent act against Diane and inform client of his intention to reveal the confidential communications. In this case, the facts indicate that Lawyer did this by trying to dissuade Paul and telling him that she would report his threatening comments to criminal authorities. However, as discussed above, given all of the circumstances this disclosure may not have been reasonable.

Attorney/Client Privilege – Under the attorney-client privilege, a lawyer may not reveal information intended by the client to be confidential which is given in order to get legal advice. However, in both California and under the ABA Model Rules, there is an exception where disclosure of confidential information obtained during the course of the attorney-client privilege is permitted to prevent death or serious bodily injury. This analysis while similar to the analysis above and the question is whether the statements made by Paul were for the purpose of legal advice; it seems like he was just telling Lawyer what he was planning to do so. The statements may not even be covered by the Attorney/Client privilege. Furthermore, these statements may fall within the exception for threats of death or serious bodily injury if the threat that Paul made against Diane was credible.

73

Duty to uphold justice – Under their duty to uphold justice under both the ABA Model Rules and the California Code, a lawyer is permitted to disclose client confidences where necessary to prevent reasonably certain death or substantial bodily harm. Lawyer will argue that this is why the disclosure was made. However, if this disclosure was unreasonable, this duty will not protect Lawyer from breaching her duty of confidentiality and potentially the Attorney-Client privilege.

Conclusion – Lawyer may have violated her duty of confidentiality and the attorney- client privilege under both ABA Model Rules and the CA Code if it is found that the threat made by Paul against Diane was not a credible one and just made in the heat of the moment without any reasonable chance of actually carrying it through. However, in her defense, Lawyer may argue that she did not disclose the identity of who was on their way to hurt Diane because she just told the dispatcher that “someone was on the way.” However, this will not be dispositive on this issue of whether she breached ethical duties.

74

Answer B to Question 5 1. Denial of Diane’s neighbors’ application for permanent injunction

Permanent injunction A permanent injunction is a court order mandating a person to either perform or refrain from performing a specific act. A permanent injunction is granted after a full trial on the merits. In order to obtain a permanent injunction, a claimant must establish the following elements.

a. Inadequate legal remedy alternative

A claimant must first establish that any legal remedy alternative is inadequate. In this case, the neighbors will argue that a money damages remedy would be inadequate because it would necessitate the filing of multiple suits. The harm that Diane is inflicting by constructing the dam -- i.e., stopping the flow of the water to neighbors downstream who rely on the stream to irrigate their crops and fill their wells -- affects multiple parties and is ongoing, therefore giving rise to multiple suits. Moreover, the neighbors will argue that a money damages remedy would be inadequate because it would be difficult to assess damages. It may be difficult, for instance, to establish how much damages they will sustain as a result of not being able to irrigate their crops. It may also be difficult to determine how much it would cost to obtain such water from other sources. Finally, the dam may be the neighbors’ only source of water, and, therefore, the award of any amount of money damages may be inadequate (i.e., the stream is unique). Therefore, the neighbors will likely satisfy this element.

b. Property right/protectable interest

Traditionally, permanent injunctions only protected property rights. However, the modern view holds that any protectable interest is sufficient. In this case, the neighbors likely have a property right in the stream to the extent that the stream flows through their respective properties. Even if they do not have a property right, however, they still have

75 a protectable interest stemming from their right to use water from a stream that runs through their property. Thus, this element is likely satisfied.

c. Feasibility of enforcement

There is usually no enforcement problem in the case of negative injunctions (i.e., court orders mandating that a person refrain from performing a specific act). Mandatory injunctions (i.e., court orders mandating that a person perform a specific act) present greater enforcement problems. For instance, a court may be unwilling to grant a mandatory injunction if: (a) the mandated act requires the application of taste, skill or judgment; (b) the injunction requires the defendant to perform a series of acts over a period of time; or (c) the injunction requires the performance of an out-of-state act.

In this case, the neighbors seek both a negative injunction (i.e., order requiring Diane to immediately stop construction of the dam) and mandatory injunction (i.e., order requiring Diane to remove the dam). There will be little enforcement problem in ordering Diane to immediately stop construction of the dam. There will likewise be little enforcement problem in ordering Diane to remove the dam since both Diane and the dam are within the court’s territorial jurisdiction, and the injunction does not require Diane to perform an out-of-state act. Therefore, the neighbors will satisfy this element.

d. Balancing of hardships

The court will balance the hardship to the neighbors if a permanent injunction is not granted against the hardship to Diane if a permanent injunction is granted. Unless the hardship to Diane greatly outweighs the hardship to the neighbors, a court will likely not grant a permanent injunction. In this case, Diane will suffer little hardship if the permanent injunction is granted because the pond was intended to be used for a free summer day camp. Therefore, the only economic harm she will suffer as a result of this injunction is the money she has already expended in constructing the dam and any additional amount she will incur in removing the dam if the injunction is granted.

76

However, the neighbors will suffer substantial harm if the injunction is not granted and the dam is completed. They rely on the stream to irrigate their crops and to fill their wells and will likely suffer substantial damage if they either cannot obtain substitute water from another source or must pay significant amounts to obtain any substitute. Thus, the hardship to the neighbors if a permanent injunction is not granted greatly outweighs the hardship to Diane if a permanent injunction is granted, and a court is more likely to grant the injunction.

e. Defenses

Diane may raise the defense of laches and argue that the neighbors delayed in bringing the permanent injunction action, thereby prejudicing her. The laches period begins the moment the neighbors know that one of their rights is being infringed upon. In this case, the neighbors knew six months before they filed an application in state court for a permanent injunction that Diane was constructing a dam and that such construction infringed on their right to obtain water from the stream. By waiting these six months to bring suit, Diane incurred substantial construction expenses in building the dam that could have been avoided if the neighbors had brought the suit sooner.

Thus, Diane will likely be able to successfully assert this laches defense.

In the end, a court may still grant the neighbors the injunction and order Diane to remove the dam. However, the court may require the neighbors to compensate Diane for any construction expenses that could have been averted if the neighbors brought the suit sooner.

77

2. Denial of Paul’s application for permanent injunction

Claim preclusion

Once a court renders a final judgment on the merits with respect to a particular cause of action, the plaintiff is barred by res judicata (i.e., claim preclusion) from trying that same cause of action in a later suit. I will examine each element of claim preclusion, in turn, below:

a. Final judgment on the merits

The court must have rendered a final judgment on the merits in the prior action. For federal court purposes, a judgment is final when rendered. For CA state court purposes, a judgment is not final until the conclusion of all possible appeals. In this case, Paul is filing his case in federal court. Since judgment was rendered by the state court in the prior action, the judgment is considered final.

A judgment is “on the merits” unless the basis for the decision rested on: (a) jurisdiction; (b) venue; or (c) indispensable parties. In this case, the state court’s decision did not rest on any of these grounds. Therefore, the judgment was on the merits.

b. Same parties

The cause of action in the later suit must be brought by the same plaintiff against the same defendant. In this case, Paul was one of the plaintiffs in the prior state court case, and the suit is brought against Diane, who was the same defendant in that prior case. Therefore, this requirement is also met.

78

c. Same cause of action

The cause of action in the later suit must be the same cause of action asserted in the prior suit. In general, if causes of action arise from the same transaction or occurrence, a claimant must assert all such causes of action in the same suit. However, under CA’s “primary rights doctrine,” a claimant may separate the causes of action into separate suits so long as each suit involves a different primary right (e.g., personal injury vs. property damage).

In this case, Paul is asserting the same permanent injunction claim based on nuisance and taking grounds that he asserted in the prior state court action. He is also requesting the same relief as in the state court case. He is not asserting a different primary right, and, thus, the “primary rights doctrine” is inapplicable. Therefore, this requirement is likewise met.

d. Actually litigated or could have been litigated

The same cause of action must have either actually been litigated or could have been litigated in the prior action. This requirement is met because the permanent injunction cause of action based on nuisance and taking grounds was actually litigated in the prior action.

In the end, Paul will [be] barred by res judicata (i.e., claim preclusion) from trying the permanent injunction cause of action against Diane in federal court, and the court was correct in granting Diane’s motion to dismiss.

79

3. Lawyer’s ethical violations

Confidentiality

Under both ABA and California rules, a lawyer has a duty not to reveal any information related to the representation of a client. However, several exceptions may nonetheless permit a lawyer to reveal such confidential information. First, a lawyer can reveal confidential client communications if the client gives the lawyer informed consent to do so. In this case, Paul has not given Lawyer such informed consent, and, therefore, this exception does not apply. Second, a lawyer can reveal confidential client communications if he is impliedly authorized to do so in order to carry out the representation. Again, this exception does not apply here. Third, under the ABA rules, a lawyer can disclose confidential client communications if he reasonably believes it is necessary to prevent a person’s reasonably certain death or serious bodily injury. Under the CA rules, however, a lawyer can disclose such information only to prevent a criminal act that is likely to lead to death or serious bodily injury. The lawyer must first make a good faith effort to convince the client not to commit the criminal act and, if the client refuses, then the lawyer must inform the client of his intention to reveal the client’s confidences.

In this case, Paul told Lawyer that he “will take care of Diane in my own way” after becoming infuriated with the court’s ruling on his permanent injunction application. On the one hand, Paul’s statement is too unclear and ambiguous to provide any indication of what specific harm he intended to inflict on Diane. On the other hand, Lawyer will argue that he reasonably believed that Paul intended to inflict serious bodily harm on Diane, as evidenced by his infuriation after the ruling. Lawyer was so convinced that Paul intended serious harm to Diane that he told the 911 dispatcher that Paul was “on his way to hurt Diane.” In the end, a disciplining body would likely hold that Lawyer was reasonable in his belief that Paul intended to cause death or serious bodily injury to Diane and, therefore, his disclosure of Paul’s confidential communications was permissible. The killing or injuring of a person also constitutes a criminal act, and since

80

Lawyer first made a good faith effort to dissuade Paul from committing any harm against Diane, Lawyer’s revelation of this confidential information would also not subject Lawyer to discipline in CA.

Fourth, under the ABA rules only (i.e., CA has no equivalent rule), a lawyer may disclose confidential client communications to prevent a crime of fraud that is likely to produce substantial financial loss to a person, so long as the client was using the lawyer’s services to perpetrate the crime or fraud. In this case, Paul threatened to “take care… of that dam.” While this threat may result in substantial financial loss to Diane, the threatened act did not involve the use of Lawyer’s services. Therefore, this exception does not apply. Nonetheless, as discussed above, Lawyer should escape discipline for his revelation of client’s confidential communications under the “death or serious bodily injury” exception.

81

ESSAY QUESTIONS AND SELECTED ANSWERS JULY 2011 CALIFORNIA BAR EXAMINATION

This publication contains the six essay questions from the July 2011 California Bar Examination and two answers to each question that were written by actual applicants who passed the examination after one read.

The selected answers were assigned good grades and were transcribed for publication as submitted, except that minor corrections in spelling and punctuation were made for ease in reading. The answers are reproduced here with the consent of their authors.

Question Number Contents Page

1 Criminal Law and Procedure 4

2 Civil Procedure 18

3 Contracts 32

4 Professional Responsibility 46

5 Real Property 57

6 Community Property 70

2

Question 2

Doctor performed surgery on Perry’s spine to insert a metal rod designed by Bolton, Inc. (Bolton). Shortly after the surgery, Perry developed severe back pain at the location where the rod was inserted. Within the applicable statute of limitations for a tort action for negligence, Perry sued Doctor in federal district court, alleging that she was negligent in using Bolton’s rod for the kind of back condition from which he suffered. Personal jurisdiction, subject matter jurisdiction, and venue were proper.

During a deposition, Perry’s attorney asked Doctor to state whether she had performed any other spine surgeries using Bolton’s rods and, if so, whether any of those surgeries had resulted in complications. Doctor’s attorney objected to the questions on the ground that the information requested had nothing to do with whether Doctor was negligent as to Perry, and Doctor refused to answer. After the attorneys properly met and conferred concerning Doctor’s refusal, Perry’s attorney filed a motion to compel Doctor to answer the questions.

Shortly after the statute of limitations had run, Perry learned through a newspaper article that Bolton had been sued by several patients who alleged that they suffered severe back pain after Bolton’s rod was inserted into their spines during surgery. Perry immediately sought and obtained leave to amend his federal complaint to join and include a claim against Bolton, alleging that it had negligently designed the rod. Bolton immediately filed a motion to dismiss Perry’s claim against it on the ground that the statute of limitations had already run.

Perry also learned that Doctor had lost a lawsuit brought by another patient with a back condition like his who had also alleged negligence by Doctor for inserting Bolton’s rod into his spine. Perry filed a motion for summary judgment against Doctor on the basis of preclusion.

1. How should the court rule on Perry’s motion to compel Doctor to answer? Discuss.

2. How should the court rule on Bolton’s motion to dismiss Perry’s claim on the ground that the statute of limitations had run? Discuss.

3. How should the court rule on Perry’s motion for summary judgment? Discuss.

17

Answer A to Question 2

Perry v. Doctor 1. Perry's Motion to Compel Doctor to Answer

Discovery provides fact-gathering tools for parties to obtain relevant evidence to the case. The scope of discovery is broad but not limitless. A party may only discover relevant evidence/information or facts reasonably calculated to lead to relevant evidence/information. However, a party may not discover privileged information. Therefore, the scope is broader than evidence admissibility as not only relevant evidence is discoverable, but also those that provide a good lead to relevant information. Deposition is one of these fact-gathering tools.

During a deposition, typically there is not judge present, but attorneys should still make proper objections for the future purpose of excluding any answers at trial. If an attorney does not make an objection during deposition to a question or an answer, it is considered waived and the same question cannot be objected to in the future. If the opposing attorney later wishes to admit the objected question and/or answer at trial, the judge will determine whether the attorney's objections at the deposition should be sustained or overruled. At the deposition, because there is no judge present to determine the objection, the witness must still answer because, as stated above, all relevant evidence or facts reasonable calculated to lead to relevant evidence/information is discoverable. One deposed does not have to answer a question unless the answer would reveal privileged information.

Here, Perry's attorney asked Doctor to state whether she had performed any other spine surgeries using Bolton's rods, and if so, whether any of those surgeries had resulted in complications. Doctor's attorney objected on the basis of relevance - asserting that the information requested had nothing to do with whether Doctor was negligent as to Perry, so Doctor refused to answer. The question asked is a relevant question because Perry is suing Doctor for negligence. In this case, Doctor performed surgery on Perry's spine to insert a metal rod designed by Bolton. Shortly after the surgery, Perry developed severe back pain where the rod was inserted. If Doctor has

18

performed similar surgeries with the same Bolton's rods in the past and those have also resulted in complications, that would be relevant to the question of Doctor's duty and breach of duty. For example, if Doctor had performed similar surgeries and had resulted in complications, then a reasonable doctor with similar skill, knowledge and experience as another doctor in the same field of profession may decide to no longer use Bolton's rods or to improve or change his technique so as to avoid future surgical complications. The answer to this question could reasonably lead to Perry's attorney finding other patients for relevant information and may also discover the techniques used during those surgeries. Thus, the information is relevant and reasonably calculated to lead to relevant information. Thus, this information is discoverable, and the Doctor must answer. Because the attorneys properly met and conferred concerning Doctor's refusal to answer and could not come to an agreement, the motion to compel was properly filed. The court should grant Perry's motion to compel Doctor to answer.

Doctor may argue that revealing such information would violate doctor-patient confidentiality, thus privileged information. However, the question simply asked whether Doctor had performed other spine surgeries using Bolton's rods, and whether any of those surgeries had resulted in complications. The answer only requires a yes or no answer. The answer would not require Doctor to reveal any patient's names or medical conditions. Thus, Doctor's argument would fail.

The court should grant Perry's motion to compel.

2. Motion to Dismiss Perry's Claim on the ground that Statute of Limitations had run

Assuming that this is a diversity of citizenship case in federal court (because negligence is typically not a federal question), the Federal court must apply the Erie doctrine where Federal Rules of Civil Procedure (FRCP) for procedure and apply state law for substantive law. Courts have established that in a diversity case, the state statute of limitations must be used as that is considered substantive law. Under FRCP, a party has 14 days to amend a complaint after original filing.

To amend a complaint after the statute of limitations had run, the complaint must relate back to the original complaint that was filed before the statute of limitations had

19

run. To relate back to the original complaint while adding a defendant on the amended complaint, 3 elements must be satisfied: 1) the claim arises out of the same transaction or occurrence as the original complaint, 2) the new party knew of the original action within 120 days of filing, and 3) the new party, but for the mistake, knew that they should have been named as the original party.

Here, the original complaint was filed in federal district court within the applicable statute of limitations for a tort action for negligence. Perry alleges that Doctor was negligent in using Bolton's rod for the kind of back condition from which he suffered.

Same transaction or occurrence

The amended complaint includes a claim that alleged that Bolton negligently designed the rod. Because Perry was suing Doctor for negligence for using Bolton's rod during surgery for Perry's kind of back condition, the new claim against Bolton arises out of the same occurrence because they both arose out of Doctor's surgery and inserting Bolton's rod. Thus, this element is satisfied.

New party knew of original action

It is unclear whether Bolton knew of the original action where Perry sued Doctor for negligence. If doctors are sued based on products they used on patients, it is not unusual that doctors would seek indemnification or contribution from the manufacturers of those products. Thus, if Doctor informed Bolton of Perry's lawsuit (or if Bolton somehow was aware of it) within 120 days of filing, then this element is satisfied. Otherwise, this element is not satisfied.

But for the mistake, Bolton knew they should have been named

There is no indication that Perry had originally wanted to file a claim against Bolton. Perry's original claim was that Doctor negligently used Bolton's rods for his type of injury, thus alleging that Bolton's rods were wrongly used. Perry did not allege that anything was actually wrong with the rod itself. Therefore, there appears to be no mistake regarding the identity of the defendant. In fact, there is no facts to suggest that Perry had even considered suing Bolton for negligence until Perry learned through a

20

newspaper article that Bolton had been sued by several patients who alleged that they suffered severe back pain after Bolton's rod was inserted into their spines during surgery. Clearly, Perry did not make a mistake as to the defendant when he filed the original claim prior to reading this newspaper. Only after reading the newspaper did he "immediately sought" to amend his complaint. The evidence shows that there is no mistake as to the identity of the defendant in Perry's suit.

Even if Bolton was aware of the suit, no indications on the claim would lead Bolton to believe that Doctor had originally meant to sue Bolton instead of Doctor. Therefore, this element is not satisfied.

The court should grant Bolton's motion to dismiss because the applicable statute of limitations had run and the amended complaint does not relate back.

3. Perry's Motion for Summary Judgment

Motion for summary judgment will be granted if the court determines there is no dispute of fact in the case. The court may look at evidence when making such a determination.

Claim Preclusion (res judicata)

To assert claim preclusion, 3 elements must be satisfied: 1) same claimaint vs. same defendant in both case #1 and #2, 2) case #1 ended in a valid final judgment on the merit (which means it did not end based on jurisdiction, venue or indispensable party), and 3) the claimant is asserting the same claim as case #1 (same claim usually means arises out of the same transaction or occurrence).

The first lawsuit was brought by another patient, not Perry. Thus, the first element requiring the same claimant and defendant fails because Perry was not the plaintiff in the first case, as he is in the second case. Although it appears that case #1 ended in valid final judgment on the merits, case #1 did not assert the same claim because it is not the same transaction or occurrence. The previous patient's claim arises under his individual surgery, and Perry's claim arises out of his own separate surgery. Thus, claim preclusion should not be asserted.

21

Issue preclusion (collateral estoppel)

To assert collateral estoppel, 5 elements must be satisfied: 1) case #1 ended in a valid final judgment on the merits, 2) the issue was actually litigated in case #1, 3) the issue was essential to the judgment (if the issue was decided differently, the case would have ended differently), 4) collateral estoppel is being used against one who was a party in case #1, and 5) collateral estoppel is being used by one who was a party in case #1 (satisfies mutuality requirement in those jurisdictions who require it), one who was not a party in case #1 but is a defendant in case #2 if plaintiff actually litigated the issue in case #1, and one who was not a party in case #1 but is a plaintiff in case #2 if it is fair. Collateral estoppel may be used by nonparties in case #1 because many jurisdictions have found that not complying the mutuality requirement does not violate due process.

Valid Final Judgment

No facts suggest that the first case did not end in final valid judgment on the merit. No facts state that case #1 ended based on jurisdiction, venue or indispensable party. Thus, if it did not end in one of these bases, then it ended in valid final judgment, and this element is satisfied.

Issue actually litigated

The facts state that "Doctor had lost a lawsuit brought by another patient... who also alleged negligence for inserting Bolton's rod into his spine." Therefore, it appears that the issue of negligence was actually litigated. If it was, this element is satisfied.

Issue essential to judgment

Because the previous patient brought an action based on negligence, the issue of negligence was likely essential, and if the court or jury in case #1 had found Doctor not to be negligent, then the outcome of case #1 would have been different. Thus, this element is satisfied.

Used against party in case #1

22

Perry is asserting issue preclusion against Doctor, who was the defendant in case #1 because in the previous case, Doctor was sued by another patient. Doctor is a current defendant in Perry's case and was a defendant in case #1. Thus, this element is satisfied.

Used by nonparty in case #1 but plaintiff in case #2

For Perry to assert issue preclusion, the use of issue preclusion must be fair. Here, Perry would argue that it is fair because the previous plaintiff/patient's injuries had a back condition like Perry's and Doctor inserted the same Bolton's rod into his spine, just like Doctor did with Perry. However, this argument would likely fail. Doctor would argue that although the previous patient in case #1 had a "back condition like" Perry's, medical conditions/injuries, especially back injuries, are almost never exactly the same. Its causes may be different and its symptoms may be different, which would call for different treatment. Thus, even if Perry and the previous patient had similar injuries, its causes, symptoms and other factors may require Doctor to use different technique or treatment. Or even if the same technique was used, each patient may react different based on the patient's physiology even without any negligence on the part of Doctor. Therefore, it would not be fair to preclude Doctor from litigating the issue of negligence in Perry's case based on Perry's injuries/condition, causes of Perry's injuries/medical condition, and techniques used during Perry's surgery. Because it would be unfair to preclude Doctor from litigating the issue of negligence in Perry's lawsuit, this element is not satisfied.

Thus, the court should deny Perry's motion for summary judgment.

23

Answer B to Question 2

1-Motion to compel Doctor (D) to answer Scope of discovery- relevance

During discovery, both parties to a lawsuit may engage in discovery through depositions, interrogatories, requests for production, requests for admissions, and other discovery devices any evidence that is relevant to the lawsuit. Relevance is a low standard and it just requires that the evidence sought to be discovered be likely to lead to the discovery of any admissible evidence relevant to a claim or defense in the subject case. Here, Perry (P) is bringing a lawsuit against D for negligence. Negligence is a tort action that requires the plaintiff to establish 1) duty, 2) breach, 3) actual causation, 4) proximate causation, and 5) damages.

Here, P is seeking discovery of whether D had performed any other spine surgeries using Bolton's rods, and if so, whether any of these surgeries resulted in complications. Although D is arguing that this information has nothing to do with whether D was negligent as to Perry, this evidence is relevant to the issues of duty and breach- which P will have to establish as part of his prima facie negligence lawsuit.

Although this information does not involve P, it is relevant to duty because it helps determine what standard of care D should be held to. A physician is typically held to the standard of care of an average member of his profession in good standing. Thus, D will be held to the standard of care of an average back surgeon in good standing. Here, if D in fact used Bolton's rods before and these surgery's resulted in complications, this would indicate that D should have warned P about these complications. An average back surgeon in good standing would warn his patients of complications that occurred when the doctor performed similar surgeries on other patients.

This information is also relevant to breach. In order to establish breach, a plaintiff has to establish that the defendant fell below the applicable standard of care. Here, if other spine surgeries using the same rod had led to complications, this would be relevant to whether D fell below his standard of care because either he did not inform P of these

24

complications (which he should of done) or because he in fact used this rod for the spine surgery knowing that it had a potential to lead to complications.

Thus, the evidence that P is seeking is relevant to his negligence theory and should have been discoverable.

Privilege/ work product

Relevant evidence is discoverable unless there the party against whom the discovery is sought can claim a privilege such as doctor-patient confidentiality or work product privilege. Here, D will have to answer P's request unless he can claim either of these privileges.

Although many jurisdictions, including CA, recognize the doctor-patient privilege, the federal courts do not. Here, P is suing D in federal district court thus the doctor-patient privilege does not apply and D will not be able to assert it to avoid his discovery obligations to P.

Work product privilege protects the work of the attorney and parties that is down in anticipation of litigation. Here, there is no indication that D's attorney complied a list of other spine surgeries in anticipation of this litigation, thus D will not be able to claim the work product privilege.

Conclusion

Because the evidence as to other spine surgery complications is relevant to P's negligence claim against D and not subject to any privilege, the court should grant P's motion to compel D to answer his deposition question.

2-Motion to dismiss on ground that statute of limitations (SOL) had run

Generally, a plaintiff must file his complaint with all claims and all defendants within the applicable SOL. There are 2 limited exceptions, outlined below, where there plaintiff may 1) add a new claim and 2) add a new defendant after the SOL has run. In these situations, the new claim/ new defendant will "relate back" to the original complaint and the date that this original complaint was filed. This way, if the original complaint was

25

filed within the applicable SOL, the plaintiff will be able to avoid the SOL problem with his new claim/ defendant.

Here, P filed a suit against doctor within the applicable SOL, thus whether P can add the claim against Bolton depends on whether it "relates back."

Relation back- Amendment of pleadings to add a claim

A plaintiff may amend his complaint to add a new claim after the SOL has run if the claim arises out of the same transaction or occurrence as his original claim against the original defendant. Here, P wants to add a claim against Bolton based on negligent design of the rod. P's original claim is against the doctor for negligence in using this rod. Thus, P's claim against Bolton arises out of the same transaction or occurrence as his original complaint- both the new claim against Bolton and the original claim against D arise out of the back surgery/ rod insertion that led to P's severe back pain. Thus, P will be able to amend his complaint to add this claim.

Relation back- Amendment of pleadings to add a defendant

A plaintiff may amend his complaint to add a new defendant after the SOL has run only in very limited circumstances. The plaintiff must establish 1) that his claim against this new defendant arises from the same transaction or occurrence as the original complaint, 2) that the new defendant knew about the original action within 120 days of its filing, and 3) that the defendant knew that, but for a mistake, he would have been originally named in the plaintiff's original complaint.

Here, P wants to include a claim against Bolton, alleging that it had negligently designed the rod that D placed in his back during the spin surgery. P's original claim is against the doctor for negligence in using this rod. Thus, P's claim against Bolton arises out of the same transaction or occurrence as his original complaint- both the new claim against Bolton and the original claim against D arise out of the back surgery/ rod insertion that led to P's severe back pain. Thus, this first element is satisfied.

Here, P will also have to establish that Bolton knew about his claim against D within 120 days of its filing. Here, there is no indication that Bolton received a copy of P's complaint

26

against D or had any notice that P brought a claim against D as a result of his surgery. Thus, unless P can establish that Bolton knew about the lawsuit, he will not be able to establish this element.

Here, P will also have to establish that Bolton knew that he made a mistake and that he would have originally named Bolton but for the mistake. Here, P will try to argue that Bolton had been sued by several other patients who alleged that they suffered severe back pain after Bolton's rod was inserted during spine surgery. Thus, P will argue that Bolton knew that P should have filed the lawsuit against it. However, P will not be able to establish this element. P did not make a mistake and negligently name the wrong defendant- rather, he named Doctor who is likely a proper defendant and then subsequently named Bolton after he learned more information. He did not even learn this information through discovery/ deposition of Doctor- he learned it by reading a newspaper article. This is not a situation where the plaintiff completely puts the wrong name in the applicable line of his complaint. P did not make a mistake at the time of his complaint and rather learned about a potential claim against Bolton too late. He will be barred by the SOL.

Conclusion

Here, P's new claim will relate back to his original complaint. However, his addition of Bolton as a new defendant will not relate back to the original complaint and P will not be able to add his claim against Bolton. Thus, the court should grant Bolton's motion to dismiss P's claim on the ground that the SOL had run.

3-Motion for summary judgment based on preclusion

A motion for summary judgment requires the moving party to establish 1) there is no genuine dispute of material fact and 2) he is entitled to judgment as a matter of law. Preclusion is a common ground for a motion for summary judgment because it involves the judgments of prior lawsuits so there is genuinely no dispute as to material fact (the outcome of these lawsuits).

Res judicata/ claim preclusion

27

Res judicata (RJ) bars a subsequent lawsuit (lets call it case 2) when there is a prior lawsuit (call it case 1) and 1) case 1 and case 2 involve the exact same parties (the exact same plaintiff and the exact same defendant), 2) case 1 ended in a final judgment on the merits, and 3) case 2 involves the same transaction or occurrence as case 1.

Here, P will not be able to assert RJ against Doctor. Here, case 1 is the lawsuit brought by another patient against Doctor. Case 1 involved a negligence action for inserting a Bolton rod into his back. Although P's lawsuit (case 2) is very similar, RJ will not apply because P was not a party to the prior case 1. RJ requires the exact same persons to be parties to both the first case and the second case. Although D was a party to the first case, P was not, thus he will not be able to assert RJ against D.

Collateral estoppel/ issue preclusion

Collateral estoppel (CE) bars a subsequent lawsuit (case 2) when 1) case 1 and case 2 involve the same issue, 2) this issue was actually litigated and decided in case 1, 3) this issue was essential to the judgment in case 1, 4) issue preclusion is being asserted in case 2 against a party who was a party in case 1, and 5) traditionally, is being asserted by a party who was a party in case 1 (mutual collateral estoppel) but modernly, does not to be asserted by a party who was a party in case 1 (non mutual collateral estoppel).

Here, case 1 (the lawsuit by the other patient) and case 2 (P's negligence suit against D) will likely involve similar issues. They are both negligence suits so they will both have issues such as 1) what was the defendant doctor's standard of care? 2) did the defendant breach this standard of care by installing a Bolton rod in his patient's spine, 3) did the insertation of the Bolton rod cause the patient to suffer subsequent back pain, etc. Thus, case 1 and case 2 will involve many of the same issues, and this first element will be satisfied.

Because Doctor lost the first negligence lawsuit, many of these issues will also have been litigated and decided, thus there are a number of issues which will have been actually litigated and decided in case 1, thus P will likely be able to satisfy the second element of CE.

28

Similarly, many of these issues would have been essential to the judgment in case 1. A plaintiff has the burden of establishing all of his prima facie negligence elements so each of these issues would have been essential to the first patient prevailing in his negligence suit against Doctor, thus this third element will likely be satisfied.

P is also asserting issue preclusion in case 2 against Doctor, who was a party in case 1. Thus, issue preclusion is being asserted against a party who was a party in case 1 and the fourth element is satisfied.

For the fifth element, traditionally, mutual collateral estoppel was required and CE could only be asserted by a party who was also a party to case 1. However, unlike the fourth element (which is required by due process), due process does not require the party who is asserting CE to be a party to case 1. Thus many jurisdictions allow nonmutual use of collateral estoppel. The standard that must be met depends on whether the party is asserting CE as a plaintiff or as a defendant. If the party is asserting it as a defendant (defensive collateral estoppel) the court will apply CE to bar further litigation of this issue as long as the plaintiff had a full and fair opportunity to litigate the issue in case 1. However, if the party is asserting it as a plaintiff (offensive collateral estoppel) the court will be more reluctant to apply CE and will look at a number of factors- 1) did the defendant have a full and fair opportunity to litigate the issue in case 1, 2) could this new plaintiff have joined case 1, 3) could the defendant have foreseen multiple lawsuits, and 4) are there any inconsistent judgments so that assertion of CE could be unfair to the defendant.

Here, P was not a party to case 1 however he still may be able to use nonmutual collateral estoppel since most courts have got rid of the mutuality requirement. P is a plaintiff and he is the one asserting CE against D. Thus, P is trying to make offensive use of CE. The court will look at a number of factors- whether Doctor had a full and fair opportunity to litigate the first lawsuit against the other patient. Whether Perry could have joined the first negligence lawsuit involving the Bolton rod- did P know of this claim at the time it was brought? Whether the doctor could have foreseen that there would be multiple lawsuits like this- here multiple patients had sued Bolton from back pain they suffered so D likely could have foreseen that plaintiffs would bring lawsuits against him

29

as well for use of the rod. And finally, whether there are inconsistent judgments against Doctor. Here, this appears to be the only other lawsuit against this particular doctor involving negligent use of the rod thus unless there are other lawsuits where the Doctor prevailed on this issues, there are unlikely to be inconsistent judgments.

Conclusion

The court should dismiss Perry's motion for summary judgment as to his claims of res judicata but likely should grant his motion as to his claims of collateral estoppel for a number of negligence issue (outlined above) depending on the factors (outlined above).

30

ESSAY QUESTIONS AND SELECTED ANSWERS JULY 2012 CALIFORNIA BAR EXAMINATION

This publication contains the six essay questions from the July 2012 California Bar Examination and two answers to each question that were written by actual applicants who passed the examination after one read.

The selected answers were assigned good grades and were transcribed for publication as submitted, except that minor corrections in spelling and punctuation were made for ease in reading. The answers are reproduced here with the consent of their authors.

Question Number Contents Page

1 Civil Procedure 4

2 Community Property/Professional Responsibility 22

3 Evidence 40

4 Contracts 58

5 Wills and Succession 71

6 Criminal Law and Procedure 85

2

Question 1

Pam and Patrick are residents of State A. While visiting State B, they were hit by a truck owned and operated by Corporation, a freight business.

Corporation is incorporated under the laws of Canada and has its headquarters there, where its President and Secretary are located. State B is the only state in which Corporation conducts its business. Corporation’s drivers and other employees work out of its warehouse in State B.

Pam and Patrick jointly filed a lawsuit against Corporation in federal district court in State A. In their complaint, Pam demanded damages for personal injury in the amount of $70,000 and for property damage in the amount of $10,000; Patrick demanded damages in the amount of $6,000.

Corporation filed a motion to dismiss the complaint for lack of personal jurisdiction. The federal district court denied the motion. After trial, it entered judgment for Pam in the amount of $60,000 and for Patrick in the amount of $4,000.

Corporation has appealed on the grounds of lack of subject matter jurisdiction and lack of personal jurisdiction.

How should the court of appeals rule on each ground? Discuss.

4

ANSWER A TO QUESTION 1

1. Did the Federal District Court of State A have Personal Jurisdiction over Corporation?

Waiver? Personal Jurisdiction is waived if not challenged. Here, Corporation ("Corp") filed a motion to dismiss for lack of personal jurisdiction ("PJ") at trial. Therefore, Corp did not waive its right to appeal based on lack of PJ.

Personal Jurisdiction Personal jurisdiction is the power of a court to have jurisdiction over an individual or entity. Here, a corporation. The exercise of personal jurisdiction must comport with the requirements of Due Process.

TRADITIONAL BASIS Traditionally, PJ could only be exercised if the defendant consented to suit in the forum, was served in the forum, or was domiciled in the forum. Here, there are no facts to indicate that Corp consented to jurisdiction ("JDX") because they did not make a general appearance, or in any way consent. Further, Corp is not domiciled in the forum. A corp is domiciled where it has its principal place of business, based on nerve center, and where it is incorporated. Both of those are in Canada for Corp. Finally, facts do not state where Corp was served or if they had an agent for service of process in State A, but assuming that they were not served in state A, there is no traditional basis in state A.

LONG ARM STATUTE If there is no traditional basis for the exercise of Personal jurisdiction, the court will next look to the state's long-arm statute to determine whether the court has jurisdiction to reach out to another state, or country to exercise jdx over the defendant. Here, there

5

are no facts to indicate that state A has a long-arm. If it did, the federal district court would have jdx to the same extent as the state.

CONSTITUTIONAL ANALYSIS To comport with due process, personal jurisdiction is only proper if the defendant has such minimum contacts with the forum state that the exercise of jdx comports with traditional notions of fair play and substantial justice.

MINIMUM CONTACTS Minimum contacts requires a showing of purposeful availment and foreseeability.

PURPOSEFUL AVAILMENT A party purposefully avails itself of the forum state if it has taken advantage of the benefits and protections of that state's laws. Here, Corporation is incorporated in Canada and has its headquarters there. Further, its warehouse is in state B. Further, Corp oprates a freight business and was driving in state B when the Accident occurred, and state B is the only state in which Corp conducts its business. There are no facts to show that Corporation had any contact at all with state A. Therefore Corp will argue it did not purposefully avail itself of the privileges and benefits of State A.

Foreseeability. Because State B is the only state in which corp does business, it will argue that it was not foreseeable that it would be haled into court in state A. P and P could argue that a trucking company should foresee being sued anywhere, but if the trucks are only on the road in state B, this argument will not likely prevail. It was not foreseeable that Corp would be sued in state A.

Relatedness of the claim to the contact The court will look at the quality and nature of the contacts. There is general jurisdiction if the defendant's contact is so systematic and continuous that he is essentially at home

6

in the forum. There is specific jurisdiction if the contact is less than systematic and continuous, but the claim arises out of the defendant's contact with the forum. Here, there is neither general or specific jdx because the claim neither arises out of the contact with the forum nor is Corp "essentially at home" in the state A because its contact there is not systematic and continuous.

Specifically, the accident occurred while Pam and Patrick were visiting in State B, not state A, and therefore the claim does not arise out of contact with State A, and there can be no Specific Jdx as a result.

Additionally, Corp only does business in State B, has its warehouse in state B and is incorporated and has its president, secretary and headquarters in Canada. Therefore there is no general jdx because there is no contact with State, and certainly not systematic and continuous contact.

Therefore, there is neither specific nor general Jurisdiction.

Fairness The fairness factors include the convenience of the parties and witnesses as well as the forum state's interest. The court will also look at the interstate judicial system's interest. Although state A has an interest in providing a forum for redress for its citizens, and Pam and Patrick are State A citizens, State B has a stronger interest because that is where the accident occurred, on its roads. Further, because Corp operates in State B only, state B has a strong interest in adjudicating the claims against its citizens for their conduct while in the state. As to convenience, any and all witnesses and evidence would be located in State B, rather than the forum, state A.

Therefore, the fairness factors are in favor of not finding PJ over Corp.

7

RULING: Therefore, Under a Due Process Analysis, The court of appeals should rule that there was no personal Jurisdiction over Corp.

2. Did the Federal District Court have Subject Matter Jurisdiction over the matter? Federal courts are courts of limited jdx and must have jurisdiction under arising under/federal question jurisdiction or diversity jurisdiction. In some cases, the court may be able to exercise supplemental jurisdiction. Federal District Court must have had jurisdiction over each and every claim in the matter. Here, both Pam and Patrick brought claims. Therefore, each claim is considered separately below.

WAIVER? Here, it does not appear that Corporation contested subject matter at trial. However, subject matter is not waived if the party fails to raise it at trial, and may be raised at any time, even on appeal. Therefore, Corporation could appeal based on this ground.

SUBJECT MATTER JURISDICTION OVER PAM'S CLAIM May have SMJ under either federal question or diversity. Here, the claim arises from personal injury, a tort claim, which is a state claim. Therefore, Pam must show diversity jurisdiction.

Diversity jurisdiction requires complete diversity between the parties (Strawbridge v. Curtiss) and an amount in controversy in excess of $75,000.

COMPLETE DIVERSITY Complete diversity requires that all plaintiffs are citizens of different states than all defendants. Or, as in the case here, that the suit be between a citizen of a state, and a foreign citizen.

8

A natural person is a citizen of the state in which she is domiciled. Domicile is physical presence plus intent to remain indefinitely. Here, facts state that Pam is Resident of State A. Therefore, Pam will be domiciled in state A.

A corporation is a dual citizen of every state in which it is incorporated and the state which is its principal place of business. ("PPB") PPB is determined by the "nerve center," or the place from which corporate managers run the corporation. (Hertz v. Friend).

Here, Corp is incorporated under the laws of Canada and thus is a citizen of Canada. Further, Corp has its "nerve center" in Canada because that is where its headquarters is located and where its President and Secretary are located. Although Corp keeps a warehouse in state B and its drivers and other employees work out of the warehouse in state B, no facts indicate that any direction of corporate activity occurs here. Therefore, this is the muscle center, not the nerve center, and the Supreme Court ruled that the Nerve center is the PPB.

Therefore, Corp will be deemed a citizen of Canada, meaning that it is a foreign citizen. Because Pam is a citizen of State A and Corp is a Foreign Citizen of Canada, there is complete diversity between the parties.

AMOUNT IN CONTROVERSY The amount in controversy must exceed $75,000 exclusive of interests and cost. The plaintiff's good faith claim will control, unless it is clear to a legal certainty that plaintiff cannot recover the required amount (in excess of $75,000). Here, Pam demanded damages of $70,00 for personal injury and $10,000 for property damages. Neither amount alone satisfies the amount in controversy.

AGGREGATION Generally, aggregating claims is not required. However, a single plaintiff may aggregate all claims against a single defendant. This means that Pam can add together

9

her claims against Corp. Therefore, adding Pam's claims together, her good faith claim was for $80,000. Because there are no facts to indicate this amount was not in good faith, or that there is a legal certainty prohibiting Pam from this recovery (such as a statutory damages cap), Pam has met the amount in controversy.

RESULT IF PLAINTIFF RECOVERS LESS THAN THE AMOUNT IN CONTROVERSY: If the plaintiff recovers less than the amount in controversy, that will not defeat diversity jdx, because the good faith claim controls. However, in such a case, the plaintiff may be required to pay the defendant's fees in the litigation. Therefore, because Pam recovered on $60,000 that will not defeat diversity, but she may be liable for costs. RULING:

The federal district court had subject matter jurisdiction over Pam's claim by virtue of diversity jurisdiction. Therefore, the court should deny the appeal based on lack of SMJ over Pam's claim.

SUBJECT MATTER JURISDICTION OVER PATRICK'S CLAIM As above, this is a tort claim, not arising under federal law, and therefore the court will not have "federal question" jurisdiction. Therefore, Patrick will have to meet the requirements of diversity jurisdiction for the federal district court to have had SMJ. COMPLETE DIVERSITY

Like Pam above, Patrick is domiciled in state A and will therefore be a citizen of state A. Under the analysis above, Corp is a foreign citizen of Canada. Therefore, as above, there is complete diversity.

AMOUNT IN CONTROVERSY Patrick requested only $6000 in damages. This is less than $75,000 and therefore does not meet the amount in controversy. Patrick may not aggregate his claim together with Pam, because plaintiffs may not aggregate claims with other plaintiffs. Therefore, the court did not have diversity jdx over Patrick's claim.

10

SUPPLEMENTAL JDX Where the court has jurisdiction over one claim in a matter, it may exercise supplemental jurisdiction over other claims that arise from a Common Nucleus of Operative Fact. The common nucleus test is generally considered broader than the same transaction or occurrence test, and therefore any party that would meet the Same Tran. and Occ. test will meet the Common Nucleus of Operative Fact test.

Here, Pam and Patrick are both suing for injuries and damages arising from the same car accident. While visiting State B, they were hit by a truck owned by Corp, the same truck, in the same accident. The witnesses to both will be the same, as will the evidence. Therefore, Patrick's claim arises from a Common nucleus of operative fact with Pam's claim, and the federal district court could exercise supplemental jdx over Patrick's claim.

DIVERSITY LIMITATIONS ON SUPPLEMENTAL JDX However, where the underlying claim is in diversity, the court cannot exercise supplemental jdx over a claim by a plaintiff that would defeat complete diversity. Here, Patrick is a plaintiff. However, if supplemental jdx is exercised over Patrick’s claim it will not defeat complete diversity because all Plaintiffs will still be citizens of State A, and all Defendants of Canada.

Where the supplemental claim does not meet the amount in controversy, but will not destroy complete diversity, the court may exercise supplemental jurisdiction over the claim. Here, Patrick's claim did not meet the amount in controversy, but will not destroy complete diversity and therefore the court may exercise supplemental jdx over the claim.

DISCRETION In some cases, a federal district court should exercise discretion not to exercise supplemental jdx, such as where there is a novel or complex issue of state law, or state claims predominate, or all federal questions have been dismissed. On these facts, this

11

is a tort claim for personal injury and therefore not novel or complex. Further, the claim is in diversity and not federal question, and thus there is no concern about the federal claims being dropped out.

This is not a claim over which the court should decline supplemental based on the discretionary factors.

RULING The Federal district court had subject matter jurisdiction over Patrick's claim based on supplemental jurisdiction. Therefore, the appeals court should deny the motion on the basis of lack of subject matter jurisdiction.

12

ANSWER B TO QUESTION 1

Pam and Patrick v. Corporation Pam and Patrick have raised a claim against Corporation (C) in federal district court in State A. Corporation attempted to dismiss the case based on lack of personal jurisdiction (PJ) and subject matter jurisdiction (SMJ). These motions were denied, Pam and Patrick were awarded damages in the case, and Corporation has appealed the case on the grounds listed above. The following considers how the court of appeal should rule on these claims.

Subject Matter Jurisdiction Subject matter jurisdiction (SMJ) considers whether the court has the power to hear the particular case. This case was brought in federal court; federal courts are courts of limited power, unlike state courts, which can generally hear any case save for several exclusively federal categories. In order for federal courts to have proper SMJ over a case, the case must either be based on a federal question, or meet the requirements for diversity of citizenship jurisdiction. Each of these will be examined in turn to see if the federal courts have jurisdiction over this matter.

Federal Question A case may properly be held in federal court when the case is based on a federal question. This requires that the plaintiff assert a claim arising under the federal constitution or a federal law. The "well pleaded complaint rule" dictates that the claim be asserted in plaintiff's complaint. It is not enough that a federal issue generally be raised by the case, nor that the defendant will defend on the grounds of a federal law. Here, the case involves personal injury damages for the injuries that Pam and Patrick suffered when they were hit by a truck owned and operated by Corporation. Thus, it appears that the case is just a simple tort case, which would be based on state law, and not on the constitution or federal law.

Thus, there is no federal question here.

13

Diversity in Citizenship However, the federal courts have another means of jurisdiction available, in the form of diversity of citizenship. To be valid, all plaintiffs must be "diverse" in citizenship from all defendants, and the amount in controversy must exceed $75,000.

Diversity There is an absolute diversity requirement, wherein each plaintiff must be entirely diverse in citizenship from each defendant. The federal rules allow for diversity between citizens of different states, or citizens of a state and a foreign country. Citizenship for individuals is based on their domicile, which is evidenced by physical presence and intent to remain. For corporations, citizenship is determined by place of incorporation, and principal place of business, which is where the owners, directors, and management manage and direct the company's affairs.

Here, Pam and Patrick are residents of State A. Though the facts do not give us any hints into whether they have the intent to remain there, it is reasonable to presume that they did have that intent. Thus, their citizenship is State A.

C is incorporated under the laws of Canada and has its headquarters there, where its President and Secretary are located. Thus, the place of incorporation and the principal place of business is in Canada. Of note, Corporation's drivers and other employees work out of its warehouse in State B. Several years ago, this may have met the "muscle" test, and thus demonstrated citizenship in State B for C; however, this test has been done away with. Nonetheless, there still would be diversity in citizenship even if C was a citizen of State B.

Thus, there is diversity in citizenship, because Pam and Patrick are citizens of State A, and C is a citizen of Canada. Because the rules of civil procedure allow for diversity between residents of a state and a foreign country, there is proper diversity.

Amount in Controversy Next, the amount in controversy must exceed $75,000 excluding interest and attorney's fees. The court will examine this based off of a good faith pleading of damages by the

14

plaintiff. To reach this amount in controversy, any single plaintiff may aggregate as many claims together to meet the minimum requirement. However, multiple plaintiffs may not aggregate claims in order to reach the minimum requirement.

Here, Pam demanded damages for personal injury in the amount of $70,000, and for property damage in the amount of $10,000. This is an example of aggregation by one plaintiff against one defendant. This is proper. Further, because $70,000 plus $10,000 equals $80,000, it exceeds the amount in controversy requirement.

Patrick demanded damages in the amount of $6,000. This would not be able to be aggregated with Pam's claims in order to reach the amount in controversy; however, because Pam has reached the amount all on the basis of her own claims, this does not impact Patrick's claim. We will need to examine whether Patrick's claim can be joined, however. I will do this later under supplemental jurisdiction (see below).

The conclusion is that the amount in controversy is met, as Pam's claims exceed the required $75,000 minimum amount.

The Effect of Receiving Less Than $75,000 At End Of Trial C may argue that SMJ was invalid because Pam and Patrick ended up receiving less than $75,000 in damages at the end of the trial. This is incorrect. The mere fact that the parties recovered less than $75,000 at the end of the trial does not mean that the court loses jurisdiction, or never had it in the first place. All that is required is a good faith claim exceeding $75,000. Thus, this will have no effect on the question of SMJ.

Supplemental Jurisdiction: Pat's Claim of $6,000 As discussed above, Pat's claim alone did not meet the amount in controversy requirement. Each and every claim must meet the requirement in order to satisfy SMJ. When the amount in controversy is not met, we can look to supplemental jurisdiction to see if the claim can nonetheless get into federal court.

15

Supplemental jurisdiction requires that the claim contain a common nucleus of fact with the other claims asserted. If the claim arises out of the same transaction or occurrence, then this test is always met. Supplemental jurisdiction cannot be invoked when it would defeat complete diversity in a diversity case. Here, Pat's claim is based on the exact same incident as Pam - the accident with C's truck. Thus, it is the same transaction or occurrence, and will be able to be heard. The federal courts do have discretion to not hear these claims, but it is likely that they would hear this to get the whole case out of the way at the same time. Further, adding Pat's claim does not defeat diversity, because he is a citizen of State A.

Thus, Pat's claim can properly be heard in federal court.

When Can SMJ Be Asserted? Finally, we must consider at what point can SMJ be raised as an issue. Some claims must be asserted before certain stages of the trial in order to be preserved, and if not raised, then they are waived. SMJ, however, is never waived, as it is a strict requirement that the federal courts have subject matter jurisdiction. Thus, it is of no concern that C apparently has not raised the SMJ issue prior to the appeal; they can still properly raise it.

Conclusion: The federal court system has proper SMJ over Pam and Patrick's claim.

Personal Jurisdiction Personal jurisdiction (PJ) considers whether this state can properly hear this claim against this defendant. It asks whether the state has the power to force the defendant to come into the state to defend the claim. To examine whether PJ exists over C in State A, we must look to the traditional bases of exercising jurisdiction, the state long-arm statute, and to the constitutional limitations on exercising PJ.

16

Traditional Bases of Exercising Jurisdiction Traditionally, PJ can be asserted over a defendant if the defendant (1) is domiciled in the state; (2) consents to jurisdiction; or (3) is served with process while in the state. As discussed above, C is not domiciled in State A, but is rather domiciled in Canada. Further, it does not appear that C has consented to jurisdiction in State A in any way (though we will later talk about the need to timely raise the argument of lack of personal jurisdiction). And finally, there is no indication that C was served in State A.

Thus, the traditional bases of exercising jurisdiction seem to not be present.

Long-Arm Statute A long-arm statute is a state statute that states when the state can reach and "grab" an out-of-state defendant, and force the defendant to defend in the state court. Some of these long-arm statutes require that the defendant commit a tortuous act in the state, or break a contract in the state, while others simply grant the state the ability to reach out to grab defendants to the full extent as allowed by the U.S. Constitution.

Here, the facts do not mention the reach of State A's long-arm statute. It is reasonable to assume that it reaches the constitutional limits. Thus, we must examine the constitutional limits of PJ.

Constitutional Limitations To exert PJ over an out-of-state defendant, the constitution requires certain minimum contacts with the forum state such that maintenance of the suit there does not offend traditional notions of fair play and substantial justice. To determine if this is true in this case, we can break the above test down into three sections: minimum contacts, relatedness of the claim to the contact, and fairness.

Minimum Contacts The constitutional requires the defendant to have some minimum contacts with the forum state in order for the state to exert jurisdiction. The defendant must have

17

purposefully availed himself in the state, such that being subject to a claim in that state would be foreseeable.

Purposeful Availment Purposeful availment requires that the defendant commit a voluntary act in the forum state. Defendant must avail himself in some way to the state, whether it be by using the state's roads, or attempting to make money in the state.

Here, C is incorporated in Canada, and has its principal place of business there. It conducts business solely in State B, which is also where it has a warehouse. Further, the accident occurred in State B. It is possible that C drives on State A roads from time to time, but the facts do not give this information. Also, there are no facts which say that C ships goods to State A, or otherwise tries to make money there. Simply put, on these facts, there seem to be no contacts whatsoever with State A, other than that Pam and Patrick are residents of State A.

The court of appeal should find that there was no purposeful availment.

Foreseeability The minimum contacts must be sufficient enough to make it foreseeable that defendant would be "haled into court" in the forum state. Here, as discussed, there appears to be nothing that C did that would make it foreseeable that they would end up in State A. The mere fact of driving on State B's roads does not make it foreseeable that they would end up in State A's court. I suppose if State A were located directly adjacent to State B that it would perhaps be more foreseeable, but again, the facts do not share that information. A case against C in State A court was not foreseeable.

Relatedness of the Claim to the Contact The more related the claim is to the contact with the forum state, the more likely the court will be to allow for jurisdiction over the defendant.

18

If the claim arises directly out of the contact with the forum state, this gives rise to specific personal jurisdiction. Here, there was no contact with State A, and so there cannot be specific jurisdiction.

Next, general personal jurisdiction may exist if the defendant consistently and regularly conducts activity in the forum state, such that he is "essentially at home there." Merely selling goods in a state does not give rise to general PJ, there must be an actual physical presence. Here, C is not in State A whatsoever, or so it seems. Thus, it is not essentially at home in State A. It may essentially be at home in State B, where it has a warehouse, but this does not affect the discussion of whether State A has jurisdiction. The claim is not related to C's contact with State A, as C has no contact with State A.

Fairness Finally, the court will look to see if holding the suit in the state meets general standards of fairness. Under this, the court considers convenience to the parties and the witnesses, the forum state's interests, and the plaintiff's interests.

Convenience Under the convenience factor, the court will look to see how convenient it is to hold the case in the forum state, based on a variety of factors including where the parties are, where the witnesses are, where the evidence is, etc. If the inconvenience to the defendant grossly impacts his ability to defend against the case, the court will likely dismiss for lack of PJ.

Here, the accident occurred in State B, so any witnesses are likely in State B. It is unknown where the wreckage is located, but the vehicles are likely also in State B. Thus, a good portion of the pertinent materials needed would be in State B. Further, C has no connection with State A, and will have to travel there to defend against the suit. This is likely not entirely burdensome, because they are a corporation, and likely would have the resources to get there.

19

However, it was likely entirely inconvenient to have the case in State A, based on where the evidence, witnesses, and the defendant was located.

State's Interests Next, the court will look to see if the forum state has a strong interest in providing a forum for the claim. Here, State A is interested in providing a forum for its residents; it wants to be sure that they are compensated for their injuries. However, the accident occurred on State B's roads, and so State B would have more of an interest, because it wants to be sure that dangerous drivers are kept off of their roads. In the end, a court would likely find that State A has a limited interest in holding this case.

Plaintiff's Interests Finally, the court looks to the plaintiff's interests in having the case in the forum state. It is likely that Pam and Patrick have suffered some injuries and thus would prefer to not have to travel. However, they had already been in State B on vacation, and could likely travel there again if needed. The court generally will be deferential to the plaintiff's choice of forum, however.

In the end, it is likely that it is simply not fair to have C defend in State A court.

When Can PJ Be Asserted? On a final note, PJ must be asserted either in a 12b motion prior to the answer, or along with the answer. If not, it is waived. Here, it appears that C raised the PJ motion at some point early on, and thus likely did not waive it, so that it can be heard on appeal. Some courts require that a party immediately appeal a decision on PJ by way of an extraordinary writ.

20

Conclusion: The Court of Appeal should hold the court had SMJ over the matter, but not PJ. Thus, provided that PJ has not been waived, it should dismiss the case. If it has been waived, the court should reject the PJ argument as well.

21

THE STATE BAR OF CALIFORNIA

OFFICE OF ADMISSIONS 180 HOWARD STREET • SAN FRANCISCO, CALIFORNIA 94105-1639 • (415) 538 - 2303 1149 SOUTH HILL STREET • LOS ANGELES, CALIFORNIA 90015-2299 • (213) 765 – 1500

ESSAY QUESTIONS AND SELECTED ANSWERS

FEBRUARY 2013

CALIFORNIA BAR EXAMINATION

This publication contains the six essay questions from the February 2013 California Bar Examination and two selected answers to each question.

The answers received good grades and were written by applicants who passed the examination. The answers were prepared by their authors, and were transcribed as submitted, except that minor corrections in spelling and punctuation were made for ease in reading. The answers are reproduced here with the consent of their authors.

Question Number Contents

1 Criminal Law and Procedure

2 Professional Responsibility

3 Remedies

4 Torts

5 Civil Procedure

6 Business Associations

Question 5

In March 2008, Pat, a citizen of State A, learned that Devon Corp. (“Devon”), a citizen of State B, may have been illegally releasing toxic chemicals into the air near her home.

In February 2011, Pat sued Devon in federal court, alleging a cause of action for negligence and seeking damages for a persistent cough. The court had subject matter jurisdiction over Pat’s lawsuit.

During discovery, Pat requested Devon to produce all documents relating to reports by local residents about foul odors coming from its plant. Devon objected to Pat’s discovery request, contending that the plant’s odors came from legally produced and harmless chemicals, and that therefore the request sought irrelevant information. In further response, Devon provided a privilege log that listed a document described as a summary of all communications with local residents concerning odors that emanated from the plant. As a basis for refusing to disclose the document, Devon claimed the summary was protected from disclosure under the work product doctrine because it had been created by its counsel, who therein described the underlying facts of the residents’ comments as well as counsel’s thoughts about them. Pat filed a motion to compel Devon’s production of the documents she requested. The court denied Pat’s motion.

In October 2012, while the lawsuit was still pending, Pat learned from a scientific report in a newspaper that the chemicals Devon released cause lung cancer.

In November 2012, Pat amended her complaint to add a cause of action for strict liability and sought to require Devon to pay for preventive medical monitoring of her lungs.

Devon moved to dismiss Pat’s strict liability cause of action on the basis that the applicable three-year statute of limitations had run.

1. Did the court correctly deny Pat’s motion to compel? Discuss.

2. How should the court rule on Devon’s motion to dismiss? Discuss.

ANSWER A TO QUESTION 5

1. The trial court incorrectly denied Pat's motion to compel. The scope of discovery is whether the request is reasonably calculated to lead the discovery of admissible evidence. As a general matter and absent any other exceptions, evidence is admissible if it is relevant. Relevance means it has any tendency to make the existence of any fact, that is of consequence to the determination of the action, more or less probable than it would be without the evidence.

Here, Pat requested Devon to produce all documents relating to reports by local residents about foul odors from its plant. Devon objected to the discovery request on the grounds that the plant's odors came from legally produced and harmless chemicals. Pat's lawsuit against Devon is brought under negligence theory and concerns Devon's release of toxic chemicals into the air. Pat's request is within the permissible scope of discovery. Although Devon contends that the odors are legal and harmless, that is not conclusive. During litigation, Pat may gather evidence to support her belief that Devon has been illegally releasing toxic chemicals. She is not required to merely accept Devon's assertion that it is not acting illegally. The reports by local residents may lead to relevant, admissible evidence. If Pat learns that other residents have likewise experienced a persistent cough or other symptoms, or developed cancer, she can use their testimony to rebut Devon's contention that the odors are harmless. Additionally, the reports of local residents are relevant to show that Devon had notice of the harmful effects of the chemical/odors on local residents. Moreover, the evidence could support Pat's assertion that her persistent cough was a reasonably foreseeable result of the chemicals/odors because Devon knew that it the chemicals had similar effects on other residents. Therefore, Pat's document request should be granted unless a privilege applies.

In response to Pat's discovery request, Devon produced a privilege log listing a document described as "a summary of all its communications with local residents concerning odors that emanated from the plant," claiming it was privileged under the

work product doctrine. When a discovery request is within the permissible scope of discovery, but it seeks protected or privileged information, the responding party must provide a privilege log describing the privileged document with particularity and asserting why it is privileged. If the summary is in fact privileged, then Devon properly complied with the discovery rules by responding with a privilege log identifying its existence and explaining why it is not required to disclose it.

The work product privilege applies to all materials prepared by an attorney, or a client at the attorney's request, in anticipation of litigation. As the summary was prepared by Devon's counsel, the first requirement is satisfied. However, the facts do not state whether it was prepared in anticipation of litigation. If Devon's counsel prepared the summary before any litigation concerning the toxic chemicals began, then it may not be covered. Pat learned that Devon may be illegally releasing toxic chemicals in 2008, and did not sue until 2011. If there had been previous complaints, Devon very well may have prepared the summary in anticipation of future litigation, even if not specifically for Pat's case. In those circumstances, the work product privilege would nonetheless apply even if it was made before Pat's lawsuit was initiated.

Not all aspects of the work product privilege are absolute. Any mental impressions, opinions, theories of the case, and related information is absolutely privileged and is never discoverable. However, the remaining aspects of a document may be disclosed if the requesting party establishes: (1) there is a substantial need for the information; and (2) he or she cannot obtain the information from any other source. First, Pat can likely establish that she has a substantial need for the information. As explained above, this information will help support her claim that Devon acted negligently, and rebut Devon's contention that the chemicals/odors are harmless. However, Pat may have more difficulty meeting the second requirement. Devon could argue that Pat could simply interview local residents to determine whether they complained to Devon. However, the court will likely find that this would be an undue hardship because Devon could provide Pat with the names of residents who complained and what their complaints were, without requiring Pat to undergo all that effort. Based on the above analysis, the

underlying information in the summary is discoverable. The communications between local residents and Devon do not fall under the work product privilege because they were not made in anticipation of litigation. Rather, they were likely routine business records. Therefore, if the actual reports of communications that were used to compile the summary are separately available, the court should have ordered that the separate reports be produced to Pat. Then, Pat would receive the information she needed and no privileged information would be disclosed. Conversely, if there are no such separate individual reports in existence, then the court may order Devon to produce the summary with counsel's thoughts redacted from the document.

In sum, the court incorrectly denied Pat's motion to compel. First, the documents requested are within the permissible scope of discovery. Second, although the summary of the communications with residents may be privileged under the work product doctrine, the individual separate reports would not be and could have been produced. Finally, if there are no individual separate reports for each resident, then the court should have ordered that Devon produced the summary with counsel's mental impressions redacted because Pat has a demonstrated a substantial need for the information and that she is unable to obtain the information from another source.

2. The court should deny Devon's motion to dismiss. Civil Rule 15 allows a plaintiff to amend her complaint once before the answer is filed or anytime thereafter with leave of court. Rule 15 requires a court to freely grant leave to amend a complaint as justice requires. When a complaint is amended to include a new claim, it relates back to the date of the original filing as long as the claim arises out of the same transaction or occurrence. Here, Pat seeks to amend her complaint to add a cause of action for strict liability. Her strict liability claim arises out of the same occurrence -- Devon's alleged illegal release of toxic chemicals into the air -- as her negligence claim. Accordingly, her cause of action will relate back to the date of the filing of her complaint in February 2011. Pat discovered Devon's illegal release in March 2008, so her strict liability claim accrued, at the earliest, in March 2008. Accordingly, her strict liability claim was timely filed within the 3-year statute of limitations.

Further, Pat's additional request for relief -- that Devon pay for preventative monitoring of her lungs -- is valid. A party may amend his or her request for damages in the complaint. This new claim for damages relates to Pat's new strict liability claim.

Therefore, the court should deny the motion to dismiss and allow Pat to amend her complaint in the interest of justice because she just discovered the scientific report regarding lung cancer.

ANSWER B TO QUESTION 5

Denial of Pat's Motion to Compel

The Scope of Discovery

The scope of discovery under the federal rules includes all materials that are 1) relevant and 2) not privileged.

As to relevance, an item is relevant if it has a tendency to make the existence or nonexistence of a fact of consequence to the action more or less probable than it would be without the item.

As to privilege, the most commonly asserted privilege objections in discovery are attorney-client privilege and work product privilege. The attorney client privilege protects confidential communications between an attorney and her client from disclosure in discovery, and the work product privilege protects materials prepared by a party in anticipation of litigation. Materials protected by the attorney client privilege are absolutely privileged from disclosure in discovery.

Materials, for which the work product privilege is claimed, however, may sometimes be required to be disclosed. If the party seeking discovery can show that 1) the claimed work product materials contain information which is not reasonably available to him by any other means, and 2) his interests would be substantially prejudiced if he were not allowed access to those materials, the court may order disclosure. However, even if the disclosure of work product is ordered pursuant to this standard, the court may not order the disclosure of an attorney's mental impressions or legal theories, because such items are absolutely protected.

Devon's Relevance Objection

In response to Pat's request for Devon to produce documents relating to reports by local residents about foul odors from Devon's plant, Devon objected and refused to produce such documents on the basis that the odors came from legally produced and harmless chemicals and therefore the request sought irrelevant information. Such documents are properly discoverable because they are relevant and not privileged. Information about reports of odors from the plant by local residents are relevant to Pat's claim that the plant was illegally releasing toxic chemicals into the air, because it is more probable that the plant was in fact releasing chemicals if local residents reported that they smelled odors. Such reports may also be relevant to the issue of the quantities, types, and times the chemicals were released into the air, which is relevant to Pat's claim that she had sufficient exposure to the chemicals to cause her persistent cough.

Devon's claim that the documents are not relevant because the odors were "legally produced" and "harmless" should have been rejected by the court. A party may not avoid discovery by self-serving claims as to what its documents would show. Moreover, the issues at the heart of this claim are precisely whether 1) the odors were legally produced, as Devon claims, or illegally produced, as Pat claims, and 2) the chemicals are toxic, as Pat claims, or harmless, as Devon claims. Devon must produce documents that show what chemicals were released and how they were being produced so that Pat and her experts can evaluate for themselves the nature of the chemicals.

Therefore, to the extent Devon claimed a relevance objection to Pat's request, the Court should have overruled that objection and ordered Devon to respond in full to the request.

Devon's Work Product Privilege Objection

Devon has also produced a privilege log indicating that it has a summary of all communications with local residents concerning odors emanating from the plant, and has claimed that the summary is protected by the work product privilege because it was created by Devon's counsel. The mere fact that a document was created by counsel does not mean that it is protected by the work product privilege. Devon must also show that the document was prepared in anticipation of litigation. If Devon's counsel prepared the document, for example, as part of a report that was required to be given to the EPA on a routine basis, it would not be protected by work product. Devon bears the burden of showing that the document is entitled to work product protection.

In addition, even if the document is work product, Pat may be able to discover it if she can show that she cannot get the information by any other means, and she would be substantially prejudiced without it. This is a very fact specific showing. Pat's alternative means of finding out what residents have complained to the plant about regarding odors would be to walk the streets and interview the neighborhood, hire an investigator, place an ad seeking responses with such information, etc. Depending on the size of the area at issue, that may not be reasonably feasible or particularly productive. Moreover, it is possible that some residents who have been extremely bothered have moved out of the area entirely and would not be accessible through such an investigation. The best source of the information is likely what is contained in the plant's summary of complaints, and it would be very difficult for Pat to collect that information otherwise.

To the extent that the document contains verbatim reports of residents’ complaints, the court should compel Devon to release it. To address Devon's claim that the document also contains counsel's thoughts about the residents' complaints, that information is mental impressions, and is absolutely protected against disclosure. The court should order Devon to produce the document for in camera review, so that the court can determine to what extent it does in fact contain such information. The court

could also order Devon to disclose the document with the work product material redacted.

Ruling on Devon's Motion to Dismiss

The issue here is whether the court should grant Devon's motion to dismiss the amendment to Pat's complaint adding a claim for strict liability and medical monitoring as barred by the statute of limitations, or whether the complaint relates back to the timely filed original complaint.

Relation Back Standard

An amended complaint filed after the statute of limitations has run "relates back" to the original complaint, and therefore is not time-barred, if: 1) the original complaint was timely filed; and 2) the new claims in the amended complaint arise out of the "same transaction or occurrence" as the claims in the original complaint.

Was the original complaint timely filed?

Here, it appears the original complaint was timely filed because Pat discovered her injury in March of 2008 and filed the complaint in February of 2008 for negligence. If the three year statute applies to personal injury complaints whether asserted under negligence or strict liability claims, the original complaint was timely filed within 3 years, and the first part of the relation back test is satisfied.

Do the new claims arise out of the same transaction or occurrence?

As to the question of whether the claims arise from the same transaction or occurrence, the answer is likely yes. Pat's negligence claim relates to the occurrence of Devon's release of chemicals into the air near her home. Her strict liability and medical monitoring claims arise from the same event - Devon's release of chemicals. She is

simply pleading a new theory of liability and requesting an additional remedy for the same conduct by Devon that was at issue in her original complaint.

Devon may argue that, even if the strict liability claim relates back, to the extent that Pat is making a claim for medical monitoring in her amended complaint, it does not arise out of the same transaction or occurrence because it concerns Pat's fear of lung cancer, not her persistent cough. However, a court would likely reject this argument, especially because Pat only recently learned of the potential for the chemicals to cause lung cancer by the Nov. 2012 news article, and filed her amended complaint within a month of learning that information.

Prejudice to Devon

Devon may argue its interests would be prejudiced by permitting the late amendment because it has been engaging in discovery for nearly two years on the basis of the allegations in the original complaint. However, a court would also likely reject this argument because Pat's allegations against Devon in both the original and the amended complaint concern the health effects of the released chemicals, and therefore the scope of the discovery and the preparation Devon must do to defend is not significantly changed by the amended complaint.

In sum, because the original complaint was timely filed, the amended complaint arises out of the same transaction or occurrence as the original complaint, and Devon would not be prejudiced in having to defend against the new claims, the court should deny Devon's motion to dismiss the amendment as time-barred.

The State Bar Of California Committee of Bar Examiners/Office of Admissions

180 Howard Street • San Francisco, CA 94105-1639 • (415) 538-2300 845 S. Figueroa Street • Los Angeles, CA 90017-2515 • (213) 765-1500

ESSAY QUESTIONS AND SELECTED ANSWERS

FEBRUARY 2014

CALIFORNIA BAR EXAMINATION

This publication contains the six essay questions from the February 2014 California Bar Examination and two selected answers for each question.

The answers were assigned high grades and were written by applicants who passed the examination after one read. The answers were produced as submitted by the applicant, except that minor corrections in spelling and punctuation were made for ease in reading. They are reproduced here with the consent of the authors.

Question Number Subject

1. Professional Responsibility

2. Community Property

3. Civil Procedure

4. Real Property

5. Constitutional Law

6. Remedies Question 3

Paul, a resident of State A, had worked as a manager at the only hotel in State A owned and operated by Hotel, Inc. (Hotel), a large national chain. Paul’s compensation was $100,000 per year. Hotel was incorporated in State B, where the majority of its hotels are located. Hotel’s main corporate offices are located in State C.

Hotel terminated Paul’s five-year employment contract when it had two years remaining. Paul immediately found new employment with compensation of $90,000 per year.

Paul timely sued Hotel in state court in State B, alleging wrongful termination of his employment contract. In his complaint, he sought reinstatement or, in the alternative, damages of $200,000 for the two years remaining on his employment contract at the time of termination. In State B, the measure of damages for wrongful termination of an employment contract is the amount a plaintiff would have earned absent the termination, less what the plaintiff actually earned during the post-termination contract period.

After the complaint was served on Hotel at its main corporate offices in State C, Hotel timely removed the case to federal district court in State B. Paul then filed a motion in federal district court to remand to state court. The federal district court denied the motion. Paul appealed the denial to the federal court of appeals.

Paul meanwhile filed a motion in the federal district court for an injunction requiring Hotel to reinstate him to his job. The federal district court granted Paul’s motion and issued the injunction. A month and a half later, Hotel appealed the injunction to the federal court of appeals.

1. Did the federal district court correctly deny Paul’s motion to remand the case to state court? Discuss. 2. How should the federal court of appeals rule on Paul’s appeal? Discuss. 3. How should the federal court of appeals rule on Hotel’s appeal? Discuss. QUESTION 3: SELECTED ANSWER A

1. Did the Federal Court properly deny Paul's Motion to Remand?

The answer to this question is no. The remand should have been granted.

(a) Diversity Jurisdiction

The issue is the nature of the federal court's jurisdiction in this case. Article III of the Constitution limits the jurisdiction of the federal courts to specific types of jurisdiction, that is, jurisdiction based on diversity of citizenship and jurisdiction based upon questions of Federal law. There are no Federal laws raised in this fact pattern, so the basis for jurisdiction must be diversity of citizenship. The other areas of justiciability will not be discussed here as they appear to be satisfied (i.e. Paul has standing to bring a claim that ripe for resolution, satisfying standing, mootness, ripeness, and a case or controversy, because he was terminated, harmed and the harm is complete).

To establish diversity jurisdiction, every plaintiff must be diverse from every defendant, based upon a party's domicile if it is an individual person, or a corporation's state of incorporation and/or principal place of business, if it is a corporation. An individual may only have one domicile; a corporation may have up to two: its state of incorporation and where it conducts its principal place of business.

Here, Paul is a resident of State A. Hotel, Inc. is incorporated in State B, so Hotel is a resident of State B. Hotel's "main corporate offices" are located in State C. If we assume that "main corporate offices" is equivalent to the corporation's "nerve center" or locus of operations, then Hotel is also a resident of State C.

Hence, it is a citizen of State A v. a citizen of State B and State C, even though there is only one defendant. The Federal diversity statute (28 USC 1332) not only has a citizenship test; it also has an amount in controversy test. This means that the amount in controversy for a cause of action (or aggregated causes of action) must be greater than $75,000, exclusive of interest and costs. If a plaintiff alleges such an amount, the court will generally not look behind that amount, unless it amounts to a legal certainty that plaintiff cannot recover it.

Here, Paul has asserted a claim for damages as a result of the breach of employment contract for $200,000 for the two years remaining on his employment contract or, in the alternative, he seeks reinstatement, which means he wants his old job back.

We are told that the law in State B is that the measure of damages for wrongful termination of an employment contract is the amount a plaintiff would have earned absent the termination, less what the plaintiff actually earned. Here, Paul has mitigated as he was contractually required to do and found new employment for $90,000 per year. But this will not defeat the amount in controversy for several reasons. First, Paul seeks as one of his remedies reinstatement to a job valued at $100,000 per year for two years — thus readily satisfying the "value of his complaint" in excess of $75,000. In addition, we do not know to a "legal certainty" that Paul will actually be employed by the new employer at $90,000 per year for the next two years. He might be fired again or he may not like the job and quit — perhaps it isn't a truly comparable position. And finally, it isn't even abundantly clear that State B law will apply. Paul was working in State A and he is a resident of State A. Generally speaking, the law of the state in which an employee performs services governs employment contract disputes (unless provided otherwise in the employment agreement). Accordingly, the amount in controversy is satisfied.

(b) Removal A party is permitted to remove an action to federal court from state court pursuant to 28 USC 1441 if federal jurisdiction exists and the party acts to effectuate removal within thirty days after receiving notice (not service) of a state court action upon which removal may be based. A removal is effective upon the filing of the notice of removal in state court. The action will then proceed as an originally filed federal action thereafter, except certain pleading timing rules are modified. But there is one other requirement not met here: only nonresident defendants may remove.

Hotel is a resident of both State B (where it is incorporated) and State C (its principle place of business). The case was filed in state B state court. Because Hotel is a resident of State B, Hotel cannot remove a case from State B court to a Federal court in State B, and the court should have granted Paul's motion to remand.

2. Paul's Appeal of the Denial of Remand

The Federal court of appeals should dismiss Paul's appeal because a denial of remand is not a final judgment from which an appeal may be taken to the Federal Circuit Court of Appeals ("CTA"). It would rather be deemed to be an interlocutory appeal. With a few limited circumstances, one of which is discussed below, a litigant may only prosecute an appeal to the CTA upon the issuance of a final judgment by the district court (i.e., final judgment after trial, a dismissal of a case pursuant to Fed R. Civ P 12b6 or a Fed. R. Civ. P. 56 summary judgment motion). Exceptions may exist, i.e., the district court may give permission to a litigant to file an appeal prior to a final judgment if it appears that it would be more efficient to proceed in that manner, i.e., the CTA may upon ruling, direct the district court to proceed in a particular manner that might impact the trial of the case. A denial of a motion to remand, however, is not an immediately appealable order and absent permission from the district court, it will not be entertained by the CTA. Paul otherwise must wait until the matter is concluded.

3. Hotel's Appeal of the Injunction

The CTA will deny Hotel's appeal as untimely. But it otherwise would have likely sustained Hotel's appeal on substantive grounds. A party has thirty days to appeal from a district court's judgment or appealable order to the CTA. Here, we are told that Hotel did not appeal the injunction until a month and a half after its issuance. Hotel was late. Injunctions are an exception to the final judgment rule — if a party is mandated by a federal court to do something, that party may immediately (or within thirty days) appeal that order. This order is in the nature of a mandatory injunction.

A mandatory injunction differs from a prohibitory injunction in that the federal court mandates or orders a party to act. A prohibitory injunction is an injunction based upon a directive to a party to cease doing something. In this case, the court has ordered Hotel to reinstate Paul to his position as manager at Hotel A. This effectively requires Hotel to reinstate an employee it has just fired.

Courts typically do not grant mandatory injunctions for personal services contracts, like the one at issue here. There are several reasons for this view. It is simply not feasible to enforce these orders: requiring an employer to have an employee on the premises who may or may not discharge his work obligations may impact the employer's business. This is particularly apt here where the employee is a manager at a hotel — he may well come into contact with the public and may be in a very sensitive position. Second, a mandatory injunction in this context inserts the court directly into the business of the employer — it just isn't feasible for a court to determine whether the employee is being treated well, paid right, advanced, compensated, managed properly, carrying out the Hotel's instructions, etc. Setting those policy issues aside, however, there is an adequate money damages remedy here — a written employment contract with two years remaining, and clearly sets compensation for those two years (regardless of mitigation issues or which state's law applies). And there is no irreparable harm — again, even on balance of all the interests here, this a simple breach of employment contract case, remedied easily through money damages. Balancing the harms, as the court would do on an injunction, clearly favors Hotel: it is far more burdensome to require Hotel to take back Paul and supervise him where Paul can ultimately recover a money judgment against a presumably solvent defendant, a large national hotel chain. QUESTION 3: SELECTED ANSWER B

Denial of Motion to Remand

The federal court erred in denying Paul's motion to remand to state court.

In order for a case to be removed to federal court, the federal court to which the case is removed (the district court for the district in which the state court is located) must be a court in which the case originally could have been filed. This requires that the court have subject matter jurisdiction, personal jurisdiction over the defendant, and that venue be appropriate in that court.

Subject matter jurisdiction can be based either on diversity jurisdiction or federal question jurisdiction. Federal question jurisdiction exists when, from the face of a well- pleaded complaint, plaintiff's claim "arises under" federal law. Here, there is no basis for federal question jurisdiction. Paul's claim is one purely in contract, based on a state law cause of action for wrongful termination. There is no indication whatsoever from the facts that his claim arises under federal law or that he is enforcing any federal right. Accordingly, there is no federal question jurisdiction.

Accordingly, the federal court can have jurisdiction only if there is diversity jurisdiction. Diversity jurisdiction is present in a suit between citizens of different states where the amount in controversy is at least $75,000. First, it does appear that Paul and Hotel, Inc. are citizens of different states. An individual is a citizen of the state in which he is domiciled, i.e., where he resides and intends to remain permanently. Here, the facts indicate that Paul is a resident of State A, and there is nothing to indicate that he resides in any other state or does not intend to remain permanently in State A; thus, he is a citizen of State A for purposes of diversity jurisdiction. A corporation, by contrast, is a citizen of both (1) the state in which it is incorporated and (2) the state in which it has its principal place of business, i.e., its "nerve center" from which responsible managers control the corporation's affairs. Here, Hotel is incorporated in State B, and its main corporate offices are in State C. There is no indication from the facts that Hotel has its principal place of business in another state, and it thus appears that its principal place of business would be its corporate offices in State C. (It is certainly clear that Hotel does not have its principal place of business in State A, in which it has only a single hotel.) Thus, Hotel appears to be a citizen of both State B and State C for purposes of diversity jurisdiction. Therefore, Paul and Hotel are citizens of different states, and there is the requisite "complete diversity" for purposes of diversity jurisdiction.

However, it is unclear that the requisite $75,000 amount in controversy is satisfied on these facts. Paul's complaint advances claims for both damages and injunctive relief. When damages are at issue, the plaintiff's good faith assessment of damages will govern the amount-in-controversy analysis, unless it appears clear to a legal certainty that those damages are unobtainable. With respect to injunctive relief, the monetary value of the relief can be based on the greater of either (1) the benefit to the plaintiff or (2) the harm to the defendant.

Here, Paul's claim for damages does not appear to satisfy the amount-in-controversy. A plaintiff seeking damages is entitled only to his expectation damages (as well as incidental damages). Every plaintiff has a duty to mitigate damages, which Paul did here by obtaining another job at a rate of $90,000 per year. Accordingly, expectation damages — which are designed to return the plaintiff to the position he would have occupied absent the breach of contract — would be only $10,000 per year, or $20,000 total. It appears that State B has adopted this measure of damages and, under the Erie doctrine, State B's substantive law should apply to this case, since a federal case sitting in diversity is obligated to apply a state's substantive law. There is no federal law in direct conflict with State B's rule of damages; accordingly, it is necessary to consider whether applying State B's damages computation methodology would be outcome determinative, whether it would lead to forum shopping (causing plaintiffs to flock to federal court), and whether the state has a particular interest in having its law applied. It is clear based on these factors that the damages methodology is substantive in nature and should be applied — the amount of damages that a plaintiff can recover in a matter are certainly outcome determinative, are highly likely to affect a plaintiff's choice of forum, and reflect the state's policy determination as to the matter at issue. In any event, the value of Paul's monetary damages claim is the same — and short of the amount in controversy requirement — whether one applies general contract law principles or State B's damages rule.

However, the injunctive relief Paul seeks does satisfy the amount-in-controversy requirement. If the injunction were granted, the benefit to Paul would only be the additional $20,000 in income over the next two years; but if granted, the harm to Hotel would be in the amount of $200,000, i.e., the two years of additional salary that it would be required to pay to Paul. Accordingly, the $75,000 amount in controversy is satisfied.

Nevertheless, although both requirements for diversity jurisdiction are met here, the federal district court erred in denying Paul's motion to remand. A defendant (or defendants) may not remove a case from state court to federal court if any defendant is a citizen of the forum state. Here, as discussed above, Hotel is a citizen of State B. Thus, Hotel was not entitled to remove the case, removal was procedurally improper, and the district court should have granted Paul's motion to remand.

Paul's Appeal

The Court of Appeals should refuse to hear Paul's appeal.

As an initial matter, a grant of a motion to remand is never appealable. Here, by contrast, the district court denied Paul's motion. However, under the final judgment rule, one has a right to appeal as of right only from a final judgment of the district court. A final judgment is one that is final as to all claims at issue and all parties involved, i.e., one that leaves nothing more for the trial court to do. A denial of a motion to remand is not such a final judgment, and is thus not appealable as of right. Moreover, there is no indication in the facts that any of the exceptions to the final judgment rule apply here. For example, the Court of Appeals may choose to hear an appeal based on the "collateral order doctrine" where a decision of the district court is collateral to the merits of the case, involves an important issue of law that has been finally decided, and the party appealing would be effectively precluded from achieving review of the decision absent an immediate appeal. Here, however, while the district court's denial of Paul's motion to remand is collateral to the merits of the case and final, Paul will have the opportunity to obtain appellate review of that decision through a normal appeal at the conclusion of the matter. Accordingly, the collateral order doctrine does not apply. Nor is there any indication from the facts that the district court has certified this ruling for immediate appeal under the Interlocutory Appeals Act (or that the Court of Appeals has accepted such an appeal), or that any statutory exception to the final judgment rule applies.

Hotel's Appeal

The Court of Appeals should dismiss Hotel's appeal, because it is untimely.

As an initial matter, the order granting Paul's requested injunction is an appealable order. Although the grant of this motion would otherwise not be a final judgment under the final judgment rule, there is an exception for orders granting, modifying, or dissolving injunctions. Accordingly, though the case remains pending, the district court's order granting Paul's motion for an injunction was an appealable order.

However, Hotel, Inc. did not timely appeal from the district court's order granting Paul's injunction. A party must appeal within 30 days of an appealable order. Here, the facts indicate that Hotel, Inc. noticed its appeal "[a] month and a half later." The facts do not contain any indication of excusable neglect on Hotel's part, and Hotel's appeal should thus be dismissed as untimely.

The State Bar Of California Committee of Bar Examiners/Office of Admissions

180 Howard Street • San Francisco, CA 94105-1639 • (415) 538-2300 845 S. Figueroa Street • Los Angeles, CA 90017-2515 • (213) 765-1500

ESSAY QUESTIONS AND SELECTED ANSWERS

FEBRUARY 2015

CALIFORNIA BAR EXAMINATION

This publication contains the six essay questions from the February 2015 California Bar Examination and two selected answers for each question.

The answers were assigned high grades and were written by applicants who passed the examination after one read. The answers were produced as submitted by the applicant, except that minor corrections in spelling and punctuation were made for ease in reading. They are reproduced here with the consent of the authors.

Question Number Subject

1. Contracts

2. Real Property

3. Civil Procedure

4. Remedies

5. Business Associations

6. Wills/Trusts QUESTION 3

In March, while driving her car, Diana struck and injured Phil.

In April, Phil filed a complaint against Diana in federal district court properly alleging diversity jurisdiction and seeking damages for negligence for physical injury.

In May, Diana filed an answer denying negligence.

In June, during discovery, Diana filed a motion asking the court to order (1) a physical examination and (2) a mental examination of Phil. Over Phil’s objection, the court ordered him to submit to both examinations.

In July, Diana served Phil with a notice to depose Laura, a physician who treated him after the accident. Phil objected on the grounds that (1) Laura could not be deposed because she was not a party, and that (2) deposing her would violate the physician- patient privilege. The court overruled Phil’s objections.

In September, a few weeks before trial, Phil decided to file a demand for a jury trial. Diana immediately filed a motion to strike the demand. The court granted Diana’s motion.

1. Did the court err in granting Diana’s motion to order (a) the physical examination and (b) the mental examination of Phil? Discuss.

2. Did the court err in permitting Diana to depose Laura? Discuss.

3. Did the court err in granting Diana’s motion to strike Phil’s demand for a jury trial? Discuss. QUESTION 3: SELECTED ANSWER A

Applicable law

Under the Erie doctrine, a federal court sitting in diversity jurisdiction must apply the substantive laws of the state where it sits and the procedural laws of the federal system, generally the Federal Rules of Civil Procedure and in most cases the Federal Rules of Evidence. Whether or not a rule is substantive or procedural is a balancing test that depends on whether 1) the rule is outcome determinative, 2) the federal court's interest in applying their own rules, and 3) whether or not application of the federal rule will result in forum shopping.

Whether or not a party may obtain an order for a physical or mental examination is a rule of discovery that is procedural and governed by the Federal Rules of Civil Procedure which will apply in this case.

a) Diana's motion for a physical examination of Phil

Under the Federal Rules of Civil Procedure, a party may obtain a mental or physical examination of the other party if 1) that party's physical or mental condition is in controversy, and 2) good cause exists for ordering the examination. Good cause will generally be found to exist if the examination in question is not overly intrusive and it is relevant, measured in terms of its logical and legal relevance as well as how relevance is defined under the Federal Rules of Civil Procedure with regard to its discoverability. Evidence is logically relevant if it tends to make the existence of a fact of consequence more or less likely. Evidence is legally relevant if its probative value is not substantially outweighed by its prejudicial effect. And evidence is relevant and discoverable if it is reasonably likely to lead to the discovery of admissible evidence.

Phil's suit against Diana is one for personal injury stemming from her alleged negligence. In a negligence suit, the plaintiff must prove duty, breach, cause, and damages. Because damages are a required element, the injury and the extent of the injury suffered by a party will always be in controversy in a personal injury suit. Additionally, good cause exists for ordering the physical examination here. It is not overly intrusive as Phil has already likely sought out and received medical treatment for his injuries of a similar nature in this case. Additionally, it is logically and legally relevant and relevant under the Rules' definition for discovery because it is reasonably likely to lead to the discovery of admissible evidence. The examining physician may have a different opinion as to the nature and extent of injuries suffered by Phil.

For these reasons, the court did not err in granting Diana's request for a physical examination of Phil. b) Diana's motion for a mental examination of Phil

With regard to Diana's motion for a mental examination of Phil, the rules are the same as for a physical examination. Under the Federal Rules of Civil Procedure, a party may obtain a mental or physical examination of the other party if 1) that party's physical or mental condition is in controversy, and 2) good cause exists for ordering the examination. However, the calculus here for a mental examination is much different.

Phil's suit against Diana is for personal injury. His physical condition is relevant because it is a fact in controversy as damages are an element of negligence. Phil's mental condition, however, does not appear to be in controversy. Phil's suit is not for infliction of emotional distress or any other cause of action where his mental condition would be a fact in controversy. If Phil suffered from some sort of mental disease or defect that made him comparatively or contributorily negligent or that affected his abilities to perceive or recall, such that Diana could impeach his credibility, then Phil's mental condition could theoretically be in issue. However, that does not appear to be the case here. There is nothing to indicate that Phil's mental condition is in controversy. Additionally, a mental examination is an intrusive procedure that should not be granted unless necessary to establish a claim or defense, neither which requirement is met in this case. Good cause for granting Diana's request for a mental examination thus cannot be said to exist. For these reasons, the court erred in granting Diana's request for a mental examination of Phil.

2)

Whether the physician-patient privilege applies

Under the Federal Rules of Evidence, there is no physician-patient privilege. There are only privileges for spousal communications, spousal immunity in criminal cases, penitent-clergy, and patient-social worker.

However, as discussed above, under the Erie doctrine, a federal court sitting in diversity jurisdiction must apply the substantive laws of the state where it sits and the procedural laws of the federal system. Generally the Federal Rules of Civil Procedure and in most cases the Federal Rules of Evidence are procedural. However, whether or not a testimonial privilege applies is a rule of substantive law and a federal court sitting in diversity must apply the law of the state in which it sits regarding testimonial privileges.

The federal court sitting in this case must apply the state law regarding the doctor- patient privilege. Generally the doctor-patient privilege covers confidential communications between a doctor and a patient for the purposes of obtaining medical treatment. If the state in which this federal court sits acknowledged the doctor-patient privilege then Phil's communications to his doctor would generally be privileged.

However, there is generally an exception to the privilege when the patient-plaintiff's physical condition is in controversy. As stated above, this is a personal injury suit and damages are a necessary element of the negligence claim so Phil's physical condition is in actual controversy. For that reason, even if the doctor-patient privilege applies, Phil's communications to Laura would likely be outside the privilege and would not prevent Diana from deposing Laura. Whether Laura cannot be deposed because she is not a party

As with the standard for granting a physical or mental examination of a party, whether a party can be deposed is a discovery rule and is thus procedural and governed by the Federal Rules of Civil Procedure.

The Federal Rules of Civil Procedure allow a party up to 10 depositions in a case. Each deposition must be no longer than 1 day of 7 hours. A party may depose another party at any time simply by providing reasonable notice. A party may depose a non-party, but it must be done on subpoena to the non-party and must provide reasonable notice and accommodations.

Under the Federal Rules of Evidence, Laura may be deposed even though she is not a party to the litigation. So Phil's objection is not correct.

However, there is no indication in the fact pattern that Diana obtained a subpoena or served it on Laura prior to deposing her. Diana cannot simply serve Phil with a notice of subpoena in order to depose a non-party.

Nonetheless, a party's objection to discovery must be stated accurately and with particularity. Phil may have waived his valid procedural objection to Diana's deposition of Laura by not correctly stating the grounds for his objection.

In sum, Diana may depose Laura under the Federal Rules of Civil Procedure, even though she is a non-party. However, Diana must do so on subpoena and notice to Laura, which Diana failed to do in this case. However, Phil incorrectly stated the basis for his objection to Diana deposing Laura and in so doing likely waived his otherwise valid procedural objection to the deposition. Thus, the court did not err in permitting Diana to depose Laura. 3)

Under the 7th Amendment to the Constitution, a party is entitled to a jury trial in all suits for damages at law. Phil's suit against Diana is a personal injury suit for damages at law and not for some form of equitable relief like an injunction so Phil is entitled to a jury trial in his suit against Diana (as is Diana). However, under the Federal Rules of Civil Procedure, a party must file a demand for a jury trial within 14 days of the filing of the answer to the complaint. A party may file a motion to strike all or a portion of the other party's pleading within 30 days of receiving that party's pleading.

In this case, Diana filed an answer to Phil's complaint denying negligence back in May. Phil did not file his demand for a jury trial until September and only a few weeks before trial. For this reason, Phil's demand is untimely and absent good cause for the delay in this case, which does not seem likely, Phil has waived his right to demand a jury trial. Since Diana immediately filed her motion to strike in response to Phil's demand, it was timely and should be considered and granted by the court.

For this reason, the court did not err in granting Diana's motion to strike Phil's demand for a jury trial. QUESTION 3: SELECTED ANSWER B

Preliminary matters

Applicable Law After having been injured by Diana (D), Phil (P), filed a complaint in April against D in federal district court properly alleging diversity jurisdiction and seeking damages for negligence for physical injury. As such, because the complaint was filed in federal court, the Federal Rule of Civil Procedure (FRCP) govern the rules applicable to the proceedings and the actions of the courts and the parties in the suit.

(1) The Court Properly Granted Diana's Motion to Order (a) the physical Examination if she properly established good cause, but erred in granting (b) the mental examination

(a) The Physical Examination

Scope of Discovery Discovery is the process by which parties obtain information from the other party. The FRCP provides for a broad scope of discovery, and the information needs only to be relevant to the cause of action. In fact, any information that would reasonably lead to the discovery of admissible evidence is discoverable. In other words, the information does not have to be admissible evidence to be produced, but only to reasonably lead t such information. Here, the dispute between P and D involves a car accident where D struck and injured P. As a result, P filed an action against D for negligence for physical injuries. Therefore, any information that would relate to the accident, the physical condition of P, which is at issue here, will be admissible. Here, D has filed a motion, seeking a court order directing the court to order a physical examination of P. Such examination is relevant because here, the physical condition of P is at issue since the lawsuit involves damages for personal injury. As such, this information is discoverable and within the scope of discovery. Physical Examination Requirements Physical Condition at Issue In order for a party to obtain an order for a physical examination, the FRCP requires, first, that the physical condition be at issue. Here, P's condition is at issue because, as explained above, the lawsuit between P and D is about a car accident where D stuck and injured P. P is seeking damages. A physical examination will be useful to determine the extent of the injury cause by the accident to P, and will therefore be useful to determine the extent of damages, if any. Also, such physical examination will also determine if the physical injuries suffered by P were the result of the accident.

Court order and Showing of Good Cause The FRCP requires that a court grant a motion to order a physical examination only when the moving party establish good cause to do so. Here, the facts are not clear on whether D established such good cause. A showing of good cause will require D to show that there is no other means to obtain the information that the physical examination would provide and establish the reasons to do so. Here, as explained above, a physical examination will be useful to determine the extent of the injury caused by the accident to P, and will therefore be useful to determine the extent of damages, if any, especially if there is no other information available. Also, such physical examination will also determine if the physical injuries suffered by P were the result of the accident. However, if the deposition of D is ordered (see below), then the showing of good cause for a physical examination will harder to establish because there would already be available information related to the physical condition of P after the accident. If ordering the deposition fails, however, this might constitute a good cause to order the examination because no information related to P's physical condition would therefore be available.

(b) The Mental Examination

Scope of Discovery Discovery is the process by which parties obtain information from the other party. The FRCP provides for a broad scope of discovery, and the information needs only to be relevant to the cause of action. In fact, any information that would reasonably lead to the discovery of admissible evidence is discoverable. In other words, the information does not have to be admissible evidence to be produced, but only to reasonably lead t such information. Here, the dispute between P and D involves a car accident where D struck and injured P. As a result, P filed an action against D for negligence for physical injuries. Therefore, any information that would relate to the accident, the physical condition of P, which is at issue here, will be admissible. Here, a request for a mental examination is not likely to lead to any relevant admissible information. In fact, here, the mental condition of P is not at issue, only his physical condition because he is seeking damages for personal injury as a result of the accident. As such, this demand does not fall within the scope of discovery.

Mental Examination Requirements Again, a court will issue an order for mental examination, only when this condition is at issue and when the moving party has established good cause to do so. Here, as explained above, the mental condition of P is not at issue and there is no reason why the court would order such examination. Not only does it fails to show good cause but would also be highly prejudicial to P.

(2) The Court Erred in Permitting to Depose Diana only if a Subpoena was not Issued, and P's argument that the Deposition would lead to the discovery of Privileged information fails

Scope of Discovery Discovery is the process by which parties obtain information from the other party. The FRCP provides for a broad scope of discovery, and the information needs only to be relevant to the cause of action. In fact, any information that would reasonably lead to the discovery of admissible evidence is discoverable. In other words, the information does not have to be admissible evidence to be produced, but only to reasonably lead t such information. Here, the dispute between P and D involves a car accident where D struck and injured P. As a result, P filed an action against D for negligence for physical injuries. Therefore, any information that would relate to the accident, the physical condition of P, which is at issue here, will be admissible. Here Diana (D) is a physician who treated P right after the accident. Her deposition will be useful because it will lead and explain what was the physical condition of P right after the accident and will help in determining the extent of the injury as well as damages, if any.

Deposition of Third Party - Subpoena To The Third Party The FRCP allows deposition of non-party to the case and provides for a maximum of 10 depositions, no longer than 7 hours each. There can also be only one deposition per person. When the deposition involves a non-party, i.e. someone not named in the lawsuit, then the requesting party must request the court to issue a subpoena in order to depose the third party. Here, D served P with a notice to depose Laura (L), the physician who treated him after the accident. The FRCP allows "notice" only when the discovery tools are used by party against another party. When a third party is involved, a subpoena is required, which D shall have done to properly depose her. In fact, not only did she fail to notice Laura personally, but she also failed by the means she used. As such, P is wrong when he says that a third party cannot be deposed. A third party can be deposed but here the court erred in granting the discovery request because the third party, Laura, was not properly notified.

Limit of The Scope of Discovery = Privileged Communication The broad scope of discovery is limited by privileged information. In fact the FRCP provides that discovery means: discovery of any "non privileged" information. As such, whenever a privileged communication is involved, the scope of discovery may be limited. Here, P is asserting the Physician-Patient Privilege. As explained in the preliminary considerations, the FRCP apply here. The FRCP, and the Federal Rules of Evidence, do not recognize a Physican-Patient Privilege. As such, whether this argument will fail or prevail depends on which law the Federal District Court will apply. Diversity Cases - Erie Doctrine - Application of State Law Privilege The lawsuit filed by P against D was filed in federal district court, and properly alleged diversity jurisdiction. Under the Erie Doctrine, Courts sitting in diversity jurisdiction will apply the federal procedural law, and the substantive law of the state. Whether a law is substantive or procedural depends on whether it is outcome determinative or not. State Law regarding privileges have been held to be outcome determinative and therefore, substantive law for purposes of Erie Doctrine. Here, assuming that the state law of the seat of the federal action recognizes the physician-patient privilege, the federal court will have to apply it and such privilege might limit the scope of discovery.

Physician-Patient Privilege Privilege The physician-patient privilege is a privilege usually applied by states specifically recognizing such privilege. Under the physician-patient privilege any communication between a physician and his patient, made for the purpose of diagnosis or treatment, is privileged. The patient is the holder of the privilege and can oppose to the revelation of such information. Here, deposing L will likely lead to revealing privileged information: P saw L for purposes of diagnosis and treatment after the car accident and therefore, such communications are likely privileged.

Exceptions The Physician-Patient privilege does not apply in several circumstances, and especially when the physical condition of the patient is at issue. Here, as explained, P's physical condition of D is at issue: the lawsuit involves a car accident where D struck and injured P and P is seeking damages for physical injury. As such, the privilege does not apply and P will fail in his argument that the deposition of L will lead to violate the physician- patient privilege because here, the privilege does not apply.

(3) The Court Properly granted Diana's Motion to Strike Phil's demand for a jury trial In September, a few weeks before trial, P decided to file a demand for jury trial. D immediately filed a motion to strike the demand. The court was absolutely right in granting the motion. 7th Amendment Right to a Jury Trial The 7th Amendment of the U.S. Constitution provides a right for a jury trial in federal civil case (does not apply to the states through the 14th Amendment) when the damages at law involved exceed $20. Here, P is seeking damages for personal injury under a negligence action. Negligence is an action recognized in common law and the damages required are legal damages and likely to involve more than $20, since they stem from the personal injury suffered after the car accident. Therefore P was entitled to a jury trial, but only as long as the demand was timely filed.

Notice to Opposing Party and Timely Demand P made his demand for a jury trial about 3 weeks before trial. A demand for jury trial must be noticed to other party and promptly filed. The FRCP requires that a demand for a jury trial be filed by the Plaintiff 14 days after the complaint is filed, at the very latest and be properly notified to the opposing party. Here, P made his demand only 3 weeks before trial, after all the pleadings were closed. As such, this was not a timely demand and the Court was absolutely right to grant D's motion to strike P's demand for a jury trial.

Evidence

ESSAY QUESTIONS AND SELECTED ANSWERS JULY 2001 CALIFORNIA BAR EXAMINATION

This publication contains the six essay questions from the July 2001 California Bar Examination and two selected answers to each question.

The answers received good grades and were written by applicants who passed the examination. The answers were prepared by their authors, and were transcribed as submitted, except that minor corrections in spelling and punctuation were made for ease in reading. The answers are reproduced here with the consent of their authors.

Question Number Contents

1. Civil Procedure

2. Real Property

3. Evidence

4. Constitutional Law

5. Torts

6. Wills/Trusts

1 QUESTION 3

Walker sued Truck Co. for personal injuries. Walker alleged that Dan, Truck Co.'s driver, negligently ran a red light and struck him as he was crossing the street in the crosswalk with the "Walk" signal. Truck Co. claimed that Dan had the green light and that Walker was outside the crosswalk. At trial, Walker called George Clerk and the following questions were asked and answers given:

17. Would you tell the jury your name and spell your last name for the record, please? A. George Clerk. C-l-e-r-k. [1] Q: Where were you when you saw the truck hit Walker? A: I was standing behind the counter in the pharmacy where I work. [2] Q: What were the weather conditions just before the accident? [3] A: Well, some people had their umbrellas up, so I’m pretty sure it must have been raining. [4] Q: Tell me everything that happened. [5] A: This guy rushed into my store and shouted, "Call an ambulance! A truck just ran a red light and hit someone." Q: What happened next? [6] A: I walked over to the window and looked out. I said, "That truck must have been going way over the speed limit." Then I called an ambulance. Q: Then what happened? [7] A: I walked out to where this guy was lying in the street. Dan, the driver for Truck Co., was kneeling over him. A woman was kneeling there too. She spoke calmly to Dan and said, "It's all your fault," and Dan said nothing in response.

At each of the seven indicated points, what objection or objections, if any, should have been made, and how should the court have ruled on each objection? Discuss.

21 ANSWER A TO QUESTION 3

For ease of reference, D will be Truck Co., C will be Clerk, and W will be Walker.

"Where were you when you saw the truck hit Walker?" The objections that can be raised to the question include: Argumentative, assumes facts not in evidence, and lack of foundation.

Assumes facts not in evidence. A lawyer may not use his/her questions on direct examination to argue the facts or issues of a case. The lawyer must ask questions and allow the witness to testify. Although we have no background evidence, we know Clerk (C) has not yet testified that he saw the truck hit Walker. Thus, the question assumes facts not in evidence, and an objection should be sustained.

Lack of personal knowledge/foundation A witness may only testify based upon his/her personal knowledge, and the lawyer must present the basis for the witness's knowledge before a witness may testify as to facts in the trial relating thereto. Here, all we know is the name of this witness. We do not know where he was, who he was, or even whether he observed any accident. This assumes not only that he saw the accident, but that the truck hit Walker -- which has not yet been established. Thus an objection for lack of personal knowledge is sustainable.

Argumentative A lawyer may not use his/her questions on direct examination to argue the facts or issues in a case. The lawyer must ask questions and allow the witness to testify. An argumentative question is one which argues the facts or issues of the case rather than just eliciting a direct response. This question is argumentative in that it assumes as the truck "hit" Walker rather than Walker "walking out in front of" the truck. Any objection should be sustained.

"What were the weather conditions like just before the accident?" The statement could be objected to based on lack of personal knowledge. The attorney has not laid a foundation that C had an opportunity to observe the weather conditions on that day. However, it could also be argued that it is within a witness's personal knowledge to remember what the weather conditions were like that day, so it is arguable that the statement did not need a foundation to be laid. Thus, an objection may be proper here, but it is not likely to be sustained unless the witness actually does not have personal knowledge (see below).

The statement could also be objected to on the basis of relevance. A statement is relevant if it makes some fact more or less likely. Although the weather conditions do not appear to make a difference in the accident claims (red light/green light issue), it could be relevant to show the ability of each party to see one another. Thus, the weather conditions are probably relevant, and the objection should be overruled.

22 "Well, some people had their umbrellas up . . ." Here, a motion to strike should be made because the answer is speculation. A motion to strike must be made immediately after a witness's response, and can only be made when the original question did not obviously contemplate an objectionable response. If granted, the jury will be instructed not to consider that portion of the witness's answer. A witness must base his testimony on personal knowledge, and cannot speculate as to the conditions surrounding his/her answer. As discussed above, the weather conditions may be within C's personal knowledge. However, upon his answer, it becomes obvious that the questions actually led him to speculate and base his answer on something other than personal knowledge -- he made an inference that it was raining because of the umbrellas. W's attorney may argue that this is not speculation but rather based on personal knowledge because he remembers the umbrellas, and as such if anything only the portion about the "must have been raining" must be stricken. The court will probably agree, and only strike the parts based solely on speculation. Thus, the failure to object in the first place is excusable, the motion to strike is proper, and it should be sustained in part.

"Tell me everything that happened." An objection should be made that the question calls for the witness to give a narrative account. The lawyer interrogating the witness on direct examination must ask specific questions and lead the witness through his or her testimony. This question calls for a narrative by the witness, and as such it is an improper question. The objection should be sustained.

The "call an ambulance" statement An objection should be made based on hearsay. Hearsay is an out-of-court statement offered to prove the truth of the matter asserted. A statement can be words or conduct. If a statement is found to be hearsay and does not fit into a hearsay exception, it must be excluded from evidence. Here, the statement is hearsay because it was made out of court by a "guy" -- a declarant who is not testifying at trial and it is being offered for its truth -- that a truck ran a red light and hit someone. It could be argued that the statement is being offered for the nonhearsay purpose of showing its effect on the listener, C, in which case it would not be hearsay, because it would not be offered to show the truth that the truck ran the light but to show the effect the statement had on C. However, this argument will fail because what C did is not relevant in this case.

Likely, W's counsel will argue that the statement is a present sense impression or an excited utterance. A present sense impression is a statement that is made contemporaneous with an observation or a physical condition that is so trustworthy because there is not much time for contemplation to lie. It must be very contemporaneous, and very little time can lapse. Here, this statement would be admissible if it were made while the accident was happening, but the lapse of time between the declarant's coming in and the accident is not established as short and we do not know what he was doing at that time. Thus, it may not be contemporaneous enough to come in a present sense impression.

23 However, it will likely come in as an excited utterance. An excited utterance is a statement made under the stress of excitement of some event. Here, the time period can be longer than in a present sense impression so long as the stress of excitement remains. It seems apparent that the declarant was still under the stress of excitement when he made the statement, as he was exclamatory in doing so. Also, a short period of time passed -- as no ambulance had yet been called, so this makes it more likely that he was under the stress of excitement. Watching a car accident is definitely stressful and exciting. Thus, it is likely that the statement is trustworthy enough to come in under the excited utterance exception, and the objection should be overruled.

"That truck must have been going way over the speed limit." Hearsay. D's counsel will object based on hearsay. This is an out-of-court statement made to prove the truth of the matter asserted. Even though the statement was made by the witness, the statement was made out of court, and as such it still is classified as hearsay. C can testify on the stand as to what he or she recalls of the events, but C cannot testify as to what he/she said about them then unless they fall into a valid hearsay exception. This statement could probably not be classified as an excited utterance because C did not observe the events and there is no indication that he was particularly excited about what occurred. Further, it cannot be classified as a present sense impression unless there would be some foundation laid as to why he thought that (e.g., what did you observe, etc.) and then it could be argued that the statement was made as a present sense impression of what it was that he saw. (However, this may still be an impermissible opinion; see below.) It could not be argued that it is an effect of hearsay scenario (see above) because it does not demonstrate why he called the ambulance, his action, but rather is being offered to show that the truck was speeding -- the truth. Thus, the objection should be sustained based on hearsay grounds.

Calls for an opinion. The statement itself is an opinion statement, and lay witnesses may not testify as to their opinions unless they have personal knowledge, the information in the opinion cannot be derived from a better source and will be helpful to the trier of fact, and it is not scientific or technical in nature. Here, the statement is not based upon personal knowledge (at least not from the foundation we have here), and as such it is an impermissible opinion. Not only could the hearsay statement not come in, but the statement made by the witness on the stand himself [sic] could not come in either. C did not observe the events; rather, C only observed the aftermath. Thus, he did not know that the truck was speeding and was basing this information on evidence not offered forth as a foundation. Thus, although his statement would be permissible if he actually saw the truck speeding, because he did not he has no basis for knowledge of this fact and his opinion is inadmissible. The objection should be sustained.

Woman's statement to Dan: "It's all your fault." Hearsay! D's counsel is likely to claim that this is a hearsay statement. Woman's statement is an out-of- court statement offered to prove the truth of the matter asserted. As such, her statement itself

24 is hearsay unless it can fall into one of the exceptions. Here, the statement is not offered for a non-hearsay purpose, so it must fall into an exception. Because of the time lapse, present sense impressions and existed utterance exceptions are probably not viable. However, C's counsel can argue that D's response to the statement, his failure to respond, is an adoptive admission. An admission is deemed to be "not hearsay" by the Federal Rules of Evidence, and as such they are not subject to hearsay objections even though they go to the truth of the matter asserted. An admission is a statement made by a party offered by his/her opponent in a case. Here, it must first be determined whether Dan is a party opponent in the case against Truck Co. Dan may be a party due to the doctrine of vicarious admissions. If an employee is acting within the scope and course of his or her employment, then all admissions made by that employee are imputed to the employer. Here, we do not have definite facts as to the activity that D was engaged in at the time of the accident; however, if it is found that D was acting in the scope and course of his employment then any statement he made concerning the accident can be considered an admission and be vicariously imputed to his employer, Truck Co.

However, it must also be shown that D's failure to respond was an adoptive admission. An adoptive admission is an admission by silence, and it is only allowed when relating to an accusatory type statement that is made that would likely invoke a denial or response by the party, when the party does not deny it, and when the party is physically and mentally capable of denying it. D's counsel will argue that it is not necessarily true that a person would deny liability in this case. D will claim that Dan was stunned and was unable to mentally grasp what was going on. Thus, he would lack the mental capacity to deny the statement. Further, D's counsel will argue that many people know that it is not in their best interest to deny or admit liability at the scene of the accident, and that they should just keep quiet. Thus, the average person would not be expected to deny the statement, but silence would in fact be appropriate. Thus, although D is probably liable for anything he did say as a vicarious admission, this statement does not qualify as an adoptive admission and the objection should be sustained.

Improper Opinion Counsel may claim that this is an improper opinion because no foundation was laid as to whether W saw the accident or not. This would be sustainable.

25 ANSWER B TO QUESTION 3

1) Where were you . . . Assumes facts not in evidence. The question asks where George was when he saw the truck. This assumes that George did in fact see the truck. There is no foundation for this assertion in the testimony at this point.

A judge would find that the question was improper and would probably ask to rephrase the question. Such question would be relevant because it would indicate facts about George's ability to perceive the action.

2) What were the weather conditions . . . Relevance In order to be admitted, the testimony must be relevant. Relevant testimony is usually admitted. Relevant evidence is evidence that tends to make a fact more or less likely.

Truck would say that the statement was irrelevant because the issue is not loss of control of the vehicle but running the red light or not. Therefore, driving conditions were irrelevant. W would respond that the information was relevant to determining if Dan could see the red light and whether George could see the incident. It seems that George did not see the incident.

A court would find that the question would be relevant because it would shed light on Dan and W's ability to see the signals.

3) Umbrellas up Speculation Truck would say that George was speculating whether it was raining or not. He lacked personal knowledge of whether it was raining but speculated that it was raining from open umbrellas. W would say that George may have had personal knowledge about the umbrellas and that would be sufficient.

A court would find that the fact was not established by his personal knowledge and hence the raining part would not be allowed.

Relevance There would be the same objection as for question 2.

4) Tell me everything Calls for a narrative. Truck would say that the question was too open-ended. The point of direct exam is to ask specific questions and not allow ramblings that may lead to inadmissible evidence. Here, there are no bounds to the way that George could answer.

26 A court would find that there was a call for a narrative and would ask for a more specific question.

5) Guy's shouted statements Hearsay Hearsay is an out-of-court statement that is being offered to establish the truth of the matter asserted. Here, Truck would say the statement of the guy would be an out-of-court statement offered to prove that indeed the truck ran the red light. In fact, it was being offered for the truth, so if there is no exception, then it would be struck as hearsay.

Excited utterance W would say that it was an excited utterance. An excited utterance is a statement made about a startling event that was made under the excitement of that event. W would say that seeing the accident would be sufficient excitement and that it was indeed made under the influence of that excitement. Here, there is an urgent call for an ambulance -- this would indicate that the statement about the red light was also made under the excitement.

A court would find that there was an excited utterance.

Present sense impression W would say that there was a PSI. There must be a statement about what someone currently is sensing. Here, the guy is making a statement about a past sensation -- seeing the accident.

A court would not find PSI.

A court would allow the testimony as an excited utterance.

6) Truck over the speed limit Personal Knowledge -- speculation Truck would say that George lacks personal knowledge. He did not see the accident and therefore cannot make an assessment of its speed.

Here, a court would say that he did not have personal knowledge and would disallow the statement.

Lay opinion. Truck would say that it was an improper opinion. Lay opinions are allowed if they are helpful, do not require expertise, and can be made on the facts.

Here, it would be helpful to know the speed. However, he did not have the fact because of lack of personal knowledge. Generally, there is lay opinion allowed for estimations of speed.

27 Here a court would find that it was an inappropriate opinion because there was no personal knowledge.

Hearsay Truck would say that it is hearsay because it was offered to prove that it was speeding. This was an out-of-court statement even by the guy testifying.

A court would find that it was offered to prove the truth and not admit it because there was no exception.

7) Woman's statement; Dan's silence Hearsay. Truck would say that it was offered to prove that it was Dan's fault.

There would be no exception because it was not under excitement (she said it calmly) and it was an opinion based on a recollection that would not allow a PSI.

A court would not allow it to be admitted.

Admission by silence and vicarious liability W would say that it is not hearsay at all because it is an admission by a party opponent. Dan's silence would be a hearsay statement as an admission of his guilt. He would be subject to the rule about admissions of party opponents because he was working for Truck and the comment was in the scope of his employment.

It would be an admission if: 1) a reasonable person would respond and 2) he had an opportunity to respond. Here, he could say something but did not. Also, with an accusation like that, he should have denied it. He would say he did not have to.

A court would allow the admission by silence because it was not hearsay -- as an omission by a party opponent. His statement would be inadmissible because he was in the scope of employment.

28 California Bar Examination

Essay Questions and Selected Answers

February 2002 Question 6

Phil sued Dirk, a barber, seeking damages for personal injuries resulting from a hair treatment Dirk performed on Phil. The complaint alleged that most of Phil’s hair fell out as a result of the treatment. At a jury trial, the following occurred:

A. Phil’s attorney called Wit to testify that the type of hair loss suffered by Phil was abnormal. Before Wit could testify, the judge stated that he had been a trained barber prior to going to law school. He took judicial notice that this type of hair loss was not normal and instructed the jury accordingly.

B. Phil testified that, right after he discovered his hair loss, he called Dirk and told Dirk what had happened. Phil testified that Dirk then said: (1) “I knew I put too many chemicals in the solution I used on you, so won’t you take $1,000 in settlement?” (2) “I fixed the solution and now have it corrected.” (3) “Don’t worry because Insco, my insurance company, told me that it will take care of everything.”

C. Phil produced a letter at trial addressed to him bearing the signature “Dirk.” The letter states that Dirk used an improper solution containing too many chemicals on Phil for his hair treatment. Phil testified that he received this letter through the mail about a week after the incident at the barbershop. The court admitted the letter into evidence.

D. In his defense, Dirk called Chemist, who testified as an expert witness that he applied to his own hair the same solution that had been used on Phil and that he suffered no loss of hair.

Assume that, in each instance, all appropriate objections were made. Did the court err in:

1. Taking judicial notice and instructing the jury on hair loss? Discuss.

2. Admitting Phil’s testimony regarding Dirk’s statements? Discuss.

3. Admitting the letter produced by Phil? Discuss.

4. Admitting Chemist’s testimony? Discuss.

79 ANSWER A TO ESSAY QUESTION 6

Phil v. Dirk

This question raises issues under the Federal Rules of Evidence. (FRE)

1. Judicial Notice.

Under the FRE, judges may take Ajudicial notice@ of certain types of facts. To take judicial notice, the fact must be of the type that (sic) well-established and commonly known , including certain scientific facts B for example, that water freezes at 32E. In a civil case, if a fact is judicially notice (sic) and the judge so instructs the jury that fact is conclusively established.

Here, the judicial notice was improper. It is not commonly known or well-established that the type of hair loss suffered by Phil (P) was abnormal.

Proving that P's hair loss was abnormal was part of P's case-in-chief to establish negligence. The judge cannot use his personal experience to judicially notice a material fact. The instruction was error.

The court erred in taking judicial notice.

78 2. Phil's Testimony Regarding Dirk's Statements.

Presentation. Witnesses are Acompetent@ to testify only if they have personal knowledge of the subject matter of their testimony. Here, Phil called Dirk and heard

Dirk's statements himself, so Phil has personal knowledge.

Relevance. Only relevant evidence is admissible. Evidence is relevant if it has a tendency to prove a fact in issue. Here, Dirk's testimony tends to prove his negligence, so it is relevant.

Exceptions to Relevance: Substantial Risk of Prejudice

However, not all relevant evidence is admissible. A court may exclude relevant evidence if Aits probative value is substantially outweighed by the risk of prejudice.@ This rule is within the court's discretion. It would not apply here though because several Public Policy Exceptions apply: (1) Offers of settlement. Offers to settle claims will be excluded due to the public policy of encouraging settlements. The rule only applies if there is an actual claim however: that is there is a dispute as to (1) liability or (2) amount.

Here, D will argue that his statement regarding paying $1000 was clearly an offer of settlement and should have been excluded. Although D admitted he Aknew he put too many chemicals in@ the amount was still in dispute. Also, Phil had called him to complain about the hair loss, suggesting that Phil was threatening suit. This testimony should have been excluded.

79 (2) Remedial Measures. Evidence of remedial measures taken after the incident are not admissible for public policy reasons of encouraging remedial actions.

Here, D's statement clearly shows the taking of remedial action and may be excluded.

Evidence of remedial measures may be, however, admissible to prove ownership and control, or to rebut proof that greater care could not be taken. If D presented evidence that the chemicals he used were proper and could not be changed, then D's statement that he Afixed the solution@ could be admitted to rebut, for that purpose only. The courts should have allowed the testimony but given a limiting instruction in purpose.

(3) Liability Insurance.

Evidence of liability insurance is also excluded for the public policy to encourage the purchase of insurance. It generally is inadmissible, except to show ownership and control. D's statement is only about liability insurance, and ownership and control is not at issue.

The court should have excluded this testimony.

Hearsay. Hearsay is an out of court statement offered to prove the truth of the matter amended. D may argue that the testimony should have been excluded as hearsay.

80 However, all three of D's statements are admissible because they are non-hearsay as an admission of a party opponent. The court erred in allowing the testimony only on the public policy grounds discussed above.

3. Admitting Phil's Letter.

Form: P's testimony about the letter is proper since he has personal knowledge.

He did not testify about whether the handwriting and signature were

actually D's. He could only do this if he had personal knowledge of D's

writing, or if the letter was a response letter to something he had written.

Presentation: Foundation, Authentication, Best Evidence Rule.

Foundation: P's testimony about receiving the letter a week after the incident at

D's barbershop laid a proper foundation for the document.

Authentication: Documents must be authenticated before they can be admitted

into evidence. They can be authenticated by testimony of a witness with

personal knowledge about the document. P's testimony is sufficient.

Best Evidence Rule: This Rule requires that where the party is trying to prove the

contents of the document, the Aoriginal@ document must be submitted, or if

it is not the original document, an explanation that is satisfactory must be

given as to why the original document is not submitted. Here, P is trying

to prove the contents of the letter. P is not testifying about what the letter

81 said, but is actually introducing the letter into evidence. Since he is

submitting the original, the rule is satisfied.

Relevance: The letter is relevant because it tends to prove that D was negligent,

the issue in the case.

Hearsay: D may object that the letter is hearsay. However the statement in the

letter is an admission by a party - opponent and is non-hearsay. The court

did not err in admitting the letter.

4. Chemist's Testimony.

Form: D can produce Aexpert witnesses@ to testify in aid of his case.

Presentation: Expert witnesses must meet several requirements before they can

testify. The testimony must be helpful to the factfinder and based on

scientific evidence. The expert himself must be qualified, may rely on

treatises or other scientific, well-established bases of information, and

must have personal knowledge of the facts of the case being discussed

(must make himself or herself aware of the facts).

Here, Chemist does not meet these requirements.

82 Chemist's testimony is not based on sufficiently scientific evidence. Conducting one experiment upon himself does not qualify as scientific and is not helpful to the fact- finder.

It is unclear whether Chemist is qualified to testify to this matter, whether he knows about chemical effects on hair loss for example. He did not mention relying on scientific evidence or treatises to conduct his experiment. He seems to have personal knowledge of the facts of the case if he knows the chemical solution used on Phil, but this is insufficient to qualify as an expert witness.

The court erred in admitting Chemist's testimony.

83 ANSWER B TO ESSAY QUESTION 6

I. Judicial Notice and Jury Instruction

A. Judicial Notice

A judge may take judicial notice on his own initiative. However, a judge may only take judicial notice of things of common knowledge, or that may be ascertained by reference to sources of undisputed accuracy.

Here, the judge took notice of the fact that the type of hair loss was not normal.

He based this, not on common knowledge, or on reference to a source of undisputed accuracy, but on his own personal knowledge. This was not a proper basis for judicial notice. Therefore the court erred.

B. Jury Instruction.

The instruction itself, other than the error in the judicial notice, would not have been in error. The judge may instruct the jury that something has been judicially noticed. In a civil case such as this one, it would be conclusive. However, because the judicial notice was in error, so was the instruction.

C. Misconduct

A judge may not offer expert testimony in a trial over which he presides. His actions should subject him to discipline.

84 II. Phil's Testimony re: Dirk's Statements

A. Statement One

1. Logical relevance

The statement is relevant because it has Aany tendency@ to show that D was negligent and liable.

2. Legal relevance

Offers of settlement and negotiations are inadmissible to prove negligence or liability, due to a public policy of encouraging such measures. Here, D's first statement was a negotiation and an offer. However, spontaneous offers made when no case is pending are admissible. Here, all we know is that P called D to tell him what had happened. It appears no claim was pending, so admissible.

D may also argue it was an offer to pay medical expenses, inadmissible because of a policy encouraging such measures. However, it is doubtful that P was going to seek medical care B there is no mention of physical injury. Further, the exception only applies to the offer, and not surrounding statements, so the statement about too many chemicals would come in.

85 3. Hearsay

D's statement was made out of court, and is offered for its truth, so it may be hearsay.

However, under the FRE, admissions of a party opponent are admissible.

Here, D is a party, and the statement is offered by P, his opponent. It is an admission because it is an acknowledgment of a fact in issue, namely D's liability. Thus, it is not hearsay.

If D testifies that he didn't use too much, the statement will also come in to impeach him as a prior inconsistent statement, so long as D's given a chance to explain or deny it.

Finally, it might also be a state of mind statement of D's intent to pay P

$1000, which is an exception to hearsay.

Thus, the statement was admissible and in error.

B. Statement Two

1. Logical relevance

If he Afixed@ it, it must have been Abroken@ B negligence.

86 2. Legal relevance

Evidence of subsequent remedial measures are inadmissible to show liability or negligence due to a policy of encouraging such measures. Here, D said he fixed the solution after P's harm. Thus, it was a subsequent remedial measure.

However, if D denies the solution was his, or that a fix was possible, it will be admissible. Further, if there are no longer any samples of the solution used on P, the statement can come in to explain why D's new solution isn't defective, or that D destroyed evidence.

3. Hearsay

See above.

Party admission, so not hearsay. May be inconsistent, depending on D's testimony, Also could be present sense impression, if D made it while fixing the solution.

Therefore, the statement was admissible only if an exception to the bar on subsequent remedial measures applies. Probably an error to admit.

87 C. Statement Three

1. Logical relevance

Saying that the insurance company would cover it shows that D was liable.

2. Legal Relevance

Evidence of liability insurance is inadmissible to show liability or liability to pay, due to a policy encouraging insurance.

Here, D said his insurers would cover it. Therefore it is inadmissible unless used to prove ownership and control, which appears to be undisputed.

3. Hearsay.

Even if not barred by public policy, there are two levels of hearsay. The statement from D to P is an admission, as discussed above. However, the statement from the insurance company to D (Athey told me@) is also an out of court statement offered for its truth, and not under any exception, unless there are circumstantial guarantees of trustworthiness, , and notice, none of which are present here.

Thus, the court erred in admitting the statement.

88 III. The letter

A. Logical Relevance

It goes to show D was negligent.

B. Legal relevance

It may be more prejudicial than probative, if D's statement, above, came in. It might be unnecessary cumulative evidence that would not add much and would waste the jury's time.

C. Authentication

A document must be properly authentic. Here, it is a letter allegedly from D. P needed to authenticate.

P could have authenticated by having someone familiar with D's handwriting or signature testify it was his, or by having an expert or the jury compare it to a sample of

D's handwriting or signature.

Or, if P had written a letter to D, and received this one in response, it could be authenticated. A response to their phone call was not sufficient. Since P did none of this, not authenticated, and error to admit.

89 D. Hearsay

Party admission B see above.

IV. Expert

An expert may give testimony on any subject beyond common experience that is helpful to the trier of fact.

The expert must be qualified, express reasonable certainty about their opinion, and have a proper factual basis, such as hypotheticals, things generally relied on by such experts, or personal knowledge. An expert may testify to ultimate issues.

The hair chemicals and effects appear to be beyond common experience. It would be helpful to the trier of fact to see if the expert found the chemicals to make him lose hair (that's why it's relevant).

However, it's unclear that Chemist was qualified as an expert in the subject of chemicals and hair loss. If he was, an opinion based on his experiment would be admissible. However, here, the expert gave no opinion as to any issue in this case, but merely testified about what he did to himself. The jury may not know enough to tell whether the experiment shows that P's claim has no merit. They needed an expert opinion to show them the two were comparable. Because he gave no opinion, he did not express any certainty.

90 Finally, if the Aexperiment@ is not let in as expert testimony, it must be authenticated. To have proper foundation for an experiment, there must be evidence that the experiment was conducted with the same materials, under the same conditions as the events at issue. No foundation was laid here.

Thus, admission as expert opinion or as evidence in its own right was in error.

91 ESSAY QUESTIONS AND SELECTED ANSWERS

FEBRUARY 2003 CALIFORNIA BAR EXAMINATION

This publication contains the six essay questions from the February 2003 California Bar Examination and two selected answers to each question.

The answers received good grades and were written by applicants who passed the examination. The answers were prepared by their authors, and were transcribed as submitted, except that minor corrections in spelling and punctuation were made for ease in reading. The answers are reproduced here with the consent of their authors and may not be reprinted.

Question Number Contents Page

1. Civil Procedure

2. Wills/Real Property

3. Criminal Law & Procedure/Evidence

4. Professional Responsibility

5. Constitutional Law

6. Community Property QUESTION 3

Don was a passenger in Vic’s car. While driving in a desolate mountain area, Vic stopped and offered Don an hallucinogenic drug. Don refused, but Vic said if Don wished to stay in the car, he would have to join Vic in using the drug. Fearing that he would be abandoned in freezing temperatures many miles from the nearest town, Don ingested the drug.

While under the influence of the drug, Don killed Vic, left the body beside the road, and drove Vic’s car to town. Later he was arrested by police officers who had discovered Vic’s body. Don has no recall of the events between the time he ingested the drug and his arrest.

After Don was arraigned on a charge of first degree murder, the police learned that Wes had witnessed the killing. Aware that Don had been arraigned and was scheduled for a preliminary hearing at the courthouse on that day, police officers took Wes to the courthouse for the express purpose of having him attempt to identify the killer from photographs of several suspects. As Wes walked into the courthouse with one of the officers, he encountered Don and his lawyer. Without any request by the officer, Wes told the officer he recognized Don as the killer. Don’s attorney was advised of Wes’s statement to the officer, of the circumstances in which it was made, and of the officer’s expected testimony at trial that Wes had identified Don in this manner.

Don moved to exclude evidence of the courthouse identification by Wes on grounds that the identification procedure violated Don’s federal constitutional rights to counsel and due process of law and that the officer’s testimony about the identification would be inadmissible hearsay. The court denied the motion.

At trial, Don testified about the events preceding Vic’s death and his total lack of recall of the killing.

1. Did the court err in denying Don’s motion? Discuss.

2. If the jury believes Don’s testimony, can it properly convict Don of: (a) First degree murder? Discuss. (b) Second degree murder? Discuss.

-18- Answer A to Question 3

1. Did the court err in denying Don’s motion?

The issue here is whether the court properly denied Don’s motion to exclude evidence of the courthouse identification.

Right to Counsel:

Don’s first ground for having the identification evidence excluded is that the procedure violated his federal constitutional rights to counsel.

Sixth Amendment: The Sixth Amendment of the US Constitution, which is applicable to the states through the Due Process Clause of the Fourteenth Amendment, affords citizens the right to counsel during all post-charge proceedings. The Sixth Amendment right to counsel only applies after a Defendant has been formerly charged. Here, Don was arraigned and therefore the Sixth Amendment right to counsel for his post-charge proceedings applies.

Don is arguing that the identification should be excluded on the grounds that it violated his federal constitutional grounds that the identification procedure violated Don’s federal constitutional rights to counsel. However, Don’s attorney was present with him during the identification. Don is going to argue that they were not made aware of the identification and given an opportunity to object to it. His lawyer was told of the identification and its methods, however, it is unclear as to when the attorney was advised of this information. It seems more likely that he was told after the identification had already been made.

However, the Sixth Amendment right to counsel does not apply to identifications of the suspect, since it’s not a proceedings for purposes of the Sixth Amendment right to counsel.

Fifth Amendment: Miranda warning: Miranda warnings also afford the defendant of right to counsel. This right is to have an attorney present during all interrogation or questioning by the police. Miranda warnings are given to someone upon arrest. They include the right to remain silent and that everything said can be used in court against him, the right to have an attorney present and the right to have an attorney appointed by the court if the arrestee cannot afford one. [In] this case the right to counsel issue did not arise as a Miranda violation, since there was no questioning or interrogation of the police, and the Defendant has already been arraigned.

-19- This case involves the Sixth Amendment right to counsel in all post charge proceedings. There are certain occasions where there is no right to counsel, for example, a photo identification of a suspect, taking of handwriting or voice samples, etc.

Because the identification of a suspect by a witness does not afford the Si[x]th Amendment right to counsel, and because Don’s lawyer was actually present with him during the identification, the court was probably correct in denying Don’s motion to exclude the evidence on this ground.

Due Process:

Don’s second ground for having the identification evidence excluded is violation of due process of law.

Identification

The police may use different methods wherein witnesses can identify suspects as the crime doer. These methods include photo identification, lineups and in-court identifications. The identification process must be fair to the suspect and not involve prejudice and therefore not violate his due process rights. For example, the lineup must include others of similar build and appearance as the suspect.

The police in this case were going to have the Wes [sic]identify Don (or the murderer) through photo identification. However, they took him to the courthouse knowing that Don was having his preliminary hearing that day. The photo lineup did not have to be at the courthouse, in fact it is usually at the police station. This questions the officers’ conduct and intent. Don is going to argue that this was done with the express purpose of having Wes see him at the hearing and associate him to the crime. This is prejudicial to Don and a possible due process violation.

The police will argue that it was mere coincidence that they ran into Don in the courthouse and that their intent was to have Wes identify the murderer [sic] through a photo identification. They will further argue that Wes told the officer he recognized Don as the killer without any request by the officer. Therefore his identification was spontaneous and not prompted. Therefore it did not violate Don’s due process rights.

However it is very suggestive to a witness to see a defendant charged with the crime and make the identification that way. If Wes had identified Don independent of that situation then the identification would have been valid and there would be no due

-20- process violation. However, that was Wes’ first and only identification of Don, and Don is going to argue that it was prejudicial and violated due process of law.

Officer’s testimony

Don is further claiming in his motion to exclude that the officer testifying to the identification would be inadmissible hearsay.

Relevance:

For any testimony or evidence to be admitted it must first be relevant. Here the officer’s testimony will be established as relevant since it involves a witness’ identification of the defendant as the murderer.

Hearsay:

Hearsay is an out-of-court statement made by a declarant that goes to the truth of the matter asserted. Hearsay is inadmissible generally because of the Defendant’s right to confront and cross-examine witnesses. The officer is going to testify that he heard Wes tell him that he recognized Don as the killer. The statement was made out of court and goes directly to prove that Don is the killer. Therefore officer’s testimony is hearsay. The question then is, is it admissible hearsay? There are exceptions to the hearsay rule depending on whether the declarant is available or unavailable to testify. There is no indication whether Wes is available or unavailable so we must look at the possible exceptions to the hearsay rule.

Present Sense Impression: Present sense impression is an exception to hearsay. This is when a declarant is expressing a present impression at that moment without an opportunity to reflect. The State will argue that Wes, upon seeing Don, merely expressed that he recognized him as the murderer. It was an impression at the present he was expressing. However this exception will probably not apply in this case since [sic].

State of Mind: The state of mind exception is a statement by the declarant that reflects the declarant’s state of mind. For example, if the declarant said he was going to Las Vegas this weekend, that statement would be admissible to show that defendant intended on going to Las Vegas for the weekend. This is an exception to hearsay and would be admissible. The state of mind exception does not apply to this case.

Excited Utterance: A statement made when the declarant is an excited state caused by

-21- an event and has not had a chance to cool down. Nothing in the facts here indicate that Wes’ identification of Don was an excited utterance and therefore this exception does not apply.

Admission by Party Opponent: Statements made by the opposing party are usually admissible as an exception to hearsay. Here, since the statement the officer is going to testify to is not that of Don’s but rather Wes, the exception does not apply here [sic].

Declaration Against Interest: When a declarant makes a statement that goes against his own interests, that statement is admissible as an exception to the hearsay rule. Again, Wes’ statement was not against his own interest but against Don’s interest and therefore this exception is not applicable here.

None of the other exceptions, including dying declaration, business record, are applicable here. It appears as though the officer’s testimony is inadmissible hearsay. Therefore the court erred in denying Don’s motion on this ground.

2. (a) First Degree Murder

Under common law, murder was homicide with malice aforethought. There were three types: murder, voluntary manslaughter and involuntary manslaughter. Statutes have categorized murder into de [sic].

The issue here is that if the jury believes Don’s testimony, can Don be convicted of first degree murder[?]

Murder is the killing of another human being. It requires an actus reus (physical act) and a mentus rea (state of mind). The defendant must have the requisite state of mind in conjunction with a physical act to be guilty of murder. The state of mind does not have to be the specific intent to kill; it could be a reckless disregard or an intent to seriously injure or harm.

First degree murder is murder with premeditation or murder during the commission of violent felony (felony murder).

Premeditation: Premeditation and thus first degree murder, is a specific intent crime. Premeditation involves the prior deliberation and planning to carry out the crime in a cold, methodical manner.

-22- In this case there are no facts to indicate that Don planned or premeditated Vic’s murder. In fact, according to the facts, Don was intoxicated and has no recollection of the killing.

Intoxication: There are two states of intoxication, voluntary and involuntary. Voluntary intoxication involves the voluntary ingestion of an intoxicating substance. It is not usually a defense to murder. Voluntary intoxication can be a defense to specific intent crimes, if it was not possible for the defendant to have the state of mind to form intent.

Involuntary intoxication is the involuntary ingestion of an intoxicating substance, such as with duress, without knowing of its nature, prescribed by a medical professional, etc.

In this case, Don was intoxicated since he ingested the hallucinogenic drug. Although Don was aware of what he was taking when he took it, he will argue that he was forced to take it under duress. Since Vic threatened Don that he would abandon him in freezing temperatures far from any town, Don was forced to take the drug. Although involuntary intoxication is not a defense to murder, it is a proper defense to the specific intent required for premeditation and thus first degree murder.

Since Don did not premeditate the murder nor have the specific intent for premeditated murder, he cannot be convicted of first degree murder.

Felony Murder: Felony murder is murder committed during the commission of an inherently dangerous felony. There are no facts to indicate that Don was committing an inherently dangerous felony, independent of the murder itself. Therefore felony murder probably does not apply in this case and Don cannot be convicted of First degree murder.

2. (b) Second Degree Murder

Second degree murder is all murder that is not first degree and is not made with adequate provocation to qualify for Voluntary Manslaughter. Second degree murder does not require specific intent.

The issue here is if the Jury believes Don’s testimony, can Don be properly convicted of Second degree murder?

Don is going to use the defense of intoxication. Although intoxication is not a defense to murder, involuntary intoxication can negate a required state of mind. Since it will

-23- probably be determined that Don’s intoxication was involuntary due to duress (see discussion above), Don will argue that he did not have the state of mind required to commit second degree murder. He will be compared to a person who is unconscious. An unconscious person cannot be guilty of murder. Don will argue that he was so heavily intoxicated that he has no recollection of the occurrences and therefore could not have had even the general intent to kill or seriously injure.

Voluntary manslaughter: in order for a murder charge to be reduced to voluntary manslaughter there must be adequate provocation judged by a reasonable standard and no opportunity to cool down and the defendant did not in fact cool down. Nothing in these facts suggests that Don acted under the heat of passion or was provoked in any way. In fact Don does not remember the killing and therefore there is no evidence of provocation.

Since was [sic] involuntarily intoxicated, he could not have the requisite state of mind for murder. Therefore he cannot be convicted of either first degree or second degree murder.

-24- Answer B to Question 3

I. Court’s Denial of Don’s (D’s) Motion

A. Violation of D’s right to counsel

The Sixth Amendment guarantees defendants the presence of counsel at all critical stages of a criminal proceeding which results in imprisonment, as well as providing that the police may not elicit information from a defendant in the absence of counsel once criminal proceedings have been initiated against the defendant, usually in the form of an arraignment. Among those stages of a criminal proceeding which are considered critical are a preliminary hearing, at trial, when making a plea, at sentencing, and at any lineup or show-up conducted following the filing of charges against the defendant.

In this instance, the identification of D occurred after he was arraigned, and thus D did have a right to have counsel present during any lineup or show-up. However, this right to counsel does not extend to photographic identifications, which are not considered adversarial proceedings, but instead only to in-person lineups or show-ups. Thus, the police in this instance will claim that they simply took Wes (W) to the courthouse for the express purpose of having him attempt to identify the killer from photographs of several suspects, something for which D was not entitled to the presence of counsel, and the fact that W witnessed D emerging from the courthouse was not part of their plan, and something for which they should not be held responsible. Further, the police will refer to the fact that when D emerged from the courthouse they made no request that W identify D, but rather W made such an identification completely of his own volition.

D’s counsel will most likely argue that the police were well aware that D would be at the courthouse at that particu[la]r time, and that bringing W to the courthouse ostensibly to view photographs was in reality simply a veiled effort to conduct a one-on-one show-up in which W could identify D, and that D thus had the right to counsel at such a proceeding.

In this instance, the court did not err in denying D’s motion based on grounds that the identification procedure violated D’s Sixth Amendment right to counsel. The Sixth Amendment guarantees the right to counsel at any post-charge lineup or show-up in part to ensure that the defendant’s attorney will be aware of any potentially unfair methods utilized in the identification process, and can refer to these inequities in court. Because D’s counsel was in fact present when W saw and identified D, D’s attorney would be able to raise any objections he had to the identification, and thus D was not ultimately denied his right to counsel. Thus, even if the court were to find that the police bring[ing]

-24- W to the courthouse amounted to a show-up in which D was entitled to the presence of counsel, D was with his attorney when the identification was made, and therefore his right to counsel was satisfied.

B. The identification as violative of due process of law

The Due Process Clause of the 14th Amendment, made applicable to the federal government by the Fifth Amendment, ensures that the prosecution bears the burden of proving each element of a criminal case against defendant beyond a reasonable doubt, and also guarantees that a defendant will be free from any identification which is unnecessarily suggestive or provides a substantial likelihood of misidentification.

In this instance, D’s attorney would probably contend that the police bringing W to the courthouse on the date of D’s prelimi[na]ry hearing to view photographs of suspects in fact raised a substantial probability that W would in fact observe D emerging from the courthouse, which is exactly what occurred. D’s attorney would contend that any identification made in this context is extremely suggestive, as the fact that D is emerging from a court of law and was in the presence of an attorney places D in a situation in which he appears to be of a criminal nature, and is likely to lead an eyewitnesses to mistakenly identify D based solely on these circumstantial factors. Further, D’s attorney would argue that the situation was unnecessarily suggestive because the witness could believe the fact that criminal proceedings had already been initiated against D, thus warranting his appearance in court, sufficient evidence, perhaps even in the form of testimony by other eyewitnesses, exists which incriminates D, and may make W more likely to believe that D was the man he had seen commit the killing.

The court probably did not err in denying D’s motion based on the fac[t] that W’s identification was violative of due process of law. The 14th Amendment guarantees against unnecessarily suggestive identifications, or identifications posing a substantial likelihood of misidentification, are intended primarily to remedy lineups in which a criminal defendant is placed in a lineup with other individuals to whom he bears no physical similarities whatsoever. It is unlikely that a court would find that a witness seeing an individual emerging from a courthouse would be so prejudicial as to lead to an unnecessarily suggestive identification.

-25- C. Hearsay

Hearsay is an out-of-court statement being offered to prove the truth of the matter asserted. In this instance, the officer’s planned testimony that W had identified D at the courthouse would qualify as hearsay, as the officer would be testifying to a statement made by W ou[t] of court in order to prove that W identified D.

However, instances in which a witness has previously identified a suspect are admissible as exceptions to the hearsay rule even if the defense is not attacking the identification. Such statements of prior identification are considered to possess sufficient guarantees of trustworthiness that the party against whom they are offered is not denied his Sixth Amendment right to confront witnesses against him. Therefore, the court did not err in denying D’s motion to exclude the evidence of the courthouse identification because the officer’s testimony would in fact not be inadmissible hearsay. II. Crimes for which D may be properly convicted

A. First degree murder

In order to convict a defendant of first degree murder, the prosecution must prove beyond a reasonable doubt that the defendant unlawfully killed a human being with malice aforethought, and that the killing was either premeditated and deliberate or was committed during the commission or attempted commission of an inherently dangerous felony (felony murder). In order to prove malice aforethought, the prosecution must show that defendant acted with an intent to kill, an intent to inflict serious bodily harm, acted with a depraved and malignant heart, or was guilty of felony murder.

In this instance, D’s acts appear to be both the actual and proximate cause of Vic’s (V’s) death, as the facts indicate that D killed V and dumped his body beside the road. However, D would probably be found not to possess the requisite intent to kill or to inflict serious bodily harm by way of his raising the excuse of involuntary intoxication. Intoxication, whether voluntary or involuntary, may be raised to negate the presence of an essential element of a crime, generally intent. In this instance, D’s intoxication would be involuntary, as he did not wish to take the hallucinogenic drug V offered, but was forced to when he feared that if he did not, he would be abandoned in freezing temperatures and his life would be in jeopardy. Ingesting a drug under such circumstances is the virtual equivalent of being unknowingly slipped the drug, or being forced to ingest the drug upon threats of death. As such, D was involuntarily intoxicated, and his intoxication resulted in his having no recall of the events between the time he ingested the drug and his arrest. D thus will be found not to have posssessed the requisite intent to kill or intent to inflict serious bodily harm necessary for a finding of first

-26- degree murder. Further, even if D were not able to rely on the excuse of intoxication in order to negate a requisite mental state, there is no evidence that the killing was premeditated or deliberate, and because it did not occur during the commission or attempted commission of an inherently dangerous felony, there is no basis for finding D guilty of first degree murder.

2. Second degree murder

The jury most likely could not properly convict D of second degree murder, either. Second degree murder also requires the prosecution to prove beyond a reasonable doubt that the defendant intentionally killed a human being with malice aforethought, though it relieves the prosecution of proving the additional elements of premeditation and deliberation or felony murder.

In this instance, D’s involuntary intoxication resulting from his unwillingly ingesting a[n] hallucinogenic drug should sufficiently relieve him from being found guilty of second degree murder, as it negates the requisite mental states of intent to kill or intent to inflict serious bodily harm as discussed above. Further, D should not be convicted under a theory of depraved or malignant heart, as such a finding requires proof of reckless conduct which created a substantial and unjustifiable risk of death or serious bodily harm. A defendant must be consciously aware of the risk he is creating to be guilty of a depraved heart killing, and D’s involuntary intoxication would most likely relieve him of guilt, since he had no recall of the events between the time he ingested the drug and his arrest, and would most likely not be considered to have appreciated the risk of his conduct.

If D were found to have been intoxicated voluntarily, rather than involuntarily, he could be properly convicted of second degree murder for V’s killing. However, if the jury believes D’s testimony that he only ingested the hallucinogenic drug because he feared if he did not he would be left out in the cold and could potentially die, they must find that D was involuntarily intoxicated, which would relieve him of guilt for second degree murder.

-27- ESSAY QUESTIONS AND SELECTED ANSWERS

JULY 2003 CALIFORNIA BAR EXAMINATION

This publication contains the six essay questions from the July 2003 California Bar Examination and two selected answers to each question.

The answers received good grades and were written by applicants who passed the examination. The answers were prepared by their authors, and were transcribed as submitted, except that minor corrections in spelling and punctuation were made for ease in reading. The answers are reproduced here with the consent of their authors and may not be reprinted.

Question Number Contents Page

1. Corporations 1

2. Remedies 10

3. Evidence 18

4. Torts 27

5. Professional Responsibility 42

6. Wills 50

i Question 3

Dan was charged with aggravated assault on Paul, an off-duty police officer, in a tavern. The prosecutor called Paul as the first witness at the criminal trial. Paul testified that he and Dan were at the tavern and that the incident arose when Dan became irate over their discussion about Dan’s ex-girlfriend. Then the following questions were asked and answers given:

17. What happened then? [1] A: I went over to Dan and said to him, “Your ex-girlfriend Gina is living with me now.” Q: Did Dan say anything? [2] A: He said, “Yeah, and my buddies tell me you’re treating her like dirt.” [3] Q: Is that when he pulled the club out of his pocket? A: He sure did. Then he just sat there tapping it against the bar. [4] Q: Tell the jury everything that happened after that. [5] A: I said that he was a fine one to be talking. I told him I’d read several police reports where Gina had called the police after he’d beaten her. Q: Do you believe the substance of those reports? [6] A: You bet I do. I know Gina to be a truthful person. Q: How did Dan react to this statement about the police reports? A: He hit me on the head with the club. Q: What happened next? [7] A: I heard somebody yell, “Watch out– he’s gonna hit you again!” I ducked, but the club hit me on the top of my head. The last thing I remember, I saw a foot kicking at my face. Q: What happened then? [8] A: Dan must have kicked and hit me more after I passed out, because when I came to in the hospital, I had bruises all over my body.

At each of the eight points indicated by numbers, on what grounds could an objection or a motion to strike have properly been made, and how should the trial judge have ruled on each? Discuss.

18 Answer A to Question 3

1. The evidence is relevant. Logical relevance consists of a tendency in reason to support or contend a fact or issue of consequence in the case. Here, the statement is offered to show Dan’s motive for attacking Paul. The statement is also legally relevant, meaning that it is not excluded on any extrinsic policy grounds and its probative value is not substantially outweighed by its prejudicial impact, confusion of the issues, misleading the jury, waste of time, etc.

The defense will likely object to hearsay. Hearsay consists of an out-of-court statement offered for the truth of the matter asserted. The statement is not an admission. An admission occurs when a party to the action admits a fact of relevance to the action. Here, Paul is not a party to the action. He is merely a witness for the prosecution. The prosecutor will argue that the statement is not offered for the truth of the matter asserted, but to show its affect[sic] on the recipient’s state of mind. In other words, we don’t care if the exact words themselves are true (whether Gina is in fact living with Paul), we are trying to explain why Dan would have become incensed enough to attack Paul.

The trial court should rule that the statement is not hearsay because it is not being offered for the truth of the matter asserted.

2. The evidence is relevant. It is logically relevant because it is being offered to show Dan’s angry state of mind. It is legally relevant because there are no policy reasons for excluding it, and its probative value is not outweighed by its prejudicial impact.

The defense will object on hearsay grounds. The prosecutor will argue that Dan’s statement is an admission. It is a statement by a party, and it tends to admit that Dan was in fact angry with Paul, a fact of consequence in this action. Admissions are not hearsay under the federal rules. However, contained within Dan’s statement is another hearsay statement, a statement by Dan’s buddies.

Thus, the defense would object to hearsay within hearsay. The prosecutor should respond that the statement by the buddies is not being offered for the truth of the matter asserted, but to show Dan’s state of mind. In other words, it is immaterial whether Paul was in fact treating Gina like dirt, what matters is that Dan was told he was, and this made Dan angry.

Because the prosecutor has an adequate response to both hearsay objections, the statement should be admitted.

3. The defense will object to this question as leading. Leading questions are not allowed on direct examination. Because Paul is being directly examined by the prosecutor, the prosecutor may not lead Paul, subject to certain exceptions inapplicable here, such as Paul being declared a hostile witness, foundational questions, etc. This is a question of consequence in the matter, and the prosecutor’s question suggests the answer sought by the prosecution. As such, it is leading, and should have been objected to and sustained

19 by the judge.

The question did call for relevant evidence. The evidence called for was both logically relevant (whether Dan had a club with him and brandished it) and legal relevance (no policy reasons and it is very probative).

4. The defense could object to this question on a variety of grounds, but would probably object to the question as vague and calling for a narrative response. Under the federal rules, direct questioning of witnesses is to proceed by question-and-answer. The attorney is supposed to give some structure to the question-and-response process. He may not simply ask an open-ended question a broad [sic] answer and allow the witness to answer as he sees fit. He also may not ask a question that has no degree of specificity with respect to the information sought. Here, “Tell the jury everything” provides no guidance to the witness as to the information sought.

5. “I said that he was a fine one to be talking.”

This statement is relevant. It is logically relevant because it tends to show further Dan’s anger towards Paul and it is legally relevant, because it is probative and there are no policy reasons for excluding it.

The defense will object that the statement is hearsay. It is not an admission, because Paul is not a party to this action. The prosecutor will again argue that it is not being offered for its truth, but simply to show its effect on Dan. In other words, it is not offered to show whether Dan has a right to be talking or not, but to show further its effect on Dan’s state of mind and why Dan became angry enough to attack Paul. This is a close call, but the judge should probably admit the statement because it is not offered for its truth.

“I told him I’d read several police reports where Gina had called the police after he’d beaten her.”

The defense will object to relevance. The statement is logically relevant (it tends to show Dan’s violent nature). However, it is not legally relevant. Its probative value is substantially outweighed by the danger of prejudice. Here, there is no evidence that Dan was convicted of abusing Gina. This would be admissible. Here, the jury may be misled into thinking that Gina’s calls are sufficient proof of Dan’s guilt, and this is improper prejudice. The judge should exclude the evidence on legal relevance grounds.

The defense will also object on the grounds of the best evidenced rule. This rule requires that the contents of a writing be introduced where: (1) the writing is of consequence in the matter; or (2) a witness’s knowledge comes from the writing and the witness testifies as to the actual contents of the writing. Here, Paul is testifying to the contents of the police reports. He’s testifying that the reports stated that Gina called the police and told them that Dan beat her. The prosecutor must introduce an original or accurate copy (unless he establishes they were unavailable) of the reports into evidence to show this evidence. The

20 best evidence rule objection should be sustained.

The defense may also object on the grounds of hearsay within hearsay. The entire second sentence is an out-of-court statement by Paul, and is thus hearsay (not an admission because Paul is not a party). Unlike previous statements by Paul, this statement is arguably being offered for the truth of the matter asserted. It is being offered to show that Dan beat Gina. The prosecutor might argue that this is untrue, we don’t care whether Dan beat Gina, we only care that Dan was upset about being accused in public. The judge might accept this as justification or it might not (even if it accepted the prosecutor’s motivation, the jury might still take it to show that Dan is a “girlfriend beater”, and this further supports exclusion of the evidence as more prejudicial than probative).

The police reports and the statements within the police reports also constitute hearsay. While the police report is admissible as an official record, because the statements are written by individuals with a duty to accurately convey the information in them, Gina’s statements are still hearsay. They do not fit within the official record or business record exception, because Gina is not under a duty to convey the information. She had no obligation to make the calls to the police. Thus, Gina’s statements are hearsay and must be excluded. (Note: the federal catch-all exception would also not allow introduction of Gina’s statements.)

Finally, the defense might object that this is character evidence. Character evidence is evidence of one’s proclivity to act in conformity with a specific character trait on the occasion in question. Here, the defense will argue that the evidence is being introduced to show that Dan has a character trait of violence, and that he acted in conformity with that trait here. This argument should be sustained. The prosecution may not introduce affirmative evidence of specific acts until the defendant has opened the door to such evidence, by either supporting his own character, or attacking the victim’s character. Here, there is no evidence that either has occurred, and the evidence should be excluded as improper character evidence.

6. This evidence is relevant. It is logically relevant, because, if admitted, it would bolster any statements by Gina in the case. There are no policy grounds for its inclusion, and it is probative.

However, the defense will object that this is improper character evidence. A party may not bolster or support the credibility of its own witness (a hearsay declarant is a witness, and may be impeached or have her character attacked as any other witness) until the witness’s credibility has been attacked. Here, the prosecutor has offered opinion evidence by Paul to support Gina’s credibility and character trait for truthfulness. The prosecutor may not do this until and unless the defense attacks Gina’s credibility.

The defense should object on character grounds and the judge should sustain the objection.

21 7. “Watch out–he’s gonna hit you again!”

The statement is relevant. It is logically relevant because it tends to show that Dan hit Paul, and more than once. It is legally relevant because its probative value outweighs its prejudicial impact and there are no policy grounds for its exclusion.

The defense will object to this statement as hearsay. It is an out-of-court statement, and it is being offered for the truth of the matter asserted. However, the federal rules allow the admission of certain out-of-court statements that are admittedly hearsay when the circumstances surrounding the statement inherently support the reliability of the statement made. This applies to excited utterances, present sense impressions, statements of physical condition, and present state of mind. Here, the prosecutor will argue that the statement is an excited utterance. The statement was made spontaneously while under the stress of excitement, so there was little chance to fabricate the substance of the statement. Even though we do not even know the identity of the declarant, the statement is admissible. (Note: The statement would also qualify as a present sense impression, as it was made concurrently with one’s sensory (visual) inputs and thus is inherently reliable because there was no time to consider what one was saying).

“The last thing I remember, I saw a foot kicking at my face.”

The defense might object to this statement as not based on personal knowledge and lacking foundation, meaning that the statement is made under circumstances that indicate that Paul may not have the best recollection of the events. However, this is not a valid objection. The defense should cross-examine Paul about his ability to accurately recall these occurrences, however.

8. This statement, if admitted, is relevant. It is logically relevant because it indicates further malicious attacks by Dan and damages. It is legally relevant because it is probative and no policy grounds exist for its exclusion.

The defense will object that the testimony is not based on personal knowledge, is speculative, and there is a lack of foundation to support the statements. Paul has not indicated he personally observed the kicking, he is merely speculating that that is what occurred. Without more foundation, this objection should be sustained. Paul’s statement about the bruises all over his body, however, are based on personal knowledge and admissible.

22 Answer B to Question 3

Î “Your ex-girlfriend is living with me now.”

Relevance - to show that D became angry because P was living with D’s ex-girlfriend.

Hearsay - Hearsay is an out-of-court statement offered at trial to prove the truth of the matter asserted.

D could object on hearsay grounds because the statement was made by P, outside of the court.

However, the prosecution could successfully argue that the statement is not being offered for its truth. The prosecution is offering the statement to show that it provoked a reaction in D which led to his assault on P. It is offered for its effects on the listener, not for its truth.

Admission– The prosecutor may also argue that if the statement were considered hearsay it would still be admissible under the exception for admissions of a party-opponent. This argument would lose, however, because P is not a party. He is a complaining witness but the government is the party.

Ï “Yeah, and my buddies tell me you’re treating her like dirt.”

Relevance - to show D’s anger over P’s treatment of the ex-girlfriend.

Hearsay - An objection could be made because this is an out-of-court statement offered for its truth.

Not for its truth - However, the prosecution could successfully argue that the statement is not offered for its truth but rather to show D’s state of mind, or motive for the alleged assault.

Exceptions

Admission - Even if considered hearsay, the statement is admissible because it is being offered against a party (D) by his opponent (the prosecution).

State of Mind - In addition, the statement would be admissible to prove D’s state of mind when the statement was made. The statement tends to show that D was feeling ill will towards P and that this motivated the assault.

Ð “Is that when he pulled the club out of his pocket?”

Relevance - to show that D assaulted P with a club. Leading Question - A leading question is one that suggests the correct answer to the

23 witness. Leading questions are permissible where questioning a hostile witness, clarifying background information, or where a witness has a difficulty remembering.

Here, P is not a hostile witness, he was called by the prosecution as their first witness. As none of the other circumstances are present, the leading question here (it suggested the right answer was yes) was impermissible and should have been disallowed.

Assumes Fact Not in Evidence - The question is also objectionable because it assumes facts not in evidence, namely, that D had a club, and that D the club [sic] from his pocket.

Ñ “Tell the jury everything that happened after that.”

Narrative - This question is objectionable because it calls for a narrative. The lawyer must interrogate the witness, not merely call him to the stand and let him tell a story.

Compound - The question could be construed as compound because it calls for the witness to answer what should have been multiple questions all at once.

æ a. “I said he was a fine one to be talking.” b. “I told him I’d read several police reports where Gina had called the police after he’d beaten her.”

a. Is hearsay because it is an out-of-court statement offered for its truth and no exceptions apply.

a. Is also objectionable because it is irrelevant–it has no tendency in reason to make a material fact more or less probable.

403 - Undue Prejudice - Even where evidence is relevant, it may be excluded by the court due to its probative value being outweighed by its danger of unfair prejudice, confusing the issues, misleading the jury, or delay.

Here the evidence should be excluded under 403. The probative value is slight if existent and the danger of confusing the issues (D assaulting P versus D assaulting Gina) is great.

b. The Police Reports

Relevance - to show that D is a violent person or to show the effect this statement had on D.

Hearsay - The statement was double hearsay: Î It is the statement by P at the bar Ï relaying the content of police reports. In order for double hearsay to be admissible there must be an exception or exclusion for each level of hearsay.

Î P’s Statement - The prosecutor could argue it is non-hearsay because it is offered to show its effect on D, not for its truth.

24 Ï The Police Reports - Police reports may be admissible as business records if made by someone in the course of their employment with a duty to make such recordings. However, police reports are not admissible as a business record in a criminal case, as we have here. Further the reports contain a hearsay statement by Gina, who was under no duty to make accurate statements.

Therefore the statement should be stricken.

Best Evidence Rule (BER) - Where the contents of a document are at issue or a witness testifies to something known only from reading a document, the BER requires production of the original document or a valid explanation for its absence.

Here, P testified to contents of police reports. His only knowledge appears to derive from reading the reports. Thus the BER requires their production or an explanation.

Character Evidence - Evidence to show conduct in conformity therewith is inadmissible unless the defendant first opens the door by bolstering his own credibility.

Here, the defendant D has put on no evidence. Also, the prosecution could only rebut by opinion or reputation evidence, not by extrinsic evidence of specific acts, as P testified to.

Relevance - To show Gina told the truth and that therefore D is a violent person.

Personal Knowledge - P lives with Gina and is thus familiar with her character for truthfulness.

Ó “I know Gina to be a truthful person.”

Improper Bolstering of a Witness/Declarant

P improperly testified to Gina’s character for truthfulness. A party may bolster the credibility on a witness/declarant with reputation/opinion evidence of truthfulness only after the credibility of the witness has been attacked.

Here, Gina has not testified, nor did D attack her credibility as a declarant, thus the testimony should be stricken.

Ô “Watch out–he’s gonna hit you again!”

Relevance - to show D attacked P

Hearsay - out-of-court, and offered for its truth, therefore it is hearsay.

Exceptions

25 Excited Utterance - A statement made concerning a startling event while under the stress of the exciting event is admissible as a hearsay exception.

Here, the statement concerned a startling event, an assault with a club, while the declarant was under the stress of the event. The statement appears to have been made in between blows and under great excitement.

Present Sense Impression - A statement made describing an event while the event is occurring or immediately thereafter is admissible as an exception to the hearsay rule.

Here the out-of-court statement was made while the declarant was observing the attack on P. Therefore the statement is admissible.

Õ “D must have kicked me more after I passed out . . . “

Relevance - to show D assaulted P.

Lack of Personal Knowledge - A witness may only testify to things they have personal knowledge of.

Here, P testified to what happened after he had passed out. A person obviously has no personal knowledge of events taking place while they were unconscious. Thus the testimony should have been stricken.

26 ESSAY QUESTION AND SELECTED ANSWERS

JULY 2004 CALIFORNIA BAR EXAMINATION

This publication contains the six essay questions from July 2004 California Bar Examination and two selected answers to each question.

The answers received good grades and were written by applicants who passed the examination. The answers were prepared by their authors, and were transcribed as submitted, except that minor corrections in spelling and punctuation were made for ease in reading. The answers are reproduced here with the consent of their authors.

Question Number Contents Page

1. Criminal Law 1

2. Constitutional Law 12

3. Wills/Trusts 22

4. Evidence 33

5. Professional Responsibility 44

6. Tarts 51

i Question 4 Victor had been dating Daniel’s estranged wife, Wilma. Several days after seeing Victor and Wilma together, Daniel asked Victor to help him work on his pickup truck at a nearby garage. While working under the truck, Victor saw Daniel nearby. Then Victor felt gasoline splash onto his upper body. He saw a flash and the gasoline ignited. He suffered second- and third-degree burns. At the hospital, he talked to a police detective, who immediately thereafter searched the garage and found a cigarette lighter. Daniel was charged with attempted murder. At a jury trial, the following occurred: a. Tom, an acquaintance of Daniel, testified for the prosecution that Daniel had complained to Tom that Victor had “burned” him several times and stated that he (Daniel) would “burn him one of these days.” b. Victor testified for the prosecution that, while Victor was trying to douse the flames, Daniel laughed at him and ran out of the garage. c. At the request of the prosecutor, the judge took judicial notice of the properties of gasoline and its potential to cause serious bodily injury or death when placed on the body and ignited.

In his defense, Daniel testified that he was carrying a gasoline container, tripped, and spilled its contents. He denied possessing the lighter, and said that the fire must have started by accident. He said that he ran out of the garage because the flames frightened him. d. On cross-examination, the prosecutor asked Daniel, “Isn’t it true that the lighter found at the garage had your initials on it?” The prosecutor urged the jury to consider the improbability of Daniel’s claim that he had accidentally spilled the gasoline. e. During a break in deliberations, one juror commented to the other jurors on the low clearance under a pickup truck parked down the street from the courthouse. The juror measured the clearance with a piece of paper. Back in the jury room, the jurors tried to see whether Daniel could have spilled the gasoline in the way he claimed. One juror crouched under a table and another held a cup of water while simulating a fall. After the experiment, five jurors changed their votes and the jury returned a verdict of guilty.

Assume that, in each instance, all appropriate objections were made. 1. Should the court have admitted the evidence in item a? Discuss. 2. Should the court have admitted the evidence in item b? Discuss. 3. Should the court have taken judicial notice as requested in item c? Discuss.

33 4. Should the court have allowed the question asked in item d? Discuss. 5. Was the jury’s conduct described in item e proper? Discuss.

Answer A to Question 4

4)

A. Tom’s (T) Testimony Re Daniel’s (D) Statement

The issue is whether T’s testimony regarding Daniel’s prior statement that D would “burn him (Victor- V) one of these days” is admissible against D.

Logical Relevance

Evidence is logically relevant if it has the tendency to make any fact of consequence in the case more probable or less probable than it would be without the evidence. Here, the main issue of the case is whether D tried to murder V. The statement that D would burn V at some point is relevant to prove that D acted intentionally, rather than accidentally, as claimed.

Legal Relevance

Evidence must be discretionarily relevant and there must not be any extrinsic public policy reasons against its admission. The judge has the discretion under FRE 403 to exclude relevant evidence if the probative value is substantially outweighed by the danger of unfair prejudice, jury confusion, misleading or waste of time, among other reasons.

Here, the statement that D would “burn” V is probative of D’s motive for acting and for rebutting D’s claim that it was an accident, however it is also highly prejudicial to D. All evidence is prejudicial to one party, however, and 403 will only exclude it if the prejudice substantially outweighs probativeness, which is the not the [sic] case here.

As there are not public grounds for excluding the evidence, it would be logically and legally relevant.

Presentation

T testified apparently in the prosecution’s case-in-chief. Because T was the person spoken to, he has personal knowledge of the statement and, so long as he could communicate it and appreciate his oath to tell the truth, would be competent to testify.

Hearsay

34 Hearsay is a statement made by the declarant other than at trial that is introduced for the purpose of proving the truth of the matter asserted in the statement. Hearsay is inadmissible unless it falls within one of the hearsay exceptions within the federal rules. Here, the statement was made out[-]of[-]court by D in a conversation with T.

Truth/Non-Hearsay

The prosecution will argue that it is not introducing the statement for it’s [sic] truth, but rather as circumstantial evidence of D’s state of mind, which is not hearsay under the rules. The prosecution will claim that the statement indicates that D had a grudge against T and his state of mind was one of hatred or disdain. Because attempt is a specific intent crime, this non-truth assertion could be relevant to show that D had the intent to harm T. This argument has merit, however, it would be better for the prosecution’s case if it can get the statement in for the truth.

Admission by a Party Opponent

A statement by a party opponent is not hearsay under the federal rules and comes in for the truth. It need not be against interest when made and may be based on hearsay. In this case, D made the statement to T and it could come in against him as non-hearsay under the FRE.

Hearsay Exceptions Present Intent

A statement made by a person showing an intent to do something is an exception to the hearsay rule and may be admissible to show that the declarant actually followed through with the act in question. Though more commonly associated with statements like “I’m meeting Joe at 10 on Tuesday” to show that the meeting with Joe happened, here it could be admissible to show that D followed though with what he said he was going to do and actually burned V.

B. V’s Testimony that D Laughed While V was Trying to Douse Flames Relevance

V’s testimony is logically relevant because it tends to prove that D acted with an intent to harm V in that, if he hadn’t meant for V to catch on fire he would not have been laughing and he would try to help V. Also, it contradicts D’s claim that he ran out of the garage frightened.

V’s testimony is prejudicial against D, as it tends to paint him as quite the villain, however it is not unduly so and it does not substantially outweigh the probative value. No public policy considerations apply. Accordingly the evidence is relevant.

35 Presentation

V is testifying in the prosecution’s case[-]in[-]chief. As the victim, V was present at the accident and has personal knowledge of the events, although V could be subject to impeachment regarding his ability to really perceive what was happening (he was on fire, after all). However, V has personal knowledge and is competent to testify so long as he has memory, can communicate and can appreciate the requirement of telling the truth.

Hearsay

As mentioned, hearsay is an out[-]of[-]court statement made by the declarant for the purpose of proving the truth of the matter asserted in the statement.

Statement

The issue here is whether D’s laughing was a statement. Assertive conduct is treated like a statement and subject to all the hearsay rules. Generally, assertive is that which tends to substitute for a statement, such as nod of the head instead of “yes” or pointing in a direction instead of “turn left.” Because it has the effect of a statement, assertive conduct is treated like a statement.

D will argue that the laughing is assertive conduct and thus inadmissible to prove the truth, that D laughed, unless it fits within a hearsay exception or may be non-hearsay. He will argue that it is the equivalent of a statement such as “this is great” or “I said I would burn you.”

The prosecution will counter with the argument that it was merely laughing and, unlike assertive conduct such as pointing or nodding, there is no way to determine what was meant by it so it cannot be assertive. It is more likely that a judge would overrule an objection by the defense and that V’s testimony comes in and is not hearsay.

Exceptions/Non-hearsay

Even if the judge were to reject the prosecution’s argument, the statement could come in as an admission by a party opponent, as discussed earlier. Alternately[sic], it could be admissible as an excited utterance because the laughter was made while D was under the stress of the excited event and arguably related to the startling event.

C. Judicial Notice of the Properties of Gasoline

It is proper for a court to take judicial notice of things that are easily proven or of common knowledge in the community. If evidence is required to demonstrate the fact in question, judicial notice may not be proper. The effect of judicial notice in a criminal case

36 is to satisfy the prosecution’s burden of proof, but the jury may elect to disregard the judicially noticed fact and decide otherwise.

The issue is thus whether the properties of gas and its potential to cause serious bodily injury or death when placed on the body and ignited was proper. On the one hand, most adults drive and are familiar with gas stations and the warnings that are all over the station regarding no flames. One the other hand, most people have not played around with gasoline and matches and are not likely familiar with the effects it can have on the body- how long it will burn, how much gas needs to be on the person, when it will explode, etc. This is important because if there is a certain amount of gas required, D could argue the amount spilled on V was insufficient.

While it may have been proper to take judicial notice of the flammable quality of gasoline, the effects of its ignition are not so likely common knowledge. Accordingly, the judge erred in taking judicial notice of this fact and should have required the prosecution to present expert testimony regarding the specific potential of gas to cause serious injury or death when placed on the body and ignited.

D. Cross-Examination of D re Lighter Relevance

The question tends to prove ownership of the lighter and refute D’s claim that he did not own it/impeach him on that issue. It is highly probative and, while somewhat prejudicial, the prejudice does not substantially outweigh the probative value. There are no policy[-]based reasons for exclusions and, accordingly, the evidence is properly admissible.

Form Leading A question that suggests the answer is a leading question and is generally not allowed. Here, the prosecutor’s question suggests that the lighter had D’s initials on it, and is thus leading. Leading questions are allowed, however, on cross-examination, preliminary matters, hostile witnesses and witnesses who are having trouble remembering. Accordingly, because this was cross[-]exam, the leading question was proper.

Assumes Facts in Evidence

The question assumes that, one, there was lighter [sic] found that has been introduced, which on these facts has not been introduced into evidence. The lighter could be an exhibit and would have to be introduced by someone with knowledge[,] who could authenticate the lighter and indicate the chain of custody. After this a proper foundation would be laid and the prosecution could ask the question.

The lighter may self-authenticate, however, as sort of a label, but that is generally

37 reserved for commercial items.

Best Evidence

The initials on the lighter could be considered a writing and the question is aimed at oral testimony to prove its contents. The best evidence rule requires that, before testimony regarding contents may be given, the original, in this case the lighter[,] must be produced or a decent reason for its absence must be given. Here, there is no indication that the lighter has been introduced and thus the content of it, the initials, could not be testified to by D if his only knowledge of the content came from the lighter.

E. Jury’s Conduct

Juries are prohibited from conducting independent investigations of the case, and such conduct may result in a mistrial for the defendant. Here, one juror when [sic] and measured the clearance on a pickup and the jury tried to re-enact the “accident” in the jury room. Jurors are not restricted to what they can do in the jury room and may use any means to explore and discuss the facts. The only real issue is whether the measuring of a, not D’s, pickup truck was independent investigation, plus it was done while the jury was in recess and should not have been discussing the case.

It is likely that the act by the juror was impermissible independent investigation, because he went outside the evidence presented in court. Accordingly, the case should be declared a mistrial unless it can be shown that it was harmless error.

Harmless Error

An error is harmless if, even without the error, there is no reasonable doubt that the case would have come out differently. Here, the independent investigation resulted in a demonstration that changed the minds of 5 jurors, which would have resulted in a hung jury. On the other hand, the jurors, in their deliberations may have eventually decided to act out the event and could have guessed at the clearance of the truck and come to the same conclusion. Although a jury is not allowed to testify regarding what happens in the jury room, unless 3 or more of the 5 would not have eventually changed their minds the error would be harmless. Because it is likely that the jury would have eventually acted out the incident, the error is likely harmless and the juror misconduct, though improper, will not have an effect on the outcome of the case.

38 Answer B to Question 4

1) The issue is whether the ct should have admitted T’s testimony that D complained of being “burned” by V & that he would “burn” V one day.

Relevance

Evidence is relevant if it tends to make any fact of consequence in a proceeding more or less probable. Here this evidence is relevant b[e]c[ause] it tends to make it more likely that D was the one who caused the fire that burned V & more likely that it was not an accident as D claims it was, but deliberate.

However, even evidence that is logically relevant may be excluded if the court finds that its probative value is substantially outweighed by its unfair prejudicial effect to the party ag[ains]t whom it is offered. Here, T’s st[ate]m[an]t is offered ag[ains]t D to show the fire wasn’t an accident. It’s [sic] probative value is great b[e]c[ause] it is a st[ate]m[en]t that D himself said he wanted to “burn” V & V was in fact literally burned in a fire D claims he started by accident. It is very prejudicial to D b[e]c[ause] it tends to completely negate D’s accident defense. However, it is not unfavorably prejudicial - it doesn’t increase the chances that the jury will convict just b[e]c[ause] D is a bad guy, rather it goes right to the central issue in the case of whether the fire was deliberate or accidental. Thus, the court shouldn’t exclude it on this basis.

Character Evidence in Crim Case

The prosecution cannot present evidence of bad character of a Î in a criminal case unless character is directly at issue or unless the Î initiates by putting a pertinent trait of his own or the victim’s character substantively at issue. In addition, the prosecution can’t use evidence of specific instances, only reputation or opinion evidence to establish character.

Here, this testimony about D is being offered in our attempted murder case where character is not an element of the crime. It is being offered in the prosecution’s case[-]in[-]chief, it is arguably character evidence b[e]c[ause] the statement about D casts D in a bad light b[e]c[ause] it makes him look like a vindictive person out to get V b[e]c[ause] he feels V has “burned” him by dating his estranged wife. In addition, it is evidence of a specific instance where D told T something, not evidence of D’s reputation for vindictiveness or violence or T’s opinion to that effect. Thus, it seems at first glance that it is barred by the rules against character evidence in criminal cases offered by the prosecution.

However, the prosecution is entitled to offer evidence of specific instances by theÎeven if it reflects negatively on the Î’s character if offered for a non-character if offered for a non- character purpose such as sharing motive or intent to commit a crime.

Here, D’s st[ate]m[en]t about feeling burned by V is relevant to show he had a motive to

39 harm V deliberately. This is especially true in light of the fact V was seeing D’s estranged wife b[e]c[ause] that gives meaning to what D meant when he said he felt burned. In addition, his saying he was going to burn V someday is evidence of intent to do the instant crime. Thus, T’s st[ate]m[en]t is admissible even if it is specific instance that reflects badly on D’s character offered in the state’s case[-]in[-]chief.

Personal Knowledge

Witnesses can only testify as to matters of which they have personal knowledge. Here T has personal knowledge of D’s st[ate]m[en]t because it was made directly to him.

Hearsay (HS)

HS is an out[-]of[-]court st[ate]m[en]t offered for the truth of the matter asserted therein.

D’s st[ate]m[en]t was made to T out of court before the burning incident took place. It is being offered to show that D wanted to burn V & had motive to do so. Thus, it is hearsay & should be excluded unless an exclusion or exception applies.

Party Admission

St[ate[m[en]ts by a party, offered against a party[,] are deemed non-HS under the criminal law & the FRE.

Here the statement is by D - the Î in this case & it is being offered ag[ains]t him. Thus it is non-HS and can come in.

St[ate]m[en]t ag[ains]t Interest

Statements by any person that are against their penal, property, or civil liability interest at the time made are admissible even if HS as long as the declarant is unavailable at trial.

Here D’s st[ate]m[en]t was arguably ag[ains]t his penal interest when made b[e]c[ause] it clearly showed he had intent to do harm to V. However, D is not unavailable b[e]c[ause] he has taken the stand in this case & has waived his privilege against self[-]incrimination w[ith] respect to his motive using an accident defense. Thus this exception doesn’t apply.

State of Mind of Declarant

St[ate]m[en]ts offered as direct evidence of a declarant’s state of mind are admissible HS. Here, this st[ate]m[en]t is being offered to show that D had a motive & intent to hurt V & the st[ate]m[en]t is precisely about D having had that state of mind. Thus it is admissible under this exception.

The court didn’t err in admitting T’s st[ate]m[en]t.

40 2) V’s Testimony Relevance

V’s st[ate]m[en]t about D laughing and running out while V was burning is relevant b[e]c[ause] it tends to show D wanted V to burn & again makes his accident defense less likely.

However, the probative value of this is low, the mere fact that D didn’t help V & may have laughed doesn’t necessarily mean D deliberately set the fire, although his general animosity towards V may have led him to laugh at V’s misfortune & leave instead of helping him. On the other hand, the potential the jury will convict D b[e]c[ause] he was coldhearted & callous & not just b[e]c[ause] he actually deliberately set the fire is great. Thus the court should use its discretion to exclude this testimony.

Character Evidence

This was evidence of a specific instance where D laughed & declined to help V, & it reflects very poorly on his character. It was offered by the prosecution in its case[-]in[-]chief. Thus it should be excluded as impermissible character evidence b[e]c[ause] it doesn’t seem relevant to any noncharacter purpose & will only inflame the jury against D b[e]c[ause] he acted in a morally reprehensible way by laughing & turning his back on V.

Hearsay

A nonverbal act can be a st[ate]m[en]t for the purposes of the HS rule if it is intended as an assertion. Here D laughed & walked out on V. This may arguably be intended as an assertion by D to V of his hatred for V & his delight that V was burning. Thus, it might be subject to exclusion as an out[-]of[-]court st[ate]m[en]t.

However, even if this argument were accepted[,] it would come in under the party admission exclusion b[e]c[ause] it was conduct by D, and is being offered ag[ains]t him.

3) Judicial Notice

Judicial notice of adjudicative facts (facts that have to be proven in a case) is proper when the facts noticed are either Î notorious facts commonly known to the public or Ï facts capable of ready & accurate verification.

Here the prosecution formally requested the court notice the fact that gasoline has certain chemical properties & has potential to cause serious injury or death when placed on the body & lit.

These facts will probably qualify under both categories. It is common knowledge that gasoline is highly flammable & even if lay people weren’t aware of all its properties these are scientific facts capable of ready verification. In addition, it is common knowledge that

41 person’s[sic] can be seriously hurt or killed if doused w/ gasoline that is then ignited. Moreover, that is again something capable of verification by expert testimony/scientific experiment. Thus, it was proper for the judge to notice these facts.

Effect of Notice

Since this was a criminal case, the effect of this notice was to relieve the prosecution of its burden of proving these facts & the jury could be told that the prosecution had met its burden but didn’t have to accept it as conclusively proven.

4) Question on X

Every party has an absolute right to cross any live witness - even if that witness is the Î in a criminal case.

Here D took the stand & testified, thus he is subject to cross[-]examination on matters relating to his testimony on direct, or else his direct must be stricken.

D testified that he didn’t have any lighter w/ him when he was in the garage. Thus it is proper for the prosecution to question him about the lighter on X.

Impeachment

Any witness can be impeached w/a prior inconsistent statement that is materially different from his testimony at trial.

Here D, [sic] testified that he didn’t have any lighter when at the garage. The prosecutor is asking him about the lighter found at the scene that has his initials on it which clearly states that the lighter was in fact his. Since this is materially different from what D said at trial, the prosecution is entitled to use it to impeach D & discredit his testimony.

In addition, b[e]c[ause] the prior inconsistent st[ate]m[en]t of D’s initials written on his lighter qualifies as a party admission, (st[ate]m[en]t by D, offered ag[ains]t him), the prosecution can use it as substantive evidence that the lighter did in fact belong to D.

5) The issue is whether the jury’s conduct during deliberation was proper.

Jurors are not permitted to conduct independent investigations of the facts. Rather they are supposed to look at the facts presented by the parties & to apply the law as instructed by the judge.

Here, the jurors took their own initiative to go out & measure a truck that wasn’t even the truck involved in the accident, & to reenact the accident themselves in the jury room. This was prohibited conduct, & in a criminal case could be grounds for mistrial if it had a

42 ESSAY QUESTIONS AND SELECTED ANSWERS

JULY 2005 CALIFORNIA BAR EXAMINATION

This publication contains the six essay questions from the July 2005 California Bar Examination and two selected answers to each question.

The answers received good grades and were written by applicants who passed the examination. The answers were prepared by their authors, and were transcribed as submitted, except that minor corrections in spelling and punctuation were made for ease in reading. The answers are reproduced here with the consent of their authors.

Question Number Contents Page

1. Community Property 1

2. Contracts/Real Property 8

3. Corporations/Professional Responsibility 20

4. Evidence 29

5. Remedies 45

6. Professional Responsibility 55

i Question 4

Dan was charged with arson. The prosecution attempted to prove that he burned down his failing business to get the insurance proceeds. It is uncontested that the fire was started with gasoline. At a jury trial, the following occurred:

The prosecution called Neighbor, who testified that fifteen minutes after the fire broke out, he saw a blue Corvette speed from the scene.

The prosecution next called Detective Pry. Pry testified that he checked Motor Vehicle Department records and found that a blue Corvette was registered to Dan. Pry also testified that he observed a blue Corvette in the driveway of Dan’s house.

The prosecution then called Scribe, the bookkeeper for Dan’s business. Scribe testified that, two months before the fire, Dan told Scribe to record some phony accounts receivable to increase his chances of obtaining a loan from Bank. Scribe then testified that she created and recorded an account receivable from a fictitious entity in the amount of $250,000, but that Bank denied the loan anyway. Scribe further testified that, two days after the fire, Dan again told her to create some phony accounts receivable, but that she refused to do so.

The prosecution called Jan, the night janitor at Dan’s business, to testify that the evening before the fire, as Jan was walking past Dan’s office, Jan heard a male voice say, “Gasoline is the best fire starter.” Jan knew Dan’s voice, but because the office door was closed and the voice muffled, Jan could not testify that the voice was Dan’s.

Assume that, in each instance, all appropriate objections were made.

Should the court have admitted:

1. Detective Pry’s testimony? Discuss. 2. Scribe’s testimony? Discuss. 3. Jan’s testimony? Discuss.

29 Answer A to Question 4

4)

State v. Dan

Admissibility of Detective Pry’s Testimony

Logical Relevance

To be admissible, evidence must first be relevant. A piece of evidence is logically relevant if it has any tendency to make a fact of consequence in the case more or less likely to be true than it would be without the evidence.

Detective’s Pry’s testimony regarding what he learned from checking the DMV records is admissible because it tends to make it more likely that Dan was the one who committed the arson. Neighbor has already testified that he saw a blue [C]orvette speeding away from the scene of the arson. It is likely that the [C]orvette was driven by the one who had committed the crime. Therefore, if Dan also drove a blue [C]orvette, it would tend to make it more likely that Dan is guilty of the crime.

Detective Pry’s testimony regarding the blue [C]orvette that he observed on Dan’s driveway is also admissible. Since a witness saw a blue [C]orvette speeding away from the scene, the fact that Dan owns and possesses a blue [C]orvette makes it more likely that he committed the crime. The officer’s testimony regarding seeing the car in Dan’s driveway is also relevant because it tends to support the theory that Dan still possessed the car and had not sold it to someone else before the crime was committed.

Therefore, Detective Fry’s testimony is logically relevant.

Legal Relevance

To be admissible, evidence must also be legally relevant. Evidence may be excluded if its probative value is substantially outweighed by the risk of undue prejudice to the jury.

Here, the evidence is legally relevant. The evidence has some probative value in making it more likely that Dan was the one who committed the arson, and there is little risk of undue prejudice. Evidence is only prejudicial if it is likely to lead the jury to draw improper conclusions about he defendant’s guilt or innocence. The fact that Dan possessed a blue [C]orvette like that driven from the crime scene may hurt Dan’s case, but it will be because the jury drew the reasonable conclusion that Dan may have been driving the car scene [sic] by neighbor, not because of any prejudicial effect.

Thus, the evidence is legally relevant.

30 Personal Knowledge

For the evidence to be admissible, Detective Pry must be competent to testify regarding it. A witness is competent if he has personal knowledge about the facts that he is testifying to.

In this case, [P]ry is competent to testify to the fact that he saw a blue [C]orvette in Dan’s driveway, because he observed that himself and had personal knowledge of it. However, Pry’s testimony regarding the DMV records will be inadmissible because Pry’s only knowledge that the [C]orvette was registered to Dan came from the DMV records, and the DMV records have not been produced at trial, under the best evidence rule described below.

Best Evidence Rule

Under the best evidence rule, if a witness’s sole knowledge of facts comes from a written document, then the fact must be proved from the written document unless the absence of the document is explained and excused.

On these facts, Pry’s only knowledge of the fact that a blue [C]orvette was registered to Dan came from reading the DMV records. Therefore, the best evidence rule applies and Dan’s ownership of the car must be proved with the DMV records themselves, rather than by Detective Pry’s testimony regarding the contents of the records.

For this reason, Detective Pry’s testimony regarding the contents of the DMV records should not have been admitted into evidence. Instead, the prosecution should have proved Dan’s ownership of the car by introducing the DMV records themselves into evidence.

Hearsay

Another objection that Dan could make to the admission of the evidence is hearsay. Hearsay is an out[-]of[-]court statement offered into evidence to prove the truth of the matter asserted. The DMV records are hearsay because they are the out[-]of[-]court statements of DMV employees who prepared the report and it is being offered to prove the truth of the matter asserted – namely, that Dan was the registered owner of a blue [C]orvette.

Therefore, the evidence will be inadmissible unless a hearsay exception or exemption applies.

Business Records Hearsay Exception

Under the business records hearsay exception, the records of a business may be admitted into evidence if they were regularly prepared in the ordinary course of business by business employees with a duty to the business to maintain accurate records. Business is defined

31 to not only include for-profit businesses but also nonprofits and government agencies.

The DMV records could be admitted into evidence under the business records hearsay exception. As part of its business of regulating motor vehicles, the DMV regularly maintains records of the cars that are registered as owned by a certain person. These reports are prepared by DMV employees who have a duty as part of their job to maintain accurate records. Therefore, the statements in the DMV report are admissible under the hearsay exception for business records.

Government Records Hearsay Exception

The contents of the DMV records could also be admitted under the hearsay exception for government records. For this hearsay exception to apply, the records must have been maintained by a government agency and must be: (1) a record of the activities of that agency, (2) a report prepared in accordance with a duty imposed by law, or (3) a report of an investigation duly authorized by law. Government records of the police investigation regarding a crime are not admissible against the defendant in a criminal trial, but other government records are admissible.

In this case, the DMV records would qualify as records of the activities of the agency. When a person buys a car, they go to the DMV and register as the owner of the car, and the DMV makes the appropriate changes in its records. Therefore, it would qualify as a record of the activities of the DMV. It would also qualify as a report prepared in accordance with a duty imposed by law because the DMV is likely under a duty imposed by the state legislature to maintain vehicle ownership records.

Therefore, the contents of the DMV report would also be admissible under the hearsay exception for government records.

Conclusion

Detective Pry’s testimony regarding observing a blue [C]orvette in Dan’s driveway is admissible because it tends to make it more likely that Dan committed the arson and Pry had personal knowledge.

However, Pry’s testimony regarding the contents of the DMV records should have been excluded because the best evidence rule required that the records themselves be produced rather than allowing someone else to testify to their contents. The prosecution should have instead introduced the DMV records themselves into evidence. The records would have been admissible under the hearsay exceptions for business records and government records and could then have been considered by the jury to help establish Dan’s guilt.

Admissibility of Scribe’s Testimony

Logical Relevance

32 Scribe’s testimony is logically relevant because it tends to establish motive. If the jury believes Scribe’s testimony, then it will establish that Dan’s business was failing and that his previous desperate attempts to obtain financing through fraudulently obtained bank loans had failed. This would make it more likely than it would otherwise be that Dan would turn to other illegal measures, such as committing arson, to escape his precarious economic situation.

So the evidence is logically relevant.

Legal Relevance

Although Scribe’s testimony is logically relevant, it could still be excluded at the discretion of the judge if its probative value was substantially outweighed by the risk of improper prejudice.

Dan would argue that this testimony is highly prejudicial and should be excluded. The testimony involves prior bad acts of Dan – specifically by inducing Scribe commit [sic] fraud in connection with a bank loan by falsifying the accounts receivable of the business and trying to do so a second time. Thus, Dan would argue, the evidence would be highly prejudicial because it would lead the jury to draw the improper inference that because Dan had done other bad things in the past, he was just a bad guy and is likely guilty of this crime as well.

However, the prosecution could successfully counter by pointing out that while the evidence does present some risk of undue prejudice, it is also quite probative of the issue of Dan’s guilt. Scribe’s testimony established that Dan was desperate for money because of his failing business and had resorted to illegal conduct in the past to try to get money. This established motive and makes it much more likely than would otherwise be the case that Dan is the one who committed this arson.

Although the evidence does present some risk of undue prejudice, it does not substantially outweigh the high probative value of the evidence. Therefore, Scribe’s testimony is legally relevant and should not be excluded on this basis.

Character Evidence

Another issue presented by Scribe’s testimony is character evidence. Character evidence – evidence of prior bad acts of the accused offered to prove the bad character of the defendant to show that he acted in conformity with the bad character – is generally inadmissible in a criminal case. However, character evidence may still be admitted if it is offered for some other purpose, such as to show motive, intent, modus operandi, or common plan or scheme.

Here, the evidence of Dan’s prior activities in connection with falsifying the company’s records is admissible for the non-character purpose of establishing motive. The evidence

33 is not being offered to prove that Dan is a bad guy in general. Rather, it is being offered for the specific purpose of showing that Dan had a strong motive to burn down his business because he was in financial trouble and his other efforts to obtain funding had failed.

Therefore, the judge should admit Scribe’s testimony. However, the judge should also issue a limiting instruction informing the jury that they may only consider Dan’s prior bad acts in establishing motive and may not infer from them that he had a bad character and so is likely to be guilty for that reason.

Personal Knowledge

Scribe is competent to testify regarding what he heard and did because he had personal knowledge of it. Scribe was there when Dan told him to falsify the books and did so himself, so Dan has personal knowledge.

Thus, this requirement for admissibility is satisfied.

Hearsay

A final issue is whether Scribe’s testimony is inadmissible hearsay. Hearsay is out[-]of [- ]court statement offered to prove the truth of the matter asserted, and is normally inadmissible unless an exception to the hearsay rule applies or the statement is exempted from the definition of hearsay under the Federal Rules of Evidence (FRE).

The prosecution would argue that Scribe’s testimony regarding Dan’s out[-]of[-]court statements and Scribe’s out[-]of[-]court statement is not hearsay at all, because it is not being offered for its truth. A statement is not considered hearsay if it is being offered for some purpose other than its truth, such as to prove the mind of the speaker and the listener. Under this argument, Dan’s statements to Scribe are not being offered to prove that the bank loan was really rejected, but to show that Dan believed that the business was desperate for money and was willing to do anything to get funds. Similarly, Scribe’s out [- ]of[-]court statement refusing to falsify the books a second time is being offered for the non- hearsay purpose of proving the listener’s state of mind – that Dan knew his fraud scheme would not work and thus was likely to try some other way to get money.

Because the prosecution has a strong argument that Scribe’s testimony is not hearsay at all, the testimony should be admitted into evidence.

Hearsay Exemption for Admissions by a Party Opponent

Furthermore, with regard to Dan’s statements to Scribe, the statements will be admissible for their truth because the hearsay exemption for admissions by a party opponent applies.

34 Under this hearsay exemption, the statements of an adverse party in a proceeding are not considered hearsay, regardless of when they were made.

Thus, in this prosecution the prosecutor is offering the evidence against Dan, so Dan is an adverse party. Therefore, Dan’s statement are [sic] not considered hearsay and are admissible for their truth.

Conclusion

In summary, Scribe’s testimony should be admitted. The evidence is relevant to proving Dan’s motive to commit the crime, a non-character purpose. And the conversations between Dan and Scribe are admissible because they are being offered for a purpose other than their truth and the hearsay exemption for party admissions applies.

Admissibility of Jan’s Testimony

Logical Relevance

Jan’s testimony is relevant because it tends to make it more likely that Dan committed the arson. As Jan walked by Dan’s office she heard someone say “gasoline is the best fire starter”. Because the statement was made in Dan’s office, it was likely made either to Dan or in Dan’s presence. Therefore, it establishes that Dan had knowledge regarding the means to commit the crime, which makes it more likely that he did in fact commit the arson. It also makes it more likely that Dan would have chosen gasoline if he were to commit arson, which matches up with the fact that the fire was indeed started with gasoline.

Of course, the conversation could have been perfectly innocent. Dan could have been seeking or obtaining advice on the best way to BBQ, or he could have not even been there are [sic] the time. But to be relevant, evidence must only have some tendency to make a fact of consequence more or less likely to be true. Because the evidence has some tendency to make it more likely that Dan committed the arson, it is logically relevant.

Legal Relevance

The evidence is also legally relevant. As discussed above, even relevant evidence can be excluded if its probative value is substantially outweighed by its prejudicial effect.

Here, the evidence is legally relevant because it has substantial probative value and poses little risk of undue prejudice. The fact that the defendant may have given or received advice on the best way to start a fire the night before the defendant’s business burned down, coupled with the motive established by Scribe’s testimony, is strong evidence of guilt. In contrast, there is little risk of undue prejudice. The defense will be able to argue that Dan was not present at the time the statement was made or that it was innocuous when they present their case.

35 Therefore, the evidence is legally relevant.

Personal Knowledge

Jan is competent to testify to what she heard because she had personal knowledge of it. She was there that night and heard the statement made.

Authentication

To be admissible, documentary evidence must be authenticated as being what it purports to be. For a voice recording by someone, this would normally mean that a witness who knows the person’s voice must testify that the voice on the tape is the voice of the person. Dan would argue that Jan’s testimony is inadmissible because Jan could not testify that the voice was Dan’s.

However, the authentication requirement will not apply to bar admission of this evidence. First, the evidence is testimonial, rather than documentary, so authentication requirements would not apply. More importantly, it is irrelevant whether Dan was the one who made the statement. The statement could have been made by someone else in Dan’s presence, for example if Dan sought the advice of someone in determining what the most effective way would be to commit the arson. Therefore, the statement is relevant even if it was not made by Dan and for this reason need not be authenticated as Dan’s.

The defense can argue that Dan was not present when the statement was made or that it was innocuous, but deciding those issues will be up to the jury, not the judge.

Hearsay

A final objection Dan might make to admission of Jan’s testimony is hearsay. Hearsay is an out[-]of[-]court statement offered to prove the truth of the matter asserted. Dan would argue that Jan’s testimony is hearsay because she is testifying about what she heard someone say in Dan’s office.

However, the hearsay objection will be rejected here. A statement is only hearsay if it is offered for its truth. An out[-]of[-]court statement is still admissible for other purposes. Here, it is irrelevant whether the statement is true. Whether gasoline is in fact the best fire starter has no bearing on the case. The significance of the statement is to establish either the speaker’s or the listener’s state of mind, which are both permissible non-hearsay purposes. If Dan made the statement, then it tends to establish that he had knowledge about how to commit the crime, which would help show his guilty [sic]. Similarly, if someone else made the statement to Dan, it would be relevant to show that Dan heard the statement and thus had obtained advice on the best way to start a fire, which again would be relevant to guilty [sic].

Although Dan may not have been in the office at the time, the fact that the statement was

36 made in Dan’s office, a place where people would not normally be without Dan being there as well, justified the judge in concluding that there was sufficient evidence to find that Dan either made the statement or was present when it was made.

Conclusion

Jan’s testimony should be admitted into evidence because it is relevant to establish Dan’s guilt, Jan had personal knowledge of the statement, and it is being offered for a non- hearsay purpose.

Answer B to Question 4

4)

37 Detective Pry’s Testimony about DMC [R]ecords

Logical/Legal Relevance

Relevant evidence is generally admissible. In order to be relevant the evidence must have any tendency to make a fact more or less likely than that fact would be without the evidence proffered. The prosecution is offering this evidence to prove that the car seen speeding away from the scene of the arson was owned by Dan (D). This evidence is logically relevant as it tends to prove identity of the arsonist.

Some logically relevant evidence will still be excluded if there are public policy reasons for the exclusion of that evidence. If the probative value of relevant evidence is substantially outweighed by the prejudicial nature of the evidence then the evidence will be excluded. D’s attorney would argue that lots of people own blue [C]orvettes and thus using the [C]orvette to identify D as the guilty party is prejudicial. D’s attorney would lose however because ownership of a car seen speeding away from the scene of a crime is not prejudicial and any possible prejudice resulting from the inference that D was driving the [C]orvette as it speed [sic] away does not substantially outweigh the probative value that this evidence possesses as far as identifying the arsonist.

Witness Competency

A witness is competent to testify if the witness has personal knowledge and is capable of understanding the oath or affirmation required of all witnesses.

Pry would be a competent witness because he read the dmv [sic] report and thus has personal knowledge of its contents.

Best Evidence

When a witnesses [sic] sole source of knowledge is from the contents of a document, and the witnesses [sic] testimony is being elicited in order to establish the contents of that document as true the best evidence rule requires the profferor of that evidence to either produce the document or explain why the document was not produced before allowing the witness to testify as to the contents of that document.

The defense’s objection to Pry’s testimony on the contents of the DMV printout should have been upheld as officer Pry’s sole source of knowledge regarding D’s ownership of a blue [C]orvette was from the DMV printout. Pry did not explain why he was not able to produce the dmv[sic] record. Without the DMV record or a reasonable explanation concerning why it was missing Pry’s testimony should have been excluded.

Hearsay

Hearsay is an out-of-court statement offered for the truth of the matter asserted. Pry’s

38 testimony about the DMV printouts[sic] contents would also be hearsay because it is a statement made by the employee transcribing data into the DMV database that is being offered to prove that D owned a blue [C]orvette. Because this statement was hearsay it should have been excluded unless on of [sic] the exceptions or exemptions from the hearsay rule applied.

Exemptions/Exceptions

Official Document

Official certified documents from public agencies charged with complying [sic] the information contained in the document are exempt from the hearsay rule. Because the prosecution failed to produce a certified record from DMV this exception to the hearsay rule would not have been available.

Business Record Exception

A record that is made in the ordinary course of a business by an employee with a duty to accurately report such information can be admitted in lieu of the employee’s testimony. Since there was no dmv[sic] record being offered this exception would not have applied.

Presumption that owner was driver of a vehicle

A presumption can be raised that the driver of a car was the owner of the car. However in criminal trials the burden of proof is on the prosecution to prove each element of a crime and the identity of the person committing the crime beyond a reasonable doubt and thus the prosecution would not have been able to use the testimony regarding dan’s[sic] ownership of a blue [C]orvette to raise a presumption that dan[sic] was driving the [C]orvette on the night of the arson.

Because of the best evidence and hearsay problems, Pry’s testimony about the DMV printout should have been excluded.

Detective Pry’s Testimony about Corvette in Driveway

Logical/Legal Relevance This evidence is logically relevant because it makes it more likely than not than D owned a blue [C]orvette which was seen sp[e]eding away from the scene of the arson. This evidence would not be excluded due to legal relevance for the same reason the DMV printout testimony would not have been excluded for legal relevance reasons.

Witness Competency

The officer is competent to testify about seeing a blue [C]orvette in D’s driveway because the officer has personal knowledge regarding what the officer saw in D’s driveway.

39 Presumption that D was the driver of the blue [C]orvette on the night of the arson

The prosecution would still not be able to use the driver presumption because this is a criminal case.

Scribe’s Testimony re: phony accounts receivable for bank loan

Logical/Legal Relevance

This evidence would be logically relevant to show that D needed money because he falsified account records to receive a bank loan which was denied. The defense would argue that this testimony is highly prejudicial and that its prejudicial effect outweighs its probative value substantially because the jury is likely to convict D for arson based on the fact that his [sic] is a dishonest person and not based on whether he actually committed the arson. While this evidence is highly prejudicial, the court was right to admit it as it goes to the d’s[sic] motive in starting the fire.

Witness Competency

Scribe would be a competent witness because he or she had personal knowledge about D’s request to make false accounts receivable.

Hearsay

This testimony would be hearsay because Scribe is testifying about a statement made by D out of court to prove the[sic] D had Scribe create false records in order to get a loan from the bank. The prosecution would argue that this is not hearsay because the evidence is not offered to prove that D tried to get a loan by false pretenses but that he had a motive to burn down his building for the insurance proceeds because he was denied a loan and thus was in need of money.

The court properly admitted this as non-hearsay if it allowed it in for the limited purpose of showing that D had a motive to burn down the building to collect insurance proceeds.

Admission of a party opponent made by an agent

A statement made by a party offered against the party by the opposing party that is adverse to the party’s interest is admissible as non-hearsay. The statement did not have to be against the party’s interest at the time that it was made. The prosecution would argue that D’s request that Scribe falsify accounts receivable is a party admission exempt from the hearsay rule because it is a statement made by D that is now relevant to his culpability for the crime of arson. The statement would be admitted under this exemption to the hearsay rule because D made the statement and it is being offered by the opposition against D.

Present sense impression

40 S’s testimony would not be excepted from the hearsay rule under the exception for a present sense impression as D’s statement to falsify records was not made contemporaneous to d[sic] observing the falsification of the records.

Excited Utterance

It would also not qualify as an excited utterance because there is no evidence that D experienced a traumatic or exciting event around the time that his instructions were made.

Present Intent to engage in future conduct

Since D was instructing S to destroy the records it is unlikely that the prosecution could have this statement admitted as a present expression of intent to engage in future conduct to prove that the future conduct was engaged. D did not make a statement concerning conduct that he was about to engage in or planned to engage in in [sic] the future.

Double Hearsay

S’s testimony about transcribing false accounts receivable would be double hearsay because S is testifying to an out[-]of[-]court statement that he made in response to a request that his boss made to prove that S engaged in the conduct alleged by the hearsay statement.

Vicarious Admission

S’s statement would be admitted as a vicarious admission so long as transcribing records was [sic] part of the duties that S performed. As D[‘]s agent S’ testimony would be vicariously attributed to D.

Character evidence

Character evidence is not allowed in a criminal trial by the prosecution to show that the defendant acted in conformity with his character unless and until the defendant offers evidence of his good character. Character evidence is however admissible to show motive, intent, a common plan or scheme, identity or opportunity.

D would argue that this evidence was offered to show that D is of bad character and likely to commit fraud and thus it should be excluded as impermissible character evidence.

The prosecution would argue that this evidence is being offered to show that D had the motive to commit an arson in order to collect the insuranc[e] proceeds on his failing business. Because the falsified accounts receivable are not required to prove that D did not get a loan from the bank which is the evidence that really tends to show that D had a motive to burn down his failing business for insurance proceeds the court should have excluded the portion of Scribe’s testimony concerning the falsified records as impermissible

41 character evidence.

Scribe’s Testimony Re: phony accounts receivable two days after fire

Logical/Legal Relevance

This testimony is not logically relevant because Scribe did not offer any reason related to the arson for falsifying the accounts receivable. While the prosecution may argue that D was falsify[sic] the records to get a bigger insurance payoff, Scribe’s testimony does not suggest that this is the case. Even if the court did find the evidence to be logically relevant for showing that D was attempting to increase the amount of payoff from the insurance company, this testimony should have been excluded because its prejudicial value substantially outweighs its probative value. Without some testimony concerning why D asked Scribe to falsify the accounts receivable after the fire this testimony tends to suggest to the jury that it should convict D for being a dishonest guy generally instead of for committing the specific crime charged.

Witness Competency

Scribe would be competent because he or she had personal knowledge of what was said.

Hearsay

This testimony would be hearsay as was the prior testimony regarding false accounting records if it was admitted to show the truth of the statement – that D wanted to falsify accounts receivable. The prosecution could still argue that it was being offered to show motive which would be for a reason unrelated to the truth of the statement.

Admission

This testimony would also be an admission of D because it was made by D and is being offered against him and thus it is exempt from the hearsay rule.

The court should not have admitted this evidence because of its potential lack of logical relevance, it [sic] highly prejudicial nature in light of its relatively low probative value.

This testimony would also not fit under the exceptions to the hearsay rule for present sense impressions, excited utterances, or a present statement of intent to engage in future conduct for the same reasons the first statement regarding the falsification of accounting records would not fit under these exceptions.

Jan’s Testimony re: “Gasoline is the best fire starter”

Legal/Logical Relevance

42 This evidence is legally relevant because it tends to show that D knew gasoline was the best fire starter and since it is undisputed that gas was used to start the fire at the business it would tend to show that D committed the arson.

Witness Competency

J is famil[i]ar with D’s voice and he heard the statement[;] thus he would be competent because he has personal knowledge of the statement and is potentially capable of authenticating the identity of the speaker, a problem which will be dealt with more extensively below.

Hearsay

[T]his statement would not be hearsay because the purpose for its admission is not to prove that gasoline is the best fire starter. The prosecution wants this evidence in to show that D had knowledge that gasoline starts fires since gasoline was used to start this fire. Even if the statement was found to be offered for its truth a hearsay exemption would apply.

Party Admission

D is a party and the statement is being offered against him and thus so long as he can be identified as the speaker this statement would be admissible as a party admission.

Authentication of Voice

When the identify of a speaker is in issue because the speaker was not visible to the person hearing the speech the voice must be authenticated. A voice may be authenticated by the person who heard the voice so long as that person is familiar with the voice. Even if the hearer is not necessarily familiar with the voice of the speaker other facts can be admitted to establish the speaker’s identity.

J is familiar with D’s voice[,] however J is unable to authenticate the speaker’s identity as that of D because the door was closed and the voice was muffled. However the prosecution would argue that there are enough circumstantial factors available that the jury should be allowed to decide whether or not the voice was D’s. Such evidence exists from the fact that J was passing D’s office and that the voice was a male voice coming from D’s office. This should be sufficient to allow this testimony to go to the jury because J’s testimony is enough to allow the jury to determine whether D was in his office.

The judge properly admitted J’s testimony as either non-hearsay because it was not admitted to prove the truth of the matter asserted or as a party admission.

43 ESSAY QUESTIONS AND SELECTED ANSWERS

FEBRUARY 2007 CALIFORNIA BAR EXAMINATION

This publication contains the six essay questions from the February 2007 California Bar Examination and two selected answers to each question.

The answers selected for publication received good grades and were written by applicants who passed the examination. These answers were produced as submitted, except that minor corrections in spelling and punctuation were made during transcription for ease in reading. The answers are reproduced here with the consent of their authors.

Question Number Contents Page

1. Real Property 1

2. Corporations/Professional Responsibility 9

3. Criminal Law and Procedure 18

4. Wills and Succession 29

5. Constitutional Law 37

6. Evidence 46

i Question 6

Officer Will, a police officer, stopped Calvin, who was driving a rental car at five miles an hour over the speed limit. Calvin gave legally valid consent to search the car. Officer Will discovered a substantial quantity of cocaine in the console between the two front seats and arrested Calvin. After being given and waiving his Miranda rights, Calvin explained that he was driving the car for his friend, Donna. He said that Donna was going to meet him at a particular destination to collect her cocaine, which belonged to her. Hoping to obtain a favorable plea bargain, Calvin offered to cooperate with the police. The police then arranged for Calvin to deliver the cocaine. When Donna met Calvin at the destination, she got into the car with Calvin. She was then arrested. Each was charged with and tried separately for distribution of cocaine and conspiracy to distribute cocaine.

Donna’s trial began while Calvin’s case was still pending.

At Donna’s trial, the following occurred: (1) The prosecutor called Officer Will, who testified to Calvin’s statements after his arrest concerning Donna’s role in the transaction. (2) The prosecutor then called Ned, an experienced detective assigned to the Narcotics Bureau, who testified that high level drug dealers customarily use others to transport their drugs for them.

In the defense case, Donna testified that she was not a drug dealer and that she knew nothing about the cocaine. She stated that she was merely meeting Calvin because he was an old friend who had called to say he was coming to town and would like to see her. (3) Donna further testified that when she was in the car with Calvin, she found a receipt for the rental car, which showed that Calvin had rented it six months prior to his arrest. She offered a copy of the receipt into evidence. The court admitted the document in evidence. (4) On cross-examination, the prosecutor asked Donna whether she had lied on her income tax returns.

The prosecutor had no evidence that Donna had lied on her income tax returns, but believed that it was likely on the basis that drug dealers do not generally report their income. Donna denied lying on her income tax returns.

Assuming that, in each instance, all the appropriate objections were made, should the evidence in numbers 1, 2, and 3 have been admitted, and should the cross-examination in 4 have been allowed? Discuss.

Answer A to Question 6

6)

QUESTION 6

(1) Should Officer Will’s Testimony Have Been Admitted?

Relevance

In order for Officer Will’s testimony to be permitted it must be relevant. Federal Rules of Evidence (FRE) 401 provides that relevant evidence is evidence having any tendency to make a fact of consequence to the determination of the action more or less likely to be true than without the evidence. Officer Will’s testimony was relevant because Calvin’s statements, that he was driving the car for his friend Donna and that she was going to meet him at a particular destination to collect her cocaine, had a tendency to make the fact of consequence that he was a coconspirator with Donna for distribution of cocaine more likely. Therefore, the evidence was relevant.

FRE 403

Although relevant, Donna may argue that it should have been excluded on FRE 403 grounds. FRE 403 provides that relevant evidence should be excluded if its probative value is substantially outweighed by the danger of unfair prejudice, waste of time, confusing/misleading the jury, or cumulative evidence. Donna will argue that there are no reliable ways of showing that this statement was true and therefore the probative value of the evidence is substantially outweighed by risk of unfair prejudice because a jury will hear the statement and automatically want to convict her. However, witnesses or convicts are often allowed to testify and any evidence against the truthfulness of the statement would be go to [sic] weight of Officer’s testimony.

Hearsay

Donna will argue that the testimony was impermissible hearsay. Hearsay is an out-of-court statement made by the declarant, while not present in court, offered for the truth of the matter asserted. A statement is defined as an oral/written assertion or assertive conduct. Officer Will is testifying about Calvin’s comments to him when he was arrested. The statements that he was driving the car for his friend Donna and that he was going to meet her at a particular destination to collect her cocaine are all offered to prove that indeed the car was being driven for Donna and he was meeting her because it was her cocaine. So it is an out-of-court statement offered for the truth of the matter asserted because it is an oral assertion by Calvin out of court. Therefore, it is hearsay and is not admissible unless it comes in under an exception.

Coconspirator Admission

Prosecutor would argue that the statement was a valid party-opponent admission. Party-

47 opponent admissions are categorically non-hearsay and are exempted from the hearsay rule’s exclusionary effect. A party-opponent admission can be done by a statement by a coconspirator during the course of the conspiracy and in furtherance of the conspiracy. Donna would argue that Calvin had already been arrested and therefore this was not made in furtherance of the conspiracy. Therefore, Calvin’s statements could not come in under this exemption, 801(d)(2).

Statement Against Interest

The prosecutor could also have argued that this was a statement against interest and hence is an exception to the hearsay rule. Statement against interest comes in under FRE 804, which requires unavailability of the witness, which can include witnesses not testifying because of self-incrimination. Here Calvin would not be testifying because of such self-incrimination and he is not present at the trial, so therefore he is considered unavailable. Statement against interest excepts statements from the hearsay rule that are so contrary to declarant’s criminal liability that a reasonable person would not have made such a statement unless it were true. The prosecutor would argue that a reasonable person would not admit his involvement in the transportation of cocaine unless it were true and therefore this falls within the exception since Calvin would be subject to criminal liability. Therefore, the evidence could potentially be admitted under this exception.

Confrontation Clause

Donna would argue that regardless the statement should not be introduced because it violates the confrontation clause. The 6th Amendment Confrontation Clause provides that an accused has the right to confront his accusers. For this reason, hearsay testimony used against an accused is often not permitted. The Supreme Court has determined that testimonial evidence can in no circumstance be used against an accused without the right to cross-examination at trial or a prior proceeding with the same motive to develop such testimony. Testimonial evidence are all [sic] statements made that a reasonable person would believe would be used by the prosecution against another at trial. Usually there requires at least statements to the police. So when Calvin spoke to the police officer and made statements about the culpability of Donna, he was giving testimonial evidence since it could have been foreseen by telling the police it could be used against her. Furthermore, he did so in hoping to get a plea bargain and hence it shows that it was testimonial. Therefore, since he cannot testify because of self-incrimination/presence in court and Donna had no opportunity at any time to cross-examine him, the police officer’s statement regarding Calvin’s statements should be excluded.

(2) Should Ned’s Testimony Have Been Admitted?

Relevance

It must be determined whether Ned’s testimony was relevant to the case. Federal Rules of Evidence (FRE) 401 provides that relevant evidence is evidence having any tendency to make a fact of consequence to the determination of the action more or less likely to be true than without the evidence. Ned’s testimony was relevant because it had a tendency to show that Donna was

48 potentially a high level drug dealer because she had someone else transport the cocaine and hence it is more likely that she should be convicted of distributing cocaine. Therefore, it should be allowed in as relevant.

Expert Testimony

Although relevant, the testimony must be valid expert testimony in order to be allowed in. Pursuant to FRE 104, a court must make the preliminary fact determination of whether an expert is qualified to give expert testimony. Under 104, evidence is not relevant if dependent on a conditional fact unless that condition is found to exist. A judge need only find sufficient evidence to show existence of the condition to allow the question to go to the jury for credibility and weight. Here the judge would consider the Daubert factors that were incorporated into the FRE on expert testimony in order to determine whether it should be allowed in.

The factors require that the expert testimony be based on the knowledge, experience, and training of the expert, be beyond the normal experience of an average lay juror, be helpful to the determination of the action, and based on proven and reliable data and methods, and be an application of such methods and data to the underlying facts of the case. Here Ned was an experienced detective in the Narcotics Bureau and hence had the knowledge, experience and training. His testimony was beyond the normal lay juror because it involved high level drug dealers’ actions. Furthermore, it was relevant and helped to determine what Donna was guilty of. Finally, it was based on reliable data of customary experience in the field that high level drug dealers customarily use others to distribute the drugs. Therefore, the testimony should be allowed in.

(3) Should Donna’s Testimony and Receipt Be Admitted?

Relevance

Federal Rules of Evidence (FRE) 401 provides that relevant evidence is evidence having any tendency to make a fact of consequence to the determination of the action more or less likely to be true than without the evidence. Donna will argue that the evidence is relevant because it makes the existence of the fact that Calvin had control over the cocaine more likely here and hence she was not involved. Therefore, it will be allowed in as relevant, unless there are other problems.

Best Evidence

Prosecutor will argue that this is not the best evidence. The Best Evidence Rule requires that one cannot testify to the contents of a writing unless the writing is presented. However, a copy is permissible. Therefore, since Donna brought a copy of the receipt that she found in the car, it should be allowed in.

Hearsay

Prosecutor will object on grounds of hearsay. Hearsay is an out-of-court statement made by the declarant, while not present in court, offered for the truth of the matter asserted. A statement is defined as an oral/written assertion or assertive conduct. Therefore since the document asserts that Calvin rented the car, it is offered for the truth of the matter and must come

49 in under an exception.

Business Records

Donna will argue this is a business record and should be admitted. Business records are records or documents made during the normal course of business with guarantees of trustworthiness. There needs to be some type of testimony demonstrating that this was in the normal course of business, and hence Donna would have needed some type of custodian or the person who entered the information testify to those facts. Therefore, the evidence will be excluded.

(4) Should the Cross-Examination Have Been Allowed?

Relevance

Prosecutor would argue that the question was relevant as to whether Donna was telling the truth. Federal Rules of Evidence (FRE) 401 provides that relevant evidence is evidence having any tendency to make a fact of consequence to the determination of the action more or less likely to be true than without the evidence. If Donna had lied in the past on her income tax then it would be more likely that she would lie at trial because she is dishonest. Therefore, it is relevant to the case.

Character Evidence of a Witness

Donna would initially argue that this is improper character evidence. Character evidence is evidence of a trait or character offered to prove action in conformity therewith. Character evidence is not allowed unless it falls under one of the exceptions to character evidence. Here this falls under the exception to character of a witness. Therefore, it is governed by 607 and 608 of the FRE.

FRE 607 allows an opposing party to generally impeach to show bias or lack of credibility of a witness. FRE 608 allows a party to use character evidence for the purpose of impeachment. However, if one wants to impeach by specific instances of conduct one can only do so by inquiring on cross-examination and not through extrinsic evidence.

Furthermore, it must bear on the truthfulness of the witness. The prosecutor’s question about whether Donna lied on her tax return was valid because it was merely a question, and no extrinsic evidence was offered. It beared [sic] on whether she was telling the truth at trial after saying she knew nothing about cocaine and only met Calvin in order to see an old friend. Therefore, it was proper use of specific instances of conduct through cross-examination of a witness.

50 Answer B to Question 6 6) 1. Will’s testimony of Calvin’s statements were NOT properly admitted. (a) Relevance Evidence is generally admissible if it is relevant, meaning that it tends to make a material fact more or less likely to be true. Here, Will’s testimony of Calvin’s statements would make Donna’s alleged involvement more likely to be true, and thus is logically relevant. However, evidence should not be admitted under Federal Rules of Evidence (FRE) 403 if its prejudicial effect substantially outweighs its probative value. Setting aside the Confrontation Clause question (discussed below), this evidence is prejudicial against Donna but is also very probative as to the central issue of the trial – whether Donna is guilty of distribution of cocaine and conspiracy to distribute. As such, the prejudicial effect does not substantially outweigh the probative effect, and testimony should be admitted absent other reasons for preclusion.

(b) Competence A witness’s testimony is admissible if he is competent to testify. A witness is competent if (1) he had personal knowledge of the fact he is testifying to, and (2) he takes an oath or affirmation to tell the truth. Here, Will was present when Calvin made the statement about Donna’s role in the transaction, and thus has the required personal knowledge. Assuming he took the proper oath at trial, Will is competent to testify as to what Calvin had said.

(c) Hearsay Hearsay is not admissible unless an exemption or exception applies. Hearsay is an out-of-court statement offered for its truth. Here, Calvin’s statement about Donna was made outside the court proceedings and was offered by the prosecution to prove that Donna indeed was involved in the cocaine transaction. Thus, it is hearsay. This issue here is whether a proper exemption or exception applies. The prosecution will argue that this declaration is (1) a coconspirator admission and (2) a statement against interest. Coconspirator statements are exempted from the hearsay rule under FRE 801(d), and can be admitted as substantive evidence. Here, Calvin was allegedly a coconspirator with Donna. If the judge finds by a preponderance that the two were indeed coconspirators, Calvin’s statement against Donna can be admitted, subject to the Confrontation Clause limitations, discussed below. On the other hand, a statement against interest is a hearsay exception, allowing admission for a statement made by an unavailable declarant which was against the declarant’s own penal, proprietary, or other interest. To apply, the declarant must be unavailable by reason of privilege, absence [sic] from the jurisdiction, illness, death, or stubborn refusal to testify. However, if the declarant’s “unavailability” is procured by the party seeking to offer his statement, or if the party acquiesced in a plan to make the declarant unavailable, with the result that he is in fact made unavailable, the right to use such declarations is forfeited. Here, Calvin has the Fifth Amendment privilege against self- incrimination to refuse to testify in Donna’s trial, and, assuming he exercised that privilege, and that his absence from Donna’s trial is not encouraged or induced by the prosecution,

51 Calvin is properly deemed unavailable. Nevertheless, Calvin’s statement identifying Donna’s role in the transaction was made with the intent to push responsibility onto Donna, in an attempt to either secure a favorable plea bargain with the prosecution, or convince the arresting officer that Calvin was not in fact involved in the transaction at all. Statements like these which are made for the purpose of currying favor with the prosecution are not against the declarant’s penal interest and cannot properly be admitted under the “statement against interest” exception.

(d) Confrontation Clause Even though Calvin’s statement is exempted form the hearsay rule as a coconspirator admission, it may not be admitted against Donna in her trial without Calvin actually testifying. Under the Sixth Amendment, the [sic] criminal defendant has a constitutional right to confront witnesses against him. In a recent Supreme Court case, Crawford v. Washington, the court held that hearsay statements that are testimonial in nature cannot be admitted against a criminal defendant unless the defendant had either (1) a prior opportunity to cross-examine the declarant, or (2) a present opportunity to cross- examine the declarant at trial as a witness. A statement is “testimonial” if the declarant reasonably could foresee that it would be used against the criminal defendant in her prosecution. Here, Calvin told a police officer that Donna was the person who owned the cocaine, and thus could reasonably foresee his statement would be used to prosecute Donna, making it testimonial. If Calvin is not now produced as a witness at Donna’s trial, and subjected to Donna’s cross-examination, his out-of-court statement could not be constitutionally admitted against Donna.

2. Ned’s expert testimony WAS properly admitted. As per the discussion on relevance, Ned’s testimony is generally admissible because (1) it would make the prosecution’s theory that Donna used Calvin to transport her cocaine more probable, and (2) its probative value is great, and not substantially outweighed by the risk of prejudice to Donna. In addition to taking a valid oath, an expert witness is permitted to give expert testimony where (1) the subject matter is one where expert opinion would be useful to the fact finder, (2) the witness is properly qualified as a witness, (3) the judge finds that the expert opinion is reliable, and (4) the expert opinion has proper bases. Here, whether or not drug dealers usually use others to transport drugs for them is a matter outside most average people’s ken, and thus is a subject matter where expert opinion would be useful. As an “experienced detective assigned to the Narcotics Bureau,” Ned has specialized knowledge and experience in the matter of drug dealers’ behavior patterns, and would probably qualify as an expert. The judge must also find, by a preponderance, that the expert opinion is reliable – that is, that the methodology the expert used to reach his conclusions were reliable, and that the methodology “fits” the facts in the case. Under the Daubert case, the judge can consider these following factors in considering reliability of an expert’s methodology: (1) Existence of peer review, (2) the error rate of the expert’s methodology, (3) the testability of the methodology, and (4) whether the methodology were [sic] generally accepted by experts in the field. Here, Ned’s methodology in reaching the conclusion that drug dealers

52 customarily use others to transport drugs was probably his experience in dealing with narcotics cases, and perhaps an analysis of the rate of “using others” narcotics cases to other narcotics cases. The methodology should be explained to the court, and if the judge finds it to be reliable, and that it properly “fits” with the facts of this case (alleged use of others to transport drugs for the dealer), the court will find the expert opinion reliable. Finally, expert opinion must have a proper basis – it must be based on either facts already in evidence, or facts not in evidence that are generally relied upon by experts in the field. Assuming that the data set [sic] from which Ned drew his conclusion was not admitted into evidence, it must be shown to be data relied upon by other drug dealer behavior experts in the field.

3. The copy of Calvin’s rental car receipt was NOT properly admitted. Because the evidence sought to be admitted here is a piece of writing (receipt), it must not only be relevant, but also be authenticated as the thing it is purported to be, satisfy the Best Evidence Rule, if applicable, and shown not to be barred by the hearsay rule. Here, if the receipt was believed, it would tend to make the prosecution’s theory that Donna rented the car and had Calvin drive it to distribute drugs less likely. Thus, it is relevant. Moreover, the prejudicial effect to the prosecution is not substantially outweighed by the probative value of the receipt as to who in fact rented the car. Because the receipt’s relevance is dependent upon it being the receipt recovered from Calvin’s car, it must be authenticated as such, meaning that defense must present sufficient evidence for a reasonable jury to decide that the receipt in court was the one recovered from the car. Donna can do this by establishing a substantially unbroken chain of custody, testifying that she had kept the receipt in a safe place since she personally retrieved it from Calvin’s car, that no one had the opportunity to access and materially alter it, and that the contents of the receipt were in fact substantially unaltered from when she retrieved it from the car. The best evidence rule also applies here because defense is offering the receipt for its contents. Under the rule, an original or mechanically made duplicated [sic] must be presented into evidence. Here, if Donna can show that the copy presented was mechanically made from the original receipt, the rule is satisfied. Finally, because the receipt is an out-of-court statement offered for its truth (that Calvin rented the car six months before his arrest), it is hearsay and inadmissible unless an exemption or exception applies. Here, the receipt might be admitted under the “business record” exception; if Donna could show that the receipt was made in the regular course of the car renter’s business, made in the manner such records are usually kept and at or around the time the car was rented, then the exception applies. However, this requires that a record custodian form the car rental company testify at trial as to these elements. Assuming that the defense did not present a custodian from the car rental company, the receipt cannot be deemed a business record, and cannot be properly admitted.

4. The cross-examination question to Donna probably should NOT be allowed. As discussed above, a piece of fact or, in this case, a question, that tends to make a material fact in case more or less likely to be true is relevant and generally admissible. Here, if Donna lied in [sic] her income tax return, it would make her a less credible witness, and more likely a drug dealer. Thus, the question is generally allowable as relevant. Character evidence is generally inadmissible for the purpose of showing that a

53 person acted on the particular occasion according to her propensity to act a certain way. However, character evidence on [sic] a witness’s veracity, including specific prior bad acts committed by the witness, may be used to impeach her credibility, provided that the cross- examining party has a good faith basis to believe that such prior bad acts in fact took place. Here, whether Donna lied on her tax return goes to her veracity, and thus is character evidence. The cross-examination question was presented for the purpose of impeaching Donna’s credibility, but the prosecution did not have actual evidence to believe that Donna had lied on her income tax returns. Instead, the basis for this question was a general impression that drug dealers usually do not report their income. While this impression was honestly held by the prosecution, its basis is weak as it relates to Donna, who has not even been proven to be a drug dealer. Moreover, the question creates a prejudicial effect on the jury’s mind, making them doubt the veracity of the defendant herself. As such, the prejudicial effect of this question substantially outweighs its weak probative value, and should therefore not be allowed.

54

ESSAY QUESTIONS AND SELECTED ANSWERS

JULY 2007 CALIFORNIA BAR EXAMINATION

This publication contains the six essay questions from the July 2007 California Bar Examination and two selected answers to each question.

The answers received good grades and were written by applicants who passed the examination. The answers were prepared by their authors, and were transcribed as submitted, except that minor corrections in spelling and punctuation were made for ease in reading. The answers are reproduced here with the consent of their authors.

Question Number Contents Page

1 Real Property 1

2 Torts 10

3 Evidence 22

4 Criminal Procedure/Constitutional Law 36

5 Remedies 45

6 Community Property 56

i

Question 3

Dave brought his sports car into the local service station for an oil change. While servicing the car, Mechanic checked the brakes and noticed that they needed repair. The following events occurred:

(1) Mechanic commented to Helper, “Dave had better get these brakes fixed. They look bad to me.”

(2) Mechanic instructed Helper (who did not himself observe the brakes) to write on the work order: “Inspected brakes — repair?”, which Helper then wrote on the work order. However, Helper currently does not remember what words he wrote on the work order.

(3) Many hours later when Dave picked up his car, Helper overheard Mechanic say to Dave, “I think your brakes are bad. You’d better get them fixed.”

(4) Dave responded, “I am not surprised. They’ve felt a little funny lately.”

(5) Later that day, when Helper was walking down Main Street, he heard the sound of a collision behind him, followed by a bystander shouting: “The sports car ran the red light and ran into the truck.”

The sports car involved in the accident was the one that Dave had just picked up from Mechanic. Polly owned the truck. Polly sued Dave for negligence for damages sustained in the accident. Polly’s complaint alleged that the accident was caused by the sports car running the red light because the sports car’s brakes failed. Polly’s theory of liability is that Dave knew or should have known that his brakes were bad and that driving the car under those circumstances was negligent.

Polly called Helper as a witness to testify as to the facts recited in items (1) through (5) above, and she also offered into evidence the work order referred to in item number (2). Assume that in each instance, appropriate objections were made.

Should the court admit the evidence offered in items numbers (1) through (5), including the work order referred to in item number (2)? Discuss.

Answer A to Question 3

Polly v. Dave

(1) “Dave had better get these brakes fixed”

Logical Relevance

Only relevant evidence is admissible. Evidence is logically relevant when the evidence has some tendency to make a fact of consequence to the litigation more or less probable than it would be without the evidence.

Here, Polly alleges that her accident with Dave was caused by his car’s brake failure. Thus, a statement that the brakes looked bad would be relevant for purposes of establishing that the brakes were bad. However, because Polly’s theory of liability is negligence, and that Dave knew or should have known that the brakes were bad, anything that Mechanic said to Helper is irrelevant for showing that Dave had knowledge. Thus, the logical relevance of the statement is minimal.

Legal Relevance

Otherwise legal evidence may be inadmissible where the probative value of the evidence is substantially outweighed by the risk of unfair prejudice to the defendant, confusion of the jury or the issues, or waste of time.

Nothing about this evidence would be prejudicial. However, it may confuse the jury, again because Polly’s claim is in negligence and thus any statement that Dave did not hear would have no bearing on his knowledge of the defect of the brakes.

Personal Knowledge

A witness can only testify about that which they have personal knowledge. This is true for the testifying witness, as well as for the declarant in any hearsay statement.

Here, Mechanic had personal knowledge of the condition of Dave’s brakes, because he was conducting the inspection. Further, Helper heard Mechanic’s comment, and so had personal knowledge of what Mechanic said.

Hearsay

Hearsay is an out-of-court statement, admitted for purposes of the proving the truth of the matter asserted. Hearsay is inadmissible unless exempt or unless an exception applies.

Mechanic’s comment to Helper was made out of court, and is being introduced for purposes of showing that the brakes were bad. Thus, the statement is hearsay.

Present Sense Impression

A statement made concerning one’s observations or impressions, made while or immediately after the observation or impression, is admissible as a hearsay exception.

Here, Mechanic made the statement while servicing Dave’s sport car. Thus, the “They look bad to me” statement, which concerned his impressions of Dave’s brakes, was made simultaneous to his visual inspection and thus admissible as a present sense impression.

State of Mind

A statement made concerning one’s then present state of mind is admissible as a hearsay exception.

Here, because Mechanic was a mechanic, he was aware of the dangers posed by faulty brakes. Thus, when he said that “Dave had better get these brakes fixed, “he likely had the mental thought that they posed a risk to Dave and other drivers, and was speaking as to his knowledge that Dave needed to get the brakes fixed.

Thus, the statement should probably not be admitted, because the probative value is low because the statement has nothing to do with Dave’s knowledge or lack thereof of the condition of his brakes.

(2) Work Order – “Inspected brakes – repair?”

Logical and Legal Relevance

Assuming that Dave received the work order, the “Inspected brakes – repair?” language would have a great tendency to make it more relevant that Dave had knowledge of the defective brakes than it would be without the work order. There is no risk of unfair prejudice to Dave, because there is nothing prejudicial about a work order. Further, given the highly probative value of the statement, there is no risk of confusing the jury or wasting judicial resources.

Totem Pole Hearsay

Where a piece of hearsay evidence contains other pieces of hearsay evidence, each statement must fall within an exception in order to be admissible. Here, because both the work order and Mechanic’s statement to helper, which was recorded on the order, were made out of court and are being admitted for their truth, they are hearsay. If either statement is inadmissible, the whole piece of evidence is inadmissible.

Business Record Exceptions / Work Order

Information recorded in a business record is admissible under a hearsay exception where the information was recorded by somebody under a duty to record or report such information, by somebody with personal knowledge of the information, and when the record was kept in the ordinary course of business (that is, the record may not be prepared in anticipation of litigation).

Here, Helper was assisting Mechanic, and Mechanic instructed Helper to write on the work order, “Inspected brakes – repair?,” and Helper did. Thus, Helper was under a duty to record such information. Given that this was a mechanic shop, preparing work orders is likely a part of the ordinary course of business. Further, Helper had personal knowledge of Mechanic’s statement, because he heard Mechanic say it himself and did himself record it in the work order.

Thus, if Mechanic’s statement meets an exception, the whole piece of evidence will be admissible.

Present Sense Impression / “Inspected Brakes – Repair?”

Because Mechanic made the statement as or immediately after his inspection of the brakes, it would fall under the present sense impression, because his impression was that the brakes needed repair.

State of Mind / “Inspected Brakes – Repair?”

Additionally, Mechanic would have been speaking as to his knowledge that the condition of Dave’s brakes was bad and that they required repair.

Recorded Recollection

A writing that was prepared by one with personal knowledge of the events contained in the writing, or at the instruction of the person with personal knowledge and adopted by them, and made soon after the event occurred and that was a true and accurate depiction of the events that transpired, is admissible as a recorded recollection.

Here, because Helper prepared the work order the same time as he heard Mechanic speak, the work order was likely a true and accurate record of what was said, and thus the writing will be admissible as a recorded recollection.

Best Evidence Rule Where a witness is testifying as to the contents of a writing, and those contents are in fact at issue, the best evidence rule requires that the writing be admitted into evidence unless it has been lost or destroyed not due to any intentional misconduct of the party seeking to introduce the evidence.

Here, because Helper is testifying as to the contents of the work order, if the work order is available it should be admitted into evidence as the best evidence. If the work order that was provided to Dave is being introduced for purposes of showing that he knew or should have known that his brakes were bad, the best evidence rule is definitely implicated. However, if it is unavailable, Helper would be permitted to testify as to the contents of the work order, if he remembered the words that were written (which he does not here remember).

Refreshing Recollection

If a witness did before have personal knowledge about something, and is simply unable to recall the specifics while on the stand, anything may be shown to the witness for the purposes of refreshing their recollection. Once the witness’s memory is refreshed, the item that was shown to them must be taken away, and the witness must then testify from their refreshed memory. The item shown must be provided to the other party at their request.

Here, if the work order is available, it may be shown to Helper for purposes of refreshing his recollection as to the words that he wrote on the work order.

Thus, the work order should be admitted. Helper’s testimony as to what Mechanic said should not be admitted, because it is not relevant for purposes of showing that Dave did or should have known of the condition of his brakes.

(3) “I think your brakes are bad.”

Logical and Legal Relevance

Information that Mechanic told Dave that his brakes were bad would be extremely probative for purposes of establishing that Dave knew or should have known that his brakes were bad, which is the basis for Polly’s complaint against Dave. Whether or not Dave had actual notice is very much a fact of consequence, because Polly’s entire negligence claim will turn on Dave’s knowledge of the conditions of his brakes. Thus, given the highly probative value, there is no likelihood of confusing the jury or wasting judicial resources.

Personal Knowledge

Because Helper heard the statement to Dave, he has personal knowledge of the contents of the statement.

Hearsay

Mechanic’s statement to Dave is being admitted for purposes of establishing its truth, that Dave’s brakes were bad. Thus, the statement is hearsay.

Effect on Hearer

One non-hearsay use for out-of-court statement is to show effect on the hearer – the statements are thus not admitted for the truth of the matter asserted. Here, even if Mechanic’s statement were not being admitted for its truth, it would be admissible as non-hearsay for purposes of demonstrating its effect on the hearer, or the effect on Dave, to show that he had been told that his brakes may be bad.

Thus, this statement should be admitted.

“I am not surprised. They’ve felt a little funny lately.”

Logical and Legal Relevance

Against, because Polly’s claim against Dave is in negligence, any evidence that Dave knew or should have known that his brakes were defective is highly probative of establishing that Dave was negligent, as the ordinary reasonable prudent person would either have their brakes inspected by another mechanic, have their brakes repaired, or cease driving the vehicle upon learning that their brakes were bad. Further, that Dave was not surprised to hear that Mechanic thought his brakes were bad and actually felt that the brakes felt funny himself, he had actual knowledge that they may be bad and thus any statement from Dave that they were bad should only have made it more apparent to Dave that he needed to have them repaired.

Although this statement is extremely bad evidence for Dave’s position and extremely good for Polly, the mere fact that evidence is bad for one’s case does not make the evidence unfairly prejudicial.

Personal Knowledge

Because Helper heard Dave’s statement to Mechanic, he had knowledge of its contents.

Hearsay

The statement is hearsay because it is being admitted for its truth. If Dave was not surprised to hear that Mechanic thought his brakes were bad and actually felt that the breaks felt funny, he had actual knowledge that they were bad.

Admission of a Party Opponent

An admission is a statement made by a party to the litigation being admitted into evidence against the speaker, by the opposing party to the litigation. It is non-hearsay as an exemption under the Federal rules of evidence.

Here, because Dave is a party to the litigation, and because his adversary in the litigation, Polly, is admitting the statement against him, it is an admission of a party opponent.

Circumstantial Evidence of State of Mind

Circumstantial evidence of the speaker’s state of mind, such as knowledge of circumstances, is non-hearsay under the Federal rules.

Here, the statement shows that Dave had knowledge that his brakes were or may be bad. Thus, the evidence is admissible for purposes of demonstrating Dave’s state of mind at the time he made the statement to Mechanic.

Thus, this statement should be admitted.

(5) “The sports car ran the red light and ran into the truck.”

Logical and Legal Relevance

That Dave ran the red light and crashed into Polly’s truck is extremely probative for purposes of establishing that Dave was at fault in the accident. The evidence is extremely probative for that purpose. However, it does not appear to be a very important fact of consequence that Dave ran through the red light or crashed into Polly, because in fact it seems that these facts have been established. As the real issue here is Dave’s negligence, and particularly whether he knew or did not know that his brakes were bad, it may confuse the jury to introduce evidence as to the cause of the accident.

Personal Knowledge

Because Helper heard the bystander’s exclamation, he has personal knowledge of its contents.

Further, based on the contents of bystander’s exclamation, it is apparent that he had personal knowledge of the facts exclaimed to.

Hearsay

Because of the bystander’s exclamation is being admitted for purposes of showing that Dave ran through a red light and crashed into Polly’s truck, it is hearsay.

Excited Utterance

A statement made while or immediately after an exciting event, while the declarant is still under the stress of the exciting event, is admissible under a hearsay exception.

Here, witnessing an accident is an exciting event, because it is extremely loud; whenever a person hears an automobile accident, they jump up to see if there is anything that they need to do to help those involved in the accident. As the statement was made immediately after Helper heard the sound of the collision, the declarant was likely under the stress of the event and thus is admissible as an excited utterance. Present Sense Impression

Additionally, the bystander was attesting as to what he had visually witnessed moments before his exclamation, and the statement would be admissible as a present sense impression because it related to something that the bystander had just moments before witnessed.

Thus, this statement should be admitted, because although there is a chance of confusing the jury, Polly is entitled to prove that Dave did run into her with his car and not simply litigate the matter of his negligence with regard to the brakes.

Answer B to Question 3

Polly v. Dave

Proposition 8 is a Victim’s Bill of Rights that is incorporated into the California Constitution. Therefore, in all criminal cases, all relevant evidence will be admitted, subject to a few exceptions. Here, because this is a civil case, the rules of Proposition 8 are inapplicable.

1. Mechanic’s comment to Helper, “Dave had better get these brakes fixed. They look bad to me.”

Relevance In order for evidence to be admitted, it must be logically and legally relevant to the case.

Logical Relevance Under the FRE, evidence is logically relevant if it tends to make any fact of consequence more or less probable than without the evidence. Thus, Mechanic’s comment to Helper is logically relevant because it tends to show that the brakes were defective. Under CA rules, evidence is logically relevant if it tends to prove or disprove any fact in dispute. Here, it is unclear whether or not Dave disputes that the brakes were defective. If Dave does dispute that the brakes were defective, then Mechanic’s comment to Helper does tend to prove that the brakes were defective. However, if Dave admits that the brakes were defective, but rather is arguing only that he did not know they were defective, then under California rules, this statement would not be logically relevant because it does not prove or disprove a disputed fact.

Legal Relevance Evidence is legally relevant if its probative value outweighs undue prejudice or undue delay. Here, this evidence is probative to showing that the brakes were broken. And it outweighs any undue prejudice because, even if the brakes were defective, Dave may still argue that he did not know they were defective.

Lay Testimony Here, Helper’s testimony is being introduced as lay testimony rather than expert testimony, because he is testifying to what he heard, not to any observations or work he did on the brakes. Lay testimony must be helpful and based on personal observations. Here, this testimony is helpful to showing that the brakes were broken and Helper did personally hear Mechanic’s comments. However, in order to admit this testimony, Helper must take an oath, and in California, this requires Helper to know that he has a legal duty to tell the truth.

Hearsay Dave will argue that this is hearsay, not admissible under any exception. Hearsay is any out-of-court statement offered for the truth of the matter asserted. This is hearsay because it is an out-of-court statement made from mechanic to helper, offered to prove that the brakes were broken.

Not for Truth of Matter Asserted Out-of-court statements are not offered for the truth of the matter asserted, and thus admissible, when they are offered to show: a) effect on the hearer; b) the declarant’s state of mind; c) impeach; d) legally operative language; or e) to refresh recollection. Here, there is no indication that Polly is introducing the evidence for any of these purposes.

Offered for Truth of Matter Asserted, but Hearsay Exception Additionally, out-of-court statements may be offered for the truth of the matter, but be exempt hearsay (in California, all of these are hearsay exceptions, not exemptions): a) prior inconsistent statement, under oath; b) prior consistent statement; c) prior identification; or d) admission by party opponent. Here, none of these are applicable.

Offered for Truth of Matter Asserted, and Out-of-Court Declarant is Unavailable Furthermore, hearsay may be admissible if it falls into one of the many hearsay exceptions. One category of exceptions is when the out-of-court declarant is unavailable. “Unavailable” means that the out-of-court declarant (Mechanic) is a) beyond the subpoena power of the court; b) invokes privilege; or c) is dead. Under the FRE, there are two additional times when an out-of-court declarant is considered “unavailable”: a) lack of memory; and b) refusal to respond to subpoena. Here, there is no indication that Mechanic is “unavailable”, thus, these hearsay exceptions do not apply.

Offered for Truth of Matter Asserted, and does not matter if Out-of-Court Declarant is Unavailable Additionally, there are categories of hearsay exceptions regardless of whether an out-of- court declarant is available. Here, Polly may argue that Mechanic’s statement should be admitted as a present sense impression.

Present Sense Impression An out-of-court statement is hearsay within an exception when it is a present sense impression. A present sense impression is a statement describing an event contemporaneously or immediately thereafter. In California, this exception is narrowly construed to only statements made by someone “engaging in” the activity. Here, Mechanic is not describing any event that he is engaging in or observing. Rather, he is making a comment regarding the state of Dave’s brakes. Thus, it is not hearsay within any exception.

2. Mechanic’s Instruction to Helper to write on work order: “inspected brakes – repair?”

Relevance Here, the work order is logically relevant because it tends to show that the brakes were broken. Again, if this was in dispute, then in California this would also be logically relevant. For the same reasons discussed above under section 1, this is also legally relevant.

Best Evidence Here the best evidence is arguably the work order. This is especially true since Helper is having difficulty remembering what words he wrote on the work order.

Hearsay Here, this is hearsay within hearsay because 1) Helper did not himself observe the brakes and therefore he was simply writing down what he was instructed to do; and 2) Helper’s statement in the work order is an out-of-court statement.

Mechanic’s instruction to helper Again, there is no evidence that Mechanic was unavailable to testify.

Present Sense Impression Polly may argue that this was a present sense impression. If this was made immediately following Mechanic’s inspection of the brakes, they may qualify as a present sense impression. However, in California, they would not because this comment was not made while Mechanic was engaged in fixing the brakes.

Helper’s writing in the work order Helper’s writing in the work order “Inspected brakes – repair?” is hearsay within hearsay.

Past Recollection Refreshed Polly may be able to introduce this as past recollection refreshed. Parties can use anything to refresh the recollection of witnesses. Here, Polly could show Helper the work order to refresh Helper’s memory. However, the work order could not be read into evidence. If Helper’s memory is refreshed from looking at the work order, then he can testify independently and that will be introduced. However, if Helper’s memory is not refreshed by looking at the work order, Polly’s counsel may look to past recollection recorded.

Past Recollection Recorded Past recollection recorded may be admitted if it was made at or near the time of the event while the event was still fresh. Here, it appears that the work order was made immediately after Mechanic inspected the brakes, and Helper immediately wrote it in the work order, and thus it was at or near the time of the event. Therefore, the work order can be read into evidence, but not introduced as evidence.

Business Record If Polly’s attorney wants to actually introduce the work order into evidence, the best way to do so is as a business record. A business record may be introduced if it is made by one with a business duty, it is recorded in the regular course/practice of business, at or near the time of the event, by someone with knowledge, and it is trustworthy. Here, this record was made by Helper, who has a business duty. Additionally, it is likely that these work orders are made in the regular course and practice of the business. This work order was not made in anticipation of litigation. Helper made the work order per Mechanic’s instructions, and therefore it was made by one with knowledge. And there is an overall element of trustworthiness, since neither Helper nor Mechanic were the negligent party.

Therefore, the work order should be admitted as a business record.

3. Mechanic to Dave, ”I think your brakes are bad. You’d better get them fixed.”

Relevance Here, Mechanic’s statement to Dave is relevant because it tends to prove that Dave knew about his defective brakes. And in California, it would be admitted because it is in dispute whether or not Dave was aware of his bad brakes. Additionally, this is legally relevant because its probative value is very high (it shows that Dave knew his brakes were bad) and its chance for undue prejudice or delay are low.

Lay Opinion Here, Helper may testify regarding this because this is helpful to the jury and because Helper was present and contemporaneously overheard Mechanic make this comment to Dave.

Hearsay: Effect on Hearer Here, Dave will argue that this is hearsay not within any exception. However, Polly will counter argue that this is not hearsay at all. Rather, Polly will argue that this is not offered to prove the truth of the matter asserted (that the brakes were in fact bad and that Dave should get them fixed). Rather, this is offered to show the effect on the hearer (Dave). Polly will argue that this is offered to prove that Dave knew (or should have known) that his brakes were defective, and was negligent in driving his car without fixing the problem. Thus, this testimony is not hearsay and should be admitted.

4. Dave to Mechanic, “I’m not surprised. They’ve felt a little funny lately.”

Relevance

This comment is relevant because it tends to show that Dave knew that his brakes were defective and was therefore negligent in driving the car. Additionally, this is logically relevant in California, because it is likely disputed whether or not Dave knew his brakes were defective. Additionally, it is legally relevant because its probative value outweighs any prejudice.

Hearsay

Not for Truth of Matter Asserted First, Polly will argue that this is not offered for the truth of the matter asserted, but rather to show the declarant’s state of mind (that Dave knew that the brakes were defective). Additionally, Polly may want to introduce this later on as impeachment evidence against Dave if he testifies that he did not have any idea that his brakes were defective.

Offered for Truth of Matter Asserted, but Hearsay Exemption/Exception Additionally, Polly may try to argue that this is within a hearsay exemption (FRE)/exception (CA) of a) prior inconsistent statement or b) admission by party opponent.

Prior Inconsistent Statement Here, if Dave testifies that he never knew that his brakes were acting up, Polly may be able to introduce this as a prior inconsistent statement. In California, this would be permitted as a hearsay exception because California does not require that the prior inconsistent statement be made under oath. However, under the FRE, this would not be admitted because it was not made under oath.

Admission by Party Opponent Here, Polly will try to introduce this as an admission by a party opponent (Dave) that his brakes were defective. As such, it would fall under a hearsay exemption (or exception in California). Here, this is Dave’s own admission that he knew that the brakes have been acting oddly, and therefore should be admitted as a hearsay exception.

Offered for Truth of Matter Asserted, and Out-of-Court Declarant is Unavailable Additionally, Polly may argue that this is a declaration against interest (against Dave’s pecuniary, penal, or social interest (California only)). However, this hearsay exception is only available if the out-of-court declarant is unavailable, and here, Dave is available.

Offered for Truth of Matter Asserted, and does not matter if Out-of-Court Declarant is Unavailable Additionally, this may be offered as current state of mind as a hearsay exception.

5. Bystander, “The sports car ran the red light and ran into the truck.”

Relevance Here, this statement is relevant because it shows that Dave was the one that ran the red light and hit Polly. This is likely an issue in dispute, so should also be logically relevant in California. Additionally, this is legally relevant because it has a high probative value that is not outweighed by any undue prejudice.

Offered for Truth of Matter Asserted, and does not matter if Out-of-Court Declarant is Unavailable

Present Sense Impression

A present sense impression is one that was made contemporaneously or immediately after an event that describes an event. In California, it is required that the out-of-court declarant be engaged in the event. Here, Bystander made the statement immediately after the collision and the statement is describing what Bystander saw. However, in California this would not be admissible because the bystander was not engaged in the activity. However, under the FRE, this would be admitted.

Excited Utterance An excited utterance is one regarding a startling event, relating to the startling event, and made while the out-of-court declarant is still startled. Here, the bystander was discussing a startling event (a car accident), and it was likely made while the bystander was still startled (certainly, it is startling to see a car accident and one would be startled immediately after observing one). Furthermore, the bystander’s comments are related to the startling event – the bystander is saying what happened.

Therefore, this statement should be admitted as hearsay within an exception.

.

THE STATE BAR OF CALIFORNIA OFFICE OF ADMISSIONS 180 HOWARD STREET • SAN FRANCISCO CALIFORNIA 94105-1639 • (415) 538 - 2300 1149 SOUTH HILL STREET • LOS ANGELES CALIFORNIA 90015-2299 • (213) 765 -1500

ESSAY QUESTIONS AND SELECTED ANSWERS FEBRUARY 2009 CALIFORNIA BAR EXAMINATION

This publication contains the six essay questions from the February 2009 California Bar Examination and two selected answers to each question.

The answers received good grades and were written by applicants who passed the examination. The answers were prepared by their authors, and were transcribed as submitted, except that minor corrections in spelling and punctuation were made for ease in reading. The answers are reproduced here with the consent of their authors.

Question Number Contents Page

1 Professional Responsibility 3

2 Civil Procedure 16

3 Evidence 33

4 Torts 50

5 Contracts 62

6 Business Associations 77

1 Question 3

Dustin has been charged with participating in a robbery in California on the morning of March 1.

(1) At Dustin’s trial in a California state court, the prosecution called Wendy, who was married to Dustin when the robbery took place. Dustin and Wendy divorced before the trial and Wendy was eager to testify.

During the direct examination of Wendy, the following questions were asked and answers given: (2) Prosecutor: You did not see Dustin on the afternoon of March 1, is that correct? Wendy: That is correct. (3) Prosecutor: Did you speak with Dustin on that day? Wendy: Yes, I spoke to him in the afternoon, by phone. (4) Prosecutor: What did you discuss? Wendy: He said he’d be late coming home that night because he had to meet some people to divide up some money. (5) Prosecutor: Later that evening, did you speak with anyone else on the phone? Wendy: Yes. I spoke with my friend Nancy just before she died. (6) Prosecutor: What did Nancy say to you? Wendy: Nancy said that she and Dustin had ―pulled off a big job‖ that afternoon. (7) Prosecutor: Did Nancy explain what she meant by ―pulled off a big job‖? Wendy: No, but I assume that she meant that she and Dustin committed some sort of crime.

Assuming all proper objections, claims of privilege, and motions to strike were timely made, did the court properly allow the prosecution to call the witness in item (1) and properly admit the evidence in items (2) - (7)? Discuss.

Answer according to California law.

33 Answer A to Question 3

1. In the prosecution of D for a robbery, the prosecution called W, who was D’s wife at the time of the robbery as a witness.

Spousal Testimonial Privilege

California recognizes a spousal testimonial privilege in both civil and criminal cases. Under that privilege, a person is permitted to refuse to testify against his or her spouse. However, this privilege does not bar W’s testimony for two reasons.

First, because W and D are no longer married, the privilege does not apply; the spouses have to be married at the time of the trial for the privilege to apply.

Second, the testifying spouse holds the privilege, so that if W decided to testify because she wanted to, D could not assert the privilege to prevent her from testifying. Here, W is eager to testify, and D cannot prevent her from doing so.

Thus, W was properly called as [a] witness, even though she was D’s spouse at the time of the robbery and even over D’s objection.

Confidential Marital Communications Privilege

California also recognizes a confidential marital communications privilege. That privilege protects communications that were made during marriage if those communications were made in confidence. Even though W and D are no longer married, the privilege would still apply to statements made during the marriage. Additionally, D and W jointly hold the privilege, and D can prevent W from testifying as to confidential communications. However, the privilege would not preclude W from testifying in general, so W was properly called as a witness.

34 2. Question about seeing D on the day of the robbery

Presentation

D should object that to the form of this question because it is leading. A leading question is one that suggests the answer to the witness. Leading questions are only proper on cross-examination, or an direct examination if a witness is hostile or has trouble remembering. Here, the prosecutor’s use of a leading question on direct examination is improper, and an objection to the form of the question should be sustained.

Relevance The question, though leading, is nevertheless relevant. Relevant evidence is evidence that tends to establish the existence of a material, disputed fact. Here, it is likely material whether W saw D on the day of the robbery, depending on D’s defenses and alibis about that day.

Relevant evidence is nonetheless inadmissible if its probative value is substantially outweighed by the danger of unfair prejudice, waste of time, or confusion. Nothing in W’s answer suggests these factors, and it is therefore admissible.

3. W’s answer to the question about speaking with D

Presentation

D should move to strike W’s answer because it answers questions not asked. The prosecutor’s question was simply if W spoke with Dustin on that Day. W should simply have answered yes, but instead offered ―in the afternoon‖ and ―by phone.‖ That additional material was not in response to the question and could

35 be stricken by the court. In California, both the party conducting the examination and the opposing party can move to strike a witness’s answer.

Relevance

The answer is, however, likely relevant to the existence of a material, disputed fact because it relates to where D was and what he was doing on the day of the robbery.

4. W’s testimony of D’s statement

Relevance

W’s testimony is relevant because it is offered to prove the existence of a disputed, material fact: namely, that D was going to divide up money with his friends, which suggests that he participated in the robbery.

The testimony can nevertheless be excluded if its prejudicial value substantially outweighs its probative value. Although, it’s prejudicial to D because it establishes guilt, it is not unfairly prejudicial because it does not improperly appeal to the jury’s sensitivities. Thus, the information is relevant.

Competence

Furthermore, W is competent to testify about D’s statement because she has personal knowledge of it, as she heard it.

Hearsay

D should object to this testimony on the basis that it is hearsay. Hearsay is an out-of-court statement offered to prove the truth of the matter asserted. Here, the

36 D’s out-of-court statement is being offered to prove that he was meeting up with friends to divide money, as evidence that D participated in the robbery.

Hearsay Exceptions

The prosecution should argue that a number of exceptions apply to this statement.

Admissions by Party Opponent

First, the prosecution should argue that D’s statement is admissible hearsay under California law because it is an admission by a party opponent. D, the defendant, is the prosecution’s party opponent. His statement that he was going to divide up money with friends is an acknowledgement of fact, and is, therefore, admissible hearsay as an admission from a party opponent.

Present State of Mind

Additionally, the prosecution could argue that the statement is admissible hearsay because it is not being offered to prove the truth of the matter asserted, but rather is being offered as circumstantial evidence of D’s state of mind and his intent to go see his friends to divide up money and as circumstantial evidence that he carried out that intent. A limiting instruction could be given to limit the use of the evidence for that purpose.

Present Sense Impression

California also recognizes a hearsay exception where the declarant is describing his conduct at the time he is acting. However, because this statement is one of future action, this exception would not apply.

37 Confidential Marital Communication Privilege

D should also object on the basis that this statement is privileged through the confidential marital communications privilege. As described above, this privilege applies even where the marriage has ended at trial, if at the time the statement is made the parties are married and the statement was made in reliance of the confidential nature of the marital relationship. D will argue that his statement that he was going to divide up money with his friends was intended to be confidential. Given its incriminating nature, it is likely he will win that argument. Unless W can show that there was no confidentiality because others were present when the statement was made, the court should probably grant D’s motion to exclude W’s testimony about his statement on the basis of privilege.

5. Question about conversation with Nancy

Form of Question

D could object to this question as another leading question, because it suggests the correct answer, and is improper on direct examination.

Form of Answer

D could also object to the answer and move to strike, since it offers information (―just before she died‖) that was not asked for in the question. In California, both the person conducting the examination and the other party can move to strike an answer that is nonresponsive to the question asked.

Relevance

D could argue that this evidence is not relevant to a material fact in dispute. On the face of the question, it does seem irrelevant that W’s friend Nancy died

38 shortly after they spoke. However, as explained below, at this information is probably relevant to lay the foundation to establish whether any hearsay exception (dying declaration) applied to Nancy’s statement, and so is likely admissible for that reason.

6. Testimony of Nancy’s statement

Competence

W is competent to offer this testimony because she has personal knowledge of the statement, that is, Nancy said it to her. However, she may not be competent to testify as to its meaning, as will be discussed below.

Relevance

The testimony of Nancy’s statement is relevant to a disputed material fact because it tends to establish D’s participation in the robbery and his guilt.

Hearsay

D should object to the admission of this statement on the basis that it is hearsay, that is, Nancy’s out-of-court declaration is being offered to prove the truth of the matter asserted (that she and D committed a robbery).

Dying Declaration Exception

California’s dying declaration hearsay exception applies to both criminal and civil cases and permits the admission of statements that were made while the declarant was dying, about the circumstances leading to her death. California requires that the declarant actually have died.

39 Here, Nancy actually died, and her statement was made shortly before her death. However, nothing indicates that the statement was related to the circumstances of her death. Perhaps if Nancy was injured during the robbery, the statement would be admissible, but on the facts presented currently, nothing suggests the statement was made about the circumstances of her death, and it is therefore not admissible under this exception.

Statement Against Interest

California also recognizes a hearsay exception where the declarant’s statement is against his or her financial, social, or penal interest at the time it was made. The declarant must be unavailable.

Here, Nancy is unavailable because she is dead. Additionally, the statement that she and D ―pulled off a job‖ suggests criminality on her part and is therefore, against her penal interest, and was so at the time that it was made. The statement should be admitted under this exception.

7. W’s interpretation of Nancy’s statement

Relevance

W’s comment about Nancy’s statement is relevant because it goes to prove a disputed material fact, that is, whether D committed a crime on March 1.

Form of answer

D should move to strike W’s answer because the prosecutor did not ask W what she thought Nancy meant by the statement; the prosecutor only asked whether Nancy explained what she meant, and W’s answer was therefore nonresponsive and possibly in narrative form.

40 Competence

However, D should object to W’s statement on the basis that W is not competent to interpret Nancy’s statement. W has no personal knowledge of what Nancy meant by ―pulled off a big job‖ because, as W testifies, Nancy never explained what that meant.

Lay Opinion

D could also object to W’s statement on the basis that it offers a lay opinion evidence, since W has no personal knowledge of what the statement meant when Nancy made it. Lay opinion is admissible where it is rationally based on a witness’s perception and is helpful to the jury. Here, it is unlikely that W’s statement is helpful to the jury because members of the jury are just as able to offer an interpretation of Nancy’s statement as W is. Unless W has some other basis for her opinion (i.e., Nancy and D had used those terms in the past, or that it was customary where she lived), W should not be allowed to offer her interpretation of Nancy’s statement.

Proposition 8

In a California criminal case, all relevant evidence is admissible, subject to certain exceptions (such as hearsay rules and privilege). Here, the court could determine that the evidence is admissible notwithstanding that it is an otherwise inadmissible lay opinion, if the evidence’s probative value was not substantially outweighed by its prejudicial value.

41 Answer B to Question 3

Because this is a criminal prosecution in California, Prop 8 applies. Prop 8 makes any relevant information admissible subject to unfair prejudice balancing. However, Prop 8 doesn’t apply to hearsay, rape shield, the exclusionary rule, privilege, evidence of D’s character first presented by the prosecution, and secondary evidence.

1. Spousal Privilege Testimonial Privilege In California, a witness may refuse to testify against their spouse in both civil and criminal proceedings. This privilege exists only during a valid marriage. Further, it is the [witness] spouse that holds the privilege.

Because D and W are divorced and W wants to testify, she may.

Confidential Communication Privilege All communications made during the course of a valid marriage and intended to be confidential between the husband and wife are privileged. The party spouse holds the privilege, and thus may prevent the witness spouse from testifying to these communications. The communications made during marriage remain privileged even after divorce.

Therefore, Wendy may testify to information other than confidential communications made between her and D during the marriage. The defense may not prevent her from taking the stand. The court allowed the prosecution to call the witness.

2. You did not see Dustin on ……

42 Relevance

Logical In order to be admissible, evidence must be relevant. It is relevant if it tends to make any disputed material fact of consequence more or less probable.

Here, the fact that D wasn’t in S’s presence on the afternoon in question makes it more probable that he could have been participating in a robbery. Thus, it is relevant.

Legal Although logically relevant, evidence may be excluded for public policy reasons or because the risk of unfair prejudice substantially outweighs the probative value. Neither of these apply here.

Form The prosecution should object to this question as leading. Leading questions are questions that suggest the desired answer. They are inadmissible on direct except where the witness is hostile, adverse, or needs help remembering. It doesn’t appear that any of these exceptions apply; thus, the form of the question was improper.

Competence of Witness A witness may testify only based on personal knowledge and present recollection. Here, W is testifying based on what she observed that day from present recollection. Thus, it is proper.

Therefore, the question was asked in an improper form, and any objection to form would have been granted. However, the answer would be admissible.

3. Did you speak with D on that day?

43 Relevance This information is relevant to lay a foundation for the next question. The fact that W spoke with D makes it more probable that he told her something in the phone conversation.

Further, it is neither unfairly prejudicial nor excluded for public policy reasons.

Competence Evidence is based on present recollection and personal knowledge.

4. What did you discuss? Relevance Evidence is relevant in that it makes more probable that D committed the robbery if he had money to divide up.

Hearsay Hearsay is an out-of-court statement used to prove the truth of the matter asserted. It is inadmissible unless it fits under one of California’s hearsay exceptions.

W’s response of what D said is hearsay because it is used to prove the truth of the matter asserted, i.e., that he would be home late because he had to divide some money. The prosecution is using it to show he did have some money from the robbery.

Exceptions Party Admission The statement, although hearsay, would be admissible under the party admission hearsay exception. A statement by any party is admissible hearsay regardless of whether the statement was against their interest when made.

44 Here, D’s statement that he had money to count up is an admission by a party, D, that he had some money to divide up.

Statement Against Interest Further, the statement may be admissible under the statement against interest hearsay exception. For this exception to apply, the statement must be against the declarant’s interest and the declarant must be unavailable. It is unclear if D is testifying, but if he doesn’t he is unavailable. Further, the statement could be argued to be against his interest because he is admitting he has a sum of money to divide.

Present State of Mind This exception includes statement of intent as circumstantial evidence that the intent was carried through. D’s statement of intent to meet people and divide some money may be admissible as circumstantial evidence that he did in fact do that.

Confrontational Clause Under the 6th Amendment, criminal defendants have the right to cross-examine the witnesses against them. If a statement of a hearsay declarant is admitted, the confrontation clause is violated if the declarant is not available, doesn’t testify, wasn’t subject to cross, and the statement is testimonial.

The confrontation clause doesn’t apply here because the declarant is the defendant himself and he wasn’t giving testimonial evidence.

Privilege

As discussed above, the confidential communication privilege may bar this testimony. It was made during a valid marriage and intended to be confidential.

45 Therefore, the defense may properly object to this testimony, and it should be excluded.

Therefore, the evidence would be admissible hearsay as a party admission. However, the confidential communication spousal privilege likely would aply to exclude the evidence.

5. Later that evening did you speak with anyone else….

Relevance Relevant to lay the foundation for the following question. If W spoke to Nancy, it is more likely she obtained the information she is about to testify to.

Form This answer may be non-responsive in that it goes beyond the question asked of the witness. Further, it may assume facts not in evidence as there is no indication that Nancy had died. As such, an objection to form should have been granted.

6. What did Nancy say to you?

Relevance It is relevant because it tends to make it more likely that D was in fact involved in a robbery.

Hearsay W’s testimony is an out-of-court statement by Nancy used for the truth of the matter asserted. Thus, it is inadmissible unless an exception applies.

46 Exceptions

Dying Declaration The dying declaration hearsay exception applies to statements made with belief that death is imminent and that concern the cause of circumstances of death and, under California law, the declarant must actually die. In CA, it applies in both civil and criminal cases.

The declarant actually died, but the statement didn’t involve the cause or circumstances of death. Thus, it is not applicable.

Party Admission An admission by a coconspirator may be admissible against a fellow conspirator as an exception to hearsay. The statement must be made concerning the conspiracy and during the existence of the conspiracy.

It appears that N and D were coconspirators (an agreement between two or more persons w/the intent to agree and intent to complete the target offense). However, a conspiracy ends when the target offense is completed, and thus, when the bank robbery was completed, it is unlikely N and D were coconspirators any longer. Therefore, it is not an admissible party admission.

Statement Against Interest A statement that, when made, was against the declarant’s interest may be admissible under this exception. The declarant must be unavailable for this exception to apply.

Here, the statement that N and D had pulled off a big job, depending on how interpreted, was against N’s interest when made. At the time made, it subjected her to criminal punishment because most people would interpret that as having committed a big robbery. Therefore, this exception likely applies.

47 Therefore, the statement is admissible hearsay under the statement against interest exception.

7. Did Nancy explain what she meant by ―pull off a big job‖?

Form The defense could move to strike the witness’ answers as non-responsive (except the ―No‖). The prosecution asked [for] a ―yes‖ or ―no‖ answer, and the witness responded with something in addition to ―yes‖ or ―no‖ that did not respond to the question. The prosecution didn’t ask her what she thought of what it meant. This would be granted by the court.

Competence/Opinion Testimony A witness must testify as to present recollection and personal knowledge. Here, W is testifying based on speculation and this is improper.

Further, a lay witness may give opinion testimony only if it is based on personal knowledge and helpful to the jury. Again, there is no personal knowledge and the speculation is not helpful to the jury. Thus, W’s last statement should be stricken.

48 FEBRUARY 2009 ESSAY QUESTIONS 4, 5, AND 6

California Bar Examination

Answer all three questions. Time allotted: three hours

Your answer should demonstrate your ability to analyze the facts in question, to tell the difference between material and immaterial facts, and to discern the points of law and fact upon which the case turns. Your answer should show that you know and understand the pertinent principles and theories of law, their qualifications and limitations, and their relationships to each other. Your answer should evidence your ability to apply law to the given facts and to reason in a logical, lawyer-like manner from the premises you adopt to a sound conclusion. Do not merely show that you remember legal principles. Instead, try to demonstrate your proficiency in using and applying them. If your answer contains only a statement of your conclusions, you will receive little credit. State fully the reasons that support your conclusions, and discuss all points thoroughly. Your answer should be complete, but you should not volunteer information or discuss legal doctrines which are not pertinent to the solution of the problem. Unless a question expressly asks you to use California law, you should answer according to legal theories and principles of general application.

49

THE STATE BAR OF CALIFORNIA OFICE OF ADMISSIONS 180 HOWARD STREET • SAN FRANCISCO CALIFORNIA 94105 1639 • (415) 538 - 2303 1149 SOUTH HILL STREET • LOS ANGELES CALIFORNIA 90015-2299 • (213) 765 – 1500

ESSAY QUESTIONS AND SELECTED ANSWERS JULY 2009 CALIFORNIA BAR EXAMINATION

This publication contains the six essay questions from the July 2009 California Bar Examination and two selected answers to each question.

The answers received good grades and were written by applicants who passed the examination. The answers were prepared by their authors, and were transcribed as submitted, except that minor corrections in spelling and punctuation were made for ease in reading. The answers are reproduced here with the consent of their authors.

Question Number Contents Page

1 Torts/Civil Procedure/Professional Responsibility 3

2 Professional Responsibility 16

3 Evidence 27

4 Constitutional Law 50

5 Civil Procedure/Remedies/Professional Responsibility 66

6 Criminal Law and Procedure 82

1

Question 3

While driving their cars, Paula and Dan collided and each suffered personal injuries and property damage. Paula sued Dan for negligence in a California state court and Dan filed a cross-complaint for negligence against Paula. At the ensuing jury trial, Paula testified that she was driving to meet her husband, Hank, and that Dan drove his car into hers. Paula also testified that, as she and Dan were waiting for an ambulance immediately following the accident, Dan said, “I have plenty of insurance to cover your injuries.” Paula further testified that, three hours after the accident, when a physician at the hospital to which she was taken asked her how she was feeling, she said, “My right leg hurts the most, all because that idiot Dan failed to yield the right-of-way.”

Officer, who was the investigating police officer who responded to the accident, was unavailable at the trial. The court granted a motion by Paula to admit Officer’s accident report into evidence. Officer’s accident report states: “When I arrived at the scene three minutes after the accident occurred, an unnamed bystander immediately came up to me and stated that Dan pulled right out into the path of Paula’s car. Based on this information, my interviews with Paula and Dan, and the skidmarks, I conclude that Dan caused the accident.” Officer prepared his accident report shortly after the accident.

In his case-in-chief, Dan called a paramedic who had treated Paula at the scene of the accident. Dan showed the paramedic a greeting card, and the paramedic testified that he had found the card in Paula’s pocket as he was treating her. The court granted a motion by Dan to admit the card into evidence. The card states: “Dearest Paula, Hurry home from work as fast as you can today. We need to get an early start on our weekend trip to the mountains! Love, Hank.”

Dan testified that, as he and Paula were waiting for the ambulance immediately following the accident, Wilma handed him a note. Wilma had been identified as a witness during discovery, but had died before she could be deposed. The court granted a motion by Dan to admit the note into evidence. The note says: “I saw the whole thing. Paula was speeding. She was definitely negligent.”

Assuming all appropriate objections were timely made, should the court have admitted:

1. Dan’s statement to Paula about insurance? Discuss. 2. Paula’s statement to the physician? Discuss. 3. Officer’s accident report relating to: a. The unnamed bystander’s statement? Discuss. b. Officer’s conclusion and its basis? Discuss. 4. Hank’s greeting card? Discuss. 5. Wilma’s note? Discuss.

Answer according to California law.

27

Answer A to Question 3

Preliminary Matters

Proposition 8 not applicable Proposition 8 is an amendment to the California Constitution that states, in part, that all relevant evidence is admissible in a criminal trial. However, the present action is a civil action for negligence and thus Proposition 8 does not apply.

Standard of Relevance In CA, evidence is relevant if it has any tendency to make disputed fact of consequence to the determination of the action more or less probable.

Discretion to Exclude under CEC 352 Under CEC 352, a judge has discretion to exclude evidence where its probative value is substantially outweighed by risk of unfair prejudice, waste of time, or confusion of the issues.

1. Dan’s statement to Paula about Insurance

At the scene, Dan told Paula “I have plenty of insurance to cover your injuries.”

Logical Relevance Dan’s statement is relevant in a couple of different ways. It might tend to show that D was driving negligently because he knew he was covered by insurance, and it may also show ability to pay a substantial judgment. Finally, it also indicates an admission of fault because D’s insurance company would only pay for P’s injuries if D was at fault. Thus, by admitting that his insurance would cover her, D implied he felt he was at fault. This is relevant because it tends to show that D was actually at fault and knew it immediately.

28

Legal Relevance Insurance to Prove Negligence or Ability to Pay Proof of D’s insurance to show that D was engaged in negligent conduct or that D has ability to pay a substantial judgment is inadmissible for public policy reasons. We want to encourage people to have insurance and thus we do not allow it to be used against them in court. Thus, D’s statement about his insurance should not be admitted to show that he was negligent or has the ability to pay a substantial judgment.

Use as Acknowledgment of Fault However, the statement is still relevant as an admission of fault. Thus, it should be admitted unless the court finds that the danger of undue prejudice to D substantially outweighs its probative value. The statement will be harmful to D’s case for sure, but mere harm is not substantial unfair prejudice. If D made this statement at the scene, he should be required to explain it and he can attack the probative value. The statement should have been admitted to show D believed he was at fault but it should not be admitted for the above improper purposes. A limiting instruction should have been given upon D’s request to ensure it was only used for the limited purposes of showing D believed he was at fault.

Offer to Pay Medical Expenses There is a public policy exclusionary rule for offers to pay medical expenses. Under the CEC admissions of fault made in conjunction with an offer to pay medical expenses are also inadmissible. Thus, D can argue his statement was an offer to pay P’s medical expenses. However, P can argue that a statement that his insurance would cover her medical expenses is not really an offer to pay and thus his acknowledgement of fault should not be excluded. P seems to have the better argument on this point.

Hearsay An out-of-court statement offered to prove the truth of the matter asserted is hearsay and is inadmissible unless it falls within an exception. Here, D’s statement was made out of court at the scene of the accident. However, if used to show D believed he was

29 at fault, it is now being offered to prove the truth of the matter asserted - that D has insurance that will cover P’s injuries. Thus, it is not hearsay if used for this limited purpose.

Even if offered for the truth of the matter asserted, under the CEC there is a hearsay exception for party admissions. Because D, the defendant here, made the statement, it would be admissible under the party admission hearsay exception.

Conclusion on Item #1: admission was proper for the purpose of showing that D believed he was at fault immediately after the accident but not to show that D was negligent or that D has the ability to pay a substantial judgment. The statement is non- hearsay or admissible as a party admission.

2. Paula’s Statement to the Physician Logical Relevance Paula’s statement tends to show that her right leg was injured and also tends to show how D was negligent - that he failed to yield to her right of way.

Hearsay See hearsay definition above. P’s statement to the physician was made out of court while at the hospital getting treatment. P’s statement is best divided up into two distinct portions: (1) that her right leg hurts, and 2) that Dan failed to yield to her right of way. Both portions of her statement are presumably being offered for their truth - that she suffered an injury to her right leg and that Dan didn’t yield to her right of way. As such, P’s statement is hearsay and is inadmissible unless it falls within a hearsay exception.

Portion 1 – Statement About Injury to P’s Right Leg Present Physical Condition A statement of present physical condition or of present state of mind is admissible as a hearsay exception. P’s statement to the physician described her present physical condition. At the time she was seeing her doctor, her right leg was hurting her and her

30 statement described this present physical condition. Thus, the statement is admissible as a present physical condition.

Excited Utterance An excited utterance is a statement relating to a startling condition made while the declarant is still under the stress caused by the condition. Here, P was injured in a car accident, which is a startling condition. However, the statement was made 3 hours after the car accident. Thus, P may not have still been under the stress caused by the accident at the time the statement was made. Perhaps if P’s injuries were sufficiently severe, she could make a strong argument that she was still under the stress of the accident. It’s a close call but P’s statement is probably not admissible as an excited utterance.

Statement Pertaining to Medical Diagnosis or Treatment Unlike the exception under the Federal Rules, California’s exception for a statement made in connection with the receipt of medical treatment is very narrow and only applies to a child describing an incident of neglect or . Thus, P’s statement is not admissible under California’s narrow exception.

Portion 2 – Statement about D Failing to Yield Present Physical Condition Although made in connection with her description of her present physical condition, the second part of P’s statement does not itself describe a present physical condition. Thus, it should not be admitted with the first portion under the present physical condition exception.

Excited Utterance Following the same analysis above, the second part of P’s statement may be admissible as an excited utterance. However, P would have to establish the preliminary fact that despite the passage of 3 hours she was still in a state of excitement as a result of the accident.

31

Exclusion under CEC 352 However, even if the second portion of P’s statement to the physician were admissible under a hearsay exception, it should probably be excluded under CEC 352. It’s not clear what the statement was based on. If she observed D’s failure to yield, she can testify to that directly rather than admitting it this way. Thus, the probative value is minimal since we don’t know the basis for P’s statement. And it will probably be duplicative of P’s actual testimony at trial and it’s somewhat prejudicial to D because it asserts that D breached a duty without giving him an opportunity to cross-examine P when she made the statement. Thus, the second portion of the statement should be excluded under CEC 352 even if it is found to fall within a hearsay exception.

3. Officer’s Accident Report Logical Relevance: The contents of the report tend to show that D drove out in front of P’s car and was thus negligent and that D was responsible for the accident.

Report - Hearsay The officer’s report is hearsay because it is an out-of-court statement that was made by the officer prior after [sic] the accident and it is being offered to prove its contents - that a witness saw D pull out in front of [P] and that the officer concluded that Dan was at fault.

Public Records Exception The CEC has a public records exception for records made by public employees in the course of their duties. However, the court may exclude the record if it does not appear trustworthy. Here, the police report is an ordinary record made in the course of a police officer’s duties. Thus, it may be admitted under the public records exception. However, the police report contains a statement from a bystander which is hearsay and the public records exception does not permit that statement because the bystander had no duty to communicate the information to the police officer. The business records exception does not cover records including conclusions on complex issues. If the same requirement is

32 applied to the public records exception, Officer’s conclusion that D was at fault may not be admitted under the exception.

Part A - Unnamed Bystander’s Statement Bystander’s Statement - Hearsay The bystander’s statement is hearsay because it was made out of court at the scene of the accident and it is being offered to prove its content that D pulled in front of P’s car. Thus, it is inadmissible unless it falls within a hearsay exception.

Excited Utterance See definition above. The bystander witnessed a startling event: a car accident which he apparently saw at close proximity. The police report also indicates that the officer arrived only 3 minutes after the accident and the bystander made the remark to the police officer immediately upon his arrival. Thus, it is likely that the bystander would have still been under the stress of witnessing the accident when the statement was made. Thus, the bystander’s statement falls within the excited utterance exception.

Present Sense Impression The CEC’s present sense impression exception is narrow in that it only applies to statements explaining the conduct of the declarant while engaged in that conduct. Here, the car accident wasn’t the bystander’s own conduct so the statement would not be admissible as a present sense impression.

Part B - Conclusion and Basis Lay Opinion The opinion of a lay witness is only admissible if it is a rational conclusion based on the witness’s firsthand observations, is helpful to the jury, and does not require expertise or knowledge unknown to the general public. Here, the police report explains that the officer’s conclusion as to fault is based on the bystander’s statement, interviews with both parties, and the skidmarks. The officer’s conclusion thus seems to be reasonably based on his own observations. The conclusion would also be helpful to the jury who

33 may not be able to understand the relevance of the skidmarks. However, it’s not clear exactly how the officer formed his conclusion. If the skidmarks were an important factor, the analysis would seem to require some expertise not possessed by the general public. Thus, the opinion should not have been admitted as lay opinion because it relies on the officer’s special expertise in accident reconstruction and analysis.

Expert Opinion Expert opinion is admissible if it is helpful to the jury, the witness is qualified as an expert, the expert witness is reasonably certain of his conclusion, the analysis is supported by a proper factual analysis and is the result of reliable principles reliably applied to the facts. Here, P cannot establish the admissibility of the officer’s conclusions as an expert opinion. First, the officer was never qualified as an expert and thus it is not clear whether he knows anything about analyzing skidmarks. Second, it is not clear whether the officer was reasonably certain of his conclusion or was just making his best guess based on what he observed. Third, we don’t know what method of analysis the officer used. California has retained the Kelley-Frye standard which requires that the expert’s methods be generally accepted by experts in the field. It is unclear how the officer analyzed the skidmarks and, thus, it is not possible to know if the officer’s methods were generally accepted. In conclusion, the officer’s conclusions could not be admitted as expert opinion.

Legal Relevance - CEC 352 Relevant evidence may [be] excluded where its probative value is substantially outweighed by risk of unfair prejudice. Even if the officer’s conclusions were admissible as lay opinion or expert opinion, the conclusions in the police report should be excluded under CEC 352. The report is extremely vague in stating the basis for the officer’s conclusions. For instance, it is not clear what the officer learned in his interviews of Dan and Paula that led him to the conclusion that Paula was at fault. And, as discussed above, the officer fails to describe how the skidmarks led him to conclude that D was at fault. For these reasons, the officer’s conclusions have minimal probative value. On the other hand the conclusions in the report are very prejudicial to D because they state

34 that he is at fault and he is unable to cross-examine the officer who made them since he will not be testifying at trial. Thus, the risk of unfair prejudice substantially outweighs what little probative value the conclusions offer and the conclusions should have been excluded under CEC 352.

4. Hank’s Greeting Card Logical Relevance The greeting card shows that P had a reason to rush home - to get an early start on their trip to the mountains and possibly that Hank would have been upset with P had she not hurried home. If P was rushing, it’s more likely she may have been negligent, which is relevant to D’s counterclaim and to D’s defense that P was contributorily negligent.

Hearsay See hearsay definition above. Henry’s statements in the card are out-of-court statements because he wrote them up the morning of the accident. However, it does not appear that D is offering them for the truth of the matter.

Non-Hearsay - To Show Effect on Listener Out-of-court statements are not barred by the hearsay rule if offered for some other purpose such as to prove the declarant’s state of mind or to show the effect on the listener. Here, D is not offering the greeting card to prove that they were going to the mountains for the weekend. Rather, D is offering the card to show its likely effect on Paula - that it made her want to get home quickly and that she may not have been driving carefully as a result. Thus, the greeting card should be admitted as non-hearsay for this purpose.

Authentication Physical evidence and writings must be authenticated before they may be admitted into evidence. Authentication requires such proof that is sufficient for a jury to find that the evidence is what the proponent claims it to be. Here, the greeting card was properly

35 authenticated by one of the paramedics who had seen the greeting card when treating Paula after the accident. Thus, it was properly admitted into evidence.

5. Wilma’s Note Hearsay Wilma’s note is an out-of-court statement because she wrote it down at the scene of the accident. Presumably it is being offered to prove the truth of the matter asserted, i.e., that P was speeding and that P was negligent. Because the note is hearsay, it is inadmissible unless it falls within an exception.

Excited Utterance An excited utterance is a statement relating to a startling condition made while the declarant is still under the stress caused by the startling condition. Wilma witnessed the accident, which was a startling event. According to Dan’s testimony, Wilma handed him the note immediately after the accident. Thus, it seems that Wilma wrote the note immediately upon witnessing the accident when she was probably still under the stress caused by witnessing the accident at close proximity. As such, the statement may be admitted as an excited utterance.

Lay Opinion re: Speeding Lay opinions must be based on the witness’s personal observations, helpful to the jury, and not based on special expertise. Wilma’s note contains the assertion that Paula was speeding. This is a lay opinion because it is based on Wilma’s observations (recall, Wilma states she “saw the whole thing”) and does not communicate the facts directly to the jury. We don’t know, for instance, whether Wilma was driving 80 miles per hour or 50 miles per hour. However, this type of lay opinion is usually permissible because it is helpful to the jury. The jury will understand that, under the circumstances, P appeared to be driving very fast. Thus, the opinion regarding P’s speeding should be admitted.

36

Lay Opinion re: Negligence Wilma’s opinion that P was negligent is probably not admissible. This opinion would not be helpful to the jury because it’s not clear what Wilma based this opinion on. If it was based merely on the speeding, then there’s no need to admit the conclusion regarding negligence because the opinion regarding speeding was already admitted. If it was based on other things, then it cannot be shown to be based on Wilma’s firsthand observations. Thus, the opinion regarding P’s negligence should not be admitted.

Authentication Dan, the recipient of the note, could properly authenticate it before it was admitted to evidence. Assuming that the foundation was established, the note would be admissible upon Dan’s authentication.

CEC 352 The circumstances surrounding the note are strange. Unless Wilma was mute, it is unclear why she would write out a note rather than just make a verbal statement to Dan. In addition, the note is rather conclusory and as such it does not assist the jury much in ascertaining whether or not P was driving negligently. On the other hand, there is some unfair prejudice because P has no opportunity to cross-examine Wilma or to even depose Wilma prior to trial. This is a close call, but the note should probably [be] excluded under CEC 352 because its probative value is substantially outweighed by its prejudice to Paula.

37

Answer B to Question 3

Because this case takes place in California state court, the court will use the California Evidence Code as the basis for the admissibility of evidence. Further, because this is a civil case, the rules regarding California’s Proposition 8 will not be applied to the evidence.

1. Dan’s statement to Paula about the insurance

Relevance

For evidence to be admissible, it must be factually and legally relevant. In California, factual relevance is evidence that would tend to make a matter in dispute more or less probable. Here, it is in dispute whether Dan was liable. Therefore, Dan’s statement that “he has plenty of insurance to cover the injuries” will be logically relevant to making the matter of Dan’s negligence more probable.

Legal relevance means that the probative value of the evidence outweighs any prejudicial impact that the evidence may have. While Dan’s comment may be slightly prejudicial in implicating him in the matter, it is highly probative because it establishes that he could have been liable. Therefore, the comment will be found to be legally relevant.

However, evidence can be excluded if a court finds that it has the tendency to confuse the issues and mislead the jury. The defendant’s comment could only establish that he has the ability to pay, and not that he was negligent in the accident. However, such evidence is unlikely to be confusing, and would not be subject to exclusion on this basis alone.

38

Reliability

Evidence must be reliable, and based on the witness’ personal knowledge in order to be admissible. Here, Paula heard Dan make the comment that he has plenty of insurance. Therefore, the evidence is reliable.

Evidence of Medical Insurance

According to the California Evidence Code, evidence of liability insurance is inadmissible in a civil trial to prove that the defendant was at fault or that the defendant has the ability to pay, because public policy concerns dictate that we should encourage persons to have insurance. Therefore, Paula’s testimony that Dan said he had plenty of insurance to cover the injuries should not have been admitted.

Offers to pay for injuries

In California, offers to pay another person’s medical costs are inadmissible in court to show that the defendant was at fault, or that the defendant had the ability to pay. In addition, any statements made in connection with the offer to pay for medical expenses are similarly excluded. Paula is likely introducing the evidence to show that Dan was at fault, and this is why he offered to pay her costs. Therefore, Dan’s statement that he can pay for Paula’s injuries should not be admitted.

Statements of sympathy

In a civil case, a defendant’s statements of sympathy made at the scene of the accident are inadmissible to show fault; however, any accompanying statements can be admitted against the defendant. Here, however, Dan was not making a statement of sympathy, but only stating that he had liability insurance to cover the injuries. Therefore, this rule will not be applicable to the statement.

39

Statements to settle

In California, any statements made with regards to a settlement offer are inadmissible to show guilt or liability. However, in order for this exception to apply, the plaintiff must have filed a lawsuit against the defendant. Because Dan’s statements were made at the scene of the accident, this rule will also not apply.

Hearsay

Hearsay is any out-of-court statement offered to prove the truth of the matter stated therein. Hearsay is generally inadmissible in court. In this case, Dan’s statement was made out of court, and is being offered to show that Dan was liable; therefore, it will be inadmissible hearsay unless an exception applies.

In California, an admission by a party opponent is an exception to the hearsay rule. An admission includes any statement made by the opposing party that is a prior acknowledgement of any fact in the case. Here, Dan made a prior statement that he could pay for Paula’s injuries. Therefore, the statement is an admission by a party opponent, and would fall under the hearsay exception.

However, as stated above, the evidence will be inadmissible, because of the public policy rule governing the exclusion of statements made in connection with proof of insurance and statements offering to pay for the plaintiff’s injuries.

2. Paula’s statement to the physician

Relevance

Paula’s statement to the physician is factually relevant because it shows that she suffered from physical harm, and because it establishes that Dan was negligent. Further, it is legally relevant, because while it is prejudicial to Dan in establishing that he

40 was negligent, it is highly probative because it shows that Paula suffered from physical injury, and it shows that Dan did not yield to the right-of-way, and thus was the party at fault in the accident.

Reliability

Paula has personal knowledge of the statement to the physician, because she made the statement.

Hearsay

Hearsay is any out-of-court statement offered to prove the matters stated therein. Here, Paula is introducing the evidence to show that she was injured and that she was negligent. Thus, it will be inadmissible hearsay unless one of the exceptions apply.

Statements of a past physical condition made to a doctor in the course of treatment

California will admit statements made to a doctor and that were necessary to receiving treatment. However, this exception only applies to minors who make the statements in connection to a claim of child abuse or neglect. Therefore, this exception will not apply.

Statement of a then-existing physical or mental condition

A statement made by the defendant of a then-existing physical condition is an exception to the hearsay rule. Paula can argue that her statement that her leg hurts the most was a statement of a then-existing physical condition, because her leg was hurting while she made the statement. However, the statement that Dan failed to yield to the right of way will not be admissible under this exception because it constitutes a past belief, and therefore, is not a then-existing state of mind.

41

Statement of a past physical condition if the physical condition is at issue in the case

California also permits a statement of past physical condition if it is at issue in the case. However, in order for this exception to apply, the declarant must be unavailable, and here, Paula is in the court. Therefore, this exception will not apply.

Excited utterance

The excited utterance exception permits the admission of a statement of a declarant who experienced an exciting or startling event and [is] still speaking under the stress of such excitement. In this case, Paula’s comment was made 3 hours after the accident. This suggests that the statement was too remote for Paula to still be under the excitement. Further, no statements indicate that she was still under the stress of the accident. Therefore, her statements will not be admissible as an excited utterance.

Present sense impression

A present sense impression is a statement made contemporaneously while witnessing the event. California only recognizes this exception to the extent that it applies to the conduct of the declarant, but not with regards to anyone else. Here, the statement was not made contemporaneously because it was made 3 hours after the accident. Further, it states the conduct of Dan and thus would not fall under the exception.

As a result, the court should have admitted her statement that her leg hurts the most because it was a statement of a then-existing physical condition. However, the further comment about Dan should be excluded because it is inadmissible hearsay.

42

3a. Officer’s accident report relating to the unnamed bystander’s statement

Relevance

The statement is logically relevant because the unnamed bystander’s statement establishes that Dan caused the accident. Furthermore, it is legally relevant because it is highly probative in establishing who was at fault, and this probative value will outweigh any prejudicial impact of the testimony.

Reliability

The bystander personally witnessed the scene; therefore, he has personal knowledge with regards to his statement. Further, the police officer has personal knowledge as to the matters which he entered into the police report, because he wrote the police report.

Hearsay

The police report is an out-of-court statement being offered to prove the matters stated therein. Furthermore, the bystander’s statement was an out-of-court statement that is being offered to prove the truth of the matters stated therein--that Dan was negligent. Thus, there are two levels of hearsay in the police report. Both levels of hearsay must fall within a hearsay exception in order to be admissible in court.

Excited utterance

The excited utterance exception permits the admission of a statement of a declarant who experienced an exciting event and is speaking under the stress of such excitement. The bystander made this statement three minutes after the accident occurred. It is likely that he was still under the stress of the excitement, because such a short time had elapsed, and he had run to the police officer in order to tell him the

43 statement. Therefore, the bystander’s comment will be admissible under the excited utterance exception to the hearsay rule.

Public records exception to the hearsay rule for the police reports

In California, the public records exception to the hearsay requires that the record be made by a public employee in accordance with his duties, that the matters were recorded at or near the scene of the accident, that the official had personal knowledge of the matters contained in the record, and that the record was made under circumstances indicating trustworthiness.

Here, the record was made by a public officer while he was carrying out his duties. Further, he made the report at the scene of the accident, and made the record according to his observations and interviews. Therefore, the factors indicating trustworthiness were present. As a result, the report is admissible under the public records exception.

3b. Officer’s accident report relating to his conclusion and its basis

Relevance

The conclusion and its basis are relevant to establish that Dan was negligent. Further, it is highly probative in establishing who was at fault, and the probative value of this determination far outweighs any prejudicial impact that it may have. Therefore, the evidence is admissible.

Expert witness opinion

Expert opinion is admissible in court if 1) the testimony is helpful, 2) the witness is qualified, 3) the witness is relatively certain of his statements, 4) the witness’ testimony has a sound factual basis, and 5) the opinion was reliably based on matters

44 that were reliably applied. Lay opinion is an opinion by a person that is rationally related to that person’s perception of the incident. Lay opinion does not include legal opinions of negligence and causation.

In this case, Officer is making an expert opinion because he is testifying as to the legal conclusions of the case. This is not conclusion on which a layperson would be able to testify. Therefore, Officer must establish his credentials as an expert. His testimony is certainly helpful to the jury, because it allows the jury to ascertain who was negligent. However, it is not clear if Officer is qualified to make such a legal conclusion (that Dan caused the accident) or that officer is relatively certain of his statements. Further, Officer is not present in court to be cross-examined; therefore, a judge will not be able to make the determination that Officer is competent to testify as an expert witness. While the skidmarks and the interviews may provide a sound basis to establish that Dan caused the accident, Officer has not been qualified as an expert, therefore, the evidence is inadmissible.

As a result, the police report will only be admissible as to the contents of the bystander’s comments, but not as to Officer’s conclusion and its basis.

4. Hank’s greeting card

Relevance

The statement is relevant because it establishes that Paula was in a hurry on the way home, and as a result may have been driving too quickly. Further, the greeting card is probative in establishing that Paula was at fault in the accident.

Authentication

All physical evidence must be authenticated in order to be admissible. Here, the paramedic testified that she recognized the greeting card as the same greeting card that

45 she found in Paula’s pocket. Therefore, the greeting card has been properly authenticated as belonging to Paula.

However, the note in the greeting card also must be authenticated to establish that it was indeed Hank who wrote the note. Circumstantial evidence can establish such authentication. The court may find that because it was found in Paula’s pocket while she was being treated, and was signed by a man with the same name as her husband, Hank. Therefore, the note in the card has been properly authenticated.

Hearsay

Paula could argue that the note should be excluded because it is inadmissible hearsay. However, Dan could argue that the statement in the note is not being offered for the truth of the matter. It is not being introduced to show that Paula was getting an early start on the weekend trip, but rather to show that Paula was on notice that she needed to hurry, and to show the effect on the hearer (Paula) upon hearing that she had to get an early start on her weekend. Therefore, the statement is non-hearsay because it is not being offered to prove the matters stated therein, but rather to show the effect of the card on Paula.

Dan could further argue that the statement is an admission by a party opponent. However, the statement was made by Hank, and not Paula, and, therefore, this exception will not apply.

5. Wilma’s note

Relevance

The note is highly relevant because it establishes that Paula was speeding during the accident, and thus was negligent. Further, it is probative to the issue of

46

Paula’s fault, and this probative value would outweigh any prejudicial impact that the note would have.

Authentication All real evidence must be authenticated in order to be presented in court. Here, Dan will likely authenticate the note as the same note that he received while he was waiting for the ambulance.

Reliability

Even if a court believes that Wilma saw the whole thing, the statement in the note is inadmissible lay opinion. Lay opinion must be 1) helpful to the jury, 2) based on the person’s perception, and 3) the opinion is rationally related to the perception.

Here, Wilma is making a legal conclusion as to Paula’s negligence. A layperson cannot testify as [to] legal conclusions such as negligence. Therefore, Wilma’s statement as to Paula’s negligence will be inadmissible as inadmissible lay opinion.

Hearsay

The note would also be inadmissible hearsay because it is an out-of-court statement that is being offered to prove the matters stated therein, that Paula was speeding and that Paula was negligent. The note may be admissible if it falls under any of the recognized exceptions to the hearsay rule.

Excited utterance

There are no facts indicating that Wilma wrote this note when she was under the stress of having viewed the accident. Further, it is unclear how much time had passed since the accident had occurred and Wilma wrote the note. Therefore, the statement in the note would not qualify as an excited utterance.

47

Present Sense Impression

As stated above, California only recognizes a present sense impression to the extent that it describes the declarant’s conduct. Here, Wilma is describing Paula’s conduct therefore, this exception will not apply.

48

JULY 2009

ESSAY QUESTIONS 4, 5, AND 6

California Bar Examination

Answer all three questions. Time allotted: three hours

Your answer should demonstrate your ability to analyze the facts in question, to tell the difference between material and immaterial facts, and to discern the points of law and fact upon which the case turns. Your answer should show that you know and understand the pertinent principles and theories of law, their qualifications and limitations, and their relationships to each other. Your answer should evidence your ability to apply law to the given facts and to reason in a logical, lawyer-like manner from the premises you adopt to a sound conclusion. Do not merely show that you remember legal principles. Instead, try to demonstrate your proficiency in using and applying them. If your answer contains only a statement of your conclusions, you will receive little credit. State fully the reasons that support your conclusions, and discuss all points thoroughly. Your answer should be complete, but you should not volunteer information or discuss legal doctrines which are not pertinent to the solution of the problem. Unless a question expressly asks you to use California law, you should answer according to legal theories and principles of general application.

49

THE STATE BAR OF CALIFORNIA OFFICE OF ADMISSIONS 180 HOWARD STREET • SAN FRANCISCO CALIFORNIA 94105 1639 • (415) 538 - 2303 1149 SOUTH HILL STREET • LOS ANGELES CALIFORNIA 90015-2299 • (213) 765 – 1500

ESSAY QUESTIONS AND SELECTED ANSWERS JULY 2010 CALIFORNIA BAR EXAMINATION

This publication contains the six essay questions from the July 2010 California Bar Examination and two selected answers to each question.

The answers received good grades and were written by applicants who passed the examination. The answers were prepared by their authors, and were transcribed as submitted, except that minor corrections in spelling and punctuation were made for ease in reading. The answers are reproduced here with the consent of their authors.

Question Number Contents Page

1 Torts 3

2 Professional Responsibility 22

3 Evidence 34

4 Business Associations 54

5 Criminal Law and Procedure 64

6 Community Property 78

1

Question 3

David and Vic were farmers with adjoining property. They had been fighting for several years about water rights.

In May, Vic and his wife, Wanda, were sitting in the kitchen when Vic received a telephone call. During the call, Vic became quite angry. As soon as he hung up, he said the following to Wanda: “That rat, David, just called and told me that he was going to make me sorry! He used some sort of machine to disguise his voice, but I know it was him!”

In June, Wanda and Vic passed a truck driven by David, who made an obscene gesture as they drove by. Vic immediately stopped and yelled that if David wanted a fight, then that was what he was going to get. Both men jumped out of their trucks. After an exchange of blows, David began strangling Vic. Vic collapsed and died from a massive heart attack. David was charged with manslaughter in California Superior Court.

At David’s trial, the prosecution called Wanda, who testified about Vic’s description of the May telephone call.

During cross-examination of Wanda, the defense introduced into evidence a certified copy of a felony conviction Vic had suffered in 2007.

The prosecution then introduced into evidence a certified copy of a misdemeanor simple assault conviction David had suffered in 2006.

During the defense’s case, David claimed that he acted in self-defense. He testified that he knew about two other fights involving Vic. In the first, which took place four years before his death, Vic broke a man’s arm with a tire iron. In the other, which occurred two years before his death, Vic threatened a woman with a gun. David testified that he had heard about the first incident before June, but that he had not heard about the second incident until after his trial had commenced.

Assuming that all appropriate objections were timely made, should the California Superior Court have admitted:

1. Wanda’s testimony about Vic’s statement regarding the May phone call? Discuss. 2. The certified copy of Vic’s 2007 felony perjury conviction? Discuss. 3. The certified copy of David’s 2006 misdemeanor simple assault conviction? Discuss. 4. David’s testimony about the first fight involving Vic breaking another man’s arm with a tire iron? Discuss. 5. David’s testimony about the second fight involving Vic threatening a woman with a gun? Discuss.

Answer according to California law.

34

Answer A to Question 3

1. Wanda’s testimony about Vic’s statement concerning the May Phone call:

Logical and Legal Relevance For evidence to be admissible it must be relevant which, under California law, is any evidence that has any tendency to make any fact of consequence, that is at issue, more or less probable than it would be without such evidence. In this case, Wanda’s testimony concerning the phone call is relevant, in that it goes to show that David’s intent to hurt Vic in some way prior to the June fight, a fact that is at issue, since David is claiming he acted in self-defense when he killed Vic.

Under Proposition 8 of the California Constitution (hereafter Prop. 8), any evidence that is relevant may be admitted in a criminal case. However, Prop. 8 makes an exception for balancing under California Evidence Code (hereafter CEC) 352, which gives a court discretion in excluding relevant evidence if its probative value is substantially outweighed by a risk of unfair prejudice, confusion of issues, or misleading the jury. In this case, the evidence has significant probative value, as it tends to show that David had a preexisting intent to hurt Vic and thus makes it more likely than not that he, not Vic, was the initial aggressor in the June fight that led to Vic’s death. There is no indication that such evidence poses a risk of unfair prejudice, confusion of issues, or misleading the jury, and as a result, the evidence would not be barred by CEC 352.

Personal Knowledge A witness may only testify as to those matters to which she has personal knowledge, in that she must have perceived the matter in some manner, such as by hearing or observing it. In this case, Wanda personally heard Vic’s statement concerning the phone call, and as a result, she has sufficient personal knowledge to testify.

Authentication All evidence must be authenticated, in that it must be proven to be what it purports to be. In this case, the authenticity of the phone call – namely, whether David was the person who actually made the call – comes into question, given that Vic stated David 35 was using some machine to disguise his voice. To authenticate a phone call, the person hearing it must be shown to have some familiarity with the speaker’s voice, which can be gained either from prior interactions before the trial or subsequent to the trial. In this case, David and Vic had been fighting for several years about water rights, and thus it would be likely that Vic was familiar with the sound of David’s voice. As a result, he would be qualified to make an identification of David’s voice over the phone. As a result, Vic’s statement concerning the phone call would be properly authenticated for purposes of trial.

Hearsay A statement is hearsay if it is made out-of-court and being offered to prove the truth of the matter asserted. In this case, Wanda’s statement contains two pieces of hearsay: 1) Vic’s statement made to her, and 2) David’s statements to Vic over the phone. Both are being offered to prove the truth of the matter asserted, in that Vic’s statement is being offered to show that David called him and Vic knew it was him despite the voice distortion, and David’s statement is being offered to show that David was planning to make Vic sorry.

In general, hearsay is inadmissible. However, the CEC does contain numerous exceptions to this general rule of hearsay inadmissibility that may allow these statements in. In a situation where a statement contains two levels of hearsay, such as here, both levels of hearsay must fall within an exception in order to be admissible.

Prop. 8 would not be sufficient to admit the evidence, as Prop. 8 contains an exception which requires hearsay rules to be satisfied before admitting relevant evidence.

David’s Statement to Vic: Admission of a Party-opponent: If the statement is made by one party to the case and is offered into evidence against him by the opposing party, it is an exception to the hearsay rule and is admissible. In this case, the person who made the statement is David, the party-opponent, and it is being offered against him by the prosecution. Thus, it would be admissible under the exception for statements of a party-opponent.

36

Statement Against Interest: A statement may also be admitted if it is mad by one party against their penal or pecuniary interest, and such party is unavailable. Here, David is available to testify, and there is no indication that he made the statement knowing that it was against his penal interest to do so; thus, the statement would not qualify under this exception.

Then-existing State of Mind: A statement may be admissible to show the party’s then-existing state of mind at the time the statement was made. In this case, Wanda can argue that the statement shows David’s existing state of mind at the time, namely, that he was going to make Vic sorry and intended to act on his statement. If the court finds this to be accurate, the statement would be admissible.

Vic’s Statement to David: Contemporaneous Statement: A hearsay statement is admissible if it is made describing or explaining certain conduct of the declarant while the declarant is engaged in such conduct. In this case, while the statement does describe Vic’s conduct, namely, that he was just on the phone with David, Vic made the statement about the phone call only after he had hung up, not while he was actively listening to David. Thus, the statement was not contemporaneous with Vic’s action and would not be admissible under this exception.

Excited Utterance: A hearsay statement is also admissible if it describes an exciting or startling event or condition and is made while the person is still under the stress of excitement from an event or condition. In this case, the facts indicate that Vic became quite angry during the call, thus indicating the call itself was a startling event or condition. In addition, given David’s particular statements to Vic during the call, namely, that he meant to make Vic sorry, a court most likely would find this to be a startling event or condition. Vic’s statements about the call were made to Wanda as soon as he hung up, thus indicating that he was still under the stress of the phone call – furthermore, the statements are followed by exclamation points, implying that he was still agitated from it.

37

Therefore, the statement would qualify as an excited utterance, and would be admissible.

Thus, in conclusion, the court did not err in admitting Wanda’s statement.

2. Certified Copy of Vic’s 2007 Felony Perjury Conviction:

Logical and Legal Relevance The evidence of Vic’s conviction is logically relevant to the case, as it goes to show Vic’s character for truthfulness, and thus would be used to impeach his statements to Wanda above concerning the telephone call, indicating that David did not make the call or have the intent to hurt Vic. Further, David’s preexisting intent to hurt Vic is in dispute, since David is claiming he acted in self-defense and was not the initial aggressor. Thus, the evidence is logically relevant.

The prosecution could argue that the evidence is inadmissible under CEC 352, on the grounds that it would mislead the jury by making them think that Vic’s character for truthfulness is relevant to whether he started the fight or not. However, it is unlikely a court would find that a reasonable jury would make this inference, given that the conviction was for perjury, not for a crime of violence, and it is being offered during the cross-examination of Wanda, thus indicating that it is meant to attack Wanda’s testimony, not Vic’s character for violence as a whole. Furthermore, the evidence has substantial probative value, as it tends to show that Vic is not truthful, and was therefore lying about the phone call from David – thus making David’s self-defense argument more probable. Therefore, the evidence would not be barred by CEC 352.

Character Evidence Character Evidence is any evidence offered to show that a person acted in conformity with character on a particular occasion, and is generally inadmissible. Here, the evidence of Vic’s prior conviction is being offered to show Vic’s action in conformity with character – namely, his character for lying – and thus would ordinarily be inadmissible. However, evidence of a witness’s or declarant’s character for truthfulness can be

38 offered for the purposes of impeachment to attack the witness’s or declarant’s credibility on the stand. Therefore, the evidence would not be inadmissible character evidence.

Impeachment Any party is permitted to impeach a witness in order to diminish his or her credibility for speaking the truth. In addition, a declarant, or out-of-court speaker, may be impeached in the same manner that a testifying witness may be impeached. Here, as the evidence goes to show Vic’s – the declarant in Wanda’s testimony – character for truthfulness, it would be permitted into evidence.

Under California law, the court has the discretion to allow in evidence of prior felony convictions for the purposes of impeaching if such convictions are for crimes of moral turpitude. In this case, the conviction is for perjury, or lying on the stand, which is a crime of moral turpitude, and thus the court would have the discretion to admit it for purposes of impeachment. In addition, prior convictions can be admitted in the evidence either through cross-examination or extrinsic evidence. Here, the conviction was introduced during cross-examination, but by means of extrinsic evidence – namely, the certified copy of the conviction, and therefore is a permissible means of impeachment.

Hearsay The conviction is hearsay, in that it is an out-of-court statement offered to prove the truth of the matter asserted, namely, that Vic was convicted for felony perjury in 2007. However, a judgment of a prior felony conviction is an exception to the general hearsay rule, and would thus be admissible.

In conclusion, the court did not err in admitting the conviction.

3. Certified Copy of David’s 2008 Assault Conviction:

Logical and Legal Relevance The evidence is logically relevant for two purposes – first, it goes to show that David had a character for violence, and thus acted in conformity with such character during

39 the June fight, thus negating his claim of self-defense. In addition, the evidence can be used to impeach David’s credibility on the grounds that his prior conviction speaks to his ability for truthfulness.

However, the evidence would be subject to CEC 352, particularly, the possibility of unfair prejudice. In this case, the evidence is being used to show action in conformity with character, which is an impermissible character inference and would unfairly prejudice David. In addition, as will be demonstrated, the use for impeachment is impermissible. As there is no other probative value attached to the statement, it would be inadmissible under CEC 352 for being unduly prejudicial.

Character Evidence As stated, character evidence is any evidence offered to show that a person acted in conformity with his character on a particular occasion. In a criminal case, such evidence cannot be offered by the prosecution unless the defendant “opens the door;” in other words, the defendant must put his character at issue, and the prosecution can only then rebut with character evidence. In this case, David had not yet opened the door to his character – while he did plead self-defense, it was only after the prosecution offered his assault conviction into evidence, not before. Therefore, the prosecution could not admit such evidence prior to David’s opening the door, and the evidence should have been ruled inadmissible.

Proposition 8 would not be applicable, as it contains an exception for the rules concerning character evidence.

Impeachment Under California law, a witness can only be impeached with a misdemeanor conviction if it is one of moral turpitude – otherwise, it is inadmissible. In this case, the conviction was for simple assault, which is not a crime of moral turpitude. As a result, it would be admissible.

Thus, the court erred in admitting the prior felony conviction.

40

4. David’s Testimony about the First Fight:

Logical and Legal Relevance The evidence is logically relevant, in that it goes towards David’s self-defense claim by showing Vic’s character for violence and thus indicating that Vic acted in conformity with character on this particular occasion – which is a fact at issue, since the prosecution claims that David was the initial aggressor, while David claims that Vic started the fight.

The evidence is also substantially probative, as it tends to show that Vic started the fight and thus makes David’s self-defense claim more likely than it would be without the evidence. However, it does carry a risk of unfair prejudice, in that it involves a character inference concerning Vic’s character for violence. However, as described below, the character evidence is permissible under the circumstances, and thus the evidence would not be inadmissible under CEC 352.

Character Evidence David’s introduction of Vic’s breaking a man’s arm with a tire iron is character evidence, as it is being used to show that Vic had a character for violence and acted in conformity with such character during the June fight. However, under the CEC, a criminal defendant can bring in evidence of the victim’s character for violence if he claims self- defense and wishes to show that the victim was the initial aggressor. As this is David’s purpose in bringing this evidence, since he is claiming self-defense and is brining in the evidence to show Vic’s initiation of the fight, the evidence would be admissible.

Character evidence can take the form of either reputation evidence, opinion evidence, or specific acts. Under the CEC, a defendant is permitted to use any of these methods in bringing in evidence of the victim’s bad character for violence during the direct examination. Here, David’s testimony would constitute specific acts, as he is testifying to specific acts that Vic had done in the past. Therefore, the method of character evidence used is permissible.

41

Personal Knowledge In this case, David does not have personal knowledge as to the fight. While he heard about it from someone before June, he did not personally witness it, nor is there any indication as to who he heard it from, for example, whether the person who told him was the other man involved in the fight whose arm was broken, or was from someone else. Thus, there is no indication that he has personal knowledge as to the fight, and as a result, the testimony would not be admissible.

Thus, the court erred in permitting David’s testimony into evidence.

5. David’s Testimony about the Second Fight:

Logical and Legal Relevance The evidence is logically relevant, in that it, like the testimony about the first fight, goes towards David’s self-defense claim by showing Vic’s character for violence and his action in conformity with such character on this particular occasion – a fact at issue in this case. The evidence is also substantially probative, as it tends to show that Vic, not David, started the fight and makes David’s self-defense claim more likely. In addition, as will be demonstrated below, the use of such evidence is a permissible use of character evidence, and as a result, the testimony would not be barred by CEC 352.

Character Evidence As with the first fight, David’s introduction of Vic’s prior threatening a woman with a gun is character evidence, as it is being used to show that Vic had a character for violence and acted in conformity with such character during the June fight. Yet, as indicated above, a criminal defendant can bring in evidence of the victim’s character for violence if he claims self-defense and wishes to show that the victim was the initial aggressor – which is the case here, as David is claiming self-defense and wishes to show that Vic was the initial aggressor.

As with the testimony above, this testimony takes the form of specific acts, as David is testifying as to specific violent acts that Vic took in the past, and thus is a permissible use of character evidence.

42

Personal Knowledge Here, David again does not have substantial personal knowledge to testify as to the fight. He only heard about it from someone else, and there is no indication as to whom; he did not actually perceive it himself nor hear about it directly from the victim or someone who saw it occur. Furthermore, he did not hear about the second incident until after his trial had commenced, thus running the possible risk of such evidence not being particularly reliable or truthful and being created solely for the purposes of trial. As a result, David lacked sufficient personal knowledge to testify as to the second incident, and the court erred in permitting the evidence to be admitted.

43

Answer B to Question 3

CA Constitution Truth-in-Evidence Provision In California, evidentiary rules in criminal cases are sometimes changed by the Truth-in- Evidence Provision of the California Constitution. The Truth-in-Evidence provision generally provides that all relevant evidence is admissible in California criminal trials. As state constitutional law, the Truth-in-Evidence provision overrides any contrary California Evidence Code provisions. However, the Truth-in-Evidence provision itself explicitly preserves numerous rules of the California Evidence Code, including the rule against hearsay and the CEC 352 Balancing Rule. With this general framework in mind, we can discuss the individual evidentiary items.

Wanda’s Testimony About Vic’s Statement Regarding the May Phone Call Logical/Legal Relevance Irrelevant evidence is never admissible. In California, evidence is logically relevant if it has a tendency to make a disputed fact of consequence more or less probable. However, even if evidence is logically relevant, it may still be excluded at the discretion of the court if the court finds that the probative value of the evidence is substantially outweighed by concerns of prejudice, confusion or delay. Neither the basic rule governing relevance nor the balancing rule are changed in criminal trials by Proposition 8.

Here, Vic’s statement that David planned to “make [him] sorry” is relevant because it tends to prove that David and Vic were in a feud and that David intended to hurt Vic. Thus, it tends to make more probable that David committed the later violence and strangulation to Vic. However, the fact David attacked Vic does not appear to be in dispute, because David is claiming he acted in self-defense. Thus, it is likely that Vic’s statement about the phone call is not relevant under California standards.

If it is logically relevant, it will not be excluded. The evidence is probative of David having committed intentional violence against Vic, and there is no substantial risk of unfair prejudice.

44

Personal Knowledge Wanda can only testify as to matters for which she has personal knowledge. Here, Vic told Wanda about the phone call directly; thus she personally perceived the statement by Vic and can testify about it.

Hearsay Hearsay is an out-of-court statement that is offered to prove the truth of the matter asserted. Hearsay is not admissible unless an exception to the hearsay prohibition applies. Moreover, where a statement contains multiple levels of hearsay, a hearsay exception must apply to each level for the statement to be admissible.

Vic’s Statement In this case, Vic’s statement that David called and said he would make Vic sorry is hearsay. Vic is making this statement to prove the truth of the matter asserted, i.e., that David did call and threaten Vic.

Vic’s hearsay statement, however, is likely admissible as a spontaneous statement. Under the CEC, a hearsay statement made describing a startling event while still under the stress of excitement is an exception to the hearsay prohibition. In this case, Vic described the phone call to Wanda immediately after receiving it. Moreover, the evidence indicates that Vic was still in a state of anger and excitement after receiving the phone call. Thus, Vic’s statement is a spontaneous statement.

The prosecution may also claim that Vic’s statement was a contemporaneous statement. The contemporaneous statement exception applies to hearsay statements made by a declarant to describe his conduct contemporaneously to or immediately following his actually doing it. However, in this case, Vic’s statement describes David’s conduct, not his own, and thus would not fit within the contemporaneous statement exception.

45

David’s Statement David’s statement that he would make Vic sorry is also an out-of-court statement. Moreover, it is also offered to prove the truth of the matter asserted in that it is intended to prove that David did intend to make Vic sorry.

David’s statement is admissible under the present state of mind exception. The present state of mind exception applies to statements by a declarant that describe the declarant’s state of mind at that time. The exception can be used to admit statements of the declarant’s intent in order to prove that the declarant carried out that intent. In this case, David’s statement that he “was going to make [Vic] sorry” was a statement of David’s present intent and thus fits within the present state of mind exception. It is thus admissible to prove that David later carried out actions to make Vic sorry.

David’s statement may also be a spontaneous statement. However, there is no indication that David was in a state of excitement, especially considering he initiated the call. Thus, this exception likely does not apply.

Accordingly, Vic’s statement is admissible hearsay because both his statement and David’s fit within hearsay exceptions.

Authentication of David’s Statement David’s alleged statement, however, can only be admissible if properly authenticated. To be authenticated, there must be sufficient evidence for a jury to find that David’s statement is what it was purported to be. In this case, Vic’s statement indicates that the caller used a voice-changing device, calling into possible doubt whether David actually called. However, given Vic’s belief that it was David that had called, and evidence of the feud between them, there is probably sufficient evidence for a jury to find David made the call. Thus David’s statement is authenticated.

Spousal Privileges David may claim that the evidence is not admissible because of spousal privileges. However, the spousal testimonial immunity only allows a current spouse to choose to refuse to testify against her husband. Moreover, although confidential marital

46 communications made during marriage are protected by privilege, this privilege is only held by either spouse, not an outside party. Thus, even though Vic’s statement to Wanda was a confidential marital communication, only Vic or Wanda could assert the benefit of the privilege.

Confrontation Clause Issues The confrontation Clause of the federal Constitution forbids the use of otherwise admissible testimonial hearsay evidence against a defendant if the defendant did not have an opportunity to cross-examine the hearsay declarant. “Testimonial” statements are those concerning a past event that are made to incriminate the defendant.

In this case, Vic’s statement about David is likely not “testimonial” because it was not made to police or concerning a past event. Thus, it was not a statement that was made for the purposes of incriminating David and the Confrontation Clause will not apply.

Conclusion Vic’s statement should not have been admitted because it was irrelevant, but otherwise it would be admissible hearsay.

Certified Copy of Vic’s 2007 Felony Perjury Conviction Relevance Vic’s felony perjury conviction tends to prove that Vic’s statement may have been a lie, negating [a] possible motive by David to attack Vic and strengthening his claim of self- defense. However, it is unclear whether there is any dispute about the veracity of Vic’s statement, and thus it may not be relevant under California law. Assuming, however, that the fact of the phone call is in dispute, then Vic’s prior conviction is relevant.

Authentication The copy of the conviction must be authenticated. However, under the CEC, certified copies of public records are self-authenticating, meaning that the document itself provides sufficient evidence for a finding that it is genuine, and no additional foundational evidence is necessary.

47

Hearsay – Public Records Exception The copy of Vic’s conviction is hearsay because such a document is an out-of-court statement offered to prove the truth of its contents, i.e., that Vic was convicted of perjury. However, factual records made by public officials in the regular course of their duties are excepted from the hearsay prohibition. Records of convictions are made in the regular course of public officials’ duties and thus are admissible hearsay as public records.

Character Evidence/Impeachment Evidence of a victim’s character to prove the victim acted in conformity with that character is generally inadmissible in a criminal trial. However, such evidence is permissible if first introduced by the defense or for the purpose of impeaching the victim. Moreover, Proposition 8 allows for the admissibility of the victim’s character in a criminal trial wherever relevant, subject to balancing. Moreover, a hearsay declarant can be impeached by any applicable method.

In this case, the evidence was both introduced by David and to impeach Vic, so it is admissible either because David “opened the door” or because it is impeachment evidence.

Use of Conviction However, a conviction can only be used for impeachment purposes under the CEC if the conviction is for a felony involving a crime of moral turpitude. Proposition 8 broadens this rule for criminal trials by allowing in any relevant convictions, which include misdemeanors involving a crime of moral turpitude.

In this case, Vic’s conviction was for a felony involving a crime of moral turpitude, perjury, and thus was admissible to impeach Vic’s statement.

Conclusion The conviction was properly admitted as allowable impeachment evidence.

48

Certified Copy of David’s 2006 Misdemeanor Simple Assault Conviction Relevance Evidence of David’s misdemeanor assault conviction is relevant because it tends to prove that David was an aggressive individual and may have been the aggressor in the fight against Vic. This does concern a fact of consequence that is in dispute because it undermines David’s claim of self-defense.

However, this evidence may be excluded because of its prejudicial effect. By introducing evidence of David’s conviction for a violent crime, there is a risk that the jury will decide to punish David because of this past crime or “criminal character” rather than the conduct at issue in this case. Thus, the court should have excluded this evidence because of the risk of unfair prejudice.

Authentication As with Vic’s conviction copy, David’s conviction copy is a self-authenticating document.

Hearsay The certified copy of David’s conviction is admissible under the public records exception for the reasons discussed above.

Character Evidence Generally, evidence of a defendant’s character cannot be introduced to prove the defendant acted in conformity unless first introduced by the defendant. However, where the defendant has introduced evidence that the victim has a character for violence, California law permits the prosecution to introduce evidence of the defendant’s same character trait for violence.

In this case, the prosecution may be introducing David’s prior conviction as evidence that David had a character for violence and acted in conformity on the particular occasion when he attacked Vic in June. This would be an inadmissible use of the conviction because at this point in the trial, David had introduced no evidence regarding his own character or evidence that Vic had a character for violence. However, because

49 the defendant later testified about Vic’s prior fights, the error of admitting evidence of David having a trait for violence was harmless.

The Truth-in-Evidence Provision does not change the rules regarding character evidence about a criminal defendant.

Impeachment by Conviction As discussed above, misdemeanor convictions cannot be used to impeach a witness or party. However, because of the Truth-in-Evidence provision, misdemeanors involving crimes of moral turpitude are relevant impeachment evidence.

In this case, the defendant has not yet testified, so it was improper for the prosecution to introduce the conviction in order to impeach him. Moreover, a conviction for simple assault is not a crime of moral turpitude because it does not involve lying or similar immoral conduct. Thus, the conviction is not admissible for impeachment purposes.

Other Purposes The conviction may be used for non-character and non-impeachment purposes, however. Conviction evidence can be used if it is relevant to establishing the defendant’s motive, intent, and absence of mistake, or other relevant non-character issues.

In this case, David’s prior assault conviction does not appear to be relevant for any purpose besides proving that David was a violent individual. Thus, there are no other purposes for which it may be admissible.

Conclusion David’s conviction should not have been admitted because of its prejudicial effect.

50

David’s Testimony About First Fight Relevance David’s testimony about Vic’s first fight involving the tire iron is relevant because it tends to prove that David reasonably believed Vic was violent and thus David’s actions were reasonable self-defense. The fact of David’s self-defense is in dispute.

Personal Knowledge David cannot testify on matters to which he does not have personal knowledge. Here, David is claiming that he knew about the fight, however, and thus may have had personal knowledge about Vic’s prior fight.

Character Evidence As discussed above, the defendant can open the door to prove the victim’s character. Thus, David could properly introduce evidence of Vic’s character to prove that Vic acted in conformity with that character by attacking David on the occasion at issue.

Other Purposes Furthermore, the evidence is also relevant to showing David’s reasonable belief that he was in danger.

Conclusion David’s testimony about Vic’s first fight was properly admitted.

David’s Testimony About Second Fight Relevance David’s testimony about Vic’s second fight also tends to prove Vic was an aggressor. However, its probative value is likely substantially outweighed by unfair prejudice because it tends to show that Vic is a violent individual and thus may have deserved David’s strangulation even if it wasn’t in self-defense. The probative value is limited because David did not know about this fight before his fight with Vic, and thus it cannot be probative of David’s belief regarding Vic’s nature.

51

Personal Knowledge David likely did not have personal knowledge of this incident, and thus it should not have been admitted on these grounds too.

Character Evidence David could open the door on character evidence regarding Vic.

Conclusion This evidence should not have been admitted because of its unfairly prejudicial impact.

52

JULY 2010 ESSAY QUESTIONS 4, 5, AND 6

California Bar Examination

Answer all three questions. Time allotted: three hours

Your answer should demonstrate your ability to analyze the facts in question, to tell the difference between material and immaterial facts, and to discern the points of law and fact upon which the case turns. Your answer should show that you know and understand the pertinent principles and theories of law, their qualifications and limitations, and their relationships to each other. Your answer should evidence your ability to apply law to the given facts and to reason in a logical, lawyer-like manner from the premises you adopt to a sound conclusion. Do not merely show that you remember legal principles. Instead, try to demonstrate your proficiency in using and applying them. If your answer contains only a statement of your conclusions, you will receive little credit. State fully the reasons that support your conclusions, and discuss all points thoroughly. Your answer should be complete, but you should not volunteer information or discuss legal doctrines which are not pertinent to the solution of the problem. Unless a question expressly asks you to use California law, you should answer according to legal theories and principles of general application.

53

THE STATE BAR OF CALIFORNIA OFFICE OF ADMISSIONS 180 HOWARD STREET • SAN FRANCISCO CALIFORNIA 94105 1639 • (415) 538 - 2303 1149 SOUTH HILL STREET • LOS ANGELES CALIFORNIA 90015-2299 • (213) 765 – 1500

ESSAY QUESTIONS AND SELECTED ANSWERS FEBRUARY 2011 CALIFORNIA BAR EXAMINATION

This publication contains the six essay questions from the February 2011 California Bar Examination and two selected answers to each question.

The answers received good grades and were written by applicants who passed the examination. The answers were prepared by their authors, and were transcribed as submitted, except that minor corrections in spelling and punctuation were made for ease in reading. The answers are reproduced here with the consent of their authors.

Question Number Contents Page

1 Wills and Succession 3

2 Constitutional Law 15

3 Real Property 25

4 Torts 36

5 Business Associations/ Professional Responsibility 52

6 Remedies/Evidence 65

1

Question 6

Green’s Grocery Outlet (“Green’s”) sponsors a lawful weekly lottery. For one dollar, a player picks six numbers. All persons who select the six winning numbers drawn at random share equally in the prize pool.

Each week, for the past two years, Andrew has played the same numbers—3, 8, 10, 12, 13, and 23—which represent the birth dates of his children.

On June 1, Andrew purchased his weekly lottery ticket. Barney, a clerk employed by Green’s, asked, “The usual numbers, Andrew?” Andrew replied, “Of course.”

Barney entered the numbers on the computer that generates the lottery ticket and gave the ticket to Andrew. Without examining the ticket, Andrew placed it in his pocket. Unbeknownst to either Andrew or Barney, Barney had accidentally entered the number “7" on the computer rather than the number “8.”

The winning lottery numbers that week were Andrew’s “usual” numbers. Much to his horror, Andrew discovered Barney’s error when he showed his wife the “winning” ticket. Andrew filed suit against Green’s seeking to reform his lottery ticket by changing the “7" to an “8.” Green’s cross-complained seeking rescission.

1. At trial, Green’s objects to Andrew’s testimony about (a) Barney’s question, (b) Andrew’s answer, and (c) Andrew’s attempt to explain what the phrase “the usual numbers” means. Should the court admit the testimony? Discuss. Answer according to California law.

2. How should the court rule on each party’s claim for relief? Discuss.

65

Answer A to Question 6

1. How will the court rule on Green’s objection to

a) Barney’s question “The Usual Numbers, Andrew”

Relevant All evidence must be logically and legally relevant.

Logical: Under California Rules of Evidence, evidence is relevant if it tends to prove or disprove a disputed fact. In this case, Green is disputing the fact that there is a contract or the terms of the contract. Therefore, Andrew’s testimony regarding Barney’s statement tends to prove that Andrew bought the ticket from Barney and that the terms were for the usual numbers. Andrew can show this is logically relevant.

Legal: To be legally relevant the probative value should outweigh the prejudicial effect. The probative value in this case is that this tends to show Andrew bought the ticket and that he had a usual set of numbers. While this may be prejudicial, the probative value is high and outweighs the prejudice because it establishes the facts of the situation.

Hearsay Green will object that the evidence is inadmissible hearsay. Hearsay is an out-of-court statement made by a declarant used to prove the truth of the matter asserted.

Out-of-Court Statement by a declarant In this case Barney’s question was made out-of-court and by Barney, therefore meeting this element.

Truth of the Matter Asserted The statements presented to prove what the statement is asserting. In this case Green will argue that Andrew is introducing Barney’s statement to show that Barney knew about the usual numbers and that Andrew asked for the usual numbers. 66

Act of Independent Legal Significances Andrew will argue he is not introducing to prove the truth of the matter asserted, but rather to show that there was a contract created when Andrew got the ticket. At this point this statement does not provide a contract.

Knowledge of facts stated Andrew may also be using it to prove that he always purchased the same numbers and that Barney knew about his practice or habit. It is likely that Andrew can show this is not hearsay, but being used to show Barney had the knowledge of his usual numbers.

Even if this is being introduced for the truth of the matter asserted Andrew can see if it falls under an exception to the hearsay rule.

Party-opponent admission Admissions by a party-opponent are an exception to the hearsay rule. Vicarious admissions by an agent are only attributed to the principal if the statement was made in the scope of the agency and the principal would be liable.

In this case Green will argue Barney made a mistake, but Barney was doing his job within the scope of the agency and principals are liable for the mistake of their agents.

Andrew can show this was a party-opponent admission.

Conclusion: Barney’s question is admissible evidence and the court should admit Andrew’s testimony on this issue.

67

a) Andrew’s answer

Relevant (see rule above) Logical: (See previous rule.) Green may argue that the creation of a contract is not in dispute and Andrew’s testimony only tends to prove the existence of a contract. Andrew will argue the testimony also refers to the question Barney asked and that he wanted his usual numbers. Andrew can likely show this is logically relevant because it tends to prove a disputed fact.

Legally: See previous rule: This is similar to the previous piece of evidence and tends to establish the facts of the incident and therefore the probative value outweighs the prejudicial effect.

Hearsay Green will object that this testimony is hearsay. See previous rule. Green will assert that this is an out-of-court statement by Andrew to prove that he assented to the purchase of the lottery ticket which is the contents of his statement.

Independent Legal Significance Andrew can show in this case as previously discussed that his statement created a contract and is therefore not being used to prove the truth of the matter asserted, but rather to prove the formation of a contract. Andrew’s assent in this case does form a contract and is therefore not hearsay.

Party-opponent Exception (See previous rule) In this case the statement is by Andrew and not a party-opponent because Andrew is testifying and Andrew is not the opponent against Andrew himself. So this exception does not apply.

68

Conclusion Andrew’s testimony about his own statement should be ruled admissible because it is not hearsay and is relevant.

b) Andrew’s explanation of “usual numbers” Relevant: Logical: This is the issue in dispute. Therefore Andrew’s testimony is highly relevant.

Legal: In this instance, this testimony is highly prejudicial to Green and therefore might be excluded. However it is also the main issue of the case and its probative value outweighs its prejudicial effect.

Character Evidence Evidence of a person’s character cannot be used to show they acted in conformity therewith on a particular occasion.

In this case Green will argue that the introduction of this evidence is trying to show Andrew acted similarly as he had on other occasions.

Habit Evidence that shows specific instances of conduct to prove that they have a regular habit are allowed. Andrew will argue that in this case he is establishing a habit he has had every week for the past 2 years. Andrew can likely show this is habit evidence and not character.

Parol Evidence Green may argue that the evidence violates the parol evidence rule because it is evidence prior to formation of an integrated contract to contradict the terms of that contract.

69

Andrew will likely be able to introduce this because he is trying to show a mistake and not to contradict the terms of an integrated contract. In this case there was a mistake Barney made and Andrew is trying to prove the mistake.

Conclusion The court should rule that this evidence is admissible.

2. How should the court rule on each party’s claim for relief?

Reform The court will grant reformation of a contract when each party knew what the terms were and they both had the same mutual mistake.

Green will argue that Andrew had the opportunity to look at the ticket and negligently failed to do so and therefore assumed the risk of the ticket being wrong. Andrew will argue the prior course of dealing with Barney and Green establishes that lottery ticket was supposed to contain a seven instead of an eight.

Recission The court will assert recission when there is evidence the contract was not valid or lacked assent on a material term.

Green will make the same argument that there was no meeting of the minds and as such the contract should be rescinded. Andrew will argue that this was just a transcription error and does not rise to a level warranting recission of the contract.

Conclusion The court should reform the contract because there is evidence that the mistake was mutual, but the mistake was a transcription rather than the objective belief of the parties. Both Barney and Andrew thought that the ticket should contain one number eight and not seven. The court should reform the contact.

70

Answer B to Question 6

(1) Green's (G) objections to Andrew's (A) Testimony

(a) A's testimony re Barney's (B's) question

Green will object to A's testimony re B's question as irrelevant and inadmissible as hearsay.

Under California law, evidence is relevant if it has any tendency to make a disputed fact of consequence to the action more or less likely to be true. In this case, A is suing Green for breach of contract, and there is a dispute between the parties as to the terms of that contract (i.e., the lottery numbers A picked). As a result, A's testimony about B's question is relevant because it goes to whether A & B agreed about the numbers that should be on A's lottery ticket, and if so, what A & B agreed to, both of which are disputed facts in this case.

Under California law, a relevant statement may nonetheless be excluded if it is substantially more prejudicial than probative, a waste of time, or likely to confuse the jury. The probative value of B's question here outweighs any potential prejudice or confusion.

Under California law, hearsay is an out-of-court statement offered for the truth of the matter asserted. In this case, B's question to A is an out-of-court statement because it was made before the suit on the day that A bought the lottery ticket in question. But A will argue, persuasively, that he is not offering B's question for the truth of the matter asserted. A will argue that he is offering B's statement to establish a verbal act -- the fact that B asked A the question, "The usual numbers, Andrew?" As such, the statement is being offered for a non-hearsay purpose because it is not being offered to prove the truth of the matter that Andrew asked for the usual numbers.

71

A could also argue that B's question should be admitted for the truth of the matter because B's question shows B's then-existing mental condition, an exception to the hearsay rule. A will argue, persuasively, that B's questions shows that B knew that A wanted A's usual numbers.

A could also argue that B's question is offered for the effect it had on A, the listener, another non-hearsay purpose. Under this argument, A is offering B's question to show that A inferred from B's statement that B knew A's usual numbers.

A could also argue that B's statement is admissible hearsay in California because it is an admission of a party. Green will argue that B is not a party to the case, but A can persuasively respond that Green should be bound by B's statements because B was acting within the scope of his employment when he made them, i.e., part of B's job is to sell lottery tickets to customers.

(b) A's testimony re A's answer

B will argue that A's answer is irrelevant and inadmissible hearsay.

A will argue that his answer is relevant because it goes to the disputed facts of whether A & B agreed to the numbers in A's lottery ticket, and what those numbers were. Moreover, A will argue that his answer has great probative value because [it] is directly related to a key disputed fact in the case, i.e., what numbers A & B agreed to put in A's lottery ticket. A's answer is relevant for those reasons.

B will argue that A's statement was made out of court -- on June 1 -- and is being offered to prove the truth of the matter asserted, that A asked for his usual numbers.

A will also argue, persuasively, that his answer is not offered for hearsay purpose because he is not offering it for the truth of the matter asserted. Rather, it is being offered as a verbal act -- agreement to the offer from B. Alternatively, A could argue

72 that A's answer is being offered for the non-hearsay purpose of showing the effect on the listener B, i.e., that B understood that A wanted his usual numbers.

A's answer will be admissible on these grounds.

(c) Andrew's attempt to explain what "the usual numbers" means

B will argue that A is attempting to offer parol evidence regarding the terms of the contract in violation of the parol evidence rule.

The parol evidence rule excludes evidence extrinsic to a contract where that contract is considered a final, or integrated writing. There are exceptions to the parol evidence rule, including to show a clerical error.

Here Green will argue that any testimony regarding what "the usual numbers" means is extrinsic evidence because the lottery ticket is the contract, and there is no evidence within the ticket regarding what A's usual numbers are.

A will argue, persuasively, that parol evidence should be admitted in this case to prove that B made a clerical error in entering A's numbers into the computer that generated A's ticket, the contract. A's testimony on this point will be allowed under the clerical error exception to the parol evidence rule.

(2) The parties’ claims for relief

Reformation Reformation is an equitable remedy that is available where one party can show, among other things, a unilateral mistake of material fact that caused A irreparable harm.

In this case, A will argue that he is entitled to reformation because he suffered irreparable harm as a result of B's unilateral mistake -- a clerical error in entering his

73

usual lottery numbers. A will argue that Green should be bound by B's error because B is Green's agent and was acting within the scope of his employment at the time of B's mistake. And A will argue that he was irreparably harmed by B's mistake because but for B’s mistake he would have won the lottery, and that A's harm was foreseeable because only a ticket that has all the winning numbers will win the lottery, and it is foreseeable that a clerical error in entering one number could cause a party to lose a lottery he otherwise would have won.

Green will argue that A is not irreparably harmed, because Green can refund A the price of the lottery ticket, and that there was no mistake because the numbers A paid for are the numbers that are clearly printed on his lottery ticket. Moreover, Green will argue that A does not have clean hands, because he could have and should have confirmed that the right numbers were on his ticket, and that by failing to do so, A waived his right to complain after the fact that he got the wrong numbers.

Rescission Green will argue for rescission because there was no meeting of the minds as to a material term of the contract. Rescission is an equitable remedy available where one party can show, among other things, mutual mistake of fact. Here Green will argue that there was a mutual mistake of fact as to what numbers A wanted on his lottery ticket, and that therefore there was no meeting of the minds required to form a valid contract. Green will argue that B thought A wanted the number 7 on his ticket, and A wanted the number 8 on his ticket, and that the numbers on the ticket were material elements of the contract between Green and A. As a result, there was no meeting of the minds as to a material term of the contract, and the contract should be rescinded.

A will argue that there was a meeting of the minds based on the question and answer between B and A -- "The usual numbers, Andrew?" "Of course." A will argue that B's question shows that B knew A's usual numbers and offered A a ticket with those numbers. A will argue that A accepted B's offer of those numbers, and that there was

74

consideration in A's payment of the price of the lottery ticket and Green's promise to pay A the winnings if the numbers of A's ticket matched the winning numbers.

This is a close question, but in this case, because all of the testimony discussed above is admissible and support's A's position, a court would likely find that A is entitled to reformation and B cannot rescind the contract. A wins the lottery.

75

ESSAY QUESTIONS AND SELECTED ANSWERS FEBRUARY 2012 CALIFORNIA BAR EXAMINATION

This publication contains the six essay questions from the February 2012 California Bar Examination and two answers to each question that were written by actual applicants who passed the examination after one read.

The selected answers were assigned good grades and were transcribed for publication as submitted, except that minor corrections in spelling and punctuation were made for ease in reading. The answers are reproduced here with the consent of their authors.

Question Number Contents Page

1 Trusts 4

2 Constitutional Law 16

3 Evidence 29

4 Corporations 45

5 Professional Responsibility 58

6 Real Property 70

2 Question 3

Paul sued David in federal court for damages for injuries arising from an automobile accident.

At trial, in his case-in-chief, Paul testified that he was driving westbound, under the speed limit, in the right-hand lane of a highway having two westbound lanes. He further testified that his passenger, Vera, calmly told him she saw a black SUV behind them weaving recklessly through the traffic. He also testified that, about 30 seconds later, he saw David driving a black SUV, which appeared in the left lane and swerved in front of him. He testified that David’s black SUV hit the front of his car, seriously injuring him and killing Vera. He rested his case.

In his case-in-chief, David testified that Paul was speeding, lost control of his car, and ran into him. David called Molly, who testified that, on the day of the accident, she had been driving on the highway, saw the aftermath of the accident, stopped to help, and spoke with Paul about the accident. She testified further that, as soon as Paul was taken away in an ambulance, she carefully wrote down notes of what Paul had said to her. She testified that she had no recollection of the conversation. David showed her a photocopy of her notes and she identified them as the ones she wrote down immediately after the accident. The photocopy of the notes was admitted into evidence. The photocopy of the notes stated that Paul told Molly that he was at fault because he was driving too fast and that he offered to pay medical expenses for anyone injured. David rested his case.

Assuming that all appropriate objections and motions were timely made, should the court have admitted:

1. Vera’s statement? Discuss.

2. The photocopy of Molly’s notes? Discuss.

Answer according to the Federal Rules of Evidence.

28 Question 3 Answer A

I. VERA'S STATEMENT The first issue is whether or not Vera's statement to Paul claiming that the black SUV behind them was weaving recklessly through the traffic. Evidence is admissible if it is logically and legally relevant and not subject to any restrictions in the federal rules of evidence.

A. Relevance: Logical Relevance: Evidence is logically relevant if it tends to prove any fact of consequence in the trial more or less probable. Here, Paul is suing David for injuries arising from an automobile accident. A central issue in this case will be who was at fault for the automobile accident that caused the injuries. The fact that David drives a black SUV and the fact that Vera observed a black SUV weaving recklessly through traffic tends to prove that David was driving recklessly and therefore was at fault for the accident. This evidence is logically relevant.

Legal Relevance: If evidence is logically relevant than [sic] it also must be legally relevant. Legal relevance is determined by whether the evidence is more prejudicial than probative. This requires a balancing test. Here, the evidence is probative because as mentioned it illustrates how one of the parties in this case was driving before the accident. David will argue that it is prejudicial because Vera called him "reckless" and that this statement might cause a jury to cast judgment on his driving. A judge will determine that the probative value outweighs any slight prejudice this evidence may include and is therefore legally relevant.

A court may also exclude evidence that is not legally relevant because it would waste time or confuse the jury. However, this evidence does not require any additional time to be spent to prove additional elements and is not confusing to a jury.

29 B. Lay Opinion: David will argue that the statement should be inadmissible because it contains a lay opinion as to the nature in which he was driving his vehicle. Lay opinions are admissible evidence if they are (1) helpful to the jury and (2) do not require any special analysis. Here, if Paul is suing on a negligence theory, David might argue that Vera stating that he was driving recklessly is allowing the witness to testify as to an element of the cause of action. However, David will be successfully [sic] in arguing that Vera could easily see the car driving and that her expression that the car is driving recklessly is merely her opinion on how the driver was swerving through lanes. This evidence will be rendered inadmissible because it is a lay opinion.

C. Hearsay Paul will argue that Vera's statement is inadmissible because it is hearsay. Hearsay is an out-of-court statement offered to prove the truth of the matter asserted. As a general rule, hearsay is inadmissible because the validity of out-of- court statements is questionable and unreliable. Hearsay is inadmissible unless a valid exception applies. David will argue that the following exceptions apply:

(1) Present Sense Impression: A present sense impression is when someone makes a statement about an event they are perceiving at the moment. Present sense impressions are exceptions to the hearsay rule, because they are presumed to be reliable. When someone makes a present sense impression, they have no motivation to lie or misstate what is actually occurring. The facts state that just 30 seconds after Vera made this statement that a black SUV hit Here [sic], Vera simply stated at the time of observing the black SUV that she saw that SUV weaving recklessly through traffic. Therefore, it will be admissible as a present sense impression. (2) Present State of Mind: Another hearsay exception are statements made by individuals that express their current state of mind. Here, Paul will argue that when Vera made the comments about the SUV, she was expressing what she thought

30 and felt at the time. This statement would also be admissible under the Present State of Mind exception. (3) Excited Utterance: Paul may argue that the excited utterance exception applies as well. An excited utterances [sic] is a statement made at the time of a shocking or exciting event that is made before the shock or excitement as [sic] worn off. Here, David will argue that the swerving of an SUV was not a shocking or exciting event. Further, the facts state that Vera calmly told Paul about the SUV which illustrates that she was not under the shock or excitement of any event. Therefore, the excited utterance exception does not apply. (4) Prior Statement: Prior statements made by individuals that are unavailable to testify sometimes qualify as an exception to the hearsay rule. However, the federal rules of evidence require that the prior statement be made under oath in the course of some type of previous testimony. This statement was made in the car to Paul and is therefore not a valid exception under the prior statement rule. (5) Dying Declaration: Paul may attempt to argue that Vera's statement qualifies under the Dying Declaration exception. This exception states that under some circumstances, statements made under the impression of impeding death are valid exceptions to the hearsay rule. However, the federal rules of evidence state that these statements are only admissible in criminal homicide cases. Moreover, the statement was not made with the knowledge of impending death because the car had not been hit yet and Vera did not know that she might be dying soon. Therefore, it would not qualify under this hearsay exception. (6) Federal Catchall Exception: The federal rules of evidence also allow a catchall exception for statements that are made under circumstances of trustworthiness. Paul will argue that Vera did not have any motivation to lie or to make this information up because it happened at the time of the accident. He will also argue that because Vera is dead there is no other way for this evidence to be admitted for trial. The judge would likely not apply the federal catchall exception because the Present Sense Impression exception is a stronger argument, and you only need one valid exception to admit the evidence.

31 In conclusion, Vera's statement would be admissible evidence as a present sense impression.

II. PHOTOCOPY OF MOLLY'S NOTES The issue here is whether or not the photocopy of Molly's notes that state that Paul told her he was at fault because he was driving too fast and that he offered to pay medical expenses can be admitted into evidence.

A. Capacity to Testify: A witness may testify if she has personal knowledge of the event in question, she recalls the event in question, she has the ability to communication [sic] these perceptions, and she takes an oath to tell the truth. Here, Molly has personal knowledge of the facts perceived because she was there the day of the accident, saw what happened, and remembers that she took notes describing the day's events. While she does not recall the events at this moment, this can be satisfied in other ways that are discussed below. She has the ability to communicate and presumably took an oath prior to testimony.

B. Authentication of Document Before any documents or other types of recordings are entered into evidence, they must be authenticated and the proper foundation must be laid. Here, Molly has testified that she was there on the day of the accident and they [sic] she remembers that she carefully wrote down notes of what Paul had said to her. Therefore, there is a foundation for the photocopy of the notes. Moreover, David showed Molly the copy of the notes while she was on the stand and she identified them as the ones that she took that day. This would suffice as authentication. Documents being admitted into evidence are also subject to the Best Evidence Rule. The Best Evidence Rule states that if a document is going to be admitted into evidence, then the original must be produced or the party must account for why the original cannot be produced. The federal rules of evidence have accepted photocopies of documents as satisfying the best evidence rule.

32 Therefore, the document has been properly authenticated and a photocopy will suffice as a representation of the original.

C. Relevance Logical Relevance: (See rule statement above.) Here, Paul's statements are logically relevant. They tend to prove whether or not Paul was at fault in the accident more probable than not. Whether or not Paul was at fault or not is a fact of consequence to this case since a central issue is who was at fault to the accident. Legal Relevance: (See rule statement above.) These statements are more probative than prejudicial. There are not statements that might prejudice Paul because they are statements that Paul himself stated. Offer to Pay Medical Expenses: However, there are some types of evidence that are not admissible for public policy reasons under the rule of legal significance. For example, evidence of insurance, subsequent remedial repairs, and offers to settle are inadmissible because as a society we want to promote people to carry insurance, rectify dangerous situations, and settle cases as not to clog the courts. Another such category is when one party offers to pay the medical expenses of the other party. Here, there are two statements that Paul made. The first is that he was at fault because he was driving too fast. The second is his offer to pay medical expenses for anyone injured. The ferenda rules of evidence will sever these two statements. Because the offer to pay medical expenses is inadmissible but the other statements made in connection with the offer are admissible.

D. Dual Hearsay: (See rule statement above.) The issue with the photocopy of Molly's notes is that there are two levels of hearsay. In order for a document that contains two levels of hearsay to be admissible evidence, there must be valid exceptions for both statements.

a. First Level of Hearsay: Paul's Statements.

33 The first level of hearsay is Paul's statements that he made to Molly. These statements were made at the scene of the accident presumably and thus are out of court statements. David will argue that the following exceptions apply: (1) Party Admission: An admission made by a party to the case is admissible because under the federal rules, it constitutes non-hearsay. Here, Paul admitted fault to the accident. He stated that he was driving too fast and explicitly said that he was at fault. Thus, this is a valid party admission and would be admitted as non- hearsay. (2) Statement Against Interest: Another category of non-hearsay is when a party makes a statement against interest. Statements against interest are any statements that an individual makes that are against his pecuniary interest. Here, stating that one is at fault for an auto accident would be a statement against his interest. Therefore, this exception would apply.

b. Second Level of Hearsay: Molly's notes The second level of hearsay is the notes that Molly wrote down on the paper. Molly wrote those notes on the day of the accident and not while in the courtroom. Therefore, the notes are Molly's out-of-court statements. David will argue that the evidence should be admitted because of the following two exceptions:

(1) Prior Recollection Recorded: Courts will admit prior recollection recorded if four elements are met. First, the witness must currently not be able to recall the facts that are in the writing. The facts state here that Molly testified that she has no recollection of the conversation. The second is that the writing be created by the witness or adopted by the witness. Here, Molly herself wrote down the notes. Third, the writing must have been made when her memory was still fresh. Here, Molly created the writing as soon as Paul was taken away in the ambulance; therefore, we can assume that her memory was still fresh. Fourth, the writing must have been made under reliable conditions. Here, there is no evidence of an alternative purpose that Molly created the writing except for the document [sic] the events as they occurred. If all of these elements are satisfied, the recollection may be read into evidence; however, the photocopy should not be admitted into evidence.

34 (2) Present Recollection Refreshed: A party can refresh a witness' memory with virtually any document. Therefore, if Molly did not recall the events, David could have shown Molly the document and allowed her to look over the writing. If this refreshed her memory, then she could testify as to her knowledge of the events. In this situation, the writing would normally not be entered into evidence unless the opposing party suggested that it be admitted. However, this does not apply because Molly was shown the document, but then did not review it or subsequently answer questions based off of her review.

In conclusion, the photocopy should not have been entered into evidence because even though there were valid hearsay exceptions applied, the appropriate way to admit the evidence would have been to read the evidence into the record as opposed to giving the jury the photocopy.

35 Question 3 Answer B

The case between Paul in [sic] David is a civil case, which means there are a few different rules than when you are in a criminal case. This case is about injuries arising out of an automobile accident in which Paul is suing David. At issue is going to be who is at fault for the injuries and the accident.

1. Did the court err in admitting Vera's statement? Vera's statement was made while she was a passenger in the car with Paul on the day of the accident. She stated in a calm manner that she saw a black SUV behind them weaving recklessly through the traffic.

Logical Relevance All evidence must be relevant to be admissible. This includes tending to prove or disprove a fact that is of consequence. Even if evidence is relevant it may be inadmissible if it is not legally relevant.

Here, Vera's statement is being offered to prove the identity of a vehicle that she observed driving recklessly, which is the same vehicle that David drives. It is also relevant to prove that Paul had notice/was aware of the black SUV driving radically. Additionally, it is relevant to prove that David was at fault and was driving recklessly.

So although Vera's statement has logical relevance its probative value must be determined.

Legal Relevance Evidence that is logically relevant may be excluded if it will create an unfair prejudice. The court has discretion as to whether or not to exclude the evidence. The test to determine whether the evidence should be excluded on a legal relevancy

36 ground is whether the unfair prejudicial effect substantially outweighs the probative value.

Here, the prejudicial effect will be that David will be determined to have driven recklessly by weaving in and out of traffic. However, this is highly probative and is what is at issue and being determined in the case, so Vera's statement will not be excluded on grounds of legal relevance.

Even relevant evidence that is otherwise admissible can be inadmissible when it is in violation of one of the federal rules of evidence.

One of the objections that David could make regarding the admissibility of this evidence, besides relevancy, would be hearsay.

Hearsay Hearsay is a rule which prevents out-of-court statements from being admitted into evidence, if the statement is being offered for the trust of the matter asserted. The reason hearsay evidence is prohibited is because it was not subject to cross- examination and cannot be determined if the statement was fabricated or reliable. Since the information in Vera's statement about a black SUV driving recklessly would be helpful to a jury or trier of fact and is being offered to prove that the reckless driving of the SUV did in fact take place it is being offered for its truth and should be excluded unless a hearsay exception or exemption applies.

Hearsay Exceptions Hearsay exceptions are statements that are made out of court and are admitted for their truth but we allow them in for other reasons. Here, Paul will try and argue that Vera's statement should get in under several different exceptions.

Present Sense Impression A present since impression is an exception to hearsay because it is considered to have reliability given the fact that the statement is made while or immediately after

37 perceiving an event. There seems to be little time to fabricate a statement when it is made while you are perceiving it.

Here, Paul is going to argue that Vera made the statement while still in the car when she saw the black SUV weaving recklessly through traffic. She was currently perceiving the SUV driving in such a manner and made the statement while making the observation. It is of no matter that she made the statement calmly because this does not negate that she had just observed the SUV driving recklessly.

David might try and counter that Vera did not make the statement immediately when she observed the car driving recklessly, but there are no facts to support that she didn't make the statement while she was observing. Also statements are allowed to be made immediately after observation, because there is still the indication that there is not time to fabricate. Absent any facts showing that Vera waited any amount of time after observing the SUV driving recklessly and telling Paul this statement could come in under the present sense impression.

Excited Utterance Excited utterance allows hearsay evidence to come in if the statement was made while under the stress or effect of an exciting or startling event. Here, Paul might try and claim that Vera commented on the SUV's reckless driving while she was still under the stress of the observation. However, David will have a valid argument against this contention because Vera calmly told Paul about the SUV and did not seem to be effected by it in a manner to justify an excited utterance.

Former Statement Former statements can be admitted as long as the declaring is unavailable. Unavailability of a declaring can be because of death, not able to locate after reasonable attempts, and/or incapacity. Here, Vera is dead so she is unavailable. Former statements that are made under oath at a previous proceeding can be admitted for impeachment purposes and to prove the truth of the matter asserted. Here, Vera's statement was not made under oath at a formal proceeding and could

38 only be used for impeachment. However, since there is no one to impeach because Paul is offering his case and chief [sic] as a plaintiff, thus going first, this statement cannot be admitted as a former statement even though Vera is unavailable.

Dying Declaration Dying declarations are allowed in criminal homicide cases as well [as] civil. Here, we are in a civil case so a dying declaration is allowed as long as the declaring is unavailable, they do not have to actually die, they made a statement regarding the cause of their death, and they made the statement under the belief that death was impeding or imminent. Here, there is no valid argument to support that Vera's statement was a dying declaration since she made the statement prior to Paul's car being struck by the black SUV and prior to her death. Even though Vera is now unavailable she did not make a statement thinking she was going to die or describing the cause of her death and this exception is not available for Paul to get Vera's statement admitted.

Personal Knowledge Personal knowledge is required for a witness to be able to testify as to an event. While Paul did not personally observe the black SUV driving recklessly as Vera did, he did perceive Vera's statement with one of his 5 senses and thus has personal knowledge that the statement was made and the manner in which it was made.

Hearsay Exemptions These statements are not hearsay because they are not admitted to prove the truth of the matter and are admitted for a different purpose. Here, Paul is going to argue that Vera's statement should come in as non-hearsay under several different grounds.

Effect on the hearer Effect on the hearer is not being offered to prove the truth of the matter and thus is not hearsay. This is offered to show the effect the statement had on the person hearing the statement. Here, Paul could assert this statement is being offered to

39 show that Paul was aware of a black SUV that was driving recklessly. Since Paul's driving is also being put at issue by David this is important for Paul to prove that he was on alert of the black SUV driving recklessly that struck him 30 seconds after hearing the statement from Vera.

Conclusion Because this statement could fall under the present sense impression exception and effect on the hearer exemption to hearsay this statement cannot be excluded on hearsay grounds and the court properly admitted Vera's statement.

2. Did the court err in admitting the photocopy of Molly's notes? Logical/Legal Relevancy Molly's notes are relevant to prove that Paul made a statement accepting fault and offering to pay medical bills. They are being offered by David for this matter and to prove that it is true as well. Although relevant to determine fault the evidence must also not be unfairly prejudicial.

Policy reasons to exclude relevant evidence Certain evidence although relevant will be excluded because of public policy reasons. Courts want to encourage parties to fix wrongs, settle cases, and help each other out. Here, Paul will argue that the notes should be excluded because they were an offer to pay medical bills. Offers to pay medical bills cannot be offered to show fault of a party.

Although offers to pay medical bills of the injured [sic] is not allowed into evidence under the federal rules of evidence, the FRE severs statements made in connection with the offers and allows them into evidence. Here, Paul made the statement that he was driving too fast, was at fault, and offering to pay medical expenses of anyone injured.

40 The statements regarding Paul driving too fast and being at fault will not be excluded under this policy reason but may be excluded on other grounds (see discussion below).

Error in allowing an offer to pay medical expenses So in regards to the court allowing in a photocopy of a document that included the offer to pay medical expenses there is an error because public policy seeks to keep these sorts of statements excluded.

The statement regarding Paul driving too fast and being at fault

The photocopy of Molly's notes being admitted constituted a recorded recollection and is actual evidence being admitted. All tangible, physical, non-testimonial evidence that is being admitted must be authenticated in order to be admitted.

Authentification Here, Molly is on the stand claiming that she wrote the notes immediately after the accident and that the notes are hers. This is sufficient to authenticate the notes because Molly is claiming they are what David purports them to be and she is on the stand and capable of being questioned as to the notes’ authenticity.

Refreshing Recollection Anything can be used to refresh a witness’s recollection. Here, David is attempting to use notes to refresh Molly's recollection. Witnesses must be shown whatever is attempting to refresh their recollection in order to see if the item is successful in helping them recall. Whatever is used to refresh a witness’s recollection may be offered into evidence by the opposing party.

Here, it is not Paul offering the notes used to refresh Molly's recollection into evidence; it is David, which means he is attempting to offer the notes as a recorded recollection.

41 Paul may argue that Molly was not given the notes before claiming that her memory failed and thus the rules regarding admitting record recollection evidence were not followed. Generally a witness should be given the document to review silently and then if they still cannot remember the document may be admitted into evidence. Paul may have a valid argument here since the facts do not say that this was done. It appears from the facts that Molly before even reviewing the document said she couldn't remember, then it was moved into evidence.

Record Recollection Documents offered into evidence that were used to refresh a witness’s recollection are permitted so long as the witness’s memory has failed to be refreshed, the witness is on the stand and able to be crossed and authenticate the document, the witness accurately prepared the document close in time to perceiving the events, and had personal knowledge of the thing to which they recorded information about.

Here, Molly did testify that she was unable to recall the conversation. She is on the stand and subject to cross and questioning. And she testified that she carefully wrote down the notes as soon as Paul was taken away in the ambulance; additionally she had personal knowledge of the conversation with Paul since she heard the conversation herself. Given these facts David would be able to properly admit the evidence as record recollection as long as no other restrictions exist permitting the admissibility of the evidence.

Best Evidence Rule The Best evidence rule is a rule which calls for the document itself to be admitted when someone is on the stand trying to testify as to the contents of the document. Here, Molly is trying to recall a conversation and the notes contain information about the conversation. Since the notes are her own memory and not of legal significance the best evidence rule does not apply.

However, Paul will try and assert that there is a problem with the best evidence rule as well as authentification because the actual note itself was not admitted and a

42 photocopy was admitted. Paul will try and argue that unless David can show a justifiable reason why a photocopy of the note and not the actual note was admitted there is a problem/violation with the best evidence rule. David will successfully counter that argument by claiming that a photocopy, properly authenticated, is an acceptable document to satisfy the best evidence rule.

Hearsay/ Multiple Hearsay See rule above and discussion above. Here we also have a case of multiple hearsay since there is a statement within a document both made/prepared out of court and being offered for the truth of the matter asserted. So both the statement and the document must meet their own separate hearsay exception or exemption. As discussed above the document itself can get in under the record recollection rule but there needs to be an exception for the actual statements.

Party Admission- Party admissions are considered non-hearsay and are statements offered by a party opponent made by the other party. These statements do not have to be against interest necessarily but they must be made by one party and offered by the other. Here David is attempting to offer statements that Paul made, and although not required, are against his interest and regard his fault in the accident. This could be a valid ground for admitting the statements made by Paul.

Statement against interest David may try and assert that the statements made by Paul can come in under a statement against interest exception to hearsay. However, this exception requires that the declaring be unavailable which is not the case here, since Paul is the plaintiff in the matter and is available in court.

Conclusion The court was likely proper in admitting the evidence because the document can come in under the record recollection and the statement is admissible as a party admission.

43 FEBRUARY 2012 ESSAY QUESTIONS 4, 5 and 6

California Bar Examination

Answer all three questions.

Your answer should demonstrate your If your answer contains only a statement ability to analyze the facts in question, to of your conclusions, you will receive little tell the difference between material and credit. State fully the reasons that immaterial facts, and to discern the support your conclusions, and discuss points of law and fact upon which the all points thoroughly. case turns. Your answer should show that you know and understand the Your answer should be complete, but pertinent principles and theories of law, you should not volunteer information or their qualifications and limitations, and discuss legal doctrines that are not their relationships to each other. pertinent to the solution of the problem.

Your answer should evidence your Unless a question expressly asks you to ability to apply law to the given facts and use California law, you should answer to reason in a logical, lawyer-like according to legal theories and manner from the premises you adopt to principles of general application. a sound conclusion. Do not merely show that you remember legal principles; instead, try to demonstrate your proficiency in using and applying them.

44

ESSAY QUESTIONS AND SELECTED ANSWERS JULY 2012 CALIFORNIA BAR EXAMINATION

This publication contains the six essay questions from the July 2012 California Bar Examination and two answers to each question that were written by actual applicants who passed the examination after one read.

The selected answers were assigned good grades and were transcribed for publication as submitted, except that minor corrections in spelling and punctuation were made for ease in reading. The answers are reproduced here with the consent of their authors.

Question Number Contents Page

1 Civil Procedure 4

2 Community Property/Professional Responsibility 22

3 Evidence 40

4 Contracts 58

5 Wills and Succession 71

6 Criminal Law and Procedure 85

2

Question 3

Vicky was killed on a rainy night. The prosecution charged Dean, a business rival, with her murder. It alleged that, on the night in question, he hid in the bushes outside her home and shot her when she returned from work.

At Dean’s trial in a California court, the prosecution called Whitney, Dean’s wife, to testify. One week after the murder, Whitney had found out that Dean had been dating another woman and had moved out, stating the marriage was over. Still angry, Whitney was willing to testify against Dean. After Whitney was called to the stand, the court took a recess. During the recess, Dean and Whitney reconciled. Whitney decided not to testify against Dean. The trial recommenced and the prosecutor asked Whitney if she saw anything on Dean’s shoes the night of the murder. When Whitney refused to answer, the court threatened to hold her in contempt. Reluctantly, Whitney testified that she saw mud on Dean’s shoes.

The prosecution then called Ella, Dean’s next-door neighbor. Ella testified that, on the night Vicky was killed, she was standing by an open window in her kitchen, which was about 20 feet from an open window in Dean’s kitchen. She also testified that she saw Dean and Whitney and she heard Dean tell Whitney, “I just killed the gal who stole my biggest account.” Dean and Whitney did not know that Ella overheard their conversation.

Dean called Fred, a friend, to testify. Fred testified that, on the day after Vicky was killed, he was having lunch in a coffee shop when he saw Hit, a well-known gangster, conversing at the next table with another gangster, Gus. Fred testified that he heard Gus ask Hit if he had “taken care of the assignment concerning Vicky,” and that Hit then drew his index finger across his own throat.

Assuming all appropriate objections and motions were timely made, did the court properly:

1. Allow the prosecution to call Whitney? Discuss.

2. Admit the testimony of: (a) Whitney? Discuss. (b) Ella? Discuss. (c) Fred? Discuss.

Answer according to California law.

40

Answer A to Question 3

California Proposition 8: Truth in Evidence Rule Under Proposition 8 in California, all non-privileged, relevant evidence is admissible in a criminal prosecution brought in California unless it falls within one of the specified exceptions to the rule. Evidence that is admissible under Proposition 8 is still subject to CEC 352 balancing. Here, as this case involves the prosecution charging Dean with murder, Proposition 8 will apply to admit any evidence that is relevant and is not excluded for CEC 352 balancing.

1. Allow the Prosecution to call Whitney The first issue is whether the prosecution should be allowed to call Whitney. This depends on whether Whitney ("W") can claim one of the spousal privileges: spousal communications privilege or spousal testimonial privilege.

Spousal Communications Privilege

The spousal communications privilege protects all confidential communications between spouses that are made in the course of an existing marriage and in reliance on the intimacy of the marriage. This privilege belongs to both spouses and may be claimed by either to prevent the other spouse from testifying. Moreover, the privilege exists regardless of whether the marriage has ended in divorce, so long as the communication itself was made during a period when the marriage existed. For purposes of the privilege, marriage does not end until there is a valid divorce.

Here, Whitney was called by the prosecution to testify that she saw mud on Dean's shoes. This observation occurred when Dean and W were still married as Dean and W have yet to obtain a divorce and reconciled prior to W providing any testimony. Although W and D had separated because W had discovered that D was dating another woman and W had moved out, for the purpose of this privilege, it extends for any

41 communication made prior to divorce. Finally, as W was called to testify to an observation, rather than a communication between W and Dean, it would not be protected under the communications privilege.

Thus, this privilege would not apply to prevent W from testifying as she did or to prevent her from taking the stand.

Spousal Testimonial Privilege The spousal testimonial privilege allows one spouse to refuse to testify against another spouse in any action. For this privilege to apply, a valid marriage must still exist. The privilege belongs to the testifying spouse, as the privilege is designed to protect the harmony of the marriage, which is not salvageable if the testifying spouse wishes to testify. Moreover, in California, the privilege allows the testifying spouse to avoid taking the stand entirely.

Here, W was called to the stand to testify that she saw mud on D's shoes during the night of the murder. Although W and D had been separated, because W moved out and stated the marriage was over when she discovered that D had been dating another woman and moved out, the marriage had not ended for the purposes of the privilege, which requires a valid divorce. As such, W was privileged to choose not to take the stand.

In this case, W initially was angry and was willing to testify against D and thus agreed to take the stand and testify. W actually took the stand and was sworn in, prior to the recess in which W and D reconciled and W decided not to offer testimony. Thus, the prosecution will argue that W waived the privilege because she took the stand and was sworn under oath.

By contrast, W will assert that she did not waive the privilege because, although she took the stand, she asserted the privilege the first time that she was asked a question

42

by the prosecution. W refused to answer when court resumed and the prosecutor asked W if she saw anything on D's shoes at the night of the murder.

As W asserted the privilege prior to answering any questions, the court will find that she had a spousal testimonial privilege and could not be forced to testify against D. However, W took the stand voluntarily and thus it was proper to allow the prosecution to call W because she was the holder of the privilege and had not yet claimed it. Proposition 8 does not allow privileged information to be admitted and thus will not change the outcome.

2. Admit the Testimony

(a) Whitney

The first issue is whether the court should have admitted the testimony of Whitney.

Logical Relevance Under California law, evidence is relevant if it makes a fact of consequence that is actually in dispute more or less probable then it would be without the evidence. Here, W testified that she saw mud on D's shoes. As V was killed on a rainy night, and the prosecution was arguing that D hid in the bushes outside her home and shot her when she returned from work, this evidence would make it more likely that D was present in a muddy flowerbed and committed the murder. Thus, it is relevant.

Legal Relevance Evidence is legally relevant if its probative value is not substantially outweighed by the danger of unfair prejudice, confusion of the issues, misleading the jury, waste, or undue delay.

43

Here, D will argue that the testimony about mud on his shoes is likely to confuse and mislead the jury, particularly if the prosecution has failed to establish that the mud came from a flowerbed near Vicky's home. However, as this evidence has high probative value in that it shows that D was standing outside in mud on a rainy night, it will likely be admitted. Thus, this objection will fail.

Personal Knowledge In order to be competent to testify, a witness must have personal knowledge of the facts to which she is testifying based upon her percipient observations.

Here, W saw mud on D's shoes in the night in question and thus testimony about the state of the shoes is within her perception and personal knowledge.

Spousal Communications Privilege As discussed above, this will not protect W's testimony about the mud on D's shoes as it was not a communication, but was an observation.

Spousal Testimonial Privilege As discussed above, this will protect W's testimony because she is still married to D and therefore cannot be compelled to offer evidence against him in the criminal action. Prop 8 does not change the outcome as privileged information is excluded.

Conclusion W's testimony will be excluded as a result of the spousal testimonial privilege.

(b) Ella The second issue is the admissibility of Ella's testimony.

Logical Relevance See rule above.

44

Ella's testimony that she overheard D tell W that he "just killed the gal who stole my biggest account" is highly relevant to the case. D is charged with murder and his alleged motivation for killing Vicky is that they were business rivals. The statement thus indicates that D committed V's murder, particularly because it was made on the night that V was killed. This fact is in dispute as it relates to whether or not D is guilty of the crime with which he is charged. Thus, this testimony is logically relevant.

Legal Relevance See rule above.

Although D will argue that this statement is highly prejudicial and should be excluded because it could be misinterpreted and it fails to identify V specifically, the court will likely find that its probative value in showing that D committed the murder and that he had a motivation to commit the murder far outweighs the risk of prejudice. Moreover, the information goes to the heart of D's guilt or innocence.

Thus, the evidence will not be excluded on this ground.

Personal Knowledge See rule above.

Here, Ella was standing by an open window in her kitchen, which was about 20 feet from an open window in D's kitchen. Ella could both see D and W and could hear D tell W that "I just killed the gal who stole my biggest account." Thus, Ella's testimony was based on her percipient observations as she could personally see and hear what was happening in D and W's house.

Thus, this objection will be overruled.

Hearsay

45

Hearsay is an out-of-court statement that is offered to prove the truth of the matter asserted. Hearsay is inadmissible unless it falls within an exception or is being used for a non-hearsay purpose. Proposition 8 will not apply to admit otherwise inadmissible hearsay as hearsay is an exception to Proposition 8.

Here, Ella's testimony that D told W, "I just killed the gal who stole my biggest account" is offered to show that D was in fact the person who killed V. Thus, it is an out-of-court statement offered to prove the truth of the matter asserted and is only admissible if it falls within an exception.

Party-Opponent Admission A statement by a party-opponent regarding a relevant fact of the case is admissible over a hearsay objection as it is a California exception from the hearsay prohibition.

Here, the statement that Ella testified about was a statement by D, who is the defendant in the criminal action. This statement is highly relevant to the issues involved in the case because it indicates whether or not D actually committed a murder of V, for which he is being charged.

Thus, this exception would allow the statement to be admitted.

Statement Against Interest A statement is admissible under an exception if it qualifies as a statement against interest. A statement against interest is a statement of a now unavailable witness that was against the person's proprietary, pecuniary, penal, or social interest when made and that the declarant knew was against his interest when made.

Here, D made the statement to W that "I just killed the gal who stole my biggest account." This statement would be against D's penal interest, because it could subject him to prosecution for murder. Moreover, it could subject him to social ridicule, ostracism and humiliation because he would be labeled as a murderer. D will argue

46

that the statement was not against his interest because it was made to his spouse in reliance on the confidentiality of their marital relationship and thus he did not think that it could be used against him. Moreover, he did not believe at the time it was made that it would subject him to social disgrace as he expected his spouse to maintain the confidentiality of the statement. As D likely did not know that the statement could be used against his interest when it was made, this exception likely would not apply. A declarant is unavailable if he can claim a privilege against testifying. As D can claim the privilege against self-incrimination under the Fifth Amendment, he would be considered unavailable for the purposes of this exception.

Thus, this exception would not apply because D likely did not know it was against his interest when made.

Spontaneous Statement A spontaneous statement is a statement made shortly after witnessing a startling event and while the declarant was still under the stress of excitement.

Here, D made his statement to W and said "I just killed the gal..." indicating that he may still have been under the stress of excitement from the murder. Moreover, a murder is likely a startling event, especially when it involved hiding in the bushes and shooting someone at their home and then seeking to avoid detection.

Thus, D's statement might be a spontaneous statement if he was still experiencing the stress of excitement.

Contemporaneous Statement A contemporaneous statement is a statement made at or near the time of an event that explains or describes the defendant's actions.

Here, D told W, "I just killed the gal who stole my biggest account." Because D specified that he "just" killed a gal, the statement may have been made near the time of

47

the event. Moreover, the statement describes D's own conduct in killing the gal and explains his reasons for that conduct--she "stole my biggest account."

Therefore, provided it was made sufficiently close in time, it may qualify as a contemporaneous statement.

Spousal Communications Privilege See rule above. In addition, the spousal communications privilege is waived if the privilege is not made in reliance on the intimacy of the marriage. A statement is not made in this reliance, if it is made in the presence of a third person who does not fall within the privilege. If the spouses could not have reasonably foreseen that the communication would be overheard by a third party, then the privilege is not waived and D may prevent Ella from testifying on the basis of the privilege. However, if the spouses made the statement negligently when it could be overheard by a third party, then the privilege has been waived as no reasonable efforts were made to maintain its confidentiality.

Here, D and W had a conversation in their kitchen. No one else was present in the home and D and W were having an intimate conversation as spouses, thus suggesting that the conversation was made in reliance on the intimacy of the marriage. However, D and W had this conversation while the window to their kitchen was open. This window was only 20 feet from a neighbor's window which was also open and D was talking in a sufficiently loud voice such that Ella could overhear the conversation. But, because D and W engaged in a private communication between themselves and they did not know that Ella overheard the communication, they likely were not so negligent as to waive the confidentiality of the communications. D and W could rely on the privacy of their home, even with an open window.

Thus, the spousal communication privilege will prevent this testimony.

(c) Fred

48

Logical Relevance Fred's testimony that the day after Vicky was killed he was having lunch and heard that two gangsters had "taken care of the assignment concerning Vicky" is relevant to establish that Dean was not the person who killed Vicky. As whether or not D killed Vicky is the primary issue in the murder trial, this is both highly relevant and in dispute. This objection will be overruled.

Personal Knowledge Here, Fred was having lunch at a coffee shop when he saw Hit and Gus conversing and overheard the conversation. Thus, Fred had personal knowledge regarding the statements that were made.

This objection will be overruled.

Hearsay See rule above.

Here, F is offering testimony regarding the statements of both H and G, and both of these statements must fall within a hearsay exception in order to be admitted. These statements are offered to show that F and G committed the murder of Vicky.

G's Statement

Effect on Hearer D will argue that G's statement asking whether H had "taken care of the assignment concerning Vicky" is not offered to show the truth of that statement, as it was a question, but instead to show its effect on H, who answered the question. A statement offered to show the effect on the hearer is not hearsay and is admissible over a hearsay objection.

49

Here, as this question is offered to show the effect on H in answering, it will be admissible.

H's Statement Although H merely made a gesture by drawing an index finger across his throat, such an action can qualify as hearsay if it is intended to communicate. Here, H's conduct was done in order to answer G's question regarding whether or not H had "taken care of the assignment concerning Vicky." As this was intended to communicate that H had in fact gotten rid of Vicky, it will qualify as hearsay.

Statement Against Interest Here, this statement is against H's penal interest as he would be subject to prosecution for murder if he killed Vicky. As H made this statement while at a coffee shop where other people like F were around, H would know that he could be subject to punishment for making it at the time it was made. It is unclear whether H is unavailable and the admissibility will depend on this.

Thus, this is likely admissible testimony.

50

ANSWER B TO QUESTION 3

People v. Dean

1. Did the court properly allow the prosecution to call Whitney? Spousal Testimonial Privilege The California Evidence Code (CEC) contains a spousal privilege. The spousal privilege allows a defendant's spouse to refuse to take the witness stand and testify against his or her spouse. Although Dean's trial is a criminal trial, the CEC makes no distinction between criminal and civil trials--the spouse may refuse to testify against his or her spouse in either civil or criminal trials.

The spouse and defendant must be married during the time of trial. Here, although Whitney had moved out of the house prior to Dean's trial and said the "marriage was over," there is nothing to indicate that Whitney and Dean's marriage was legally dissolved. Thus, Whitney was married to Dean at the time of trial, and therefore can invoke the spousal testimonial privilege.

The spouse--not the defendant--is the holder of the privilege. Thus, even if Dean did not want Whitney to testify against him, Whitney could if she so chose, and so long as the matter she testified to was not otherwise privileged.

Under the CEC, the witness spouse may refuse to take the witness stand completely. Here, although Whitney initially took the stand, intending to testify against Dean, she could have refused to take the stand altogether. The issue is whether Whitney could later invoke the privilege after voluntarily waiving the spousal testimonial privilege.

The CEC does not dictate that a spouse has waived the spousal testimonial privilege once he or she takes the witness stand. Here, Whitney has testified to nothing yet. Thus, although she has taken the witness stand, she is still not otherwise

51

prohibited from invoking the spousal testimonial privilege. Thus, her testimony should not have been compelled.

However, the court did not err in allowing the prosecution to call Whitney to the witness stand because Whitney initially wanted to testify against Dean. Thus, error, if any, was on the court's compelling Whitney to testify, not on the court allowing the prosecution to call Whitney to the witness stand.

2. Did the court properly admit the testimony of Whitney, Ella, and Fred? Whitney Logical Relevance To be admissible, evidence must be relevant. Under the CEC, evidence is relevant if it has any tendency to make the existence of some fact of consequence to the action more or less probable than the absence of such evidence. The CEC further requires that to be relevant, the fact must be in dispute.

Here, Whitney's testimony that she saw mud on Dean's shoes is relevant because it makes a disputed fact--whether Dean was hiding in the bushes outside Vicky's home that rainy night--more probable than the absence of the evidence.

Legal Relevance Even if logically relevant, the court may exclude evidence if its probative value is substantially outweighed by the risk of unfair prejudice, confusing the issues, or misleading the jury. Here, the probative value of Whitney's testimony is relatively high. Because Whitney is Dean's wife, her testimony tending to inculpate Dean is especially probative. That Dean had mud on his shoes the night of the murder tends to show that Dean might have been hiding in the bushes that night. There is little risk of unfair prejudice because there is nothing to indicate that Whitney's testimony that she saw mud on Dean's shoes will cause the jury to have prejudice against Dean.

52

Spousal Testimonial Privilege As discussed above, Whitney should have been able to invoke the spousal testimonial privilege because she is married to Dean at the time of trial and thus may refuse to testify against him. Although she took the stand--which California allows a spouse to refuse to do--Whitney still had the privilege to not testify against Dean.

Confidential Marital Communications Privilege Whitney may attempt to alternatively invoke the confidential marital communications privilege. Any confidential communication between spouses is privileged and inadmissible. Here, however, Whitney testified as to an observation, not a communication. Whitney merely saw mud on Dean's shoes. Whitney did not testify as to any communication Dean made to her. Thus, the confidential marital communications privilege does not apply.

In conclusion, Whitney's testimony--although relevant--should have been excluded because of the spousal testimonial privilege.

Ella

Logical and Legal Relevance Ella's testimony that Dean told Whitney "I just killed the gal who stole my biggest account" is extremely relevant. If Dean told Whitney this, it tends to make it more probable that Dean in fact did kill Vicky. The probative value is high, and there is little risk of unfair prejudice as a result of Dean's statement to Whitney.

Hearsay Ella's testimony may be objected to on the grounds that it is hearsay. Hearsay is an out of court statement being offered to prove the truth of the matter contained therein. Here, Dean's statement is out of court because it was made in his home to his wife. If offered to prove that Dean did kill Vicky, it would be being offered for its truth. Thus, the statement is hearsay by definition.

53

Nonhearsay: Declarant's state of mind Dean's statement may be offered for the nonhearsay purpose of showing his state of mind. It could be offered to show Dean's intent to kill, rather than the fact that he did kill Vicky. However, if offered only for this purpose, it would be highly prejudicial because it would be very difficult for a jury to not consider the statement as evidence that Dean actually killed Vicky. Thus, it should not likely be admissible solely for this purpose.

Admission of a party/opponent Alternatively, Dean's statement to Whitney could be offered for its truth if it comes under a hearsay exception. The CEC provides an exception to the hearsay rule for admissions made by parties and offered by an opponent. Here, Dean's statement to Whitney is a statement made by Dean--a party--and offered by the prosecution--an opponent. Thus, although hearsay, Dean's statement may be admissible as an admission--an exception to the CEC's rule against hearsay.

Confidential Marital Communications However, Dean may seek to exclude his statement to Whitney on the grounds that the statement was a confidential communication between spouses and thus is privileged. Both spouses are holders of the privilege. Here there is a twist because a third person is attempting to testify as to a confidential communication between spouses. Both Dean and Whitney did not know that Ella overheard their conversation. Thus, Dean and Whitney believed Dean's statement to be in confidence. Ella was standing 20 feet away and in the house next door when the statement was made. If Dean and Whitney's belief that the communication was confidential was reasonable, such communication was privileged. Here, it appears that Dean and Whitney's belief that their communication was in confidence was reasonable--notwithstanding the fact that Ella overheard the communication 20 feet away.

The purpose of the confidential marital communications privilege is to foster the confidence of the marital relationship, and to encourage open and honest

54

communication. Here, if Ella is permitted to testify as to Dean's statement if Dean and Whitney reasonably believed their communication was made in confidence, such an allowance would seem to go against the grain of the purpose of the confidential marital communications privilege. Spouses should not have to take every measure to ensure their communications are confidential so as to invoke the benefit of the confidential marital communications privilege. A reasonable belief that the communication is made in confidence should be sufficient. Here, the court should not allow Ella's testimony for this reason.

Logical and Legal Relevance Fred's testimony that Hit implicitly admitted to killing Vicky is relevant because it makes it more probable that Dean did not kill Vicky. Assuming that the Vicky that Gus was talking about was the same Vicky who died the day before, such evidence would be extremely probative to show that Dean was not the killer, but Hit was.

Hearsay Hearsay is an out-of-court statement. To be a statement, there must be some assertive words or conduct. Although Gus's question to hit was out of court, it was not a statement because it was not assertive. A question is not an assertion. Thus, Gus's question to Hit whether Hit had taken care of the assignment concerning Vicky was not hearsay.

The issue becomes whether Hit's drawing his index finger across his throat was assertive conduct. Taken in light of the surrounding circumstances, Hit's conduct seems to indicate that Hit acknowledged to Gus that he in fact killed Vicky. To be hearsay, the declarant need not utter actual words. Here, the judge would use his or her discretion in deciding whether Hit's conduct was assertive. The court should hold that the conduct was assertive when taken in context with Gus's immediately preceding question.

55

Because Hit's assertive conduct was made out of court, and if offered to prove the truth--that Hit did kill Vicky--it is hearsay by definition. Hearsay is inadmissible absent any exception.

Statement against Interest Dean may argue that Hit's statement was a statement against interest. However, for a statement against interest to be admissible, it must be shown that the declarant is "unavailable" to testify. No such showing has been made, and therefore Hit's statement may not be admitted as a statement against interest.

Admission Hit's statement cannot come in as an admission because Hit is not a party to the action.

Present Sense Impression/Contemporaneous Statement Hit's statement may not be admitted under the present sense impression/contemporaneous statement exception because Hit's statement was not made either while killing Vicky or immediately thereafter. Also, Hit was not describing his conduct, he merely made a motion tending to indicate that he killed Vicky. Thus, this exception does not apply.

Confrontation Clause The Sixth Amendment right to confrontation applies to the states, including California, and provides that criminal defendants shall have the right to be confronted with the witnesses against them. Here, because Dean is offering the out-of-court statement made by Hit, the Sixth Amendment right of confrontation does not apply.

Conclusion Because Hit's conduct was assertive, given the surrounding circumstances, and because it is only relevant to prove the truth of his statement--that he killed Vicky, and thus inferentially, Dean did not kill Vicky--Hit's statement was hearsay. Because no

56

exception to the rule against hearsay applies, Hit's statement should not have been admitted.

57

The State Bar Of California Committee of Bar Examiners/Office of Admissions

180 Howard Street • San Francisco, CA 94105-1639 • (415) 538-2300 845 S. Figueroa Street • Los Angeles, CA 90017-2515 • (213) 765-1500

ESSAY QUESTIONS AND SELECTED ANSWERS

JULY 2014

CALIFORNIA BAR EXAMINATION

This publication contains the six essay questions from the July 2014 California Bar Examination and two selected answers for each question.

The answers were assigned high grades and were written by applicants who passed the examination after one read. The answers were produced as submitted by the applicant, except that minor corrections in spelling and punctuation were made for ease in reading. They are reproduced here with the consent of the authors.

Question Number Subject

1. Contracts/Remedies

2. Evidence

3. Business Associations / Professional Responsibility

4. Criminal Law and Procedure

5. Trusts / Community Property

6. Torts

Question 2

Pete was a passenger on ABC Airlines (ABC), and was severely injured when the plane in which he was flying crashed because of a fuel line blockage.

Pete sued ABC in federal court, claiming that its negligent maintenance of the plane was the cause of the crash.

At trial, Pete’s counsel called Wayne, a delivery person, who testified that he was in the hangar when the plane was being prepared for flight, and heard Mac, an ABC mechanic, say to Sal, an ABC supervisor: “Hey, the fuel feed reads low, Boss, and I just cleared some gunk from the line. Shouldn’t we do a complete systems check of the fuel line and fuel valves?” Wayne further testified that Sal replied: “Don’t worry, a little stuff is normal for this fuel and doesn’t cause any problems.”

On cross-examination, ABC’s counsel asked Wayne: “Isn’t it true that when you applied for a job you claimed that you had graduated from college when, in fact, you never went to college?” Wayne answered, “Yes.”

ABC then called Chuck, its custodian of records, who identified a portion of the plane’s maintenance record detailing the relevant preflight inspection. Chuck testified that all of ABC’s maintenance records are stored in his office. After asking Chuck about the function of the maintenance records and their method of preparation, ABC offered into evidence the following excerpt: “Preflight completed; all okay. Fuel line strained and all valves cleaned and verified by Mac.” Chuck properly authenticated Sal’s signature next to the entry.

Assuming all appropriate objections and motions were timely made, did the court properly:

1. Admit Wayne’s testimony about Mac’s question to Sal? Discuss.

2. Admit Wayne’s testimony about Sal’s answer? Discuss.

3. Permit ABC to ask Wayne about college? Discuss.

4. Admit the excerpt from the maintenance record? Discuss.

Answer according to the Federal Rules of Evidence. QUESTION 2: SELECTED ANSWER A

1) Wayne's Testimony about Mac's question to Sal

Logical Relevance; in order to be logically relevant, the evidence must make a fact that is of consequence in determination of the action more or less probable than without the evidence.

Here the evidence with regard to Wayne's testimony is highly relevant in that it tends to establish that Mac's (M) supervisor Sal (S) had notice of a potential problem with the aircraft prior to flight. Moreover, the second part of the statement shows, the ABC had the opportunity to do a systems check that was part of the routine operation, but ultimately failed to do so. It thus makes it more probable that ABC's employees were negligent in maintaining the aircraft, because S had notice of a problem and took no corrective action.

Legal Relevance - relevant evidence may be excluded if its probative value is substantially outweighed by the danger of unfair prejudice, waste of time, or confusion of the issues. Here ABC will argue that the evidence is highly prejudicial to ABC since it demonstrates that one of its employees noted a problem and stated, that corrective action should be taken. This is unlikely to be well received by the court, since, it is prejudicial, but not unfairly so, since it does not tend to arise the emotions or passions of the jury. Further, the evidence is highly probative in that one of its employees noticed a potential problem and recommended corrective action. As such, the statements about Mac's statements are legally relevant with the probative value not being substantially outweighed by unfair prejudice.

Hearsay: hearsay is defined as an out of court statement offered for the truth of the matter asserted. Here the statement by M was made out of the current proceeding in court, thus it was made out of court. The first part of Mac's statement is an assertion and thus definition be considered a statement. However the second part of the statement with regard to the systems check is actually a question (further explained below), and as such is not an assertion. Accordingly it would fall outside the definition of hearsay as discussed below. Finally, both parts of the statement may be being offered for their truth. That M noticed a problem and cleared out the fuel lines, and that M asked whether they should conduct a full systems check. This would be offered to show that there was actually a problem detected in the aircraft.

Alternatively however, Pete (P) could argue that he is offering this evidence not for its truth, but only for the purpose of showing the effect on the hearer (S). As such, P is only showing that S had notice of a potential problem and failed to take corrective action. If the evidence were offered only for this purpose, it is admissible and not hearsay.

Assuming that P wants to offer the evidence for its truth (that there actually was a problem detected: a) First part of statement regarding fuel reading and clearing the gunk from the line Because the first part of the statement is hearsay, it will be inadmissible unless a hearsay exception applies, or the federal rules deem the statement Non-hearsay under an exemption.

Hearsay within Hearsay - when there are multiple levels of hearsay - each independent level of hearsay must be satisfied either by an exception or exemption.

1st Layer - The reading on the fuel gauge. ABC might try to argue that this is an independent level of hearsay, and is an out of court statement being offered for the truth of the matter asserted. This argument would be unavailing however, since gauges which simply provide readout of data (which is not entered by a human) are not considered statements under the traditional hearsay definition. As such the first layer with regard to the fuel indicator would be deemed non-hearsay and would be admissible.

2nd layer - The statement itself A statement that is made by a party opponent is admissible against that party when introduced by an opposing party. Further, within this exception, an employee's statement related to a matter of employment, while within the scope of employment are exempt from the hearsay definition under this exemption. Similarly, the statements by spokespersons or agents for an individual can be admitted under this exemption. In sum, under the FRE, statements under this exemption are deemed non-hearsay and can be offered for the truth of the matter asserted. Here the statement made by Mac is was made while he was employed with ABC and related directly to matter of his employment - the mechanical evaluation of the plane before flight. As such it would be deemed non-hearsay and admissible.

Present sense impression - a statement made while contemporaneously perceiving and event and describing that event may be admissible under the present sense impression exception. Here, the statement involves M relaying what he just read and the actions he took on the line. If it was made right after the observations, which it appears to be, it would also be admissible under the present sense impression hearsay exception. b) Second part of statement with the question regarding the systems check Here as indicated above, M is actually asking a question, as to whether they should perform a systems check As such it would fall outside the hearsay definition regarding. A statement under the hearsay definition requires an assertion. As such a question cannot be considered hearsay, and would be properly admissible.

In sum, the evidence of Mac's question is properly admissible both for its truth and for the effect on the hearer to show negligence. 2) Wayne's Testimony about Sal's answer

Logical Relevance; in order to be logically relevant, the evidence must make a fact that is of consequence in determination of the action more or less probable than without the evidence. Here the evidence is clearly logically relevant, it shows that S believed that the gunk wouldn't cause any problems, and more importantly did not take any corrective action upon hearing the findings of Mac.

Legal Relevance - relevant evidence may be excluded if its probative value is substantially outweighed by the danger of unfair prejudice, waste of time, or confusion of the issues. Here, there does not seem to be any danger of unfair prejudice, and thus is legally relevant.

Hearsay - an out of court statement offered for the truth of the matter asserted. Here the statement is made out of court and is likely being offered for the truth of the matter asserted, namely that as the supervisor, S took no corrective action with regard to the plane.

Because it is hearsay it will be inadmissible unless an exception applies.

Non-hearsay, as statement by part opponent (an employee). As defined above, the statement by S will be deemed a statement of party opponent (ABC) since it related to a matter of employment (inspecting the aircraft) and was made while S was employed with ABC. As such, it will be deemed non-hearsay and is properly admitted.

3) ABC inquiry to Wayne about college

Logical Relevance; in order to be logically relevant, the evidence must make a fact that is of consequence in determination of the action more or less probable than without the evidence. Here the evidence is relevant because it tends to impeach the credibility of Mac a testifying witness. As such it logically relevant because it may make the jury not believe his testimony, and impact the outcome of the proceeding.

Legal Relevance - relevant evidence may be excluded if its probative value is substantially outweighed by the danger of unfair prejudice, waste of time, or confusion of the issues. Here, the jury may give unfair weight to the evidence, and discredit Wayne's (W)'s testimony. However, it is unlikely a court would find this unfair prejudice, and it probative value is high, since it tends to demonstrate W has been untruthful in the past. As such it would be legally relevant.

Impeachment - prior instances of uncharged conduct - probative of truthfulness - on cross-examination a party is permitted to inquire in specific instances of uncharged prior bad acts if they are probative of truthfulness. It bears noting however, that counsel is bound by the witnesses answer and may not provide extrinsic evidence to prove up the prior bad act.

Here, ABC's counsel is asking W about a specific instance of uncharged conduct - the lying in the course of a job application. Because the lying on a job application with regard to whether W went to college links directly on W's truthfulness as a witness, it is properly admitted. Additionally, since ABC's counsel did not try to introduce extrinsic evidence of the bad act, its form of introduction into evidence was also proper.

4) Excerpt from the maintenance record

Logical Relevance; in order to be logically relevant, the evidence must make a fact that is of consequence in determination of the action more or less probable than without the evidence. Here the evidence is relevant in that it demonstrates that the fuel lines were cleaned and the preflight checks were completed. As such it is relevant, to show that proper care was taken before flight, and less likely ABC was negligent in performing maintenance. Legal Relevance - relevant evidence may be excluded if its probative value is substantially outweighed by the danger of unfair prejudice, waste of time, or confusion of the issues. Here there are no issues with danger of unfair prejudice; the evidence is also legally relevant.

Hearsay - an out of court statement offered for the truth of the matter asserted. Here the maintenance records are made out of court; they are a statement and are being introduced for the truth of the matter asserted. Specifically, that the maintenance was in fact performed. As such they will be inadmissible unless a hearsay exception or exemption applies.

- Hearsay within hearsay: here there are two levels of hearsay. The first is Mac's entries and the second is the business record itself, each must independently satisfy the hearsay exception.

Statement by Party Opponent Here the entries by Mac would fall not fall under the statement of party opponent exception because they are being offered by ABC and not P. As such an alternate exception must be used.

Business Record Exception - a report that is created within the regular course of business, is recorded contemporaneously or near after the action of the business, and has indications of reliability can be offered under the business record exception. The business records will be inadmissible if they contain entries by a person who is not under a business duty to report, or are completed with anticipation of litigation.

Here, the custodian of records is proffering the business records. The custodian testified how the records were prepared and their method of preparation. Assuming there were no indicators of untrustworthiness the records are properly admitted. It bears mentioning that the custodian can properly authenticate the signature if he was familiar with the handwriting of Sal. Additionally, the hearsay within hearsay problem is alleviated because the business record exception covers all employees who are creating and contributing to the record who fall under the business duty. As such, M's statements would be properly admitted within the business record. QUESTION 2: SELECTED ANSWER B

1. Ok to Admit Wayne's testimony about Mac's question to Sal

Relevance = The testimony is logically and legally relevant. For an evidence to be admissible, it must be relevant. To be relevant, the evidence has to have any tendency to make any fact that is of consequence to the determination of the action more or less probable than without the evidence. Here, Wayne's testimony is most likely logically relevant because Mac's question ("Shouldn't we do a complete systems check of the fuel line and fuel valves?") shows that Mac and Sal, both ABC employees, was on notice that Mac thought they should do a complete systems check of the fuel line and fuel valves. Because Mac has stated that he just cleared some gunk from the line, he probably though more gunk would exist in other parts of the fuel line and valves. If ABC employees thought this way, then this could be relevant to prove that ABC knew that plane had some fuel line blockage problem before operating.

Even if the evidence is relevant, court may not admit the evidence if its probative value is substantially outweighed by unfair prejudice, waste of time, or confusion. Here, ABC would argue that this was only a question by Mac, and it does not indicate whether Mac actually thought there would be Gunk in other parts in the fuel line and valves. ABC would further argue that this question would confuse the jury (if this is a jury trial) to think that the employees actually thought there would be gunk in other places in the fuel lines and valves. However, Wayne's testimony is relevant, and is not substantially outweighed by any unfair prejudice. Although it would prejudice ABC, it is not unfair since opposing party's evidence would most likely be prejudicial to the other party due to nature of the adversarial setting of the trial.

Hearsay = The testimony is either not hearsay or falls under an exception Hearsay is an out of court statement offered to prove the truth of the matter asserted. Statement can be a conduct or question as long as it is intended by the declarant to communicate something. Here, Mac's question was made outside of the court. Pete would argue that Mac's question is not hearsay because it is a question. However, this question appears to be communicating. Mac stated that he just cleared some gunk from the line, and asked Sal if they should do a complete systems check of the fuel line and valves. Because of his previous statement before the question, Mac's question seems to communicate to Sal that they should be doing some systems check to see if other gunk exists elsewhere. Thus, Pete's argument that this is not hearsay because it is a question will not be too good.

It is not hearsay if the purpose of introducing the statement is not to prove the truth of the matter asserted but to show effect on the listener. Here, this is double-edged sword for Pete. Pete can probably get this in if he argues that this question should be admitted to show the effect on Sal. However, he also wants this question admitted for the truth of the matter asserted to show that Mac most likely thought that gunk existed elsewhere in fuel lines and valves. Thus, Pete can use this argument, but probably is not a good one to make.

The most successful argument would be that this statement falls under a hearsay exemption of statement of party opponent. Statement of party opponent can be admissible even if it is an opinion statement. An employee's statement can be admitted against an employer if the statement was made during the employment and statement describes a matter within the scope of their employment. Here, Mac was employed as an ABC mechanic when he made his question. Also, his statement directly related to his scope of employment as a mechanic because he was talking about doing some system check on the plane. Thus, his question would be admissible as a hearsay exemption of statement against party opponent.

Pete can also use a hearsay exception of present sense impression. A statement describing a condition or event while the declarant is perceiving the condition or event or immediately thereafter is admissible under hearsay exception. Here, Mac stated that he just cleared some gunk from the line, and asking a follow up question to his work. Thus, Pete can argue that Pete was asking that question pursuant to his observation of his clearing of some gunk. ABC would argue that the question pertains to some future work that Mac is thinking about doing, so it does not relate to Mac's present sense impression of his past work completed. Even if ABC has a better argument here, this statement will pass the hearsay hurdle as a statement against party opponent.

Ok to Wayne's testimony about Sal's answer

Relevance = Sal's statement is logically and legally relevant Here, Sal's statement is logically relevant because it can show negligence of ABC. Sal was notified by Mac that the plane had some gunks, but decided not to do system check because "a little stuff" (i.e., gunks) is normal for this fuel. Pete would argue that ABC knew about the gunks and decided not to clean or do any further systems check. Thus, it bolsters Pete's claim of negligent maintenance of the plane by Mac when he was on notice that the gunk was present in the fuel line. Thus, this is logically relevant.

Additionally, this statement is not substantially outweighed by any unfair prejudice. ABC may argue that little gunks in plane is normal, and this evidence may mislead the jury to think that having little gunk would cause problems.

Although this evidence is prejudicial, this is not unfair because jury can weigh the evidence after it is admitted.

Hearsay = this is not a hearsay statement and falls under a hearsay exception Here, Sal's statement is a hearsay. His statement was made outside of the court; it was intended to communicate to Mac that little gunk is ok and that it would not cause problems; Pete is introducing this statement for the truth that Sal knew about there being some gunk and little gunk would not cause problems. Pete can argue that he is not offering this statement for the truth of the matter asserted but that Sal knew of some gunks and affirmatively decided not to conduct a system check even after being put on notice. In such a case, this statement would be admitted as non-hearsay. Like Mac's question, Sal's statement would fall under statement against party opponent. Sal made this statement when he was employed by ABC and it was within the scope of his employment as an ABC supervisor. As a supervisor, he would ordinarily make decisions on whether to do a systems check of the fuel line and valves, and his statements regarding decision not to do such check and reasoning behind such decision would be constituted as statement within his scope of employment. Thus, Sal's statement would be not a hearsay statement.

Pete can also argue that Sal's statement is then-existing state of mind hearsay exception. A statement of past mental or physical condition or then existing statement of mind is admissible even if it is a hearsay statement. Here, Sal is telling Mac to not worry because little gunk will not cause any problems. This shows Sal's lack of worry at the time the statement was made with respect to little gunk in the fuel line system. Thus, Sal's statement would also fall under this hearsay exception.

3. Ok to permit ABC to ask Wayne about college

Relevance This evidence of Wayne's lying on his job application is relevant because it goes to the credibility of the witness testifying in the court. Here, if Wayne is shown as a liar, it is relevant because then his other testimony cannot be fully trusted. Also, it is not outweighed by unfair prejudice. Jury can determine how much weight to give to a witness who has been impeached.

Leading Question ok here Leading question is permitted on direct examination in certain circumstances, but is generally allowed in cross-examination. Here, Wayne is being cross-examined, so it is ok for ABC's counsel to use leading questions.

Character Evidence vs. Impeachment = Impeachment with prior misconduct related to lying Character evidence is almost never allowed in civil cases except for few exceptions. Character evidence is given to prove that the person has acted in conformity with his character. However, under right circumstances this is ok if the purpose is to impeach the witness. A witness can be impeached with his prior misconduct related to lying. This impeachment can only be done on cross-examination and cannot be done with an extrinsic evidence. Here, Wayne is on cross-examination, so it was ok for ABC to ask Wayne about his lying on his job application about graduating from college.

4. Ok to admit the excerpt from the maintenance record

Relevance The maintenance record is relevant because it shows that preflight check was completed with all okays. The record also shows that fuel line strained and all valves were cleaned and verified by Mac. This shows proper maintenance on the part of ABC to counter Pete's negligent maintenance claim. Also, it is not substantially outweighed by any unfair prejudice.

Authentication proper When non-testimonial evidence is being introduced, it must be authenticated (i.e., prove the evidence is what it purports to be). This can be done several ways. One way is for a custodian of the record to testify to the creation or how the record gets maintained. Here, the maintenance record has been properly authenticated by Chuck, ABC's custodian of records. He testified that all ABCs maintenance records are stored in his office and discussed about the function of the maintenance records and their method of preparation. Also, facts indicate Chuck properly authenticated Sal's signature next to the entry.

Best Evidence Rule When a written document is introduced as an evidence, courts usually allow the original document or its duplicate (photocopy or another method to re-create the original) to be admissible to prove the content of the written document. However, handwritten copy is not admissible in lieu or an original or a duplicate. Although it is not clear whether the original maintenance record is being introduced, but it would be reasonable to assume that either an original or a duplicate is being introduced.

Hearsay This maintenance record is hearsay. It is made outside of the court. It was a statement intended to communicate that preflight check was completed, fuel line was strained and all valves were cleaned. ABC is offering this written statement for the truth of matter asserted so that proper maintenance has been conducted. To be admitted, it must fall under a hearsay exception.

ABC would argue that it falls under a hearsay exception of business records. To be a business record exception, it must be (1) a statement of diagnosis, opinion, condition, event, (2) kept at a regularly conducted business activity, (3) made at or near the time matter observed, (4) by personnel who had personal knowledge or gotten the information from someone who had duty to report, and (5) it is regular practice for business to make such record. Here, the maintenance records had statement of plane's condition because the maintenance was completed and the fuel line was strained and all valves were cleaned and verified. Also, it was kept at a regularly conducted business activity because it would be safe to assume that such preflight maintenance records are kept. Although it doesn't say when the record was created, it is reasonable to assume that these records are maintained as Sal and Mac do maintenance checks. Also, Sal as a manager probably has duty to report the maintenance record. Chuck also testified that all ABC's maintenance records are kept in his office, so it would be safe to assume that it is regular practice for ABC to make and keep these types of records. In conclusion, the maintenance records probably fall under business records hearsay exception.